MPRE Practice Questions

अब Quizwiz के साथ अपने होमवर्क और परीक्षाओं को एस करें!

A defendant, who has been indicted for auto theft, is represented by an attorney. The prosecutor reasonably believes that the defendant committed the offense, but, because of the defendant's youth, it is in the interest of justice to permit him to plead guilty to the lesser offense of "joy-riding" in return for an agreement by the prosecutor to recommend probation. The prosecutor has so advised the attorney, but the attorney told the prosecutor she would not plea bargain and would insist on a jury trial. The attorney informed the defendant of the prosecutor's offer and advised the defendant not to accept it. The defendant followed the attorney's advice. The attorney is a candidate for public office, and the prosecutor suspects that the attorney is insisting on a trial of the case to secure publicity for herself. Which of the following would be improper for the prosecutor? Send a member of his staff who is not a lawyer to consult with the defendant. Move the trial court to dismiss the indictment and accept a new complaint charging the offense of "joy-riding." Proceed to trial on the indictment and prosecute the case vigorously. All of the above would be improper.

Answer choice A is correct because it is not proper. MRPC 4.2 prohibits a lawyer who is representing a client from communicating about the subject of the representation with a person the lawyer knows to be represented by another lawyer in the matter, unless the lawyer has the consent of the other lawyer or is authorized to do so by law or a court order. Here, it would be improper for the prosecutor or someone working for the prosecutor to communicate with the defendant, whom the prosecutor knows is represented by counsel in the matter. Answer choice B is proper, as the prosecutor believes it is in the interests of justice to seek the conviction on the lesser offense. Prosecutors have discretion under the law regarding what to prosecute, so long as probable cause exists for the crime charged. Answer choice C is proper, as the prosecutor has discretion to decide to seek a conviction on the more serious charge of auto theft, for which probable cause exists. Answer choice D is incorrect because not all of the options are improper.

A general partnership litigation firm hires new law school graduates as associates. These new lawyers are largely left to their own resources to practice law. The firm accepts many small litigation matters and assigns them to the associates for training purposes. No senior partners are assigned to supervise this work. It is assumed that if an associate needs help on a case, he or she will seek the guidance of a more senior attorney. A client retained the firm to pursue a claim for breach of contract against the city. A first-year associate was assigned to the client's case. The associate failed to comply with the applicable 30-day notice requirement for filing a complaint against the city, and the client lost the chance to recover $5,000 owed to the client by the city. When the complaint was dismissed for failure to comply with the notice requirement, the associate instead told the client that the case was dismissed on the merits. Which of the following statements is NOT true? The law firm is subject to discipline for failure to supervise the associate. The individual partners of the firm are subject to discipline for failure to make reasonable efforts to establish a system providing reasonable assurance that all lawyers in the firm comply with the rules of professional conduct. The associate, an unsupervised subordinate lawyer, is subject to discipline for making misrepresentations to the client. Both the law firm and the associate are subject to civil liability for the client's loss.

Answer choice A is correct because it is untrue. The Model Rules impose the duty of supervision of a subordinate lawyer on the individual partners of a firm or the supervising lawyer, but not on the firm itself. MRPC 5.1. Answer choice B is a true statement (and hence incorrect) because partners in a law firm are required to make reasonable efforts to ensure that all lawyers in the firm comply with the Model Rules. MRPC 5.1(a) and 5.3(a). Answer choice C is incorrect because a lawyer owes a duty of loyalty to his client and may not misrepresent facts or withhold information to serve the lawyer's own interests. Answer choice D is incorrect, because the associate committed malpractice by failing to comply with the notice requirement, and the law firm would be liable for the associate's conduct on the basis of the doctrine of respondeat superior and the negligent supervision of the associate.

An attorney represented a father in a custody dispute with his ex-wife regarding their child. As a result of the custody proceeding, the father lost custody of his child. Subsequently, the father sued the attorney, asserting that the attorney failed to diligently represent him in the proceeding because she had taken on too many clients. At trial, the attorney seeks to disclose information she acquired from her client regarding his finances as evidence that he is suing because he is upset about the significant support payments he has to make. Is the attorney's disclosure of the financial information she acquired from her client proper? No, because her client's finances are not relevant to the dispute. No, because the information was acquired during the course of the representation. Yes, because a dispute between an attorney and a client allows for the disclosure of confidential communications. Yes, because the client, by filing suit against his attorney, consented to the disclosure of confidential information acquired by the attorney during the representation.

Answer choice A is correct. A dispute between an attorney and her client (e.g., a malpractice allegation) or between co-clients who are now adverse to each other allows for the disclosure of confidential communications relevant to the dispute. The attorney could disclose information gained during her discussions with the client that was relevant to his allegations against her. Financial information is not relevant to the dispute because the client never claimed there was any issue with attorney's fees or any other financial aspect of the case; the attorney should be able to prove that she was diligent without disclosing her client's financial information. Answer choice B is incorrect because the attorney might be able to disclose this information if it was relevant to the dispute. Answer choice C is incorrect because, while such a dispute does allow for the disclosure of certain confidential information, it only allows for the disclosure of information that is relevant to the dispute. Answer choice D is incorrect because, as noted with regard to answer choice C, the client has not consented to the disclosure of any information beyond that relevant to the dispute.

A judge is presently serving on a state intermediate appellate court. This court, in opinions written by her, has decided several controversial cases in which the court has held that the Fourteenth Amendment to the United States Constitution does not guarantee due process protection to state prison inmates who are disciplined by prison authorities for violating the prison's rules of conduct. The judge is now a candidate for election to a vacancy on the state supreme court. She is vigorously opposed by several organizations concerned with the conditions under which prisoners are incarcerated in the state's prison. The judge is scheduled to be interviewed on television and has been informed that questions will be asked of her concerning those decisions and her attitude on the subject of prisoners' rights. Which of the following is proper for the judge to say during the interview? Correct Answer: "I believe that the issues raised by the organizations opposing me are appropriate matters for legislative consideration." "In my opinion, incarceration for the commission of a crime carries with it a loss of civil liberties in prison discipline proceedings." "I am convinced I was right in those cases and will make the same decision in similar cases in the future." All of the above are proper.

Answer choice A is correct. A judge is permitted to speak at non-legal functions about the law, the legal system, and the administration of justice. Answer choices B, C, and D are incorrect, as CJC Rule 4.1 provides that a judge or candidate for judicial office must not, in connection with cases, controversies, or issues that are likely to come before the court, make pledges, promises, or commitments that are inconsistent with the impartial performance of the adjudicative duties of judicial office. This issue is likely to come before the state supreme court and the judge's statements in options B and C appear to commit her to a specific view.

A judge underwent an operation for a medical condition at a local hospital. After the operation, which was successful, the surgeon who performed the operation approached the judge about appearing in a television commercial. The purpose of the commercial was to encourage others in the community who required such an operation to have the operation done by the surgeon at the local hospital. The judge only briefly appeared in the commercial as one of three patients of the surgeon, all of whom had had the same type of operation. In the commercial, the judge truthfully stated that she was pleased to have her medical problem solved by the operation. While the judge was on-screen, she was visually identified by name and profession (i.e., "Judge ______") in a manner similar to the other two patients who appeared in the commercial. The judge did not receive compensation in any form for appearing in the commercial. Was the judge's participation in the commercial proper? No, because the judge abused the prestige of her office to advance the economic interests of the surgeon and the hospital. No, because, by revealing that the judge had the operation, the commercial reflects adversely on the judge's fitness to serve as a judge. Yes, because the judge was not compensated for appearing in the commercial. Yes, because the information about the judge, as well as the information conveyed by the judge in the commercial, was truthful.

Answer choice A is correct. A judge must not abuse the prestige of judicial office to advance the personal or economic interests of the judge or others. CJC Rule 1.3. In this instance, the judge's appearance in the commercial was intended to directly advance the economic interests of the surgeon and the hospital. Moreover, the judge not only personally appeared in the commercial, but also allowed herself to be identified as a judge, thereby lending the prestige of her office to advance those interests. Answer choice B is incorrect; since the commercial indicated that the judge's medical problem had been solved, it did not call into question the judge's fitness to serve as a judge. Moreover, the Code of Judicial Conduct is concerned with conduct by a judge that reflects adversely on the judge's fitness to serve as a judge, rather than on impairment of a judge due to a medical condition. CJC Rule 1.2, cmt. 5. Answer choice C is incorrect because the prohibition on a judge's use of the prestige of judicial office to advance personal and economic interests extends to the advancement of those interests by anyone. Answer choice D is incorrect because the prohibition focuses on the exploitation of the judge's office, not on the truthfulness of any information revealed during the exploitation process.

An attorney successfully pursued an action on behalf of a well-known client who was injured in an automobile accident. The attorney, realizing the value of the client's recommendation of her legal services, offered to forego a portion of her fee in exchange for the client appearing in a televised advertisement on behalf of the attorney. The client agreed and appeared in a commercial that was broadcast on several stations in the local television market. In the commercial, the client truthfully and honestly recommended use of the attorney's legal services for individuals wrongfully injured in an automobile accident. The attorney also appeared in the commercial and stated that her practice was limited to representing plaintiffs in personal injury actions. Is the attorney subject to discipline for her actions? Yes, because the attorney reduced the client's fee in exchange for the client's recommendation of her services. Yes, because the attorney may not state that her practice is limited to a particular field of the law. No, because the advertisement was not misleading or false. No, because the advertisement was not real time, direct contact with a prospective client to solicit paid employment.

Answer choice A is correct. A lawyer cannot generally give anything of value to a person for recommending the lawyer's services. MRPC 7.2, cmt. 5. Here, the attorney in effect gave her client a portion of her fee in the form of a rebate for recommending her services in the commercial. Answer choice B is incorrect because a lawyer may state the particular fields of law in which the lawyer does or does not practice. MRPC 7.4(a). Answer choice C is incorrect because, even though the lawyer's advertisement does not violate the Model Rule that a lawyer may not make a false or misleading statement about the lawyer or the lawyer's services (MRPC 7.1), the truthfulness of a lawyer's advertisement does not protect the lawyer from discipline for violation of another Model Rule of Professional Conduct. Answer choice D is incorrect because, even though a lawyer's advertisement does not violate the Model Rule regarding solicitation (MRPC 7.3(a)), this fact does not protect the lawyer from discipline for violation of another Model Rule of Professional Conduct.

A stock clerk employed at a warehouse witnessed, but was otherwise not involved in, an accident between a truck and a car. The accident occurred while the driver of the truck was backing into the loading dock at the warehouse. The driver of the car sued the corporate owner of the warehouse and the truck driver, who was also an employee of the corporation, alleging that the truck driver's negligence was the cause of the accident. The attorney hired to represent the corporation in this lawsuit learned that the stock clerk had witnessed the accident, and he interviewed the clerk. The clerk's version of the accident did not correspond with the truck driver's version, and in several details, it supported the car driver's explanation of the event. The attorney told the clerk not to discuss the accident with anyone, particularly the driver of the car or his attorney, unless contacted by either of them. Is the attorney subject to discipline for giving the clerk this instruction? No, because the clerk was a warehouse employee. No, because the attorney did not represent the stock clerk. Yes, because a lawyer must not obstruct another party's access to evidence. Yes, because a lawyer owes a duty of candor.

Answer choice A is correct. A lawyer generally must not request that a person other than a client refrain from voluntarily giving relevant information to another party. However, there is an exception to this rule for a person who is an employee or other agent of the client if the lawyer reasonably believes that that person's interests will not be adversely affected by refraining from giving such information. Because the clerk was a warehouse employee and, other than as a witness, uninvolved in the accident, the lawyer's direction that the clerk not give information about the event unless contacted by the car driver or his attorney is not cause for discipline. Answer choice B is incorrect because the attorney's request to the clerk, as an unrepresented third party, would have been improper had the clerk not been an employee or other agent of the attorney's client. Answer choice C is incorrect because, while a lawyer must not unlawfully obstruct another party's access to evidence, the attorney did not prevent the opposing party or his attorney from contacting the clerk or take action that would have that effect (e.g., advising the clerk to take an extended vacation in order to forestall such contact). Answer choice D is incorrect because, while an attorney does have a duty of candor, an attorney is not under a general duty to reveal facts unfavorable to a client, and in fact may be subject to discipline for breach of the duty of loyalty by voluntarily revealing such facts.

Pursuant to state law, a judge in juvenile court appointed an attorney as guardian ad litem to represent a child who was the subject of a physical abuse investigation. Since the child's family was indigent, the attorney would be compensated by the state at a rate that was less than half of his normal rate. The attorney declined the appointment on the grounds that, although he was competent to represent the child due to past representations of similarly situated clients, he had recently ceased to practice in the area of juvenile law. In addition, the attorney asserted that he was not required to provide legal services to persons of limited means at a substantially reduced fee from his standard fee and that a member of the firm with which the attorney's sister is a lawyer had been appointed to represent the alleged abuser. Is the attorney subject to discipline for his refusal to represent the child? Yes, because the attorney has a duty to accept a court appointment. No, because the attorney no longer practices in the area of juvenile law. No, because the attorney's obligation to provide legal services to persons of limited means without fee or at a substantially reduced fee is aspirational, rather than mandatory. No, because the member of the law firm with which the attorney's sister is associated is representing the alleged abuser.

Answer choice A is correct. A lawyer has a duty to accept a court appointment to represent a client unless a good cause exists. As discussed below with regard to the other answer choices, none of the reasons advanced by the attorney here constitute good cause for declining the appointment. MRPC 6.2. Answer choice B is incorrect because, while an attorney must be able to provide competent representation for the assigned client, the mere fact that the attorney does not currently practice in the area of the law to which the assigned matter relates is not a sufficient reason for declining a court appointment. Answer choice C is incorrect because, while a lawyer's obligation to provide legal services to persons of limited means without fee or at a substantially reduced fee is aspirational, rather than mandatory (MRPC 6.1), a lawyer has a mandatory duty to accept a court appointment unless good cause exists. Answer choice D is incorrect because, while a lawyer may refuse to accept a court appointment that would create a conflict of interest, the representation of an opposing party by an attorney who is related to the lawyer creates a conflict of interest that is personal to the related attorney. It is not imputed to other members of the firm with which the related attorney is associated. MRPC 1.7, cmt. 11; 6.1(a).

At the urging of his family members, a newly-licensed attorney made an appointment with his primary-care physician to discuss the attorney's binge drinking. The attorney was diagnosed by the physician as an alcohol abuser. The physician referred the attorney to a psychologist for a series of one-on-one counseling sessions spread over the next two months. Before the completion of the consulting sessions, the physician opined that the attorney's problem would materially impair the attorney's ability to represent a client. Prior to the beginning of the counseling sessions, a potential client contacts the attorney about representing her with regard to a possible adverse possession claim on certain real property with which she is concerned. Although he has never handled an adverse possession claim before, the attorney feels confident that he understands the law regarding adverse possession actions and can become familiar with the necessary procedures for filing and pursuing the action in a timely manner. In discussing the matter with the attorney, the potential client reveals sufficient information to allow the attorney to conclude that the potential client's adverse possession claim would likely ripen within the next three to four months. Eager to acquire title to the property as soon as possible, the potential client suggests that she would be willing to testify falsely that she had entered the property in question prior to the time that she had actually done so. The potential client then requests that the attorney represent her in an action to acquire title to the property by adverse possession. Must the attorney decline to represent the potential client? Yes, because the attorney's alcohol abuse presently materially impairs his ability to represent the potential client. Yes, because the potential client has suggested her willingness to engage in perjury. No, because the attorney can acquire the necessary competence to pursue the action in a timely manner. No, because the attorney has been licensed to practice law.

Answer choice A is correct. A lawyer has a duty to decline representation of a client when the lawyer's physical or mental condition materially impairs the lawyer's ability to represent the client. MRPC 1.16(a)(2). Here, the attorney's medical condition as an alcohol abuser materially impairs his ability to represent the potential client. Answer choice B is incorrect because, while a lawyer cannot represent a client who demands that the attorney engage in illegal behavior or behavior that would violate a Model Rule of Professional Conduct, such as knowingly permitting a client to commit perjury, a lawyer is not required to reject representation of a client who merely suggests such behavior. MRPC 1.16, cmt. 2. Answer choice C is incorrect because, while a lawyer can acquire the necessary competence to handle a matter provided the acquisition is done in a timely fashion (MRPC 1.1, cmt. 2), the attorney here, even after acquiring such competence, has a duty to reject the representation due to his drinking condition. Answer choice D is incorrect because, while a lawyer is properly authorized to provide legal services to the potential client by virtue of being licensed, the attorney here must nevertheless comply with the Model Rules of Professional Conduct, MRPC 5.5, cmt. 1, which impose upon the attorney a duty to reject the representation as explained with regard to answer choice A.

An attorney whose practice includes legal malpractice regularly reviews the published disciplinary actions taken by the highest court of the state against lawyers. He sends an e-mail to each lawyer who has been disciplined, offering his services to represent the lawyer in any future legal malpractice action brought against the lawyer. The e-mail does not state that it is an advertisement. In some circumstances, the attorney also calls the lawyer's office, properly identifies himself, and indicates that his purpose is to solicit business. If permitted to speak directly to the disciplined lawyer, the attorney briefly explains his practice and offers his services. Are the attorney's actions proper? Yes, because the prospective clients are all lawyers. Yes, because the attorney may use targeted electronic communication that does not involve real-time contact to solicit professional employment. No, because the attorney made live telephone contact with an individual in order to solicit the individual to utilize the attorney's legal services. No, because the attorney's e-mail failed to state that it is an advertisement.

Answer choice A is correct. A lawyer is generally prohibited from solicitation of professional employment from a prospective client that is done by live telephone. However, an exception exists when the prospective client is another lawyer. Also, a lawyer may send an electronic communication that does not involve real-time contact and that solicits professional employment from a prospective client known to be in need of legal services in a particular matter, if such communication includes the words "Advertising Material" at the beginning of the communication. However, an exception exists to including the words "Advertising Material" when the prospective client is another lawyer. MRPC 7.3. Answer choice B is incorrect because, while a lawyer may use targeted electronic communication that does not involve real-time contact to solicit professional employment, such communication must generally indicate that it is an advertisement unless, among other exceptions, the potential client is a lawyer. In addition, this answer choice fails to explain why the attorney's live phone call to the disciplined lawyer is proper. Answer choice C is incorrect because, while a lawyer may not generally make live telephone contact with a potential client in order to solicit the potential client to utilize the attorney's legal services, a lawyer may generally do so when the potential client is a lawyer. Answer choice D is incorrect because, while a lawyer who sends a targeted, but non-real-time electronic communication soliciting professional employment to a prospective client known to be in need of legal services in a particular matter must generally indicate that the communication is an advertisement, such notification is not necessary when the potential client is a lawyer.

A client has retained an attorney to represent him in a contract suit. The attorney's retainer agreement provided that the attorney's fees would be based on a fixed hourly rate, payable at the end of each calendar month. Two months before trial, the client fell behind in the payment of the attorney's monthly billing for fees. The attorney included the following statement on his last billing to the client: "Your account is more than 30 days past due. If amounts due are not paid promptly in accordance with our agreement, I will terminate the representation. If you cannot pay the amount due, I will accept an assignment of your cause of action as security for your fee to me." Two weeks after the last billing, the attorney telephoned the client and told him that he would withdraw from representation if the bill was not paid within 48 hours or adequate security given for its payment. If the bill remains unpaid or unsecured after forty-eight hours, it would be proper for the attorney to: You Selected: Upon notice to the client, move the court for permission to withdraw. Turn the client's file over to another experienced lawyer in town and notify the client that he no longer represents him. Accept an assignment of the client's cause of action as security for his fee. All of the above are proper.

Answer choice A is correct. A lawyer is permitted to move for the court's permission to withdraw from representing a client in a case when there has been non-payment of fees. Answer choice B is not proper, as MRPC 1.6 provides that a lawyer is prohibited from disclosing information relating to the representation of a client unless the client gives informed consent. Here, turning over the client's file would violate that rule. Answer choice C is not proper, as a lawyer must not obtain a proprietary interest in the cause of action or subject matter of litigation in which a client is represented, except when: (i) the lawyer acquires a lien granted by law to secure payment of a fee, or (ii) the lawyer contracts for a reasonable contingent fee, provided the case is not a criminal or matrimonial/domestic matter. MRPC 1.8(i). Answer choice D is incorrect because it includes two improper courses of action.

A businessman contacts an attorney, who was a sole practitioner, about representing him before a governmental board. Due to a conflict of interest arising from the attorney's recent service on the board, the attorney declines to represent the businessman in the current matter. Instead, the attorney offers, for a reasonable fee, to refer the businessman to another lawyer who is experienced in handling such matters. The businessman agrees and pays the attorney the agreed upon fee. The attorney supplies the businessman with the name of the other lawyer, whom the businessman then employs to represent him before the board. Is the attorney's action proper? Yes, because the referral fee was reasonable. Yes, because the attorney did not practice in the same firm as the lawyer to whom he referred the businessman. No, because an attorney may not pay another person for referring a client. No, because a referral fee constitutes an impermissible splitting of a fee between lawyers.

Answer choice A is correct. A lawyer is prohibited from charging an unreasonable fee for his services. MRPC 1.5(a). The attorney in question charged a reasonable fee to recommend another attorney. While often an attorney provides a referral to another attorney without charge to the person seeking the referral, there is not a prohibition on charging a person for providing a referral. Answer choice B is incorrect because the fact that the two lawyers in question do not practice in the same firm is irrelevant to the issue of whether the fee the attorney charged the businessman for a referral to another attorney was proper. Answer choice C is incorrect because, while it is true that a lawyer may not pay another person, including another lawyer, for referring a client, MRPC 7.2(b), that prohibition does not apply to these facts. Here, it was the potential client, not the lawyer, who was paying for a referral. Answer choice D is incorrect because the facts do not indicate that the attorney has shared the fee with the lawyer to whom he referred the businessman.

A driver consulted an attorney and asked the attorney to represent him because he was being prosecuted for driving while intoxicated in a jurisdiction in which there is an increased penalty for a second offense. The driver told the attorney that his driver's license had been obtained under an assumed name because his prior license had been suspended for driving while under the influence of alcohol. The driver asked the attorney not to disclose the driver's true name during the course of the representation and told the attorney that, if called as a witness, he would give his assumed name. The attorney informed the driver that, in order to properly defend the case, the attorney must call the driver as a witness. The attorney called the driver as a witness and, in response to the attorney's question "what is your name," the driver gave his assumed name and not his true name. Is the attorney subject to discipline? Yes, because the attorney knowingly used false testimony. Yes, if the driver committed a felony when he obtained the driver's license under an assumed name. No, because the attorney's knowledge of the driver's true name was obtained during the course of representation. No, unless the driver's true name is an issue in the proceeding.

Answer choice A is correct. A lawyer is prohibited from knowingly offering false evidence. MRPC 3.3(a)(3). Here, the attorney knew that the driver's testimony was false. Answer choice B is not correct, as the issue of how the driver obtained the license is not directly relevant. What controls whether the attorney is subject to discipline is that the attorney knew that false testimony was being offered. Answer choice C is not correct, as it does not matter how the attorney learned that the evidence being offered was false. The attorney is subject to discipline for knowingly offering false evidence. Answer choice D is not correct, as the rule governing knowingly offering false testimony does not merely relate to testimony about a specific issue in the proceeding. Knowingly offering any false testimony is prohibited.

An attorney has three paralegals who have all worked for him for several years, and each has a paralegal certificate from a highly regarded school. All of the attorney's correspondence goes through the paralegals, who then enter deadlines into the attorney's electronic timekeeping and organizational system. One of the paralegals received a notice of an arbiter's award, which contained the deadline before which an attorney could reject the award and bring the case to court. Due to an oversight, the paralegal forgot to enter the deadline into the electronic system, and the deadline passed. The arbiter's award was significantly less than the attorney could have won for the client in court. Is the attorney subject to discipline as a result of the paralegal's oversight? Yes, if the attorney failed to adequately supervise the paralegal. Yes, because attorneys are always responsible for the actions of their staff while acting in the scope of employment. No, because paralegals are certified in their own rights and are responsible for their own actions. No, and the attorney is also not civilly liable to the affected client if the client wishes to recover the difference in the amounts.

Answer choice A is correct. A lawyer is responsible for the conduct of a non-lawyer assistant that would be a violation of the Model Rules if engaged in by a lawyer if: (i) the lawyer orders the conduct, (ii) with the knowledge of the specific conduct, ratifies the conduct involved, or (iii) the lawyer is a partner or has comparable managerial authority in the law firm in which the non-lawyer assistant is employed, or has direct supervisory authority over the person and knows of the conduct at a time when its consequences can be avoided or mitigated but fails to take reasonable remedial action. Answer choice B is incorrect because an attorney is not responsible for all the actions of his employees. Answer choice C is incorrect because attorneys still have some responsibility for their subordinates under the doctrine of respondeat superior. Answer choice D is incorrect because, under the doctrine of respondeat superior, an attorney would be civilly liable for failing to object to the arbiter's award.

An attorney represented a plaintiff in an action against several defendants. The retainer agreement provided that the plaintiff would pay all costs and expenses of litigation and would, on demand, reimburse the attorney for any costs or expenses advanced by the attorney. After serving process on two defendants, the attorney had difficulty locating and serving the remaining defendants. The plaintiff approved the hiring of an investigator to locate and serve the defendants, and the attorney advanced the costs for the investigator. When the attorney asked the plaintiff for reimbursement, the plaintiff refused to pay. The attorney then told the plaintiff that she would do no more work on the case until she was reimbursed for the amount advanced. Thereafter, one of the defendants filed a counterclaim that required a responsive pleading within thirty days. Because the attorney had not been paid, the attorney permitted the time to respond to the counterclaim to expire without filing a responsive pleading, and a default was entered on the counterclaim. Later, the plaintiff reimbursed the attorney for the costs the attorney had advanced, and the attorney was successful in having the default on the counterclaim set aside. The case was tried, and the plaintiff prevailed on his complaint, and the counterclaimant recovered nothing. Is the attorney subject to discipline for not initially filing a responsive pleading to the counterclaim? Correct Answer: Yes, because the attorney neglected the plaintiff's case. Yes, unless the attorney had asked leave of court to withdraw. No, because the plaintiff breached the agreement to reimburse the attorney. No, because the plaintiff did not sustain any prejudice as a result of the attorney's action.

Answer choice A is correct. A lawyer may properly withdraw if the client refuses to abide by the terms of an agreement relating to the representation, such as an agreement concerning fees or costs; however, permission of the court must be granted. MRPC 1.16(c). Here, the attorney did not ask the court for leave to withdraw. Moreover, even if withdrawal was permitted, MRPC 1.16(d) requires a withdrawing lawyer to take all reasonable steps to mitigate the consequences to the client. Here, the attorney allowed the filing period to lapse and entry of default to occur and did not take any steps to warn the plaintiff. Answer choice B is not correct, as even if the attorney had asked leave of court to withdraw, the attorney still needed the court's permission to actually stop working on the case and needed to take reasonable steps to mitigate the consequences of the withdrawal to the client. Answer choice C is not correct, as even though the plaintiff breached the agreement, MRPC 1.16 requires court permission to withdraw and the attorney to have taken steps to mitigate the consequences of the withdrawal to the client. Answer choice D is not correct, as default was entered on the counterclaim, which does constitute a negative consequence, even though it was subsequently set aside.

An attorney specializing in bankruptcy law is serving on a special bar association committee that is examining current bankruptcy law and procedure and making legislative reform recommendations to Congress. During discussions on the proposed recommendations, it becomes clear that one of the legislative changes that the committee will be recommending would directly and materially benefit an automobile manufacturer that is one of the attorney's principal clients. The attorney discloses to the other members of the committee the fact that one of his clients would materially benefit from the recommendation, but he refuses to identify the client. Has the attorney acted properly? Yes, because only disclosure of the fact that a client could benefit is required, not the identity of the client. Yes, because the committee is only making recommendations for legislative change rather than adopting specific rules or laws. No, because the identity of the client is required to be disclosed if the proposed change would benefit the client. No, because the lawyer is not permitted to participate on the committee once he knows that the recommendations may benefit his client.

Answer choice A is correct. A lawyer may serve as a member of an organization involved in reform of the law or its administration notwithstanding the fact that the reform may affect the interests of a client of the lawyer. If the lawyer knows that the interests of a client may be materially benefited by a decision in which the lawyer participates, the lawyer is required to disclose that fact, but he need not identify the client. This effectively serves to alert the organization that the lawyer's position on an issue may not solely reflect the lawyer's personal opinion, but it may be colored by a client's interest as well. Thus, answer choice C is incorrect. Answer choice B is incorrect, as it is irrelevant that the committee is only making recommendations rather than adopting specific rules or laws. A lawyer is professionally obligated to protect the integrity of the program by making an appropriate disclosure within the organization when the lawyer knows that a private client might be materially benefited, but he need not identify the client. Answer choice D is incorrect, as a lawyer is encouraged to participate on law reform committees, so long as the requirements of the Rule are met.

A sole practitioner was appointed to represent a criminal defendant on appeal. A recently admitted lawyer who shared office space with the sole practitioner agreed to write the brief if the sole practitioner would pay him one-half of the statutory fee. The defendant agreed to the arrangement in writing, after full consultation. The recently admitted lawyer entered an appearance as co-counsel for the defendant and, with the sole practitioner's knowledge, applied for and received several extensions of time to file the brief. Subsequently, the appellate court dismissed the appeal for failure to pursue the appeal. A third lawyer was later appointed to represent the defendant, whose conviction was affirmed after the appeal was reinstated. Is the sole practitioner subject to discipline? Yes, because he neglected the defendant's case. Yes, because he shared fees with the recently admitted lawyer. No, because the defendant agreed in writing to the co-counsel arrangement. No, because the affirmance by the appellate court indicated that the defendant's appeal was without merit.

Answer choice A is correct. A lawyer who is appointed to represent a criminal defendant must competently discharge his work or associate with someone who will perform the work competently with the consent of the client. The lawyer neglected the case because he referred the case to an inexperienced attorney and was aware that the attorney was not competently representing the defendant. Answer choice B is incorrect, as the client consented to the arrangement with full information and in writing, which is allowed under the rules. Answer choice C is incorrect because the defendant's consent to the agreement did not relieve the sole practitioner of the responsibility of making sure that the appeal was timely filed. Answer choice D is incorrect because the end result in the court did not relieve the sole practitioner of his responsibility to make sure that the first appeal was done properly.

A man hired an attorney to handle his divorce proceedings. The attorney had a general practice law firm and the man became so pleased with the attorney's representation that he hired the attorney to provide business advice for him as well. The man's business was soon to merge with a competitor and the attorney helped the man broker and negotiate the merger deal. The attorney and the man executed two separate representation contracts for the two different matters, and the man paid the attorney's fees out of two different accounts: a business account for the fees related to the merger and a personal account for the fees related to the divorce. Eventually, it became evident that the merger was going much more smoothly than the divorce, and the man became displeased with the attorney's handling of his divorce case. The man soon stopped paying the attorney's bills related to the divorce, though he continued paying all fees associated with the merger. The attorney, frustrated with the man's continued refusal to pay his fees for the divorce case, sued the man for payment. During the course of the trial regarding the divorce fees, in addition to detailing the work he had provided to the man with regard to the divorce and the fees associated with that work, the attorney sought to testify to facts relevant to the merger case in the belief that reminding the man that he had sensitive information regarding the merger would make the man more likely to settle the dispute and pay his outstanding fees. Would testimony regarding the merger facts be proper? No, because the attorney can only disclose facts relevant to the fee dispute. No, because the attorney should have refused to take the man's merger business while handling his divorce. Yes, because no privilege between an attorney and a client exists if there is a fee dispute between them. Yes, because the attorney had a good faith belief that inclusion of the facts relevant to the merger would more quickly resolve the fee dispute.

Answer choice A is correct. A legitimate dispute between an attorney and a client does allow for the disclosure of confidential communications relevant to the dispute. As such, if the disclosure of information regarding work the attorney had done for the man was necessary to explain the fees, then he can disclose the information. In this case, however, disclosure of information regarding the work the attorney had done in the merger case was not necessary to explain the fees for the divorce case. They were entirely separate matters, and the merger case had no relevance with regard to the fee dispute concerning the divorce representation. Answer choice B is incorrect because an attorney can represent a client in more than one matter. Answer choice C is incorrect because even if there is a dispute between an attorney and his client, the attorney can only disclose facts relevant to the dispute. Answer choice D is incorrect because even if the threat of further disclosure of confidential communications would have encouraged the man to pay the attorney's fees, those disclosures are not relevant to the dispute, and the attorney cannot use the threat of breach of confidence for his own gain.

A defendant is charged with robbery. A police officer received an anonymous tip that a specific individual other than the defendant committed the robbery. The police officer investigated the information, but concluded that the other individual did not commit the robbery. The police officer conveyed this information to the prosecutor. Due to the existence of other evidence that linked the defendant to the robbery, the prosecutor concluded in good faith that the tip, while exculpatory, was not material, and therefore she did not have a constitutional duty to turn the information over to the defense. Consequently, the prosecutor did not reveal the tip to the defendant's lawyer, who failed to make a Brady request for exculpatory evidence. State criminal procedure discovery rules did not require the prosecutor to disclose the anonymous tip. Is the prosecutor subject to discipline for her failure to inform the defense of the anonymous tip? Yes, because the prosecutor determined that the anonymous tip was exculpatory information. Yes, because a prosecutor has a duty to search for exculpatory information. No, because the defendant's lawyer did not make a Brady request for exculpatory evidence. No, because the prosecutor acted in good faith in determining that the anonymous tip did not have to be turned over to the defense.

Answer choice A is correct. A prosecutor has an ethical duty to make timely disclosure to the defense of all evidence or information known to the prosecutor that tends to negate the guilt of the accused or mitigates the offense. MRPC 3.8(d). Since the prosecutor concluded in good faith that the anonymous tip was exculpatory, the prosecutor had a duty to disclose the tip to the defense. Answer choice B is incorrect because, while a prosecutor does have a duty to reveal exculpatory information of which the prosecutor has knowledge, a prosecutor is not under a duty to search for such information. Answer choice C is incorrect because, under MRPC 3.8(d), the prosecutor has a duty to reveal exculpatory information regardless of whether the defense requests such information. Answer choice D is incorrect because the prosecutor's good faith evaluation is part of her constitutional disclosure obligation. The ethical duty imposed by MRPC 3.8(d) is broader than the constitutional obligation. As noted with respect to answer choice A, since the prosecutor determined that the tip was exculpatory, the prosecutor was required by MRPC 3.8(d) to disclose the tip to the defense.

A corporation hired a law firm to handle all of its corporate work. The firm had not previously represented a corporation on an ongoing basis, but decided that it wanted to attract additional corporate clients. Accordingly, the partners handling the corporation's work began a practice of giving to all lawyers in the firm, on a monthly basis, detailed descriptions of the work they were doing for the corporation for the purpose of illustrating what the firm could do for corporate clients. One of the partners mentioned this practice to the corporation's management, and the corporation complained that its confidences had been violated. Was it proper for the partners to give detailed descriptions of the work being done for the corporation to other lawyers in the firm? Yes, because, absent client instructions to the contrary, lawyers may discuss client information with other lawyers in the firm. Yes, because lawyers may discuss client information with other lawyers in the firm, regardless of client instructions to the contrary, so long as the disclosure does not disadvantage the client. No, because sharing the information increased the risk that it might be improperly disclosed to third parties outside the firm. No, because lawyers may not disclose client information, even to other firm lawyers, unless the disclosure is in furtherance of the representation.

Answer choice A is correct. Although the nature of the work done for clients of the firm is confidential under MRPC 1.6 and generally may not be disclosed to third parties without client consent, "[l]awyers in a firm may, in the course of the firm's practice, disclose to each other information relating to a client of the firm, unless the client has instructed that particular information be confined to specified lawyers." MRPC 1.6, cmt. [5]. Answer choice B is incorrect; if the client instructs that particular confidential information be disclosed only to particular lawyers in the firm, the firm must abide by that instruction. Answer choice C is incorrect. To facilitate a law firm's practice, its lawyers may share client confidential information with each other, even though doing so may increase the risk that the information will be disclosed to third parties outside the firm, because clients generally understand that this is done and implicitly consent to it. Law firms take other measures to minimize the risk that the information will be disseminated to third parties. Therefore, except when the client instructs otherwise or there are special reasons to limit disclosures within the firm, client information may be shared, as in this case, among the firm's lawyers for legitimate business purposes. Answer choice D is incorrect. Disclosures of client information may be made within a law firm to carry out the representation, but that is not the only circumstance in which disclosures may be made within the firm. In general, they may also be made for the purpose of facilitating the law firm's practice, such as for billing purposes or for the education of young lawyers or, as in this case, to familiarize the firm's lawyers with the nature of each other's practice.

While using the copy machine, a transactional associate overheard two summer interns talking in the next room about a litigation associate in the firm. The interns, who did not see the transactional associate or know he was there, discussed the litigation associate's behavior and speculated that she had been drinking while at work. They said they believed that her drinking had caused her to make several mistakes in active cases. The interns never mentioned the litigation associate by name, but the transactional associate knew that they primarily worked with one attorney. He had never seen the litigation associate drinking and had always heard that her work was satisfactory, so he dismissed the discussion as mere gossip and did not take any action based on the information. Several months later, a client filed a complaint with the disciplinary board against the litigation associate that included allegations related to alcohol abuse. The disciplinary board found that the litigation associate had made errors on the client's case because she was inebriated. Would the transactional associate be subject to discipline based on his failure to report the litigation associate's alleged misconduct? No, because the transactional associate did not have actual knowledge of any misconduct. No, because the transactional associate did not personally witness any misconduct. Yes, because the transactional associate failed to report another attorney's misconduct. Yes, because the misconduct raised a substantial question as to the litigation associate's professional fitness.

Answer choice A is correct. An attorney generally must report misconduct by another attorney to the appropriate professional authority when the attorney has actual knowledge of the misconduct. Actual knowledge may be inferred from the circumstances and includes learning of misconduct from a reliable source when there is independent corroboration. In this case, the transactional associate did not have actual knowledge because there was no independent corroboration of the statements by the interns, and the transactional associate had no reason to believe they were true. Answer choice B is incorrect because an attorney may have a duty to report misconduct even if he does not personally witness it. Answer choice C is incorrect because an attorney is subject to discipline if he fails to report misconduct of which he has actual knowledge. Here, the transactional associate did not have actual knowledge of the litigation associate's alcohol abuse. Answer choice D is incorrect because although the misconduct raises questions as to the litigation associate's professional fitness, the transactional associate did not have a duty to report it because he lacked actual knowledge.

An attorney received a phone call from a client asking the attorney to represent the client's daughter, a 19-year-old college student, against a drug charge. The father told the attorney that he would pay the attorney's standard hourly rate for the representation, as well as any expenses. The following day, the attorney met with the father and daughter, and the three discussed the matter. When the daughter asked who was paying for the services, the attorney responded that they should simply focus on the case and worry about that later. As the trial date approached, the prosecution offered a plea deal. The attorney brought the deal to the daughter, who accepted it. When the father later learned of this, he was furious with the attorney for failing to include him in the discussion regarding the plea deal. Were the attorney's actions in representing the daughter proper? No, because the daughter did not give informed consent for the attorney to be paid by her father. No, because the father should have been notified of the plea deal. Yes, because a parent may pay the legal fees for his child. Yes, because the attorney was not required to inform the father of the plea deal.

Answer choice A is correct. An attorney may not accept payment for representation from someone other than the client, unless: (i) the client gives informed consent, (ii) there is no interference with the attorney's professional judgment, and (iii) attorney-client confidentiality is preserved. In this case, the daughter did not give informed consent for her attorney to be paid by her father, and thus the attorney's actions were not proper. Answer choice B is incorrect because the daughter, and not the father, was the attorney's client. Thus, the attorney was not required to include the father in the plea discussions. Answer choice C is incorrect because a parent may pay the legal fees for an adult child, as long as the child provides informed consent, there is no interference with the attorney's judgment, and the attorney-client privilege is preserved. Answer choice D is incorrect because although the attorney was not required to inform the father of the plea deal, his actions were not proper because he did not obtain the daughter's informed consent to have his bills paid by her father.

Able, Baker, and Carter had been indicted for the armed robbery of a cashier at a grocery store. Together, Able and Baker met with an attorney and asked her to represent them. The attorney then interviewed Able and Baker separately. Each told the attorney that the robbery had been committed by Carter while Able and Baker sat in Carter's car outside the store. They each said that Carter had said he needed some cigarettes and that they knew nothing of his plan to rob the cashier. The attorney agreed to represent both Able and Baker. One week before the trial, Able told the attorney that he wanted to plea bargain and that he was prepared to testify that Baker had loaned Carter the gun Carter used in the robbery. Able also said that he and Baker had shared in the proceeds of the robbery with Carter. What is the proper course of action for the attorney to take? Request court approval to withdraw as the attorney for both Able and Baker. Continue to represent Baker and, with Able's consent and court approval, withdraw as Able's lawyer. Continue to represent Able and, with Baker's consent and court approval, withdraw as Baker's lawyer. Continue to represent Able and Baker, but not call Able as a witness.

Answer choice A is correct. Attorney has a conflict of interest under MRPC 1.7 and may not continue to represent both clients. If the attorney negotiates a plea bargain for Able, she will be prejudicing Baker, whereas if she fails to negotiate the plea bargain, she will be prejudicing Able. Under MRPC 1.16(a)(1), the attorney must seek judicial permission to withdraw from the representation to avoid violating MRPC 1.7. Answer choice B is incorrect. If the attorney withdrew from representing Able, she would still have a conflict of interest under MRPC 1.7 because of the duty to protect Able's confidences. There would be a risk that, in trying to avoid abusing Able's confidences, the attorney would fail to represent Baker competently, especially when the time came to cross-examine Able. Answer choice C is incorrect because if the attorney withdrew from representing Baker but continued to represent Able, the attorney would have a conflict of interest under MRPC 1.9(a), which concerns representations adverse to a former client. By continuing the representation of Able and negotiating an agreement whereby Able would testify against Baker, the attorney would be representing Able adversely to Baker, now a former client, in the same matter in which the attorney formerly represented Baker. Answer choice D is incorrect because the attorney has a conflict of interest under MRPC 1.7 and may not continue to represent both clients.

A judge of the city's trial court has served for many years as a director of a charitable organization that maintains a camp for disadvantaged children. The organization has never been involved in litigation. The judge has not received any compensation for her services. The charity has decided to sponsor a public testimonial dinner in the judge's honor. As part of the occasion, the local bar association intends to commission and present to the judge her portrait at a cost of $4,000. The money to pay for the portrait will come from a "public testimonial fund" that will be raised by the City Bar Association from contributions of lawyers who are members of the association and who practice in the courts of the city. Is it proper for the judge to accept the gift of the portrait? Correct Answer: Yes, because the gift is incident to a public testimonial for the judge. Yes, because the judge did not receive compensation for her services to the charitable organization. No, because the cost of the gift exceeds $1,000. No, because the funds for the gift are contributed by lawyers who practice in the courts of the city.

Answer choice A is correct. CJC Rule 3.13 generally permits a judge to accept gifts incident to a public testimonial (but the judge must generally report such acceptance). Answer choice B is not correct, as the lack of compensation by the charitable organization is not relevant to the issue of whether the gift is proper under CJC Rule 3.13. Answer choice C is not correct, as CJC Rule 3.13 allows a judge to accept a gift incident to a public testimonial without a limitation on the cost of the gift. Answer choice D is not correct, as the source of the funds for the gift is not relevant. CJC Rule 3.13 allows a judge to accept a gift incident to a public testimonial.

Four years ago, an attorney represented a husband and wife, both high school teachers, in the purchase of a new home. Since then, the attorney prepared their tax returns and drafted their wills. Recently, the husband called the attorney and told her that he and his wife had decided to divorce, but wanted the matter to be resolved amicably. The husband stated that they were planning to file and process their own divorce case, utilizing the state's new streamlined divorce procedure, applicable in "no-fault" cases in which there are no minor children. The husband asked if the attorney would agree to work with them to prepare a financial settlement agreement that could be presented to the divorce court, reminding the attorney that the couple's assets were modest and that they wanted to "split it all down the middle." After considering the risks of a conflict of interest arising in this limited representation, the attorney wrote to the couple separately, and advised each that he or she might be better off with separate lawyers, but that the attorney would assist with the financial settlement agreement, charging an hourly fee of $140 per hour, provided that they were in complete agreement and remained so. The attorney advised that if a conflict developed, or if either party was dissatisfied or uncomfortable about continuing with the joint representation, the attorney would withdraw and would not represent either party from that point forward, forcing them to start all over again with separate lawyers. Finally, the attorney cautioned the husband and wife that she would be representing both of them equally, that she would not and could not favor one or the other, and that their separate communications to her could not be kept confidential from the other party. Both the husband and wife signed their individual copy of the letter, consenting to the joint representation, and returned them to the attorney. Was it proper for the attorney to accept the representation on these terms? Yes, because there was little risk that the interests of either the husband or wife would be materially prejudiced if no settlement was reached. Yes, because the attorney had previously represented the husband and wife in their joint affairs. No, because the attorney conditioned representation upon receiving a waiver of client confidentiality. No, unless the attorney advised both the husband and wife, in writing, that they should seek independent counsel before agreeing to enter into the financial settlement on the terms proposed.

Answer choice A is correct. Despite the existence of a conflict, the lawyer may represent both parties if the lawyer reasonably believes that she will be able to provide competent and diligent representation to each affected client, the representation is not prohibited by law, the representation does not involve the assertion of a claim by one client against another client represented by the lawyer in the same litigation or other proceeding before a tribunal, and each affected client gives informed consent, confirmed in writing. MRPC 1.7(b). Because all these criteria have been met, the attorney could properly accept representation. Answer choice B is not correct, as it does not matter that there was a previous joint representation, so long as the criteria of MRPC 1.7(b) are met. Answer choice C is not correct, as the attorney could properly condition the representation on receiving a waiver of client confidentiality in order to satisfy the criteria of MRPC 1.7(b). Answer choice D is not correct, as a suggestion of independent counsel is not required by the Model Rules under these circumstances.

An attorney for a defendant in a criminal case called the clerk of the court with which the case had been properly filed. The attorney sought clarification as to the procedure by which the clerk of the court, who was an elected judicial official, assigned a case to a particular judge. After learning of the procedure, which involved the assignment of cases through a random selection process, the attorney remarked that he didn't want his case assigned to a particular judge because the attorney knew that the judge had prior involvement in the matter that would disqualify the judge from hearing the case. The clerk indicated that she didn't have the authority to exclude the judge in question from the list of judges who were eligible to be assigned the case. When the case was assigned, the clerk, following established procedures, assigned the case to another judge. Was the attorney's conduct with regard to the assignment of a judge to the attorney's case proper? No, because the attorney communicated ex parte with a judicial official during the proceeding. No, because the attorney knew that the judge's prior involvement in the matter would disqualify the judge from hearing the case. Yes, because the court clerk was an elected official. Yes, because the attorney communicated with the court clerk about an administrative matter.

Answer choice A is correct. During a proceeding, a lawyer must not communicate ex parte with a person who serves in an official capacity. MRPC 3.5(b), cmt. 2. The clerk of the court was a judicial official who was dealing with the attorney's case. While a routine and customary request of such an official, such as a communication dealing with the scheduling of a hearing, is not prohibited, a communication made for the purpose of having a matter assigned to a particular judge is prohibited. Answer choice B is incorrect because, while a judge's prior involvement in a matter is grounds for recusal (CJC Rule 2.11), an attorney's knowledge of such involvement does not create an exception to the prohibition on ex parte communication. Instead, the attorney in question could have protected his client from judicial bias by requesting, had the judge been assigned to the case, that the judge recuse herself from the proceedings. Answer choice C is incorrect because, while the manner in which a judge is selected (e.g., election) can affect activities in which a judicial candidate can engage, it is irrelevant for purposes of the prohibition on ex parte communication with a judge or other judicial official. Answer choice D is incorrect because, while the prohibition on ex parte communications does not apply to communications that involve an administrative matter, a communication whose purpose is to effect the assignment of a particular judge to a matter is subject to the prohibition.

An attorney represents a plaintiff in a civil action that was filed one year ago and is about to be set for trial. The client informed the attorney that he could be available at any time during the months of October, November, and December. In discussing possible trial dates with opposing counsel and the court clerk, the attorney was advised that a trial date on October 5 was available and that the next available trial date would be December 10. Without first consulting the client, the attorney requested the December 10 trial date because she was representing a different defendant in a felony criminal trial that was set for October 20 and she wanted as much time as possible to prepare for that trial. Was it proper for the attorney to agree to the December trial date without obtaining the client's consent? Yes, unless the client will be prejudiced by the delay. Yes, because a criminal trial takes precedence over a civil trial. No, because the attorney should manage her calendar so that her cases can be tried promptly. No, unless the attorney was court-appointed counsel in the criminal case.

Answer choice A is correct. In general, the lawyer controls the means by which the objectives of representation are to be pursued, but the lawyer must reasonably consult with the client. MRPC 1.2 and 1.4. Here, the client had already told the attorney that he could be available in December. The attorney was reasonably setting the trial date to allow her to best prepare for trial. So long as the December trial date does not prejudice the client, it would be proper to accept that date without obtaining the client's specific consent. Answer choice B is not correct, as what matters here is that the attorney has discretion over the means by which the objectives of representation are to be pursued, so long as the client is not prejudiced by the decisions being made. Answer choice C is not correct, as the attorney should be managing her calendar to ensure that she provides the best representation to her clients, not merely so that cases can be tried promptly. Answer choice D is not correct, as it is irrelevant whether or not the attorney is court-appointed counsel in the criminal case. The decision regarding trial dates is within the attorney's discretion under these facts.

An attorney regularly volunteered as an intake consultant for an established program run by a legal services organization. The program provided indigent individuals with limited short-term legal assistance. If it became apparent that an individual needed long-term representation, the intake consultant would recommend that the individual consult an attorney and would provide a list of legal services organizations in the area. On one occasion, a woman who had received an eviction notice sought help from the program. The attorney gave her some general advice about how to avoid eviction and then recommended that she contact a private attorney or a legal services organization if she needed further assistance. The attorney later learned that the woman's landlord was a corporation represented in unrelated matters by partners in the attorney's law firm. Did the attorney's actions in providing legal advice to the woman subject him to discipline? No, because the attorney did not know that his law firm represented the woman's landlord at the time he provided legal advice. No, because the Model Rules of Professional Conduct regarding conflicts of interest do not apply to programs such as that in which the attorney participated. Yes, because the attorney had a conflict of interest due to his law firm's representation of the woman's landlord. Yes, because the attorney did not check to see if he had a conflict of interest before providing the woman with legal advice.

Answer choice A is correct. In this case, the attorney participated in an established program in which attorneys provide short-term limited legal services without expectation by either the attorney or the client that the attorney would provide continuing representation in the matter. An attorney who participates in such a program is subject to the Model Rules of Professional Conduct regarding conflicts of interest only if the attorney knows that the representation involves a conflict of interest for the attorney or another attorney associated with the attorney in a law firm. In this case, the attorney only learned later that his firm represented the woman's landlord, and thus he was not subject to discipline under the Rules of Professional Conduct regarding conflicts of interest for providing her with legal services. Answer choice B is incorrect because the Model Rules regarding conflicts of interest do subject an attorney in such a program to discipline when the attorney knows that a conflict exists. Answer choice C is incorrect because the Rules regarding conflicts of interest did not apply because the attorney was unaware that his firm represented the landlord. Answer choice D is incorrect because an attorney is not required to check for conflicts before providing short-term legal representation through a program such as the one here.

An attorney is a member of the bar in his state and is also licensed as a stockbroker in a neighboring state. In his application for renewal of his stockbroker's license in the neighboring state, the attorney knowingly filed a false financial statement. Is the attorney subject to discipline in his state for so doing? Yes, because his actions involve dishonesty or misrepresentation. Yes, but only if he is first convicted of a criminal offense in the neighboring state. No, because his action was not in his capacity as an attorney. No, because his action was not in his state.

Answer choice A is correct. It is professional misconduct for a lawyer to engage in conduct involving dishonesty, fraud, deceit, or misrepresentation. MRPC 8.4(c). There is no requirement that such conduct be criminal in nature or that such conduct be engaged in by the lawyer in the state in which the lawyer is licensed or in the lawyer's capacity as a lawyer. Answer choice B is incorrect because there is no requirement of a criminal conviction for the dishonest conduct. Answer choice C is incorrect because any dishonest conduct by a lawyer, even if it occurs in a non-lawyer capacity is improper under MRPC 8.4(c). Answer choice D is incorrect because the dishonest action need not have occurred in the jurisdiction where the lawyer is admitted to practice in order for a lawyer to be subject to discipline for dishonesty. MRPC 8.5(a).

An attorney entered into a written retainer agreement with a defendant in a criminal case. The defendant agreed in writing to transfer title to her automobile to the attorney if the attorney successfully prevented her from going to prison. Later, the charges against the defendant were dismissed. Is the attorney subject to discipline for entering into this retainer agreement? Yes, because the attorney agreed to a fee contingent on the outcome of a criminal case. Yes, because a lawyer may not acquire a proprietary interest in a client's property. No, because the charges against the defendant were dismissed. No, because the retainer agreement was in writing.

Answer choice A is correct. MRPC 1.5(d)(2) provides that a lawyer may not represent a defendant in a criminal case on a contingent fee basis. Here, the client's payment of a fee (title to an automobile) was contingent on a successful outcome in the criminal case. Answer choice B is incorrect. MRPC 1.8(i) provides that a lawyer may not obtain a proprietary interest in the cause of action or the subject of the litigation the lawyer is conducting for a client. But the attorney has not done so in this case because the automobile is not involved in the litigation. MRPC 1.8(i) does not prohibit a lawyer in all cases from obtaining a proprietary interest in a client's property. The fee agreement, however, was impermissible under MRPC 1.5(d)(2) because the lawyer's fee was contingent on the successful outcome in the criminal case. Answer choice C is incorrect because, even though the lawyer achieved success, he is subject to discipline for entering into an impermissible fee agreement. Answer choice D is incorrect because, although it is true that a contingent fee agreement must be in writing, this agreement was improper for a different reason.

An attorney represented the plaintiff in an automobile accident case. Two weeks before the date set for trial, the attorney discovered that there was an eyewitness to the accident. The attorney interviewed the witness. Her version of the accident was contrary to that of the plaintiff and, if believed by the trier of fact, would establish that the plaintiff was at fault. The witness told the attorney that she had not been interviewed by defense counsel. The witness also told the attorney that she was uncomfortable with testifying and that she had been thinking about taking a vacation to Europe the following week. The attorney told the witness that, since no one had subpoenaed her yet, she had no obligation to appear. He told her that trials were very difficult for witnesses and suggested that she take the vacation so that she would be unavailable to testify. Is the attorney subject to discipline? Yes, because the attorney asked the witness to leave the jurisdiction. Yes, because the attorney did not subpoena the witness knowing she was an eyewitness. No, because the witness had not been subpoenaed by the defense. No, because the attorney did not offer the witness any inducement not to appear at the trial.

Answer choice A is correct. MRPC 3.4(f) prohibits a lawyer from asking a person other than a client to refrain from voluntarily giving relevant information to another party unless the person is a relative or an employee or other agent of the client. Answer choice B is incorrect because a lawyer is under no obligation to subpoena a witness that the lawyer does not intend to call at trial. Answer choice C is incorrect because, even though the witness had not been subpoenaed by the defense, MRPC 3.4(f) prohibited the attorney from asking the witness to refrain from voluntarily giving relevant information to another party. Answer choice D is incorrect because although the attorney did not offer the witness any inducement not to appear at trial, MRPC 3.4(f) prohibited the attorney from asking the witness to refrain from voluntarily giving relevant information to another party.

Pros, an elected prosecutor in City, plans to run for reelection in six months. Last year two teenage girls were kidnapped from a shopping center and sexually assaulted. The community was in an uproar about the crime and put pressure on Pros to indict and convict the assailant. Four months ago, Deft was arrested and charged with the crimes. The trial is scheduled to begin next week. Pros met with the police chief last week to review the evidence in the case. At that time, Pros first learned that, before they were interviewed by the detective in charge of sexual assault crimes, the two victims had been tape-recorded discussing the case between themselves in an interview room. Reviewing the tape, Pros realized that the girls' descriptions of the assailant differed significantly in terms of height, weight and hair color. When officially interviewed, however, their descriptions matched almost perfectly. Deft's appointed counsel was busy handling a large case-load of indigent defendants and neglected to seek access to the prosecution's investigative file. Pros was virtually certain that Deft's counsel was unaware of the tape recording. Given the other evidence in the case, Pros reasonably believed that the girls accurately identified Deft as their assailant. Pros did not reveal the existence of the tape to defense counsel. Is Pros subject to discipline? Yes, because the tape raises a legitimate question about the victims' eyewitness identification of Deft as the assailant. Yes, unless Pros reasonably believed that the girls accurately identified Deft as their assailant. No, because under the adversary system of criminal justice, it is expected that each party will marshal the evidence best supporting its own position. No, unless Deft's counsel submitted a request for all mitigating or exculpatory evidence before the start of trial.

Answer choice A is correct. MRPC 3.8(d) requires that a prosecutor make timely disclosure to the defense of all evidence or information known to the prosecutor that tends to negate the guilt of the accused or mitigates the offense. Pros would be subject to discipline for failing to reveal the existence of the tape to opposing counsel. Answer choice B is incorrect, as any evidence that tends to negate the guilt of the accused or mitigates the offense must be timely disclosed under the Model Rules. Answer choice C is incorrect, as a prosecutor has a special duty under the Model Rules to ensure that justice is done in a criminal case and is required to make timely disclosure to the defense of any evidence or information that would tend to negate the guilt of the accused or mitigate the offense. Answer choice D is incorrect, as the ethical duty of the prosecutor applies whether or not defense counsel submits a request for mitigating or exculpatory evidence.

An attorney represented a plaintiff in an action against a manufacturer of a drain cleaner. The plaintiff's complaint alleged that the manufacturer's product exploded in use and caused her serious and permanent injuries. The jury agreed and awarded the plaintiff $5,000,000 in actual damages and an additional $5,000,000 in punitive damages. The manufacturer paid the judgment. The attorney made this recovery the cornerstone of an aggressive television advertising campaign for his law practice. In those ads, a voice-over discussed the $10,000,000 recovery obtained in the plaintiff's case. The plaintiff praised the attorney's legal skills in an on-camera statement, saying that no one would work harder on a case than the attorney. The plaintiff prepared her on-camera statement in response to the attorney's request, but without any further involvement by the attorney, and she believed it to be entirely true. Is the attorney subject to discipline for using the television advertisement described above? Yes, because the advertisement is likely to create an unjustified expectation about the results the attorney will be able to achieve and is therefore misleading. Yes, because the attorney's advertisement contains a client testimonial. No, because the plaintiff prepared the entire statement without any involvement by the attorney. No, because the result obtained in the plaintiff's case was reported accurately, and the plaintiff believed that everything she said about the attorney was true.

Answer choice A is correct. MRPC 7.1 prohibits false or misleading statements in lawyer advertising. The Comment to MRPC 7.1 notes that an advertisement truthfully reporting a lawyer's achievement on behalf of a client may be misleading if presented so as to lead a reasonable person to form an unjustified expectation that the same results can be obtained for other clients in similar matters without reference to the specific factual and legal circumstances of each client's case. An express and prominent disclaimer would be necessary to avoid such unjustified expectations here. Answer choice B is incorrect, as there is no per se ban on client testimonials in MRPC 7.1 or any of the other advertising rules. Such testimonials are prohibited only when they include false or misleading communications. Here, the misleading communication is the voice-over discussion of the result in the plaintiff's case, not the plaintiff's own statements. Answer choice C is incorrect because the attorney is responsible for the entire content of the advertisement, regardless of the lack of the attorney's involvement in creating the ad or the testimonial. Answer choice D is incorrect; MRPC 7.1 prohibits false or misleading statements in lawyer advertising. It is true that the advertisement does not contain any false statements; however, the report of the attorney's achievement on the plaintiff's behalf is misleading because it could lead a reasonable person to form an unjustified expectation that the same results could be achieved for other clients.

A corporation has applied to a bank for a $900,000 loan to be secured by a lien on the corporation's inventory. The inventory, consisting of small items, constantly turns over. The security documents are complex and if improperly drawn they could result in an invalid lien. The bank has approved the loan on the condition that the corporation and the bank jointly retain an attorney to prepare the necessary security instruments and that the corporation pays the attorney's fees. Both the corporation and the bank gave informed consent in writing to the attorney's representation of both parties. This arrangement is customary in the city in which the attorney's law office and the bank are located. It is obvious to the attorney that he can adequately represent the interests of both the corporation and the bank. Is it proper for the attorney to prepare the security documents under these circumstances? Yes, because both the bank and the corporation have given their informed consent to the arrangement. Yes, because the arrangement is customary in the community. No, because the attorney's fees are being paid by the corporation, not the bank. No, because the corporation and the bank have differing interests.

Answer choice A is correct. The attorney has a conflict of interest under MRPC 1.7(a) arising out of the representation of two clients in the same transaction — the bank, which is the lender, and the corporation, which is the borrower. But MRPC 1.7(b) allows the attorney to represent both clients with informed consent, confirmed in writing, because the attorney reasonably believes that he can competently and diligently represent them both. Answer choice B is incorrect. The fact that the arrangement is customary in the community is irrelevant. The attorney has a conflict of interest under MRPC 1.7(a) arising out of the representation of two clients in the same transaction. However, the joint representation would be permissible in this situation under MRPC 1.7(b). Answer choice C is incorrect. In a joint representation such as this one, there is no restriction against the payment of legal fees by one of the two clients with informed consent, as required by MRPC 1.8(f). Answer choice D is incorrect. Although the clients have differing interests, and there is therefore a conflict of interest under MRPC 1.7(a), MRPC 1.7(b) allows the attorney to represent both clients with informed consent, confirmed in writing, because the attorney reasonably believes he can competently and diligently represent them both.

An attorney represented a plaintiff who sued a defendant for injuries the plaintiff sustained in a car accident. Prior to trial, the attorney interviewed a witness who stated that she had observed the defendant drinking heavily hours before the accident. Unfortunately, on the eve of trial, the witness informed the attorney that she was ill and could not testify at trial. The attorney tried but could not obtain a continuance. As a result, the plaintiff's direct case rested solely on the plaintiff's testimony that the defendant was speeding and that the defendant's car crossed the middle line and hit the plaintiff's car. The defendant testified that he was driving safely in compliance with all rules and that the accident was entirely the plaintiff's fault. On cross-examination, the attorney asked the defendant, "Isn't it a fact that you were drinking prior to the accident?" The defendant answered that he had not consumed alcoholic beverages on the day of the accident. In summation to the jury, the attorney stated: "Ladies and gentlemen of the jury, you and I know that the defendant lied when he stated that he had not consumed alcoholic beverages on the day of the accident. We know that he was impaired." On which of the following grounds, if any, would the attorney NOT be subject to discipline? The attorney's question to the defendant implying that the defendant had consumed alcoholic beverages when the attorney knew that he could not offer evidence of the defendant's drinking. The attorney's statement to the jury asserting that the attorney knew that the defendant was drunk when no evidence in the record supported this allegation. The attorney's statement asserting a personal belief that the defendant was drunk and lying. The attorney is subject to discipline on all of these grounds.

Answer choice A is correct. This statement would not subject the attorney to discipline. The attorney was impeaching the defendant on cross-examination with knowledge of what the witness had told the attorney, and it is therefore permissible to imply that the defendant had been drinking, even if the attorney could not produce the witness to rebut a denial by the defendant. Answer choice B would subject the attorney to discipline, as MRPC 3.4(e) prohibits a lawyer in a trial from alluding to any matter that is not supported by admissible evidence. Answer choice C would subject the attorney to discipline, as a lawyer is prohibited from stating a personal opinion as to the justness of a cause, the credibility of a witness, or the guilt or innocence of an accused. MRPC 3.4(e).

A judge is one of three trustees of a trust for the educational benefit of her grandchildren. The trust owns 5,000 shares of stock in an oil company. The stock has been selling for the past year at $10.00 per share. The oil company is suing an oil refining company for breach of an oil refining agreement, and the case is assigned to the judge for trial. The judge believes that she can be fair and impartial. Must the judge disqualify herself from the case? Yes, because the trust has more than a de minimis financial interest in the oil company. Yes, unless the outcome of the lawsuit is unlikely to affect the value of the stock. No, unless the judge personally owns stock in either party to the litigation. No, because the judge believes she can remain impartial.

Answer choice A is correct. Under CJC Rule 2.11(A)(2), a judge must disqualify herself in a matter in which the judge knows that she shares a third-degree relationship or closer to a person who has more than a de minimis interest that could be affected by the proceedings. A third-degree relationship includes a grandchild. A de minimis interest is an insignificant interest that would not raise a question about the judge's impartiality. Here, the grandchildren's trust has $50,000 worth of stock, which appears to be more than a de minimis interest in the company, so the judge must disqualify herself. Answer choice B is not correct, because regardless of whether the outcome of the suit is unlikely to affect the stock value, CJC Rule 2.11(A)(2) requires recusal. Answer choice C is not correct, as CJC Rule 2.11(A)(2) requires disqualification under these circumstances even though the judge does not personally own stock in the company. Answer choice D is not correct, as even if the judge believes she can remain impartial, CJC Rule 2.11(A)(2) controls.

For many years, an attorney has served as outside counsel to a corporation. Shortly after a change in management, the attorney discovered what she reasonably believed to be a material misstatement in a document she had drafted that the attorney was about to file on the corporation's behalf with a government agency. The attorney advised the corporation's Board of Directors that filing the document was probably criminal. However, the Board disagreed that there was any material misstatement and directed the attorney to proceed with the filing. When the attorney indicated her intention to resign, the corporation argued that a resignation at this time would send a signal that there was a problem with the filing. The corporation urged the attorney to continue the representation, but offered to use in-house counsel to complete the work on the filing. Although she does not know for certain that filing the document is illegal, the attorney reasonably believes that it is. In any event, the attorney is personally uncomfortable with the representation and wants to withdraw. May the attorney withdraw from her representation of the corporation? Yes, because withdrawal is permitted but not required when a client insists on conduct that the lawyer reasonably believes, but does not know, will be criminal. Yes, because withdrawal is required when a client insists on conduct that the lawyer reasonably believes, but does not know, will be criminal. No, if the corporation is correct that withdrawal would breach confidentiality by sending a signal that the filing is problematic. No, if the attorney's withdrawal as outside counsel might affect the corporation's ability to complete the filing in a timely fashion.

Answer choice A is correct. Under MRPC 1.16(b), a lawyer may withdraw if the client persists in a course of action the lawyer reasonably believes is criminal or fraudulent. Here, the facts indicate that the attorney reasonably believes the action is illegal. Thus, she may properly seek to withdraw from representing the corporation. Answer choice B is not correct, as withdrawal is not required, but is permitted, under such circumstances pursuant to MRPC 1.16(b). Answer choice C is not correct, as withdrawal does not breach a lawyer's ethical duty of confidentiality, which protects the confidentiality of information learned by the lawyer in the course of representing a client. Answer choice D is not correct, as MRPC 1.16(b) allows withdrawal when a client insists on conduct that the lawyer reasonably believes, but does not know, will be criminal, without regard to the effect on the client. While a withdrawing lawyer must take steps to the extent reasonably practicable to protect a client's interests, the lawyer is not required to continue the representation.

An attorney filed an action on behalf of a client for breach of contract. In fact, the client had no legal basis for the suit but wanted to harass the defendant. In order to induce the attorney to file the action, the client made certain false statements of material fact to the attorney, which the attorney included in the complaint filed against the defendant. At the trial of the case, the client took the stand and testified as set forth in the complaint. The trial court ordered judgment for the client. After entry of judgment, the client wrote the attorney a letter marked "Confidential," in which the client admitted that she had lied to the attorney and had testified falsely in the case. Upon complaint of the defendant, who claimed the attorney had knowingly used false testimony in the case, disciplinary proceedings were instituted against the attorney. Is it proper for the attorney to use the client's letter to the attorney in the attorney's defense in the disciplinary proceedings? Yes, if it is necessary to do so in order to protect the attorney's rights. Yes, because the client had committed a fraud on the court in which the case was tried. No, because the attorney learned the facts from the client in confidence. No, if disclosure by the attorney could result in the client's prosecution for perjury.

Answer choice A is correct. Under MRPC 1.6(b)(5), a lawyer may reveal confidential information concerning the representation of a client to the extent the lawyer reasonably believes it necessary to establish a defense to a criminal charge or civil claim against the lawyer based on conduct that client was involved in, or to respond to allegations in any proceeding concerning the lawyer's representation of the client. MRPC 1.6(b)(5). Answer choice B is incorrect because even if a client has committed a fraud on the court, such conduct does not permit the attorney to breach the ethical duty of confidentiality. Answer choice C is incorrect, as MRPC 1.6(b)(5) provides an exception to the duty of confidentiality under these circumstances. Answer choice D is incorrect, as MRPC 1.6(b)(5) allows a lawyer to protect himself, even at the expense of a client's potential prosecution.

An attorney represented a buyer in a real estate transaction. Due to the attorney's negligence in drafting the purchase agreement, the buyer was required to pay for a survey that should have been paid for by the seller, the other party to the transaction. The attorney fully disclosed this negligence to the buyer, who suggested that he would be satisfied if the attorney simply reimbursed the buyer for the entire cost of the survey. Although the buyer might have recovered additional damages if a malpractice action were filed, the attorney reasonably believed that the proposed settlement was fair to the buyer. Accordingly, in order to forestall a malpractice action, the attorney readily agreed to make the reimbursement. The attorney drafted a settlement agreement, and it was executed by both the attorney and the buyer. Was the attorney's conduct proper? Yes, if the attorney advised the buyer in writing that the buyer should seek independent representation before deciding to enter into the settlement agreement. Yes, because the attorney reasonably believed that the proposed settlement was fair to the buyer. No, because the attorney settled a case involving liability for malpractice while the matter was still ongoing. No, unless the buyer was separately represented in negotiating and finalizing the settlement agreement.

Answer choice A is correct. Under MRPC 1.8(h)(2), a lawyer may not settle a claim or potential claim for malpractice liability with an unrepresented client or former client unless that person is advised in writing of the desirability of seeking independent legal counsel with regard to the settlement and is given a reasonable opportunity to seek such advice. Because the answer choice indicates that the attorney properly advised the buyer of the need for independent representation, the attorney's conduct was proper. Answer choice B is not correct, as fairness alone does not control the attorney's responsibility to the buyer. The Model Rules require advising the client in writing of the desirability of seeking independent legal counsel. Answer choice C is not correct, as factually we are told that the transaction at issue was already concluded. Answer choice D is not correct, because once an underlying matter is concluded a lawyer may settle a client's claim for malpractice without the client being independently represented by counsel, so long as the lawyer advises the client, in writing, of the desirability of seeking independent legal counsel. If the underlying matter was ongoing, then independent counsel is required.

An attorney represents a corporation which is a defendant in a product liability case. An engineer of the corporation nearing retirement was likely to be a key witness in the case, as she had been in charge of all of the corporation's product safety testing during the relevant period. The engineer had been very critical of the corporation's safety testing procedures during that period and had repeatedly complained that the product at issue had not been adequately tested. The engineer's views were reduced to writing and were well known to many employees of the corporation. Because of the early stage of the case, however, plaintiff's counsel was not yet aware of the engineer's existence or her views. Aware of the engineer's views, the corporation's attorney approached the corporation's officials and recommended that the corporation offer the engineer a special package of severance benefits if she would retire immediately and move to the Bahamas. The attorney believed that if the engineer accepted this offer, she would be beyond the subpoena power of the court in which the suit against the corporation was pending. The corporation adopted the attorney's recommendation and made the offer. The engineer accepted it. The attorney did not disclose the engineer's identity to plaintiff's counsel. Is the attorney subject to discipline? You Selected: Yes, because the attorney caused the engineer to leave the jurisdiction of the court for the purpose of making her unavailable as a witness. Yes, because opposing counsel had not yet had a reasonable opportunity to learn of the engineer's views. No, because the engineer's views were reduced to writing and are well known to many other employees of the corporation. No, unless there was a pending request for the engineer's testimony at the time the retirement offer was made to the engineer.

Answer choice A is correct. Under MRPC 3.4(a), a lawyer must not unlawfully obstruct another party's access to evidence or counsel or assist another person to do any such act. Here, by causing the engineer to leave the court's jurisdiction for the purpose of making her unavailable as a witness, the attorney was obstructing the plaintiff's access to evidence in the case and would be subject to discipline. Answer choice B is not correct, because whether or not opposing counsel had a reasonable opportunity to learn of the engineer's views is not controlling with regard to whether the attorney should be subject to discipline. What matters is that the attorney caused the engineer to leave the court's jurisdiction for the purpose of making her unavailable as a witness. Answer choice C is not correct, because although the engineer's views may have been available to the opposing party, the opposing party would not have an opportunity to fully examine the engineer on the issues. By causing the engineer to leave the court's jurisdiction for the purpose of making her unavailable as a witness, the attorney is subject to discipline. Answer choice D is not correct, as the fact that there may not have been a pending request for the engineer's testimony at the time the offer was made is not controlling. The attorney was aware that the engineer's testimony could be significant evidence in the case.

An attorney, who had represented a testator for many years, prepared the testator's will and acted as one of the two subscribing witnesses to its execution. The testator's sister and brother were his sole heirs. The will left the testator's entire estate to his sister and nothing to his brother. Upon the testator's death two years later, the executor named in the will asked the attorney to act as his lawyer in the probate of the will and the administration of the estate. At that time, the executor informed the attorney that the testator's brother would concede that the will was properly executed but intended to contest the will on the ground that he had been excluded because of fraud previously practiced on the testator by the testator's sister. The other subscribing witness to the will predeceased the testator, and the attorney will be called as a witness solely for the purpose of establishing the due execution of the will. Is it proper for the attorney to accept the representation? Yes, if there is no contested issue of fact with respect to the formal execution of the will. Yes, because the executor has no beneficial interest under the will. No, unless the attorney's services are necessary to avoid substantial hardship to the executor. No, because the attorney will be called as a witness in the case.

Answer choice A is correct. Under MRPC 3.7(a), a lawyer is permitted to act as an advocate at a trial in which the lawyer is likely to be a necessary witness if the testimony relates to an uncontested issue. If there is no contested issue of fact with respect to the formal execution of the will, then it is proper for the attorney to accept the representation. Answer choice B is not correct, as it is irrelevant that the executor has no beneficial interest under the will. A lawyer is only permitted to act as an advocate at a trial in which the lawyer is likely to be a necessary witness if (i) the testimony relates to an uncontested issue, (ii) the testimony relates to the nature and value of legal services rendered in the case, or (iii) disqualification of the lawyer would work substantial hardship on the client. MRPC 3.7(a). Answer choice C is not correct. Even though avoiding substantial hardship to the client is a ground for the attorney to properly represent the executor under MRPC 3.7(a), it is not the only ground, which the phrasing of this answer choice suggests. Answer choice D is not correct, as MRPC 3.7(a) does allow a lawyer to be a witness and act as an advocate when the testimony relates to an uncontested issue.

An attorney, who is corporate counsel for a company, is investigating a possible theft ring in the parts department of the company. The attorney knows that an employee has worked in the parts department for a long time and believes that the employee is a suspect in the thefts. The attorney believes that if the employee were questioned, she would not answer truthfully if she knew the real purpose of the questions. The attorney plans to question the employee and falsely tell her that she is not a suspect and that her answers to the questions will be held in confidence. Is the attorney subject to discipline if she so questions the employee? Yes, because the attorney's conduct involves misrepresentation. Yes, unless the attorney first advises the employee to obtain counsel to represent her. No, because no legal proceedings are now pending. No, because the attorney did not give legal advice to the employee.

Answer choice A is correct. Under MRPC 4.1, in representing a client a lawyer must not knowingly make a false statement of material fact or law to a third person. Answer choice B is not correct, as even if the attorney first advises the employee to obtain counsel, if the attorney makes a false statement, she will be subject to discipline. Answer choice C is not correct, as it is irrelevant whether legal proceedings are pending. Any false statement to a third person in representing a client violates the Model Rules. Answer choice D is not correct, as the issue of legal advice to the employee is not relevant. What matters is that the attorney made a false statement to the employee, which violates MRPC 4.1.

Attorney Alpha represents a defendant in an action for personal injuries. Alpha, pursuant to the defendant's authorization, made an offer of settlement to Attorney Beta, who represents the plaintiff. Beta has not responded to the offer, and Alpha is convinced that Beta has not communicated the offer to the plaintiff. State law authorizes a defendant to move for a settlement conference and to tender an offer of settlement. If such a motion is made and the offer is rejected by the plaintiff and the eventual judgment does not exceed the amount of the offer, the plaintiff must bear all costs of litigation, including reasonable fees, as determined by the court, for the defendant's counsel. Alpha, with the defendant's consent, filed a motion requesting a settlement conference, tendered an offer to settle for $25,000, and served copies of the motion and tender on Beta and on the plaintiff personally. Is Alpha subject to discipline for serving the plaintiff with a copy of the motion and tender? Yes, unless service of copies of the motion and tender on the plaintiff were authorized by statute or rule of court. Yes, unless Alpha first informed Beta of Alpha's intention to serve copies of the motion and tender on the plaintiff. No, because the decision to accept or reject a settlement offer rests with the client. No, because the motion and tender became public documents when they were filed in court.

Answer choice A is correct. Under MRPC 4.2, in representing a client, a lawyer or someone acting on behalf of the lawyer is not permitted to communicate about the subject of the representation with a person the lawyer knows to be represented by another lawyer in the matter, unless the lawyer has the consent of the other lawyer or is authorized to do so by law or a court order. Here, if service of copies of the motion and tender on the plaintiff were authorized by statute or court rule, Alpha would not be subject to discipline. Answer choice B is not correct, as it is not enough merely to inform the person's lawyer of the intent to communicate in order to avoid violating MRPC 4.2. Consent of the lawyer is required. Answer choice C is not correct, as MRPC 4.2 prohibits communications about the subject of the representation with a person a lawyer knows to be represented by another lawyer in the matter. It does not matter that ultimate settlement authority rests with the person. Answer choice D is not correct, as MRPC 4.2 still applies regardless of the fact that the motion and tender were public documents.

A lawyer has just joined a professional corporation engaged solely in the practice of law. The lawyer is a salaried associate and is not a member or shareholder of the professional corporation. A partner's spouse, who is not a lawyer, is vice president of the corporation and office manager. All of the other officers are lawyers in the firm. All of the corporate shares are held by lawyers in the corporation, except for ten shares held by the executor under the will of a lawyer-member who died one month previously and whose will is now being probated. The lawyer knows that the partner's spouse is an officer and not a lawyer. Is the lawyer subject to discipline? Yes, because the partner's spouse is an officer of the corporation. Yes, if a non-lawyer holds the stock as the executor of the will of the deceased member. No, because the lawyer is a salaried employee and not a member or shareholder of the corporation. No, if the partner's spouse does not participate in any decision regarding a client or a client's case.

Answer choice A is correct. Under MRPC 5.4(d), a lawyer is not permitted to practice with or in the form of a professional corporation or association authorized to practice law for a profit if a non-lawyer is an officer of the corporation. Because the lawyer knows that the partner's spouse is a non-lawyer, the lawyer would be subject to discipline. Answer choice B is not correct. A non-lawyer who is an executor may hold the stock of a lawyer in a professional corporation for a reasonable time during the administration of the lawyer's estate. Answer choice C is not correct, as the fact that the lawyer is only a salaried employee does not absolve the lawyer of responsibility under MRPC 5.4(d), which applies to all lawyers associated with the corporation, regardless of whether or not they are members or shareholders. Answer choice D is not correct, as the fact that the partner's spouse does not participate in decisions regarding clients or their cases is irrelevant. MRPC 5.4(d) controls.

An attorney regularly appears before a trial court judge who is running for reelection in six months. Over the past year, the attorney has noticed that the judge has become increasingly ill-tempered on the bench. Not only is the judge abrupt and critical of lawyers appearing before him, he is also rude and abusive to litigants. On more than one occasion, the judge has thrown his gavel across the courtroom in a fit of temper. The judge's conduct on the bench is often the subject of discussion whenever a group of lawyers meets. Some lawyers are automatically filing requests for judicial substitution whenever a case in which they are to appear is assigned to the judge. The attorney discussed the matter with her law partners, who rarely make court appearances. The attorney's law partners suggested that she, too, file a request for judicial substitution whenever one of her cases is assigned to the judge. In addition, the attorney and her law partners discussed the possibility of reporting the judge to the appropriate disciplinary authority but are concerned that this would alienate the other judges to whom their cases are assigned. The attorney has reluctantly started filing for substitution of the judge in every one of her cases to which the judge is assigned, but she has taken no further action. Is the attorney subject to discipline? Yes, because the attorney failed to inform the appropriate authorities about the judge's conduct. Yes, because, by filing automatic requests for substitution of the judge, the attorney undermined public confidence in the administration of justice. No, because the attorney has a duty to represent her clients zealously. No, because the judge is running for re-election and may not be re-elected.

Answer choice A is correct. Under MRPC 8.3, a lawyer who has actual knowledge that a judge has committed a violation of the rules of judicial conduct that raises a substantial question as to that person's fitness for judicial office must inform the appropriate authority. In this fact pattern, the judge has, on more than one occasion, thrown a gavel across the courtroom and has been critical of lawyers and rude to litigants. Such behavior crosses the line into conduct that raises a substantial question as to his ability to judge. Answer choice B is incorrect; under MRPC 8.3, a lawyer who has actual knowledge that a judge has committed a violation of the rules of judicial conduct that raises a substantial question as to that person's fitness for judicial office must inform the appropriate authority. Filing requests for substitution of the judge does not excuse the duty to report, but it really has nothing to do with public confidence in the judiciary. Failing to report the judge's behavior does affect public confidence. Answer choice C is incorrect because zealous representation of the attorney's clients does not relieve her of the duty to report the judge. Answer choice D is incorrect because whether or not the judge will be reelected does not affect the attorney's duty to report this matter to the appropriate authority.

A defendant was on trial for felony theft. The evidence against the defendant was airtight, but the defendant refused the prosecutor's plea offers. Instead, the defendant instructed his attorney to go to trial, and to "go all out" to get an acquittal. After reviewing the case, the attorney determined that he did not have a reasonable factual or legal basis on which to mount a defense, but he decided to attack the prosecution's case-in-chief by arguing that the prosecutor failed to prove all the elements of the underlying felony. The attorney followed this plan at trial and argued to the jury in his closing statement that they should acquit because the prosecution had failed to prove those elements. Were the defense attorney's actions improper? No, because the client was a criminal defendant. No, because the client has complete control over the means by which the attorney pursued the case. Yes, because the attorney had no factual basis for challenging the prosecution's case. Yes, because the attorney had no legal basis for challenging the prosecution's case.

Answer choice A is correct. While a lawyer is generally not allowed to raise a claim or defense unless there is a basis in law and fact for doing so, a defense lawyer in a criminal case may so defend the proceeding as to require that every element of the case be established beyond a reasonable doubt, even if there is not a factual or legal basis for challenging the element. MRPC 3.1. As such, answer choices C and D are incorrect. Answer choice B is incorrect because, while a lawyer must abide by a client's decisions concerning the objectives of a representation, the lawyer generally controls the means by which the objectives of the representation are to be pursued. MRPC 1.2, 1.4.

An attorney and an accountant entered into an agreement providing that each would recommend the other's services. The agreement, which did not specify a termination date, permitted the attorney and the accountant to enter into similar agreements with other parties. The agreement also placed no limitations on the number of clients that could be referred. Both the attorney and the accountant disclosed the nature and existence of this relationship to each client who is referred pursuant to the agreement. Is the agreement improper? Yes, because the agreement was of an indefinite duration. Yes, because the agreement was made with a person who is not an attorney. No, because such a reciprocal referral agreement is permitted by the Model Rules of Professional Conduct. No, because the agreement is not exclusive, and each party disclosed the nature of the agreement to clients.

Answer choice A is correct. While an attorney may enter into a reciprocal referral agreement with a non-lawyer professional, the agreement should not be of indefinite duration. (Note, too, that the agreement must not be exclusive, and that the client must be informed of the existence and nature of the agreement.) Since this agreement is of an indefinite duration, it will subject the attorney to discipline. Answer choice B is incorrect because a reciprocal referral agreement may be entered into by an attorney not only with another attorney, but also with a non-lawyer professional. Answer choice C is incorrect because, although a reciprocal referral agreement is permitted by the Model Rules of Professional Conduct, such an agreement must meet specific requirements, including that the agreement be of limited duration. Answer choice D is incorrect because, although the agreement satisfies the requirements related to exclusivity and disclosure, it fails to meet the requirement that such an agreement not be for an indefinite duration.

An attorney representing a client in a divorce action sought to recover attorney's fees and other litigation costs from the opposing party as permitted by statute. After presenting evidence that the client was entitled to recover such fees and costs, the attorney called himself to testify about the nature and value of legal services he had rendered in the case. The opposing party's lawyer, who planned to present her own evidence as to the value of such services through the testimony of another lawyer, objected to the attorney serving as a witness. The court overruled the objection. The opposing party's lawyer then moved to disqualify the attorney and have him removed from the case. Should the court grant the opposing party's motion to remove the attorney from the case? No, because the attorney may testify as to the nature and value of the legal services rendered in the case. No, because the attorney may testify regarding a matter that does not relate to the merits of the case. Yes, because the attorney cannot serve as an advocate in an action in which he serves as a witness. Yes, because the attorney may not both testify and serve as an advocate in a domestic relations action.

Answer choice A is correct. While, under the advocate-witness rule, a lawyer is generally prohibited from serving both as an advocate and as a necessary witness at the same trial, there is a special exception for a lawyer who testifies as to the nature and value of the legal services rendered in the case. MRPC 3.7(a). Answer choice B is incorrect because there is not a general exception to the advocate-witness rule based on whether the testimony relates to the merits of a case. Answer choice C is incorrect because, while there is a prohibition on a lawyer acting as both an advocate and as a necessary witness, there is, as noted above, an exception for a lawyer who testifies as to the nature and value of his legal services. Answer choice D is incorrect because, although an attorney may not take a domestic relations matter on a contingency fee basis (MRPC 1.5(c)), there is not a comparable restriction on a lawyer serving as an advocate with respect to a specific type of action, such as a domestic relations matter, in which the lawyer is a necessary witness.

During the course of a personal injury trial, the judge inadvertently learns, from a careless courthouse conversation between the plaintiff and one of his attorneys, a partner in a law firm, that they had consensual sexual relations for the first time the previous night. An associate in the partner's firm is serving as co-counsel for the plaintiff. The associate is competent to represent the plaintiff without the assistance of the partner. The client expressly requests that both lawyers be permitted to continue to represent him. Should the judge allow both lawyers to continue representing the client? No as to both, because sexual relations with the client creates a conflict of interest for the partner that is imputed to the associate. No as to the partner, because sexual relations with the client creates a conflict of interest; but yes as to the associate, because the partner's conflict is not imputed to the associate. Yes, as to both, because the client has waived any conflict of interest. Yes as to both, because the client has control over whether or not to terminate a lawyer-client relationship under these circumstances.

Answer choice B is correct. A conflict of interest arising out of a sexual relationship between a lawyer and a client is personal to the lawyer and is not generally imputed to other attorneys in the lawyer's firm. MRPC 1.8(j), (k). Answer choice A is incorrect because, as noted with regard to answer choice B above, the conflict of interest that arises from sexual relations with a client is generally personal to the lawyer who is engaged in such relations and is not imputed to other members of the lawyer's firm. Answer choice C is incorrect because a client cannot waive a conflict of interest that arises due to sexual relations between the client and the lawyer. MRPC 1.8(j), cmt. 17. Answer choice D is incorrect because, while a client is generally free to terminate a lawyer-client relationship, a client cannot mandate that a lawyer continue to represent the client when doing so would violate a Model Rule of Professional Conduct.

A plaintiff filed a personal injury complaint, and the case was assigned to a judge. After the defendant was served, a partner from a large law firm filed an appearance on behalf of the defendant. The judge's niece was a salaried associate in the estate planning department of the law firm representing the plaintiff. At the initial scheduling conference, the judge disclosed this relationship to the parties. Subsequently, the judge also disclosed that a person listed by the plaintiff as a material witness was his wife's nephew. Neither the niece nor the nephew resided in the judge's household. Neither party moved to disqualify the judge. Other than the disclosures made by the judge, there were no grounds upon which the judge's impartiality could be reasonably questioned. Should the judge disqualify himself from presiding over this action? Yes, because of the judge's relationship with a member of the law firm representing the defendant. Yes, because of the judge's familial relationship with the material witness. No, because neither party moved to disqualify the judge. No, because neither the niece nor the nephew were members of the judge's household.

Answer choice B is correct. A judge must disqualify himself in a matter if he knows that he or his spouse or domestic partner shares a third-degree or closer relationship to a material witness in a case. Since the nephew of the judge's wife falls within the third degree of relationship, the judge should recuse himself. Answer choice A is incorrect because a judge must disqualify himself in a matter if he knows that he or his spouse or domestic partner shares a third-degree or closer relationship to an attorney who represents a party in the case. A third-degree relationship includes a niece of the judge. However, the employment of the judge's niece as a lawyer by the same law firm as the attorney who represents a party does not automatically require the judge to disqualify himself unless the niece has more than a de minimis interest that could be affected by the proceedings. Since the niece was not employed by the department of the firm handling the case and since the income of the niece, as a salaried associate, would not be directly affected by the outcome of the case, the niece's interest in the case likely was de minimis. Accordingly, the judge's impartiality probably cannot be reasonably questioned on the grounds of his relationship with his niece, making B a better answer. Answer choice C is incorrect because a judge may be required to disqualify himself even if a party does not seek his disqualification. Answer choice D is incorrect because, although the standard for disqualification when a relative of the judge has an economic interest in the subject matter of the controversy or is a party to the proceeding is based in part on whether the relative resides in the judge's household, disqualification can be appropriate for a relative within the third degree of relationship even though that relative does not reside in the judge's household in certain circumstances, such as when a relative is likely to be a material witness.

A judge, prior to her recent appointment to the federal court, had been an outspoken and effective opponent of the racial segregation policies of a foreign country's government. As part of its worldwide tour, the foreign government's national soccer team scheduled a soccer match with a team in this country. Several civil rights groups have applied to the judge for an order enjoining the playing of the proposed match. The matter is now pending. Only legal issues are presented. The judge, after painstaking consideration, has privately concluded that she cannot decide the legal questions without bias against the representatives of the foreign government. However, no one has made a motion to disqualify the judge. Must the judge recuse herself in the pending matter? Yes, unless the judge believes she has greater expertise than other judges on the court in legal issues involving racial segregation. Yes, because the judge believes that she cannot be impartial. No, because the only issues presented for decision are legal questions. No, because none of the interested parties has moved to disqualify the judge.

Answer choice B is correct. A judge must recuse herself from a proceeding when the judge's "impartiality may reasonably be questioned." CJC Rule 2.11. A judge must disqualify herself in any proceeding if there is reasonable ground to believe that the judge has a personal bias or prejudice concerning a party. CJC Rule 2.11(A)(1). Recusal is required even when a party has not sought to disqualify the judge. CJC Rule 2.11, cmt. 2. Here, because the judge herself believes that she cannot be impartial, she must recuse herself from the pending matter. Answer choice A is not correct because, even if the judge believes she has the most expertise to decide the matter, she also believes that she cannot be impartial. Answer choice C is not correct, as the fact that the only issues being presented are legal questions is irrelevant. Under the CJC, the judge must recuse herself because she believes she cannot be impartial. Answer choice D is not correct, as the fact that none of the parties has moved to disqualify the judge does not control. The CJC requires the judge to disqualify herself if she believes she cannot be impartial.

An attorney is widely regarded as an exceptionally competent practitioner in the field of criminal law. A client of the attorney became the subject of a grand jury investigation in a matter that could result in a felony indictment. The client lacked sufficient funds to pay for the attorney's services beyond the grand jury stage. He asked the attorney to provide limited representation for a flat fee. Under the arrangement he proposed, the attorney would advise the client concerning the grand jury investigation, but the representation would end when an indictment was returned or the grand jury decided not to indict. The attorney fully advised the client of the practical and legal aspects of the client's proposal. Is it proper for the attorney to accept this limited representation? Yes, because the client and not the attorney suggested this arrangement. Yes, because the attorney and the client may agree to limit the scope of the representation so long as the limitation is reasonable under the circumstances. No, because the attorney should not limit the scope of the representation based on the client's ability to pay. No, because the scope of the representation may not be limited in a criminal case.

Answer choice B is correct. A lawyer and client may agree to limit the scope of the lawyer's representation as long as the client has been fully informed. The limitation on the representation must be reasonable. MRPC 1.2(c). In this case, it is entirely permissible for the client and the lawyer to agree to a limited representation that covers only the client's involvement in the grand jury proceeding. Answer choice A is incorrect, as limitations on the scope of a representation will not be judged based upon whether they were suggested by the lawyer or the client. Answer choice C is incorrect, as lawyers do not have an obligation to provide free legal services to everyone. The ability to pay is one factor in crafting a limited representation agreement and, in this case, it puts the client in a better position than if he were not represented by the lawyer at all. Answer choice D is incorrect, as the Rules do not prohibit limited representation in criminal cases.

A state does not require lawyers to participate in continuing legal education courses. Three lawyers, recently admitted to practice, formed a law partnership in that state. As they considered what expenses the partnership would pay on behalf of each lawyer, a majority decided that the firm would not pay for continuing legal education courses since they were not required by their state. One of the lawyers, who wanted reimbursement for continuing legal education courses, angrily said, "Fine. I won't attend any continuing legal education courses." Is it proper for the lawyer to refuse to attend any continuing legal education courses? Yes, unless the state offers free continuing legal education courses. Yes, if the lawyer independently undertakes continuing study and education in the law. No, because the lawyer cannot maintain competence without attending continuing legal education courses. No, unless the lawyer obtains malpractice insurance.

Answer choice B is correct. A lawyer has a duty of competence. MRPC 1.1. To maintain the requisite knowledge and skill, a lawyer should make efforts to keep abreast of changes in the law and its practice, engage in continuing study and education, and comply with all continuing legal education requirements to which the lawyer is subject. MRPC 1.1, cmt 6. Here, the state has no continuing legal education requirement. The lawyer can properly maintain her knowledge and skills through her own independent study. Answer choice A is incorrect because there is no requirement under the state's law or the Model Rules for an attorney to attend even free continuing legal education courses. Answer choice C is incorrect because the lawyer can maintain her competence through self-study, without attending continuing legal education courses. Answer choice D is incorrect because, under the facts here, it would still be proper for the lawyer to refuse to attend any continuing legal education courses even if she obtained malpractice insurance. The purchase of malpractice insurance does not otherwise relieve an attorney of the duty of competence.

An attorney met with a couple, who were close personal friends of the attorney but had never been clients, regarding the attorney's representation of the couple in an adoption proceeding. The three discussed and agreed upon the legal services that the attorney would render and the flat fee, which was reasonable, that the attorney would charge. The attorney promised to prepare and send the couple a written memorandum of their agreement, but through an inadvertent oversight failed to do so. Upon the successful completion of the adoption proceeding, the attorney sent the clients a bill. In accordance with the attorney's normal practice, the couple was charged, in addition to the agreed upon fee, litigation expenses and court costs, both of which were reasonable in amount. Is the attorney subject to discipline for his actions? Yes, because the attorney failed to put the fee agreement in writing. Yes, because the attorney failed to discuss with the couple that the couple would be responsible for the litigation expenses and court costs. No, because the attorney's failure to reduce the fee agreement to writing was inadvertent and the clients were close personal friends. No, because the attorney's fee, the litigation expenses, and the court costs were all reasonable in amount.

Answer choice B is correct. A lawyer has a duty to communicate with a client not only the fee that the lawyer will charge, but also the expenses for which the client will be responsible. This communication must take place before or within a reasonable time after the commencement of the representation. MRPC 1.5(b). Here, while the attorney did discuss the fee arrangement with the clients, the attorney failed to communicate with them regarding the expenses for which they were responsible. Answer choice A is incorrect because, while it is advisable for the attorney to set forth in writing the agreement regarding fees and expenses, it is not mandated by the Model Rules of Professional Conduct, except with regard to contingent fees. MRPC 1.5, cmt. 2. Answer choice C is incorrect because the fee agreement need not be reduced to writing; the relationship of the attorney to the client is irrelevant to the issue of the items that must be communicated to a client. Answer choice D is incorrect because, while the attorney's fee and the amount of expenses charged to a client cannot be unreasonable (MRPC 1.5(a)), the lawyer also has a duty, as noted with respect to answer A, above, to communicate to the client such fee and expenses.

An attorney prosecuted a former state official on criminal charges stemming from conduct by the former official while in office. Late one afternoon, after the trial in state court had been recessed for the day, the attorney was waiting in a light rain in front of the courthouse to catch a cab back to his hotel, which was approximately one mile away. A juror in the trial drove by and recognized the attorney. The juror stopped and offered the attorney a ride. Upon getting into the car, the attorney thanked the driver and then realized that the driver of the car was a juror in the criminal case. The attorney requested that the juror stop the car and let the attorney out. The juror stopped the car, but indicated that she had no motive other than being a Good Samaritan and asked where the attorney was attempting to go. The attorney, conscious of the fact that it had begun to rain hard, indicated the hotel at which he was staying. The juror drove the attorney to the hotel. During the short trip, they discussed the foul weather. At the hotel, the attorney got out of the car and thanked the juror for the ride. Subsequent to the conviction of the former state official, the encounter between the attorney and the juror came to light. The former state official filed a habeas corpus petition in federal court seeking a new trial due to prosecutorial misconduct based on the encounter. The federal court ruled that, after questioning the juror, the encounter did not result in juror bias and therefore did not violate the fair trial guarantee of the Due Process Clause of the Fourteenth Amendment. Is the attorney subject to discipline for his conduct? Yes, because the attorney failed to report the encounter to the court. Yes, because the attorney engaged in an ex parte communication with a juror. No, because the attorney did not initiate the contact with the juror and did not have the intent to improperly influence the juror. No, because the encounter did not result in a mistrial.

Answer choice B is correct. A lawyer is prohibited from any out-of-court communication with a juror during the trial, even when the communication is unrelated to the trial. MRPC 3.5(b), cmt. b. Answer choice A is incorrect because the Model Rules of Professional Conduct do not require a lawyer to report any inappropriate contact with a juror. Answer choice C is incorrect because the rule against ex parte communication with a juror does not contain an exception for contact that is initiated by the juror or for contact during which the lawyer does not intend to improperly influence a juror. Answer choice D is incorrect because the prohibition in the Model Rules of Professional Conduct against communication with a juror does not require that the communication result in an adverse effect on the trial itself.

An attorney, acting on behalf of a wealthy client, entered into negotiations to purchase land from its owner. Solely in order to forestall the seller from raising the asking price due to the client's financial resources, the client instructed the attorney not to reveal that she was acting on behalf of a client. Adhering to these instructions, the attorney entered into a contract in her own name to purchase the land from its current owner. Prior to the closing date, the client told the attorney that he no longer wanted the land. The attorney informed the owner, who then sold the land to another buyer at a price below the contract price. The seller sued the attorney for expectation damages based on the attorney's breach of the contract. Is the attorney subject to civil liability to seller? Yes, because an attorney acting as a negotiator is nevertheless subject to the Model Rules of Professional Conduct. Yes, because the attorney did not reveal that she was acting on behalf of a client. No, because the attorney was acting on behalf of her client. No, because of the duty of confidentiality.

Answer choice B is correct. A lawyer is subject to liability on a contract entered into with another, even though the lawyer entered into the contract on a client's behalf, if the client's existence or identity was not disclosed to the other party. Although the lawyer, as an agent acting on behalf of the client, may seek indemnification from the client, the lawyer is liable on the contract to the other party, in this case the seller of the land. Answer choice A is incorrect because, although an attorney who acts as a negotiator is subject to the Model Rules of Professional Conduct, the attorney in this case is liable to the seller because the attorney entered into a contract with the seller without revealing that the attorney was acting on behalf of a client. Answer choice C is incorrect because, although the attorney was acting as an agent on behalf of a client in entering into a contract to purchase the property, the attorney is liable to the seller for breach of contract because the seller was not aware of the agency relationship between the attorney and her client. Answer choice D is incorrect because, although the attorney did owe her client a duty of confidentiality, and would have breached that duty by revealing the existence or identity of her client, the attorney is nevertheless liable to the seller on the contract. In this instance, the attorney, as an agent, can seek reimbursement from the client, as the principal, because the attorney's liability to the seller arose from following the client's instructions, but the attorney cannot avoid liability to the seller through adherence to the duty of confidentiality.

An attorney who specializes in estate planning entered into a written agreement with an insurance agent. Under the terms of the agreement, the attorney agrees to recommend that his clients use the insurance agent to acquire life insurance whenever the attorney advises a client to have life insurance as a part of the client's estate plan. Similarly, the insurance agent agrees to recommend the attorney to her clients whenever they need the services of an estate planning lawyer. The agreement provides that the attorney and the insurance agent must each disclose to a client the existence and nature of the agreement. The agreement also provides that the attorney will not recommend another insurance agent to provide life insurance and the insurance agent will not recommend another lawyer to provide estate planning services. Was it proper for the attorney to enter into this agreement? No, because the attorney gave something of value to the insurance agent for recommending the attorney's services. No, because the agreement was exclusive. Yes, because the agreement required both the attorney and the insurance agent to disclose the existence and nature of the agreement to their clients. Yes, because a lawyer may enter into a referral agreement with a non-lawyer.

Answer choice B is correct. A lawyer may enter into a reciprocal referral agreement with another lawyer or a non-lawyer professional, but such an agreement may not be exclusive. MRPC 7.2(b)(4). Answer choice A is incorrect because, although a lawyer must generally not give something of value to another person for recommending the attorney's services, there is an exception for a reciprocal referral agreement that meets the restrictions of the Model Rule. Answer choice C is incorrect because, although the parties to a reciprocal referral agreement must inform their clients of the existence and nature of the agreement, such disclosure does not by itself meet the restrictions of the Model Rule. Answer choice D is incorrect because, even though a lawyer may enter into a referral agreement with a non-lawyer, that agreement cannot be exclusive.

A family court judge appointed an attorney to represent an indigent father in an action brought by a state agency to terminate the father's parental rights with respect to his only child. Until recently, the attorney was employed as a lawyer by the same state agency to collect court-ordered child support. This agency did attempt unsuccessfully to collect child support from the father, although the attorney was not personally involved in this attempt. Due to his employment with the agency, the attorney has personal knowledge of the father, and honestly believes him to be so repugnant that the attorney's ability to represent the father would be impaired. He also believes that it is in the best interests of the child for the father's parental rights to be terminated. Additionally, since the father is notorious in the community where the attorney practices, the attorney fears that representation of the father could have an adverse effect on the attorney's ability to attract clients. Which of the following grounds is most likely a proper basis for the attorney to decline the appointment? The attorney was employed until recently as a lawyer by a state agency that attempted to collect child support from his court-appointed client. The attorney honestly believes that the father is so repugnant that his ability to represent the father would be impaired. The attorney believes that it is in the best interests of the child for the father's parental rights to be terminated. The attorney fears that representation of the father could have an adverse effect on the attorney's ability to attract clients.

Answer choice B is correct. A lawyer may not decline an appointment by a tribunal to represent a person except for good cause. Among the situations giving rise to good cause is a client who is so repugnant to the lawyer as to be likely to impair the lawyer's ability to represent the client. Answer choice A is incorrect. Although representation of a client that is likely to result in violation of the Model Rules of Professional Conduct can constitute good cause for declining a court appointment, the mere fact that the attorney was employed by the same agency that had sought to recover child support from the intended court-appointed client does not give rise to a conflict of interest. The attorney was not personally involved in that matter. Answer choice C is incorrect because a lawyer's belief that the client in a civil matter should not or will not succeed is not sufficient to constitute good cause for declining a court appointment to represent that client. Answer choice D is incorrect because, even though representation of the court-appointed client could have an adverse financial effect on the attorney, good cause likely requires something more, a financial sacrifice that is so great as to be unjust.

The beneficiary of an estate, who was indigent, met with an attorney about the possibility of representing him in a dispute over his share of the estate. The attorney's secretary attended the meeting to take notes. After discussing the matter with the beneficiary, the attorney stated that she could not take the case because her reasonable fee would exceed the amount of the potential recovery. Later, the attorney learned from another attorney that the second attorney had agreed to represent the beneficiary. The attorney discussed the beneficiary's position with the second attorney, advising the lawyer of what she believed were some weaknesses in the case. Did the attorney who elected not to represent the beneficiary behave properly? No, because she had an obligation to represent the indigent beneficiary on a pro bono basis. No, because she breached a duty of confidentiality she owed to the beneficiary. Yes, because any duty of confidentiality was destroyed by the presence of her secretary. Yes, because she could refuse to represent the beneficiary on the basis of lack of financial benefit to the attorney.

Answer choice B is correct. A lawyer may not reveal information learned in consultation with a prospective client, even though no lawyer-client relationship is formed. The fact that the lawyer reveals such information to another lawyer or does so with a motive to aid the client does not justify the revelation, absent the potential client's informed consent. Answer choice A is incorrect because while there is an aspirational goal for a lawyer to provide at least 50 hours of pro bono public legal services each year, there is no requirement that a lawyer accept a case simply because the potential client is indigent. Answer choice C is incorrect because although the attorney-client privilege may be destroyed by the knowing presence of a third party, the secretary was a representative of the attorney and not a third party. Further, the duty of confidentiality remains intact, even if the attorney-client privilege has been destroyed. Answer choice D is incorrect because, while a lawyer generally may refuse to represent a person, including a refusal based on lack of financial benefit to the lawyer, here, the lawyer's actions are improper due to the breach of the duty of confidentiality.

An attorney, a newly licensed sole practitioner, opened a trust account at a local bank into which she deposited a check from a client that constituted an advance payment for future legal services to be rendered by the attorney. Subsequently, the attorney also deposited her own funds into the account in order to pay bank service charges on the account. The attorney did not deposit into the trust account an engagement fee received from another client. The engagement fee represented payment to the attorney for accepting the case, being available to handle the case, and agreeing not to represent another party in the case. The fee did not require the attorney to perform additional legal services. The attorney maintained records related to all account transactions in accordance with the state rules of professional conduct, which are identical to the ABA Model Rules of Professional Conduct. Is the attorney subject to discipline with regard to her actions concerning the trust fund account? No, because the attorney did not receive funds from a third party that belonged to a client. No, because the attorney properly kept her client's property separate from her own property and maintained the necessary records. Yes, because the attorney deposited her own funds into the trust account to pay the bank service charges. Yes, because the attorney did not deposit the engagement fee into the trust account.

Answer choice B is correct. A lawyer must hold funds that are in a lawyer's possession in connection with a representation separate from the lawyer's own funds. Funds must be kept in a separate client trust account, which generally must be maintained in the state where the lawyer's office is situated. MRPC 1.15(a). In addition, a lawyer is required to deposit into a client trust account legal fees and expenses that have been paid in advance by the client. MRPC 1.15(b). Here, the attorney correctly determined that the advance payment was the client's property until the attorney earned it and that the engagement fee was the attorney's property as soon as it had been paid. Answer choice A is incorrect because a lawyer is responsible for safeguarding funds or other property belonging to the client when such funds or other property are received directly from a client as well as when such funds or other property are received from a third party. Answer choice C is incorrect because a lawyer may deposit the lawyer's own funds in a client trust account for the sole purpose of paying bank service charges on the account. MRPC 1.15(b). Answer choice D is incorrect because a lawyer generally must not put his own funds into a client trust account. Thus, here, the attorney correctly did not place the engagement fee, which represented monies already earned by the attorney, into the trust account.

A trust company entered into the following arrangement with an attorney, who was newly admitted to the bar. The trust company would provide the attorney with free office space in the building in which the company had its offices. If a customer of the trust company contacted it about a will, an officer of the company, who is not a lawyer, would advise the customer and help the customer work out the details of the will. The customer would be informed that the necessary documents would be prepared by the trust company's staff. The completed documents would be submitted by an officer of the company to the customer for execution. The attorney, in accordance with a memorandum from the company's trust officer detailing the plan, would prepare the necessary documents. The attorney would never meet with the customer and would not charge the customer for these services. The attorney would be free to engage in private practice, subject only to the limitation that the attorney could not accept employment adverse to the trust company. Is the attorney subject to discipline for entering into the arrangement with the trust company? Yes, because the attorney is restricting his right to practice. Yes, because the attorney is aiding the trust company in the practice of law. No, because the attorney is not charging the customer for his services. No, because the attorney is not giving advice to the company's customers.

Answer choice B is correct. A lawyer must not assist a person who is not admitted to practice in a jurisdiction in the unauthorized practice of law. MRPC 5.5(a). Here, the trust company is practicing law by providing legal advice on wills to its customers. The attorney would be subject to discipline for helping the company in the practice of law. Answer choice A is not correct, as the attorney is not restricting his right to practice; he is aiding the trust company in the unauthorized practice of law. Answer choice C is not correct, as it is irrelevant to the ethical violation of assisting a person who is not admitted to practice in the unauthorized practice of law whether or not the attorney is paid by the client. Answer choice D is not correct, as the violation here is assisting a person who is not admitted to practice in the unauthorized practice of law. It does not matter if the attorney gave advice to the trust company's customers, only that the attorney aided in the unauthorized practice by the trust company.

An attorney, who represented a plaintiff, received a check from the defendant payable to the attorney's order in the sum of $10,000 in settlement of the plaintiff's claim against the defendant. The plaintiff had previously paid the attorney a fee, so no part of the $10,000 was owed to the attorney. Which of the following would not be proper? Endorse the check and send it to the plaintiff. Deposit the check in the attorney's personal bank account and send the attorney's personal check for $10,000 to the plaintiff. Deposit the check in a Clients' Trust Account, advise the plaintiff, and forward a check drawn on that account to the plaintiff. All of the above would be proper.

Answer choice B is correct. A lawyer who receives funds or other property in which a client has an interest must promptly notify the client and is required to pay over the funds, making answer choice A incorrect, because it is a proper course of action. MRPC 1.15 requires that a lawyer hold funds that are in a lawyer's possession in connection with a representation separate from the lawyer's own funds. Therefore, depositing the check in the attorney's personal bank account would be improper, making answer choice B correct. Answer choice C is incorrect because it, too, is a proper course of action, as it avoids commingling the client's funds with the lawyer's funds and meets the requirements of MRPC 1.15.

An attorney with many years of experience in family law matters was serving as a mediator selected by a husband and a wife in an effort to help them agree on the terms of a joint custody agreement regarding their children. After several days, an agreement could not be reached. The parties indicated that they would be giving up on mediation and turning to the court for a decision on the terms of custody. In front of the wife, the husband asked the attorney to represent him in the litigation. The attorney told the wife that her informed consent was needed in order for him to represent the husband in the matter and carefully explained the material risks and reasonably available alternatives to her. In response, the wife said "I give my informed consent to you representing this pig." Is it proper for the attorney to now represent the husband in the matter? No, because the attorney served as a joint mediator in the dispute and is never permitted to thereafter represent one of the parties in the same matter. No, because the consent given to the representation was insufficient to allow the attorney to represent the husband. Yes, because the wife specifically gave her informed consent to the representation. Yes, because this is a family law matter.

Answer choice B is correct. A lawyer who serves as a third-party neutral in a matter may not thereafter serve as a lawyer representing a client in the same matter, unless all parties give their informed, written consent. Here, the wife's consent was only oral and would be insufficient. In addition, the husband did not give his consent in writing. Answer choice A is incorrect, as a mediator may subsequently represent one of the parties in the same matter, so long as all the parties give their informed, written consent. Answer choice C is incorrect, as the wife's oral consent is not enough to allow the representation. All parties must give their informed, written consent to the representation. Answer choice D is incorrect, as it is irrelevant that this is a family law matter. Whenever a lawyer serves as a third-party neutral in a matter, representation of a party in the same matter is limited.

An attorney was passionate about civil rights, but the jurisdiction in which he practiced was less progressive than he. The attorney accepted the case of a client whose claim was not supported by law within the jurisdiction. If the client had been able to bring the claim in another state, however, his claim would likely have been successful. The attorney accepted the claim despite his knowledge that the client would lose because he was confident that the media attention would provide momentum for a change of the law. He notified the client of the likelihood of losing, but the client wished to pursue the claim regardless. Is the attorney subject to discipline for bringing this suit? No, because the client wishes to pursue the claim even if it will be unsuccessful. No, because the case, even if unsuccessful, might lead to a change in existing law. Yes, because he knows the claim will likely lose. Yes, because there is no basis of law in the jurisdiction to support the claim.

Answer choice B is correct. An attorney is prohibited from bringing or defending a proceeding, or asserting or opposing an issue in a proceeding, unless there is a basis in law and fact for doing so that is not frivolous. A good-faith argument for an extension, modification, or reversal of existing law is not frivolous. An action is also not frivolous merely because the attorney believes that the client's position ultimately will not prevail. Here, although the attorney knew that his client would lose in the jurisdiction, the ultimate purpose of the litigation was to lead to a reversal of existing law. Answer choice A is incorrect because the attorney must determine whether a claim is frivolous, independent of the client's objectives. A claim that a client wishes to pursue may, in fact, be frivolous. Answer choice C is incorrect because a claim is not frivolous merely because the client's position will ultimately not prevail. Answer choice D is incorrect because, although there was no legal precedent in the jurisdiction that would help him win the case, the claim was not frivolous in light of the attorney's attempt to make a good-faith argument to modify existing law.

An attorney represented a client who was sued following a car accident. The attorney and her client discussed the allegations in the complaint, and the client denied the plaintiff's assertion that the client had been speaking on his cell phone at the time of the accident. Relying on this information, the attorney filed a response to the complaint including such a denial. Later, the attorney, in reviewing the client's cell phone records, discovered that a call had been placed from the client's cell phone immediately before the accident. When the attorney confronted the client with this information, the client admitted that he had lied to the attorney. The attorney figured that the information would come out shortly, since the records were being produced to the plaintiff, and did not act upon it. Were the attorney's actions proper? No, because the attorney should have taken reasonable steps to confirm the client's factual assertion before including it in a pleading. No, because the attorney was required to correct any false statement of material fact he made to the court. Yes, because the attorney did not knowingly include a false statement of fact in a pleading. Yes, because the attorney was not required to confirm the client's factual assertions before including them in a pleading.

Answer choice B is correct. An attorney must not knowingly make a false statement of fact to a tribunal or fail to correct a false statement of material fact previously made to the tribunal by the attorney. In this case, the attorney was required to correct her earlier false statement regarding her client's cell phone use. Answer choice A is incorrect because an attorney is not required to confirm a client's factual assertions before including them in a pleading. Although an attorney usually files pleadings, the attorney is not usually required to have personal knowledge of matters asserted in the pleadings. Answer choice C is incorrect because although the attorney did not act improperly in including the statement in the original pleading, she acted improperly when she failed to correct it after learning that it was false. Answer choice D is incorrect because although the attorney was not required to confirm the client's assertions, she did not act properly because she failed to correct her earlier false statement.

An attorney took out an advertisement in a local newspaper, which stated that the cost of his services was "50 percent cheaper than the customary attorney fees in town." The attorney never researched the fees of other attorneys in the area, but he planned to reduce his own fees to half of any such fees if and when a potential client brought them to his attention. Would the attorney's actions subject him to discipline? Yes, because the attorney posted an advertisement that mentioned his rates. Yes, because the attorney's claims regarding his rates were unsubstantiated and specific. No, because the attorney planned to modify his rates to conform to the advertisement. No, because the attorney did not state a specific dollar amount for his rate.

Answer choice B is correct. An unsubstantiated comparison of the attorney's services or fees with the services or fees of other attorneys may be misleading if presented with such specificity as would lead a reasonable person to conclude that the comparison can be substantiated. Here, the attorney has not researched the fees of other local attorneys and could not substantiate that his fees are 50 percent lower. However, a reasonable person would conclude that, because he stated that his rate was 50 percent lower than the rates of other local attorneys, he likely had a basis for his statement. Answer choice A is incorrect because an attorney is not prohibited from advertising his fees if the advertisement is not misleading and meets the other requirements for advertisements. Answer choice C is incorrect because, although the attorney may be willing to adjust his fees to conform to the advertisement, the advertisement is nevertheless misleading due to the attorney's failure to substantiate the comparison. Answer choice D is incorrect because there is no requirement that an attorney who advertises his rates must do so by stating a specific dollar amount.

Leaving an airport, an attorney who primarily practices criminal law shared a cab with a medical doctor. The cab was involved in a collision, and the doctor was seriously injured while the attorney was only shaken up. The attorney accompanied the doctor to the hospital in the ambulance. The doctor believed that she was dying and asked the attorney to prepare a simple will for her. The attorney told the doctor, "I have never prepared a will, but hope that I can remember the basics from law school." The attorney then complied with the doctor's request. The doctor signed the will, and the two paramedics in the ambulance signed as witnesses. Was it proper for the attorney to prepare the will? Yes, unless the attorney omitted some required formality that rendered the will invalid. Yes, because the attorney provided legal services that were reasonably necessary under the circumstances. No, unless the doctor waived the attorney's malpractice liability. No, because the attorney did not have the skill required for the representation.

Answer choice B is correct. In an emergency, a lawyer may give advice or assistance in a matter in which the lawyer does not have the skill ordinarily required when referral to or consultation with another lawyer would be impractical. MRPC 1.1, cmt 3. Given the emergency situation and the medical doctor's belief that she was dying, the attorney properly provided the legal services that were reasonably necessary under the circumstances. Answer choice A is not correct, as it was proper for the attorney to act because of the emergency circumstances, and it is irrelevant to the propriety of that action that he may have omitted some required formality. Answer choice C is not correct, as the attorney could properly act in the emergency, regardless of whether the doctor waived malpractice liability. Answer choice D is not correct, as the emergency situation overrode the fact that the attorney did not have the required skill for purposes of propriety under the Model Rules.

An attorney represented a landlord in a variety of matters over several years. An elderly widow living on public assistance filed suit against the landlord alleging that the landlord withheld without justification the security deposit on a rental unit that she vacated three years ago. She brought the action for herself, without counsel, in small claims court. The attorney investigated the claim and learned that it was legally barred by the applicable statute of limitations, although the plaintiff's underlying claim was meritorious. The attorney told the landlord of the legal defense, but emphasized that the plaintiff's claim was just and that, in all fairness, the security deposit should be returned to her. The attorney told the landlord: "I strongly recommend that you pay the plaintiff the full amount with interest. It is against your long-term business interests to be known in the community as a landlord who routinely withholds security deposits even though the tenant leaves the apartment in good condition. Paying the claim now will prevent future headaches for you." Was the attorney's conduct proper? Yes, if the landlord did not object to the attorney's advice and paid the plaintiff's claim. Yes, because the attorney may refer to both legal and non-legal considerations in advising a client. No, unless the attorney's engagement letter informed the landlord that his advice on the matter would include both legal and non-legal considerations. No, because in advising the landlord to pay the full claim, the attorney failed to represent zealously the landlord's legal interests.

Answer choice B is correct. In representing a client, a lawyer is required to exercise independent professional judgment and render candid advice. In rendering advice, a lawyer may refer not only to law, but also to other considerations such as moral, economic, social, and political factors that may be relevant to the client's situation. MRPC 2.1. Answer choice A is incorrect because it is irrelevant what the landlord chose to do in response to the attorney's properly given advice. Answer choice C is incorrect, as MRPC 2.1 permits a lawyer to refer to both legal and non-legal considerations when rendering advice, and there is no requirement that a lawyer advise a client of this in an engagement letter. Answer choice D is incorrect. Although a lawyer must be dedicated and committed to the interests of the client, a lawyer has a duty to exercise independent professional judgment and render candid advice, which is what the attorney did in this case.

An attorney is a well-known, highly-skilled litigator. The attorney's practice is in an area of law in which the trial proceedings are heard by the court without a jury. In an interview with a prospective client, the attorney said, "I make certain that I give the campaign committee of every candidate for elective judicial office more money than any other lawyer gives, whether it's $500 or $5,000. Judges know who helps them get elected." The prospective client did not retain the attorney. Is the attorney subject to discipline? Yes, if the attorney's contributions are made without consideration of candidates' merits. Yes, because the attorney implied that she receives favored treatment from judges. No, if the attorney's statements were true. No, because the prospective client did not retain the attorney

Answer choice B is correct. It is professional misconduct for a lawyer to state or imply an ability to improperly influence a government agency or official. MRPC 8.4(e). Here, the attorney's statements implied an ability to improperly influence judges. Answer choice A is not correct, as the contributions are irrelevant. The professional misconduct occurs as a result of the attorney's statements to the prospective client. Answer choice C is not correct, as the attorney would be subject to discipline even if the statements were true. It is professional misconduct for a lawyer to state or imply an ability to improperly influence a government agency or official. MRPC 8.4(e). Answer choice D is not correct, as the violation of MRPC 8.4(e) occurs regardless of whether or not the prospective client retains the lawyer.

An attorney practices law in a state that has experienced a business recession and where several banks have failed and others are severely pressed to preserve their solvency. The attorney maintains a Clients' Trust Account in a bank and that account is insured by the Federal Deposit Insurance Corporation against losses up to $100,000. The attorney also maintains his regular office account in the same bank and that account is insured to $100,000. During a particularly busy time, the attorney's bookkeeper told the attorney that the balance in the Clients' Trust Account had increased to $150,000. The bookkeeper noted that the office account had a balance of $30,000. Which of the following courses of action by the attorney would be proper? Leave the Clients' Trust Account as is if the balance is likely to decrease to less than $100,000 within the next ten days. You Selected: Open another Clients' Trust Account in another bank within the state, and transfer some funds to the second Clients' Trust Account to maintain a fully insured balance in both accounts. Open another Clients' Trust Account in a bank situated in another state, which is not experiencing bank failures, and transfer some funds to the second Clients' Trust Account to maintain a fully insured balance in both accounts. Temporarily transfer $50,000 from the Clients' Trust Account to the office account so the balance in both accounts is fully within insured limits.

Answer choice B is correct. It would be proper to transfer funds to another clients' trust account at another bank to ensure that all funds were properly insured by the Federal Deposit Insurance Corporation (FDIC). Under MRPC 1.15 (a), a lawyer must hold funds that are in a lawyer's possession in connection with a representation separate from the lawyer's own funds. Funds must be kept in a separate clients' trust account maintained in the state where the lawyer's office is situated (or elsewhere with the consent of the client or third person). Answer choice A is incorrect because, given the bank failures in the state, it would be improper for the attorney to leave funds that would exceed the amount insured by the FDIC in the original bank. Answer choice C is incorrect because clients' trust accounts must be maintained in the state in which the attorney's office is situated. Answer choice D is incorrect, as client funds must be kept separate from the lawyer's own funds, so it would be improper to transfer any of the funds to the office account.

A client telephoned an attorney who had previously represented him. The client described a problem on which he needed advice and made an appointment for the following week to discuss the matter with the attorney. Prior to the appointment, the attorney performed five hours of preliminary research on the client's problem. At the end of the appointment the client agreed that the attorney should pursue the matter and agreed to a fee of $100 per hour. The client then gave the attorney a check for $5,000 to cover the five hours already worked and as an advance on additional fees and expenses. The attorney gave the check to the office bookkeeper with directions to deposit the check into the client trust account and immediately transfer $3,000 to the general office account to cover the five hours of research already conducted plus the 25 additional hours she would spend on the matter the following week. At that time, the attorney reasonably believed that she would spend 25 additional hours on the case. The bookkeeper followed these directions. The next week, the attorney worked diligently on the matter for 23 hours. Reasonably believing that no significant work remained to be done on the matter, the attorney directed the bookkeeper to transfer $200 from the general office account to the client trust account. The attorney then called the client and made an appointment to discuss the status of the matter. Is the attorney subject to discipline? Yes, because the attorney accepted legal fees in advance of performing the work. Yes, because the attorney transferred funds for unearned fees to the general office account. No, because the attorney transferred the $200 owed to the client from the general office account to the client trust account. No, because the attorney reasonably believed that she would spend 25 additional hours on the case.

Answer choice B is correct. MRPC 1.15(c) requires a lawyer to "deposit into a client trust account legal fees and expenses that have been paid in advance, to be withdrawn by the lawyer only as fees are earned or expenses incurred." Since the attorney had earned only $500, for five hours of work, at the time of receiving the client's check, only $500 should have been withdrawn from the trust account and placed in the general office account. Answer choice A is incorrect, as an attorney may accept legal fees in advance of performing legal work, as long as the lawyer complies with the requirement of MRPC 1.15(c) that the advance fees be deposited in a client trust account until earned. Answer choice C is incorrect. Under MRPC 1.15(c), a lawyer may not hold fees paid in advance in the lawyer's general office account and later transfer unearned portions to the client trust account, as the attorney did here. Rather, the lawyer must deposit the advance payment in the client trust account and withdraw the payment for fees as they are earned. Answer choice D is incorrect. Regardless of whether a lawyer expects to earn fees paid in advance, MRPC 1.15(c) requires the lawyer to deposit the advance payment in the client trust account until the fees are actually earned.

An attorney has been representing a client in a matter in litigation. During protracted pre-trial proceedings, the client complained bitterly about the time and expense involved and insisted that the attorney take steps to terminate the pre-trial proceedings. The attorney believes that to do so would jeopardize the client's interests and has so informed the client. The attorney believes that the case cannot be adequately prepared for trial without further pre-trial proceedings that will require an additional six months' delay and involve further expense. The client insists that the attorney forego any further pre-trial proceedings and set the case for trial at the earliest available date. There are several other competent lawyers who are willing to undertake the representation. Is it proper for the attorney to ask leave of the court to withdraw? Yes, because a lawyer may discontinue representation in a civil case at any time before trial. Yes, because the client's conduct makes it unreasonably difficult for the attorney to represent the client effectively and competently. No, because the attorney must follow the client's instructions. No, unless the client consents to the attorney's withdrawal.

Answer choice B is correct. MRPC 1.16(b)(6) allows a lawyer to request permission to withdraw from representing a client when the client has made representation unreasonably difficult for the lawyer. Here, the client is insisting on a course of conduct that makes continuing the representation unreasonably burdensome on the attorney, and there are other competent lawyers willing to undertake the representation. Requesting leave to withdraw would be proper under such circumstances. Answer choice A is not correct, as a lawyer may not automatically discontinue representation in a civil case at any time before trial. The Model Rules require certain conditions to be met. Answer choice C is not correct, as MRPC 1.16(b)(6) allows the attorney to request leave to withdraw when the client's instructions make continuing the representation unreasonably burdensome. Answer choice D is not correct, as client consent to the withdrawal is not required by the Model Rules.

An attorney is representing a plaintiff in a paternity suit against a defendant. Both the plaintiff and the defendant are well-known public figures, and the suit has attracted much publicity. The attorney has been billing the plaintiff at an agreed hourly fee for his services. Recently, the plaintiff told the attorney, "I'm going broke paying you. Why don't you let me assign you all media rights to books, movies, or television programs based on my suit as full payment for all services you will render me between now and the conclusion of the suit?" The attorney replied, "I'll consider it, but first you should seek independent advice about whether such an arrangement is in your own best interests. Why don't you do so and call me next week." Is the attorney subject to discipline if he agrees to the plaintiff's offer? Yes, because the amount received by the attorney would be contingent on the receipts from the sale of media rights. Yes, because the attorney has not concluded the representation of the plaintiff. No, because the paternity suit is a civil and not a criminal matter. No, if the plaintiff received independent advice before entering into the agreement.

Answer choice B is correct. MRPC 1.8(d) prohibits a lawyer from negotiating for media rights relating to representation of a client prior to the conclusion of the representation. Answer choice A is not correct, as it is irrelevant that the amount received would be contingent. What subjects the attorney to discipline is the violation of MRPC 1.8(d). Answer choice C is not correct, as MRPC 1.8(d) applies to both civil and criminal matters. Answer choice D is not correct, as MRPC 1.8(d) is an absolute prohibition on negotiating for media rights prior to the conclusion of the representation. It is irrelevant whether or not the plaintiff received independent advice before entering into the agreement.

An attorney represented a respondent in proceedings instituted by a child protection services agency to establish the paternity of a child and to recover past-due child support. The mother of the child had refused to file a complaint, had refused to retain a lawyer, and in fact had asked that the agency not file any action whatsoever. However, state law permitted the agency to commence paternity and support proceedings in its own name in such circumstances. The attorney contacted the mother without the knowledge or consent of the agency or its lawyers. The attorney identified himself to the mother as "an officer of the court" and told the mother that he was investigating the matter. Based upon what she told him, the attorney prepared and the mother signed an affidavit truthfully stating that the respondent was not the father of the child. Is the attorney subject to discipline? Yes, because the attorney acted without the knowledge or consent of the agency or its lawyers. Yes, because the attorney implied that he was disinterested in the matter. No, because all of the attorney's statements to the mother were true. No, because the attorney did not give the mother legal advice.

Answer choice B is correct. MRPC 4.3, which governs the conduct of lawyers dealing with unrepresented persons, prohibits a lawyer from stating or implying that the lawyer is disinterested in a matter in which the lawyer represents an interested party. The mother was an unrepresented person in this case. Answer choice A is incorrect. MRPC 4.2 prohibits a lawyer from communicating about the subject of the representation with a person the lawyer knows to be represented by another lawyer in the matter unless that lawyer consents; however, in this case, the mother was not represented by the agency or its lawyers or by any other lawyer. Answer choice C is incorrect because, although it may be true that all lawyers are "officers of the court" in some sense, in this context such a statement clearly implied that the lawyer was disinterested in the outcome of the case, and such statements are prohibited by MRPC 4.3. Answer choice D is incorrect because MRPC 4.3 prohibits a lawyer from giving an unrepresented person legal advice, but it also prohibits a lawyer from stating or implying that he is disinterested in a matter in which the lawyer represents an interested party, which is what the attorney did here.

An attorney and a restaurant owner entered into a reciprocal referral arrangement. The attorney agreed to prominently display ads for the restaurant in her office, and to mention the restaurant to all of her clients who requested a recommendation of a nearby place to eat. In return, the owner agreed to prominently display ads for the attorney's firm in the restaurant and to recommend the attorney to any of his customers who indicated a need for the services provided by the attorney. The reciprocal referral agreement was not exclusive, and the clients and customers would be informed of the existence and nature of the agreement. Is the attorney subject to discipline for entering into this agreement? Yes, because she asked the owner to place ads for the firm in the restaurant. Yes, because the agreement provided something of value to the restaurant owner in return for recommending the attorney's services. No, because she did not pay the restaurant owner for the referrals. No, because the agreement is not exclusive, and the clients and customers will be informed of the existence and nature of the agreement.

Answer choice B is correct. MRPC 7.2(b) prohibits a lawyer from giving anything of value to a person for recommending the lawyer's services. Subsection (4) of that rule provides for an exception for reciprocal referral agreements with other lawyers or with non-lawyer professionals. However, even if the restaurant owner were considered a "professional" under this exception, this is not a mere reciprocal referral agreement. A reciprocal referral agreement would only require each professional to refer clients to the other professional. Here, the attorney's display of additional advertising is an additional exchange of value that goes beyond a mere referral. Therefore, the attorney is subject to discipline for this arrangement. Answer choice A is incorrect because the Model Rules do not prohibit a lawyer from asking others to post advertisements concerning the lawyer's services. Answer choice C is incorrect, because the attorney's promise to display the restaurant ads and to mention the restaurant to clients seeking a recommendation is something of value. Answer choice D is incorrect because, even if a reciprocal referral agreement would be permitted with the restaurant owner, the attorney's display of additional advertising is an additional exchange of value that goes beyond a mere referral. Therefore, the attorney is subject to discipline for this arrangement.

Alpha and Beta practiced law under the firm name of Alpha & Beta. When Beta died, Alpha did not change the firm name. Thereafter, Alpha entered into an arrangement with another attorney, Gamma. Gamma pays Alpha a certain sum each month for office space, for use of Alpha's law library, and for secretarial services. Alpha and Gamma each have their own clients, and neither participates in the representation of the other's clients or shares in fees paid. On the entrance to the suite of offices shared by Alpha and Gamma are the words "Law Firm of Alpha, Beta & Gamma." Is Alpha subject to discipline? Yes, because Beta was deceased when Alpha made the arrangement with Gamma. Yes, because Gamma is not a partner of Alpha. No, because Alpha and Beta were partners at the time of Beta's death. No, because Gamma is paying a share of the rent and office expenses.

Answer choice B is correct. MRPC 7.5(d) prohibits lawyers from stating or implying that they practice in a partnership or other organization when that is not the fact. The sign at the entrance to the suite of offices represents to the public that Alpha and Gamma practice together in a single law firm and it is therefore prohibited by MRPC 7.5(d), as well as by MRPC 7.1, which prohibits false or misleading communications about a lawyer's practice. Answer choice A is incorrect, as MRPC 7.5 and its Comment permit a lawyer to use the name of a deceased member of a firm when there has been a continuing succession in the firm's identity, regardless of whether the deceased partner had given permission for the continuing use. Answer choice C is incorrect because Alpha was indeed permitted to continue to use Beta's name in his firm's name under MRPC 7.5. However, MRPC 7.5(d) prohibits him from stating or implying that he and Gamma practice together in a single law firm. Answer choice D is incorrect, as paying a share of the rent and other office expenses does not make Gamma a partner of Alpha, and MRPC 7.5(d) prohibits Alpha from stating or implying that he and Gamma practice together in a single law firm.

An associate in a small law firm represented a property owner in a suit against the former owner, a corporation, for fraud with regard to the sale of the property. The associate has learned that the largest shareholder of a subsidiary of the corporation is the managing partner of the law firm. If the property owner is successful in the action, the corporation and its subsidiary would be crippled financially, and the managing partner would likely see a significant decline in the value of his stock, which represents a substantial portion of the managing partner's retirement portfolio. Must the associate share with the property owner the information regarding the managing partner's relationship to the corporation and seek the property owner's consent in order to continue representing him? Yes, because a personal disqualification of a member of a firm is imputed to all other members of the firm. Yes, because there is a significant risk of the managing partner's relationship to the opposing party materially limiting the associate's representation of the property owner. No, because the associate, not the managing partner, is representing the property owner. No, because the associate's duty of confidentiality prevents him from disclosing information about the partner's involvement in the corporation.

Answer choice B is correct. Members of a law firm are generally treated as one lawyer for purposes of the disqualification rules. When one member of the firm is disqualified from representing a client, all members of the firm are generally disqualified. However, there is an exception when the disqualification is based on a personal interest of a member of the firm and that interest does not present a significant risk of materially limiting the representation of the client by the remaining lawyers in the firm. Here, the managing partner would be personally disqualified from representing the property owner because of the managing partner's significant financial ties to the opposing party in the lawsuit (i.e., the corporation) and the significant risk that the associate would be materially limited in representing the property owner, out of loyalty to the partner. Answer choice A is incorrect because generally a disqualification based on a personal interest of a member of the firm is not imputed to other members of the firm if it does not present a significant risk of materially limiting the representation of the client by the remaining lawyers in the firm. Answer choice C is incorrect because members of a law firm are generally treated as one lawyer for purposes of the disqualification rules. In this case, the managing partner's disqualification is imputed to the associate as discussed with respect to answer choice B. Answer choice D is incorrect because the duty of confidentiality is owed by an attorney to a client. This duty does not apply to information regarding other members of the attorney's law firm.

An attorney was employed as a lawyer by the state Environmental Control Commission (ECC) for ten years. During the last two years of her employment, the attorney spent most of her time in the preparation, trial, and appeal of a case involving the discharge by a corporation of industrial effluent into a river in the state. The judgment in the case, which is now final, contained a finding of a continuing and knowing discharge of a dangerous substance into a major stream by the corporation and assessed a penalty of $25,000. The governing statute also provides for private actions for damages by persons injured by the discharge of the effluent. The attorney recently left the employment of ECC and went into private practice. Three landowners have brought private damage actions against the corporation. They claim their truck farms were contaminated because they irrigated them with water that contained effluent from dangerous chemicals discharged by the corporation. The corporation has asked the attorney to represent it in defense of the three pending actions. Is the attorney subject to discipline if she represents the corporation in these actions? Yes, unless the judgment in the prior case is determinative of the corporation's liability. Yes, because the attorney had substantial responsibility in the matter while employed by ECC. No, because the attorney has acquired special competence in the matter. No, if all information acquired by the attorney while representing ECC is now a matter of public record.

Answer choice B is correct. Model Rule 1.11(a) prohibits a former government lawyer from representing a client in a matter in which the lawyer participated personally and substantially as a government lawyer, unless the appropriate government agency gives its informed consent, confirmed in writing, to the representation. As the attorney had substantial responsibility in the matter while employed by the ECC and there is no indication of informed consent by the ECC, she is subject to discipline. Answer choice A is not correct, as the determination of liability in the prior matter is irrelevant. What controls is that the attorney had substantial responsibility in the matter and there is no indication of informed consent by the ECC. Answer choice C is not correct, as what controls the issue is that the attorney had substantial responsibility in the matter, and that there has been no informed consent by the ECC. Answer choice D is not correct; the fact that the information is public is irrelevant as to whether the rules permit the attorney to represent the corporation. What controls that issue is that the attorney had substantial responsibility while representing the ECC, and there has been no informed consent by the ECC to the representation of the corporation.

An attorney participated in a program sponsored by a nonprofit association to provide representation at no cost to low-income individuals involved in tax disputes with the Internal Revenue Service (IRS). The attorney, who initially lacked the expertise necessary to properly represent such individuals, attended classes sponsored by an organization through which she gained sufficient knowledge of the tax law to competently represent such individuals. After satisfactorily representing several individuals referred to the attorney by the organization, the attorney learned from a friend about a store clerk with limited means who was involved in a dispute with the IRS regarding a previously claimed earned income tax credit. The attorney made a phone call to the store clerk, with whom she had had no previous contact either professionally or personally, and offered to represent the store clerk without charge in her tax dispute with the IRS. Was the attorney subject to discipline for making the phone call? No, because the attorney is required to provide pro bono legal services to those who are unable to pay. No, because the attorney did not charge for her services. Yes, because the attorney made live telephone contact with an individual in order to solicit the individual to utilize the attorney's legal services. Yes, because the attorney had no previous personal or professional relationship with the clerk.

Answer choice B is correct. Solicitation of professional employment from a prospective client that is done by live telephone contact is prohibited only when a significant motive for doing so is the lawyer's pecuniary gain. MRPC 7.3. Since the attorney offered to represent the individual without charge, this solicitation prohibition does not apply to the attorney's phone call to the individual. Answer choice A is incorrect because, although a lawyer should provide pro bono representation for persons who are unable to pay, the failure to adhere to this aspirational goal is not enforced through the disciplinary process. MRPC 6.1, cmt. 12. Answer choice C is incorrect because, while the attorney did make live telephone contact with the individual to solicit the individual to utilize the attorney's legal services, such contact is not prohibited, as noted with regard to answer choice A, since the attorney did not have her own pecuniary gain as a motive. Answer choice D is incorrect because, although there is an exception to the solicitation of legal business by a lawyer when the person solicited has a familial, close personal, or prior professional relationship with the soliciting lawyer, the solicitation prohibition does not apply when a lawyer's own pecuniary gain is not a motive for making the solicitation.

An established law firm located in a state capital focused mainly on litigation, but it also provided legal services related to lobbying. The firm created a lobbying department and hired a former state legislator, who was not an attorney, to head this department. The former legislator received as compensation a percentage of the fees from all services provided to clients that he brought to the firm, including any legal services, but he did not direct the professional judgment of the attorneys in his department, nor was he made a partner. Is the law firm's employment of the former legislator proper? No, because a non-lawyer may not hold a senior position in a law firm. No, because attorneys may not share fees with non-lawyers, as provided by this agreement. Yes, because the legislator was not made a partner of the firm. Yes, because the legislator did not provide legal services.

Answer choice B is correct. The Model Rules of Professional Conduct generally prohibit fee sharing by attorneys with non-lawyers. Because the agreement provides for the former state legislator, who is not an attorney, to share fees—including legal fees—with the partners of the firm, it is not proper. While there are some exceptions to this general rule, this agreement does not address any of those. Answer choice A is incorrect because a non-lawyer may hold a position within a law firm, so long as the non-lawyer is not a corporate director or officer, and so long as he does not occupy a position of similar responsibility, own any interest in the association, or direct the professional judgment of any lawyers in the association. Answer choice C is incorrect because though the legislator was not made a partner, the fee-sharing agreement nonetheless violates the Rule that prohibits fee-sharing with non-lawyers. Answer choice D is incorrect because although the former legislator did not perform legal services, the agreement indicates that he will share legal fees with the attorneys in the firm.

An attorney represented a plaintiff in litigation that was settled, with the plaintiff's approval, for $25,000. The attorney received a check in that amount from the defendant, payable to the attorney's order. The attorney endorsed and deposited the check in the attorney's Clients' Trust Account. The attorney promptly notified the plaintiff and billed him $5,000 for legal fees. The plaintiff disputed the amount of the fee and wrote the attorney, stating, "I will agree to pay $3,000 as a reasonable fee for the work you did, but I will not pay anything more than that." Based on the above facts, which of the following actions is proper for the attorney to take? Retain the entire $25,000 in the attorney's Clients' Trust Account until the fee dispute is settled. Send the plaintiff $20,000, transfer $3,000 to the attorney's office account, and retain $2,000 in the attorney's Clients' Trust Account until the dispute is settled. Send the plaintiff $20,000 and transfer $5,000 to the attorney's office account. None of the above is proper.

Answer choice B is correct. The attorney is required to pay the plaintiff the $20,000 undisputed portion of the settlement, and may properly transfer the undisputed fee of $3,000 to the attorney's office account. The $2,000 that remains in dispute must remain in the Clients' Trust Account until the dispute is resolved. Answer choice A is not proper, as MRPC 1.15(e) requires the attorney to promptly pay the plaintiff the undisputed portion of the settlement ($20,000). Answer choice C is not proper, as $2,000 of the $5,000 fee remains disputed by the plaintiff and must remain in the Clients' Trust Account pursuant to MRPC 1.15.

An attorney practices law in the same community as a lawyer who is running for election as a state judge. The attorney has frequently observed the judicial candidate's courtroom demeanor in litigated cases. Based on those experiences, the attorney believes that the judicial candidate does not have a proper judicial temperament. A local news reporter asked the attorney how he would rate the candidate, and the attorney responded in good faith that he believed the candidate was unsuited for the bench and lacked the proper judicial temperament for a judge. A local newspaper with a wide circulation quoted the attorney's remarks. Were the attorney's remarks proper? Yes, because the attorney was not seeking judicial office. Yes, because the attorney believed the candidate was unsuited for the bench. No, because the remarks serve to bring the judiciary into disrepute. No, because a lawyer should not publicly comment on candidates for judicial office.

Answer choice B is correct. The attorney's expression of good-faith views about a judicial candidate's qualifications was proper because "[e]xpressing honest and candid opinions on such matters contributes to improving the administration of justice." MRPC 8.2, cmt. [1]. Answer choice A is incorrect, as it would have been proper for the attorney to provide good-faith opinions about the judicial candidate's qualifications even if the attorney had also been seeking judicial office, since such opinions promote the administration of justice by assisting the voters in making informed electoral decisions. See MRPC 8.2, cmt. [1]. As a candidate for public office, the attorney would have been required to comply with applicable provisions of the Code of Judicial Conduct, but that code would not have forbidden the attorney's comments. Answer choice C is incorrect because, although false statements about judicial candidates can unfairly undermine public confidence in the administration of justice, a lawyer's good-faith expressions of opinion about the judicial candidate's temperament, based on the lawyer's experience, promote the administration of justice by enabling voters to make informed decisions, and such expressions are encouraged under MRPC 8.2. Answer choice D is incorrect because, although false public statements about a judicial candidate's qualifications are improper, not all public statements about judicial candidates are improper. On the contrary, honest and candid opinions are encouraged under MRPC 8.2.

A judge nearing the end of her term heard the morning docket calls on behalf of her sick colleague. Among the morning's items was a motion to extend the time for discovery. An attorney for each party was present, and each attorney had signed the motion, agreeing to the extension of time. The judge granted the extension. The following month the judge's term expired, and shortly thereafter the judge secured a position as a managing member of the law firm that represented one of the parties who had sought the discovery extension. The judge was not assigned to work on that party's case, was screened from receiving information about the case or participating in discussions about its handling, and did not receive any portion of the fee received by the firm from the client. Several months later, upon learning from an independent source of the judge's association with the firm, the other party to the case moved to have the firm's attorneys removed from the case. Is it proper for the firm's attorneys to continue representation of the client? Yes, because the former judge was timely screened from participation in the matter and did not receive any portion of the fee from it. Yes, because the former judge had not substantially participated in the case as a judge. No, because the former judge had personally participated in the case as a judge. No, because the firm did not give written notice to the other party in the case of the former judge's employment by the firm.

Answer choice B is correct. The fact that a former judge exercised administrative responsibility in a court would not prevent her from acting as a lawyer in a matter in which the judge had previously exercised remote or incidental administrative responsibility that did not affect the merits of the case. MRPC 1.12, cmt. 1. The judge's participation in this matter was administrative and did not affect the merits of the action. Since the judge herself would not be disqualified, her firm would be allowed to continue representation. Answer choice A is incorrect because screening is unnecessary unless the judge had substantially and personally participated in the adjudication of the case. MRPC 1.12(c). Answer choice C is incorrect because a judge's participation in the case must be substantial and personal to trigger the disqualification rule. Answer choice D is incorrect for the same reason; written notice is required only if a judge's participation is both personal and substantial. MRPC 1.12(a).

Upon learning that a judge on the local trial court is retiring, an attorney decides to run for election to the position that the judge is vacating. In this state, the judgeship is determined by a local partisan election. In accordance with the timing restrictions imposed by state law, the attorney announces his intent to seek the endorsement of a particular political party by running as a candidate in the primary of that political party. In addition, the attorney actively seeks the endorsement of various elected local officials, including seeking the endorsement of the judges who are currently up for re-election on the local trial court. Are the attorney's efforts to be elected to the judgeship proper? Yes, because the attorney, as only a candidate for judicial office rather than a sitting judge, is not subject to the Code of Judicial Conduct. Yes, because the attorney, as a candidate for judicial office, has adhered to the Code of Judicial Conduct. No, because the attorney, as a candidate for judicial office, may not seek the endorsement of a political party. No, because the attorney, as a candidate for judicial office, may not seek the endorsement of judges who are running for re-election to the same judicial office.

Answer choice B is correct. The lawyer, as a candidate for judicial office, is subject to Canon 4 of the Code of Judicial Conduct, which relates to political and campaign activities. A candidate for a judicial office that is determined by a partisan election may seek the endorsement of a partisan political organization, such as a political party. CJC Rule 4.2(C), cmt. 3. In addition, a candidate for any elective judicial office may seek, accept, and use an endorsement from any person, including candidates for the same office, which encompasses judgeships on the same court. CJC Rule 4.1, cmt. 4; 4.2(B)(3), (5), cmt. 6. Answer choice A is incorrect because, as noted with respect to answer choice B, the attorney as a candidate for judicial office is subject to Canon 4 of the Code of Judicial Conduct. Answer choice C is incorrect because, as noted with respect to answer choice B, the attorney, as a candidate for elective judicial office, may seek the endorsement of a political party. Answer choice D is incorrect because, as noted with respect to answer choice B, the attorney, as a candidate for elective judicial office, may seek the endorsement of elected officials, including judges who are up for re-election to the same judicial office.

An attorney is a senior partner at a law firm in which there are 50 lawyers. The firm pays each of its lawyers a fixed annual salary. In addition, at year's end, each lawyer receives a bonus from the profits of the firm in the proportion that the annual salary of each bears to the total of the fixed annual salaries of all lawyers. The attorney plans to introduce a new management plan under which the firm's non-lawyer office administrator would have general charge of all business matters but would not participate in any decisions involving legal judgment. The administrator would be paid a fixed annual salary and would be included as a participant in the firm's bonus plan on the same basis as the lawyers in the firm. This would usually yield a bonus of approximately one-fourth to one-third of the administrator's total annual compensation. The amount paid to the administrator will not exceed the compensation commonly paid to law office administrators within the local legal community. Is it proper for the attorney to institute such a plan? Yes, because the amount paid to the administrator does not exceed the compensation commonly paid to law office administrators within the local legal community. Yes, because an employee of the firm may be compensated based on the profits of the firm. No, because the administrator's bonus is computed on the same basis as those of the lawyers in the firm. No, because the administrator's compensation is derived from the legal fees of the firm's lawyers.

Answer choice B is correct. The plan for part of the administrator's compensation is based upon the net profits of the firm and this arrangement is expressly permitted in MRPC 5.4(a) as long as the non-lawyer is an employee. The other aspects of the arrangement do not give the administrator an ownership interest in the firm or any right to control the legal work done in the law firm, thus avoiding any violation of the Rule. Answer choice A is incorrect; the plan is proper, but the reason why it is proper has nothing to do with the compensation paid to other law office administrators in the community. This answer erroneously refers to MRPC 1.5(a)regarding reasonable fees for lawyers, which lists as one factor fees paid to other similarly situated lawyers in the community. Answer choice C is incorrect; the Model Rules do not address the basis on which net profits may be divided and distributed to non-lawyer employees. That decision is for the firm. Answer choice D is incorrect because lawyers may not share legal fees with non-lawyers, but MRPC 5.4(a) contains an exception for the sharing of net profits with non-lawyer employees.

A former employee hires an attorney to represent her in a wrongful termination case against her former employer. The client and the attorney agree in writing that the attorney's obligations to the client exclude representation with regard to any appeal from a decision by the trial court. Among the instructions that the client gives to the attorney is that she will not accept an offer for less than a specific dollar amount. Subsequently, the lawyer for the former employer conveys a settlement offer to the attorney, which is below that amount. The lawyer for the former employer indicates that the former employer will not make any further settlement offers, but will proceed to trial. Based on past dealings with the former employer, the attorney has reason to believe that the former employer will adhere to that course of action. In the exercise of his professional judgment, the attorney also believes that his client's case has only a slight chance of success at trial. Consequently, the attorney, reasonably believing that the offer is the best resolution of the case for the client, accepts the offer without consulting her. Are the attorney's actions improper? Yes, because the attorney failed to convey the settlement offer to the client. Yes, because the attorney accepted the settlement offer. No, because the attorney believed that he was acting in the best interests of the client. No, because the attorney may reasonably limit the scope of his representation of the client.

Answer choice B is correct. The ultimate decision as to whether to accept an offer rests with the client. MRPC 1.2(a). The attorney improperly undertook to make this decision on behalf of the client. Answer choice A is incorrect because, while a lawyer must generally communicate all bona fide offers of settlement to the client, a lawyer is not required to do so when the client has given the lawyer specific instructions regarding future settlement offers. MRPC 1.4(a), cmt. 2. The client in this case gave the attorney instructions that permitted the attorney to reject the offer without first consulting with the client. Answer choice C is incorrect because a lawyer must abide by a client's decision with respect to a settlement offer even though the lawyer believes that the client's decision is unwise. Answer choice D is incorrect because, although a lawyer and a client may agree to reasonably limit the scope of the lawyer's representation, the attorney's actions in this case with respect to the settlement agreement were improper. MRPC 1.2(c).

The state bar association has offered a judge and her spouse free transportation and lodging to attend its institute on judicial reform. The judge is expected to deliver a banquet speech. Is it proper for the judge to accept this offer? Yes, unless the value of the transportation and lodging exceeds $500. Yes, because the activity is devoted to the improvement of law. No, if members of the bar association regularly appear in the judge's court. No, because the bar association is offering free transportation to the judge's spouse.

Answer choice B is correct. Under CJC Rule 3.13(C), a judge may generally accept an invitation to the judge and the judge's spouse, domestic partner, or guest to attend without charge an event associated with a bar-related function or other activity relating to the law. A judge may also accept reimbursement of necessary and reasonable expenses for travel, food, lodging, or other incidental expenses, if the expenses or charges are associated with the judge's participation in extrajudicial activities permitted by the Code of Judicial Conduct. CJC Rule 3.14(A). Reimbursement of expenses for necessary travel, food, lodging, or other incidental expenses must be limited to the actual costs reasonably incurred by the judge and, when appropriate to the occasion, by the judge's spouse, domestic partner, or guest. CJC Rule 3.14(B). The judge must publicly report acceptance of such invitations and the amount of any reimbursement for attending. Here, the judge would be attending a state bar event devoted to improvement of the law, so it is proper to accept the offer. Answer choice A is not correct, as the CJC imposes no $500 limit on the food and lodging reimbursement the judge may seek. The only limit is that the amount be necessary and reasonable. Answer choice C is not correct, as there is no prohibition on attending a bar association-related event and receiving reimbursement when members of the bar association regularly appear in the judge's court. Answer choice D is not correct, as a judge's spouse may be included under CJC Rules 3.13 and 3.14.

A judge in a state criminal trial court wishes to serve as guardian of her father, who has been declared incompetent. Accepting the responsibilities of the position would not interfere with the performance of the judge's official duties. Although the position in all likelihood would not involve contested litigation, it would be necessary for the judge to prepare and sign various pleadings, motions, and other papers and to appear in civil court on her father's behalf. Would it be proper for the judge to undertake this guardianship? Yes, unless the judge receives compensation for her services as guardian. Yes, because the position involves a close family member and will not interfere with the judge's performance of her judicial duties. No, because the position will require the judge to appear in court. No, because the position will require the judge to prepare and sign pleadings, motions, and other papers.

Answer choice B is correct. Under CJC Rule 3.8(A), a judge may accept appointment to serve as a guardian for a member of the judge's family, but only if such service will not interfere with the proper performance of judicial duties. A judge may not serve in such position if the judge, as a fiduciary, will likely be engaged in proceedings that would ordinarily come before the judge, or if the ward becomes involved in adversary proceedings in the court on which the judge serves, or a court under its appellate jurisdiction. CJC Rule 3.8(B). Here, because the position involves the judge's father, a close family member, and likely will not interfere with the judge's judicial duties, it would be proper for the judge to undertake the guardianship. Answer choice A is not correct, as a judge who properly becomes a guardian may receive compensation for such service. Answer choice C is not correct, as the fact that the judge will have to appear in court is not enough to prohibit acceptance of the guardianship. Only if the guardian will have to appear in proceedings that would ordinarily come before the judge or before a court on which the judge serves would it be improper. Here, the judge is on the criminal court, so this proceeding would not ordinarily come before the judge. Answer choice D is not correct, as it is permissible for a judge acting as a guardian for a close family member to prepare and sign pleadings, motions, and other papers.

An attorney in his capacity as part-time assistant county attorney represented the county in a criminal non-support proceeding against a husband. This proceeding concluded with an order directing the husband to pay or be jailed. The husband refused to pay. The attorney, pursuant to applicable rules, is permitted to maintain a private law practice. The wife has discovered some assets of the husband. The attorney now has accepted employment from the wife to maintain a civil action against the husband to recover out of those assets arrearages due to the wife under the wife's support decree. The attorney did not obtain consent from the county attorney or from the husband to represent the wife in the civil action. Is the attorney subject to discipline for accepting employment in the wife's civil action against the husband? Yes, because the attorney did not obtain the husband's consent to the representation. Yes, because the attorney had personal and substantial responsibility in the first proceeding. No, because the attorney's responsibility in his public employment has terminated. No, because the attorney is representing the wife's interests in both the criminal and the civil proceedings.

Answer choice B is correct. Under MRPC 1.11, the attorney, who had personally and substantially participated in the first proceeding, could not accept the employment without the county's informed, written consent. Answer choice A is not correct, as the husband's consent is not needed. It is the county's consent that controls under MRPC 1.11. Answer choice C is not correct, as MRPC 1.11 imposes a duty to obtain informed, written consent from the county in order to represent the wife, even though the attorney's responsibility in his public employment has terminated. Answer choice D is not correct, as it does not matter that the attorney is representing the same side as both a county attorney and a private attorney. The county's informed, written consent is needed to permit the representation.

A client retained an attorney to recover for a personal injury. In the retainer agreement signed by the client and the attorney, the client agreed to cooperate fully and pay the attorney a contingent fee computed as a percentage of the amount of recovery after expenses: 25 percent if settled before trial, 30 percent if settled before verdict, 35 percent after verdict, and 40 percent after appeal. The attorney's representation of the client in the matter extended over a three-year period during which the attorney advanced a large amount for litigation expenses. After trial, the client obtained a jury verdict for an amount larger than either the attorney or the client had anticipated. However, the defendant filed an appeal based on questions of evidence and the measure of damages. Meanwhile, the defendant made an offer of settlement for approximately the amount the attorney had originally projected as reasonable to expect. The client, who was hard pressed financially, directed the attorney to accept the offer and settle. The attorney refused, because she was confident that there was no reversible error in the trial and that the appeal was without merit. The attorney reasonably believed that the appeal was filed solely to gain negotiating advantage in settlement negotiations. Is the attorney subject to discipline? Yes, because the attorney's percentage under the fee contract increased after appeal. Yes, because the client directed the attorney to accept the settlement offer. No, because the decision whether to settle or defend an appeal is a tactical matter for the attorney to determine. No, because evaluation of the merits of an appeal requires the exercise of independent professional judgment.

Answer choice B is correct. Under MRPC 1.2(a), a lawyer must abide by a client's decision as to whether to settle a matter. Answer choice A is incorrect. Although the percentage increase gives the attorney an incentive to continue through the appeal, this incentive does not create an impermissible conflict of interest. Answer choice C is incorrect. Under MRPC 1.2(a), a lawyer must abide by a client's decision whether to settle a matter, regardless of the procedural status. Answer choice D is incorrect. Although it is true that evaluating the merits of an appeal requires the exercise of independent professional judgment, MRPC 1.2(a) confers upon the client the ultimate authority to determine the objectives of the representation and to make certain decisions, including a decision as to whether to settle a matter.

An attorney represented both the owner of an art gallery and a publisher. The gallery owner and the publisher each made a practice of paying the attorney's fees in cash. The attorney received separate cash payments from the gallery owner and the publisher on the same day. Each payment consisted of ten $100 bills, which the attorney immediately deposited in her bank account. One week later, the attorney was contacted by United States Treasury agents, who informed her that four of the bills had been identified as counterfeit. The agents did not accuse the attorney of knowingly passing the counterfeit money but asked her who had given her the bills. The attorney was subpoenaed to testify before a grand jury and was asked who could have given her the counterfeit money. Is it proper for the attorney to provide the grand jury with the names of the gallery owner and the publisher? Yes, because negotiation of a counterfeit bill is a criminal act. Yes, because under the circumstances neither client's identity is privileged. No, because counterfeiting is not a crime that involves an imminent threat of death or serious bodily harm. No, because the attorney has no way of knowing which of the two clients gave her the counterfeit bills.

Answer choice B is correct. Under MRPC 1.6(B)(6), the attorney may provide client confidences in response to a grand jury subpoena unless the information may be protected by the attorney-client privilege, in which case the attorney must assert the privilege unless the client waives it. In this case, the clients' names are confidential under MRPC 1.6 but are not protected by the attorney-client privilege. The privilege covers only confidential communications between a client and a lawyer. It does not cover clients' identities unless disclosing their identities will implicitly reveal their confidential communications, which is not the case here. Answer choice A is incorrect. It is irrelevant that negotiating a counterfeit bill is a crime. If the attorney's information in this case was privileged, the attorney could not disclose it under the crime-fraud exception because the attorney does not know which client paid with counterfeit bills or whether the client who did so was aware that the bills were counterfeit. Answer choice C is incorrect. The fact that the information is confidential under MRPC 1.6(a) and not subject to the exception under MRPC 1.6(b)(1) that allows the disclosure of confidences to prevent reasonably certain death or substantial bodily harm is irrelevant to these facts. Answer choice D is incorrect. Under MRPC 1.6(B)(6), the attorney may provide client confidences in response to a grand jury subpoena unless the information may be protected by the attorney-client privilege, in which case the attorney must assert the privilege unless the client waives it. In this case, the clients' names are not protected by the attorney-client privilege, so the attorney may reveal their names. If the attorney's information in this case were privileged (which it isn't), then the attorney could not disclose it under the crime-fraud exception because the attorney does not know which client paid with counterfeit bills or whether the client who did so was aware that the bills were counterfeit. But because the information is not privileged, the crime-fraud exception is inapplicable.

An attorney represented a client in negotiating a large real estate transaction. The buyer, who purchased the real estate from the client, has filed suit against both the client and the attorney, alleging fraud and violation of the state's unfair trade practices statute. The attorney had advised the client by letter against making the statements relied on by the buyer as the basis for the buyer's claim. The attorney and the client are each represented by separate counsel. In responding to a deposition under subpoena, the attorney wishes to reveal, to the extent the attorney reasonably believes necessary to defend herself, confidential information imparted to the attorney by the client that will be favorable to the attorney but damaging to the client. Is it proper for the attorney to reveal such information? Yes, unless the client objects to the disclosure. Yes, because the attorney may reveal such information to defend herself against a civil claim. No, unless criminal charges have also been brought against the attorney. No, because the disclosure will be detrimental to the client.

Answer choice B is correct. Under MRPC 1.6(b)(5), a lawyer may reveal confidential information to the extent necessary to establish a defense to a criminal charge or civil claim against the lawyer based on conduct in which the client was involved. Answer choice A is not correct, as MRPC 1.6(b)(5) allows the attorney to reveal the information even if the client objects. Answer choice C is not correct, as MRPC 1.6(b)(5) allows disclosure for both criminal and civil matters brought against a lawyer. Answer choice D is not correct, as the attorney may disclose the information to the extent necessary to defend herself against a civil claim even if the disclosure will be detrimental to the client.

An attorney has been hired to represent a client in a civil commitment proceeding initiated by the state. The client is now undergoing psychiatric evaluation to determine whether civil commitment should be ordered. The client told the attorney that she intends to commit suicide as soon as the tests are completed, and the attorney believes that the client will carry out this threat. Suicide and attempted suicide are crimes in the state. Is it proper for the attorney to disclose the client's intentions to the authorities? Yes, because the information concerns a future crime and is not protected by the attorney-client evidentiary privilege. Yes, because the information concerns a future crime that is likely to result in the client's imminent death. No, unless the attorney knows that the client has attempted suicide in the past. No, because disclosure would aid the state in its civil commitment case against the client.

Answer choice B is correct. Under MRPC 1.6(b), a lawyer may reveal otherwise protected information relating to the representation of a client to the extent the lawyer reasonably believes necessary to prevent reasonably certain death or substantial bodily harm. Here, the attorney reasonably believes that disclosing the client's intention to commit suicide will save the client's life. Answer choice A is not correct. The attorney-client privilege, which covers all confidential communications between a client and his lawyer, only applies to protect information in the context of evidence or testimony by the lawyer and is not relevant to the question of whether the lawyer has violated the ethical duty of confidentiality under MRPC 1.6, which protects all information learned in the course of representing a client from disclosure. Answer choice C is not correct, as MRPC 1.6(b) allows the information to be revealed to the extent the lawyer reasonably believes necessary to prevent reasonably certain death or substantial bodily harm. Answer choice D is not correct, as MRPC 1.6(b) allows disclosure under these circumstances, even if it would aid the state in its civil commitment case against the client.

An attorney represents ten plaintiffs who were injured when a train operated by a railroad was derailed. The railroad has offered the attorney a $500,000 lump-sum settlement for the ten plaintiffs. The attorney has determined a division of the $500,000 among the ten plaintiffs with the amount paid each plaintiff dependent on the nature and extent of that person's injuries. The attorney believes the division is fair to each plaintiff. The railroad will not settle any of the claims unless all are settled. The attorney has told each plaintiff the total amount the railroad is prepared to pay, the amount that the individual will receive, and the basis on which that amount was calculated. The attorney has not told any plaintiff the amount to be received by any other plaintiff. The attorney believes that if he reveals to each plaintiff the amount of each settlement, there is danger that some plaintiffs will think that they are not getting enough in relation to the amounts others will receive and the entire settlement will be upset. Each of the plaintiffs has agreed to his or her settlement. Is the attorney subject to discipline if he effects such a settlement? Yes, because the attorney is aiding the lawyer for the railroad in making a lump-sum settlement. Yes, because no individual plaintiff knows the amount to be received by any other plaintiff. No, if to disclose all settlements to each plaintiff might jeopardize the entire settlement. No, if the amount received by each plaintiff is fair and each plaintiff is satisfied.

Answer choice B is correct. Under MRPC 1.8(g), a lawyer representing co-parties may make an aggregate settlement only if all plaintiffs consent in writing after full consultation and disclosure by the lawyer, including disclosure of the nature and extent of all claims and pleas, and the participation of each party in the settlement. Here, the attorney has not fully disclosed the settlement to all of the co-parties and would therefore be subject to discipline. Answer choice A is not correct, as a lump-sum settlement would be permissible, but only if there has been full disclosure to, consultation with, and consent from, each affected client. Answer choice C is not correct, as MRPC 1.8(g) requires full disclosure of all settlements to each plaintiff. Answer choice D is not correct, as the fairness of the settlement and the satisfaction of the client are not what control with respect to whether the attorney is subject to discipline. MRPC 1.8(g) requires full disclosure and consultation by the attorney and consent from each plaintiff.

An attorney is representing a defendant on a charge of armed robbery. The defendant claims that the prosecution witness is mistaken in her identification. The defendant has produced a witness who will testify that the defendant was in another city 500 miles away when the robbery occurred. The attorney knows that the witness is lying, but the defendant insists that the witness be called on the defendant's behalf. Is the attorney subject to discipline if she calls the witness? Yes, unless, before calling the witness, the attorney informs the court of her belief. You Selected: Yes, because the attorney knows the witness will be testifying falsely. No, unless the attorney relies on the alibi defense in her argument before the jury. No, because the defendant has insisted that the witness be called on the defendant's behalf.

Answer choice B is correct. Under MRPC 3.3(a), a lawyer is prohibited from knowingly offering false evidence. If the attorney calls the witness, knowing the witness will testify falsely, that would subject the attorney to discipline under the Model Rules. Answer choice A is not correct, as informing the court would not prevent a violation of MRPC 3.3(a) for knowingly offering false evidence. Answer choice C is not correct, as it is irrelevant whether or not the attorney relies on the testimony in her closing argument. The attorney violates MRPC 3.3(a) by calling the witness, knowing that the witness will testify falsely. Answer choice D is not correct, as a client's insistence does not trump the Model Rules. If the attorney calls a witness she knows will testify falsely, she violates MRPC 3.3(a) and is subject to discipline.

An attorney is a long-time member of the state legislature and serves on the legislative budget committee that funds the local trial courts in the state. The attorney also maintains a part-time law practice, as is permitted in the state. An influential businessperson who regularly makes significant contributions to the attorney's political campaigns asked the attorney to help his uncle, who was involved in a bitter divorce. The attorney called the trial judge sitting on the uncle's case, a personal friend of the attorney. In discussing some upcoming votes of the budget committee with the judge, the attorney mentioned that the uncle was the type of solid citizen and influential person who could help garner support for the budget and thus ensure the economic health of the judicial system. Is the attorney subject to discipline? Yes, if the trial judge ruled in the uncle's favor. Yes, because the attorney used her public position to attempt to influence a tribunal in a pending matter. No, if the attorney called the trial judge in her capacity as a legislator and not as the uncle's lawyer. No, because members of the state legislature are permitted by law to engage in part-time legal practice.

Answer choice B is correct. Under MRPC 3.5(a), a lawyer must not seek to influence a judge by means prohibited by law. In addition, under MRPC 8.4, it is professional misconduct for a lawyer to engage in behavior prejudicial to the administration of justice. Here, the attorney was using her public position as a legislator with control over trial court funding to attempt to influence the judge on a pending matter, and she would therefore be subject to discipline. Answer choice A is not correct, as it does not matter whether the judge actually was influenced and did rule in the uncle's favor. What controls whether the attorney is subject to discipline is that the attorney was using her public position to attempt to influence the judge in a pending matter. Answer choice C is not correct, as MRPC 3.5 and 8.4 apply to a lawyer regardless of whether she is acting as an advocate for the person for whom the lawyer is attempting to influence the court. Answer choice D is not correct, because it is not relevant. The attorney is not subject to discipline for engaging in part-time practice while a legislator. The attorney is subject to discipline for improperly using her position to try to influence the court on a pending matter.

An attorney has been retained to defend an adult charged with a sex offense involving a minor. The attorney believes that, in order to win the case, she must keep parents of minor children off the jury. The attorney instructed her investigator as follows: "Visit the neighborhood of those prospective jurors on the panel with minor children. Ask the neighbors if they know of any kind of unusual sex activity of the prospective juror or any member of the family. This talk will get back to the prospective jurors, and they will think of excuses not to serve. But don't under any circumstances talk directly with any prospective juror or member of the family." Is the attorney subject to discipline for so instructing her investigator? Yes, unless the prospective jurors investigated are, in fact, selected to serve on the jury in the case. Yes, because the investigation is intended to harass prospective jurors and members of their families. No, if the matters inquired into might be relevant to a prospective juror's qualifications to serve in the case. No, because no prospective juror was directly contacted.

Answer choice B is correct. Under MRPC 3.5(a), a lawyer shall not seek to influence a prospective juror by means prohibited by law. Here, the attorney has instructed her investigator to engage in conduct that would constitute intimidation and harassment of prospective jurors, which would subject the attorney to discipline for violating the Model Rules. Answer choice A is not correct, as the Model Rules apply to conduct directed at both jurors and prospective jurors. Answer choice C is not correct, as the facts demonstrate that the conduct here is intended to harass prospective jurors and convince them not to serve. Answer choice D is not correct, as the Model Rules prohibit efforts to influence a prospective juror whether or not the prospective juror is directly contacted.

A prosecutor was assigned to try a criminal case against a defendant, who was charged with robbery of a convenience store. The defendant denied any involvement, contending he was home watching television with his mother on the night in question. At the trial, a customer at the convenience store testified that he had identified the defendant in a police lineup and provided other testimony connecting the defendant to the crime. In addition, the prosecutor entered into evidence a poor-quality videotape of the robbery as recorded by the store surveillance camera. The jury convicted the defendant of the crime charged. Unknown to the defendant's court-appointed lawyer, the customer had first identified another person in the police lineup and selected the defendant only after encouragement by the detective. The prosecutor was aware of these facts but did not notify the defendant's counsel, who made no pre-trial discovery request to obtain this information. Is the prosecutor subject to discipline? Yes, unless the jury could make its own identification of the defendant from the videotape. Yes, because this information tended to negate the defendant's guilt. No, because the defendant's counsel made no pre-trial discovery request to obtain this information. No, unless it is likely that the jury would have acquitted the defendant had it known that the customer first identified someone else.

Answer choice B is correct. Under MRPC 3.8(d), a prosecutor must make timely disclosure to the defense of all evidence or information known to the prosecutor that tends to negate the guilt of the accused or mitigates the offense. Here, the information tended to negate the defendant's guilt and should have been disclosed. Answer choice A is not correct. Regardless of whether the jury could make its own identification of the defendant from the videotape, the prosecutor was required to timely disclose the facts and will be subject to discipline for not doing so. Answer choice C is not correct, as it is irrelevant whether the defendant's counsel had made a pre-trial request for the purposes of discipline under the Model Rules. MRPC 3.8(d) required the disclosure and controls the issue. Answer choice D is not correct. Regardless of the potential outcome of the case, the prosecutor was obligated under MRPC 3.8(d) to disclose the information.

A law firm has 300 lawyers in 10 states. It has placed the supervision of all routine administrative and financial matters in the hands of a non-lawyer administrative assistant. The administrative assistant is paid a regular monthly salary and a year-end bonus of 1% of the law firm's net income from fees. Organizationally, the administrative assistant reports to the managing partner of the law firm. The partner deals with all issues related to the law firm's supervision of the practice of law. Is it proper for the partner to participate in the law firm's use of the administrative assistant's services in this fashion? Yes, unless the administrative assistant has access to client files. Yes, if the administrative assistant does not control the professional judgment of the lawyers in the firm. No, because the law firm is sharing legal fees with a non-lawyer. No, because the law firm is assisting a non-lawyer in the unauthorized practice of law.

Answer choice B is correct. Under MRPC 5.4(a), fees may be shared with law firm personnel who are not lawyers through a compensation plan, even if it is based on a profit-sharing arrangement. Under MRPC 5.4(d), a lawyer may not practice in a for-profit firm if a non-lawyer has the right to direct or control the professional judgment of a lawyer. Here, if the administrative assistant does not control the professional judgment of the lawyers in the firm, then the attorney may properly participate in the use of the administrative assistant's services. The fact that the administrative assistant has a compensation agreement based on a profit-sharing arrangement is not improper. Answer choice A is not correct. The administrative assistant would potentially need access to client files to do administrative work. Such access is not improper. Answer choice C is not correct, as MRPC 5.4(a) allows legal fees to be shared with non-lawyers through a compensation plan, even if it is based on a profit-sharing arrangement. Answer choice D is not correct, as the administrative assistant is not practicing law. The administrative assistant is doing administrative work for the firm.

Although licensed to practice law, an attorney does not practice law but works as an investment broker. The attorney could have elected inactive status as a member of the bar, but chose not to do so. Recently, in connection with a sale of worthless securities, the attorney made materially false representations to an investment customer. The customer sued the attorney for civil fraud, and a jury returned a verdict in the customer's favor. The attorney did not appeal. Is the attorney subject to discipline? Yes, because the attorney was pursuing a non-legal occupation while an active member of the bar. Yes, because the attorney's conduct was fraudulent. No, because the attorney was not convicted of a crime. No, unless the standard of proof in the state is the same in lawyer disciplinary cases and civil cases.

Answer choice B is correct. Under MRPC 8.4(c), it is professional misconduct for a lawyer to engage in conduct involving dishonesty, fraud, deceit, or misrepresentation. Here, the attorney's conduct was fraudulent, and she is thus subject to discipline. Answer choice A is not correct, as a lawyer is subject to discipline for any fraudulent conduct, even if not committed in the lawyer's role as a lawyer. Answer choice C is not correct, as the fraudulent conduct need not be criminal in nature to subject a lawyer to discipline under MRPC 8.4(c). Answer choice D is not correct, as the standard of proof need not be the same in lawyer disciplinary cases and civil cases.

An attorney was retained by a passenger on a bus who had been injured in a collision between the bus and a truck. The passenger paid the attorney a retainer of $1,000 and agreed further that the attorney should have a fee of 25% of any recovery before filing suit, 30% of any recovery after suit was filed but before judgment, and 35% of any recovery after trial and judgment. The attorney promptly called the lawyer for the bus company and told him she was representing the passenger and would like to talk about a settlement. The attorney made an appointment to talk to the lawyer for the bus company but did not keep the appointment. The attorney continued to put off talking to the lawyer for the bus company. Meanwhile, the passenger became concerned because she had heard nothing from the attorney. The passenger called the attorney's office but was told she was not in and would not call back. The passenger was told not to worry because the attorney would look after his interests. After ten months had passed, the passenger went to a different attorney for advice. The different attorney advised the passenger that the statute of limitations would run in one week and, with the passenger's consent, immediately filed suit for him. The first attorney, upon the passenger's demand, refunded the $1,000 the passenger had paid. Is the first attorney subject to discipline? Yes, unless her time was completely occupied with work for other clients. Yes, because she neglected the representation of the passenger. No, because the passenger's suit was filed before the statute of limitations ran. No, because she returned the $1,000 retainer to the passenger.

Answer choice B is correct. Under the Model Rules, a lawyer must act with reasonable promptness in representing a client. MRPC 1.3. The first attorney failed to take any action on the passenger's representation for ten months, which violates the Model Rules. Answer choice A is not correct, as a lawyer cannot justify a lack of diligence based on the inability to balance other work. A lawyer must control her workload to ensure that all matters are handled competently. MRPC 1.3, cmt 2. Answer choice C is not correct, as it is irrelevant to the violation of MRPC 1.3 that the statute of limitations was not missed. The first attorney's extreme lack of promptness is a violation of the Model Rules. Answer choice D is not correct, as the refund of the retainer is irrelevant to the violation of MRPC 1.3. A lawyer is subject to discipline for a lack of promptness or diligence regardless of whether she refunds a client's payment.

Looking to earn extra money, an attorney with personal injury defense expertise signed up to participate as a legal services provider for a pre-paid legal plan administered by a local insurance company. Under the terms of the contract that the insurance company and the attorney signed, the company agreed to telephone all persons who were known to need personal injury defense representation as a result of claims filed with the company's accident insurance division and provide the attorney's name and contact information and information about joining the pre-paid plan to them. Is the attorney subject to discipline for participating in the plan? Yes, because participation in a pre-paid legal services plan is not permissible. Yes, because the company would be engaging in improper telephone contact to solicit memberships in the plan from persons known to need specific services covered by the plan. No, because an attorney is permitted to participate in a pre-paid legal services plan. No, because there would be no solicitation of persons known to need specific legal services by the attorney.

Answer choice B is correct. While a lawyer may participate in a pre-paid or group legal services plan under certain circumstances, the plan may only solicit from persons who are not known to need legal services in a particular matter covered by the plan. Here, the insurance company would be engaging in telephone solicitation on behalf of the attorney with specific persons known to need the kind of legal services the attorney would provide. Answer choice A is incorrect, as pre-paid legal services plans are permissible, so long as certain specific requirements are met. Answer choice C is incorrect, as even though such plans are allowed, this plan does not meet the requirement that the plan not solicit from persons known to need legal services in a particular matter covered by the plan. Answer choice D is incorrect, as the fact that the attorney is not personally doing the solicitation does not immunize the attorney from the insurance company's impermissible action. An attorney who participates in such a plan is required to reasonably assure herself that the plan is in compliance with the Model Rules of Professional Conduct.

An attorney represented a defendant in a personal injury case. Opposing counsel approached the attorney and offered to drop the case in exchange for a $5,000 settlement from the defendant. The attorney, eager to finish the case before trial and honestly believing that the client would be happy with the settlement, agreed on the spot. Opposing counsel asked the attorney if he needed to check with his client first, and the attorney told her that the amount was fine with his client. The two attorneys entered into the settlement agreement. Later that day, the attorney informed his client of the settlement agreement. The client, angry with the attorney's behavior, categorically rejected the settlement. The attorney informed the client that he was now bound by the settlement agreement and would have to pay. Is the lawyer correct in telling the client that he is bound by the settlement agreement and has to pay the plaintiff? Yes, because the decision to settle is a procedural tactic and is ultimately left to the attorney. Yes, because the lawyer acted with apparent authority. No, because the decision to accept a settlement offer is a decision ultimately left to the client. No, because the client did not ratify the settlement agreement.

Answer choice B is correct. While a lawyer who agrees to a settlement without the client's consent and authorization is subject to discipline, the opposing party may still enforce the settlement agreement if the lawyer acted with apparent authority. Answer choice A is incorrect because the ultimate decision as to whether to accept settlement offers rests with the client. Answer choice C is incorrect because, while a decision as to whether to accept a settlement offer is the client's decision, a lawyer acting with apparent authority may bind his client to a settlement offer. Answer choice D is incorrect because, while a client who ratifies a settlement agreement is bound by the agreement, the client will be bound anyway since the attorney acted with apparent authority.

An attorney is a candidate in a contested election for judicial office. Her opponent, a judge, is the incumbent and has occupied the bench for many years. The director of the state commission on judicial conduct, upon inquiry by the attorney, erroneously told the attorney that the judge had been reprimanded by the commission for misconduct in office. The attorney, who had confidence in the director, believed him. In fact, the judge had not been reprimanded by the commission; the commission had conducted hearings on the judge's alleged misconduct in office and, by a three-to-two vote, declined to reprimand the judge. Decisions of the commission, including reprimands, are not confidential. Is the attorney subject to discipline for publicly stating that the judge had been reprimanded for misconduct? Yes, because the official records of the commission would have disclosed the truth. Yes, because the judge had not been reprimanded. No, because the attorney reasonably relied on the director's information. No, because the judge was a candidate in a contested election.

Answer choice C is correct. A candidate for judicial office must not knowingly, or with reckless disregard for the truth, make any false or misleading statement. CJC Rule 4.1. Here, it was reasonable for the attorney to rely on the statements of the director of the state commission on judicial conduct, and therefore the attorney did not knowingly or with reckless disregard for the truth make a false or misleading statement. Answer choice A is not correct, as it was reasonable for the attorney to rely on the statements of the commission's director, and the attorney was not required by the Code of Judicial Conduct to check the official records of the commission. Answer choice B is not correct, as the fact that the judge was not actually reprimanded is irrelevant. The attorney merely needed to reasonably believe that the judge had been reprimanded in order to avoid being subject to discipline. Answer choice D is not correct, as the fact that the judge was a candidate is irrelevant. The attorney, as a candidate, must not knowingly or with reckless disregard for the truth make any false or misleading statement.

A partner in a law firm learns that the firm has recently agreed to represent a new client in a personal injury suit for a contingent fee. The partner realizes that she has a conflict with the client from her work at her previous law firm, and she tells the firm's managing partner about the conflict. The firm then properly screens the partner from the case and provides all required notice regarding the conflict to the client. The firm is eventually successful in its representation of the client and receives a significant fee as a result of the representation. In allocating the annual firm income to each partner under the firm's partnership agreement, the firm includes the fee earned in the personal injury suit. Is it proper for the partner to share in the fee from the personal injury suit? Yes, because the partner is obtaining the fee as part of her annual partnership share. Yes, because the firm properly screened the attorney from the case and provided all required notice regarding the conflict to the client. No, because the partner is not permitted to receive compensation directly related to a matter from which she was disqualified. No, but only because the fee was a contingent fee.

Answer choice C is correct. A disqualified lawyer may not receive compensation that is directly related to the matter from which the lawyer is disqualified. MRPC 1.10, cmt. 8. Answer choice A is incorrect because even though a disqualified lawyer may receive a partnership share established by prior independent agreement, she is not permitted to receive compensation that is directly related to the matter from which she has been disqualified. Here, it is clear that the partner would be receiving compensation directly related to the matter from which she was disqualified. Answer choice B is incorrect because the screening and notice requirements to avoid imputed disqualification do not create an exception to the requirement that a disqualified lawyer not receive compensation that is directly related to the matter from which she was disqualified. Answer choice D is incorrect, as the fact that the fee was contingent is irrelevant. What matters is that a disqualified lawyer is not permitted to receive compensation that is directly related to the matter from which she has been disqualified.

A manufacturer sued a company for its breach of warranty regarding machine components it furnished. A judge, who presided at the nonjury trial, sent her law clerk to the manufacturer's plant to observe the machine that was malfunctioning due to the allegedly defective parts. The clerk returned and told the judge that the machine was indeed malfunctioning and that the engineer, an employee of the manufacturer, had explained to the clerk how the parts delivered by the company caused the malfunction. There was testimony at the trial that supported what the clerk learned on his visit. The judge rendered a judgment for the manufacturer. Was the judge's conduct proper? Yes, because the judge's judgment was supported by evidence at the trial. Yes, because the judge has the right to gather facts concerning the trial. No, because the judge has engaged in ex parte contacts that might influence the outcome of litigation. No, unless the engineer was a witness at the trial and subject to cross-examination by the company.

Answer choice C is correct. A judge is required not to initiate, permit, or consider ex parte communications or other communications made to the judge outside the presence of the parties or their lawyers, with regard to a pending or impending matter. A judge is required to make reasonable efforts, including providing appropriate supervision, to ensure that this rule is not violated by court staff, court officials, and others subject to the judge's direction and control. CJC Rule 2.9(D). Here, the statement made to the clerk constituted an ex parte communication and was improper for the judge to consider. Answer choice A is not correct because regardless of whether the judgment was supported by evidence at trial, the communication and consideration of the communication was improper under the Code of Judicial Conduct. Answer choice B is not correct, as a judge is not permitted to gather facts through improper ex parte communication. Answer choice D is not correct, because even if the engineer was a witness subject to cross-examination by the company, the Code of Judicial Conduct only permits the judge to consider evidence from the engineer that is provided in court.

Three days ago, an attorney agreed on a contingency fee basis to represent, at an administrative hearing, a claimant who has been denied Social Security disability benefits. Neither the attorney nor the claimant has filed a request for a hearing with the Social Security Administration. Due solely to a backlog in hearing similar challenges, once a request for a hearing is filed, the hearing itself will not take place for at least another six months. Two days ago, the attorney learned that she had been appointed as counsel by a federal court in a class action lawsuit. The class action lawsuit has the potential for being very lucrative for the attorney, in contrast to the remuneration the attorney may receive for representation of the claimant. Not wanting to be distracted by the disability benefits matter, the attorney discussed the transfer of representation of the claimant with another lawyer in her firm who is as well qualified to handle the matter as the attorney is. The other lawyer was amenable to the transfer. However, the client refused to agree to the transfer or to the attorney's withdrawal from representation of her claim. Despite the client's refusal, the attorney has informed the client that the attorney nevertheless is withdrawing. The attorney has returned all papers related to the matter that she had received from the client. Is the attorney's withdrawal from representation of the claimant proper? No, because the attorney breached her duty of confidence to the claimant by discussing the matter with another lawyer in her firm. No, because the attorney may not withdraw from representation of the claimant without the claimant's permission when a substantial reason for the withdrawal is the potential economic benefit to the attorney. Yes, because the withdrawal will not materially harm the claimant. Yes, but only because the attorney found another qualified lawyer to represent the claimant.

Answer choice C is correct. A lawyer generally may withdraw for any reason when the client's interests will not be materially harmed, even when the client objects to the withdrawal. MRPC 1.16(b)(1). Here, the hearing will not take place for another six months, and there is no indication that the claimant's interests would be materially harmed as a result of the withdrawal from representation. As this matter was not in court, no permission of the court would be required for the attorney to withdraw. Answer choice A is incorrect because, unless the client objects, a lawyer may discuss the client's matter with another attorney in the lawyer's firm without breaching the ethical duty of confidentiality. MRPC 1.6, cmt. 5. Answer choice B is incorrect because, as noted with regard to answer choice C, when a client will not be materially harmed, the lawyer may withdraw for any reason. Answer choice D is incorrect because, while finding another qualified attorney to represent a client may constitute evidence that the client will not be materially harmed by the lawyer's withdrawal, a lawyer is not required to find substitute counsel in order to withdraw.

An attorney represented a criminal defendant in a murder trial. While interviewing the defendant in preparation for the trial, the attorney became suspicious that his client was, in fact, guilty of the murder. Throughout the attorney's extensive questioning of the client, however, the client maintained his innocence. The client insisted that the attorney allow him to testify at trial. The attorney believed, but was not certain, that his client would commit perjury if allowed to testify, but the client swore he would tell the truth. The attorney put the client on the stand and questioned him thoroughly about his involvement in the murder. The client's testimony was consistent with what he had told the attorney previously. The jury found the client not guilty. Immediately after the court entered its final judgment, the client thanked the attorney for letting him testify. He boasted to the attorney that everything he had said while on the stand was a lie and that he was actually guilty. The attorney was distraught over the defendant's behavior, but he took no action. The jurisdiction has no laws that supersede the Model Rules of Professional Conduct. Is the attorney subject to discipline for his failure to inform the court of his client's perjury? Yes, because the attorney had a reasonable belief that the client's testimony was false before the statement was made. Yes, because the attorney had an affirmative duty to take reasonable remedial measures after learning of the client's perjury. No, because the proceedings had concluded at the time that the attorney learned of his client's perjury. No, because the client was a criminal defendant.

Answer choice C is correct. A lawyer has an affirmative duty to take reasonable remedial measures if the lawyer knows that his client has offered false material evidence or has engaged in criminal or fraudulent conduct related to an adjudicative proceeding. MRPC 3.3(a)(3), (b). The duty to rectify false evidence, however, continues only until the end of the proceedings, which means that a final judgment has been affirmed on appeal or the time for appeal has expired. Here, the attorney learned that his client had committed perjury in testifying at his own trial, but the information came after the jury had rendered a not guilty verdict, from which the prosecution cannot take an appeal. Thus, the lawyer had no duty to take remedial measures. Answer choice B is therefore incorrect. Answer choice A is incorrect because, while it is true that a lawyer's reasonable belief as to the falsity of material evidence generally is a basis on which the lawyer may refuse to offer such evidence (but this does not apply with respect to the testimony of a client who is a criminal defendant), such a belief is not a basis on which a lawyer may be disciplined for failure to take remedial measures after the lawyer learns of the falsity of testimony given by his client or that his client has engaged in criminal or fraudulent conduct related to an adjudicative proceeding. MRPC 3.3(a)(3), (b). Answer choice D is incorrect because, while a lawyer may not refuse to offer the testimony of a criminal defendant that the lawyer reasonably believes but does not know is false, a lawyer is required to take remedial measures with regard to material evidence that the lawyer knows is false unless state law provides otherwise. MRPC 3.3(a)(3), cmt. 7. As the discussion above regarding answer choice C notes, however, there is no duty when the proceedings have concluded at the time the lawyer learns of the false material evidence or criminal or fraudulent conduct.

Attorney, recently admitted to practice, opened an office near a residential neighborhood and published the following advertisement in the local newspaper. COUPON Get Acquainted With Your Neighborhood Lawyer A. Attorney Suite 2 - 1100 Magnolia Avenue Sunshine City, State 01000 Telephone: (555) 555-5555 In order to acquaint you with our services, we are offering a one-hour consultation to review your estate plans, including your wills, trusts, and similar documents, all at the nominal cost of $25 to anyone presenting this coupon. Call now for an appointment. Is Attorney subject to discipline? Yes, because Attorney is soliciting business from persons with whom Attorney had no prior relationship. Yes, because Attorney requires the use of a coupon. No, if Attorney provides the services described for the fee stated. No, unless Attorney is seeking business from persons who are already represented by a lawyer.

Answer choice C is correct. A lawyer is generally permitted to advertise her services so long as the communications regarding such services are not false or misleading in violation of MRPC 7.1 or in violation of the rules against solicitation of clients under MRPC 7.3. MRPC 7.2(a). Answer choice A is not correct, as this is advertising, not solicitation under the Model Rules. "Solicitation" refers to narrower, targeted communications directed at a small group identified as requiring a specific service (such as letters sent to accident victims, in-person solicitation of employment, or real-time electronic contact). "Advertising" refers to widely distributed, public statements about the services available from a lawyer or law firm (such as phone book listings, or newspaper or television ads). Answer choice B is not correct, as there is nothing in the Model Rules prohibiting the use of a coupon as part of a lawyer's advertisement. Answer choice D is not correct, as the Model Rules permit advertisements, so long as they are not false or misleading or in violation of the rules against solicitation.

The law firm of Alpha and Beta has a radio commercial which states: "Do you have a legal problem? Are you being sued? Consult Alpha and Beta, licensed attorneys at law. Initial conference charge is $25 for one hour. Act now and protect your interests. Call at 1234 Main Street; telephone area code (101) 123-4567." Are Alpha and Beta subject to discipline for the commercial? Yes, because the qualifications of the lawyers are not stated. Yes, because the radio broadcast may encourage litigation. You Selected: No, if all the statements in the radio broadcast are true. No, unless the radio broadcast is heard outside the state in which they are licensed.

Answer choice C is correct. A lawyer is generally permitted to advertise his services through public media, including a radio commercial, so long as the communications regarding such services are not false or misleading or in violation of the rules against solicitation of clients. MRPC 7.2(a). Any advertisement must include the name and office address of at least one lawyer or law firm responsible for its content. MRPC 7.2(c). Here, the answer choice indicates that all of the statements in the commercial are true. The name and office address of the lawyers are included. Thus Alpha and Beta would not be subject to discipline. Answer choice A is incorrect, as there is no requirement under the rules that a lawyer list his credentials in an advertisement. Answer choice B is incorrect, as the Model Rules do permit advertising, so long as the advertisement is not false or misleading or in violation of the rules against solicitation. Answer choice D is incorrect, as the Model Rules do not prohibit lawyers from using a radio commercial that might be heard outside of the state in which the lawyers are licensed.

A homeowner called his cousin, an attorney, to help the homeowner secure a zoning variance in order to build a garage on his property. The attorney informed the homeowner that, because the attorney's practice was normally limited to the area of criminal law, he had no experience in obtaining a zoning variance. In addition, because the homeowner was a relative, the attorney informed him that he would waive any fee. The homeowner readily agreed to the attorney's representation. At the initial hearing on the zoning variance, the variance was denied due to the attorney's failure to follow proper legal procedures, which an experienced zoning attorney would have known. At a subsequent hearing, however, the homeowner was granted a variance as a result of the arguments made by the attorney. As a consequence of the delay, the homeowner's cost of building the garage was increased. Is the attorney subject to discipline? No, because the attorney did not charge the homeowner a fee. No, because the client was aware of the attorney's lack of experience regarding zoning law. Yes, because the zoning variance was initially denied as a result of the attorney's lack of the requisite knowledge of the zoning variance process. Yes, because the homeowner suffered harm due to the delay.

Answer choice C is correct. A lawyer is obligated to provide competent representation to a client, which includes possessing the legal knowledge reasonably necessary for the representation. Here, the attorney failed to possess such knowledge. MRPC 1.1. Answer choice A is incorrect because the payment of a fee is not necessary to create a lawyer-client relationship with the duty of competent representation. Answer choice B is incorrect because a client's awareness of a lawyer's lack of competency at the time of consenting to the lawyer's representation does not obviate a lawyer's obligation to competently provide such representation. Answer choice D is incorrect because it is not necessary that the client suffer any financial loss or that the attorney be the cause of any harm to the client for the attorney to be subject to disciplinary (as opposed to legal malpractice) action. MRPC 1.16, cmt. 1.

A plaintiff, who is not a lawyer, is representing himself in small claims court in an action to recover his security deposit from his former landlord. The plaintiff told an attorney, a close friend who lived near him, about this case, but did not ask the attorney for any advice. The attorney said, "I'll give you some free advice. It would help your case if the new tenants would testify that the apartment was in good shape when they moved in, and, contrary to the allegation of your former landlord, it was not, in fact, repainted for them." The plaintiff followed the attorney's advice and won his case. Is the attorney subject to discipline for assisting the plaintiff in preparing for his court appearance? Yes, because the attorney assisted the plaintiff in the practice of law. Yes, because the attorney offered unsolicited, in-person legal advice. No, because the plaintiff was representing himself in the proceedings. No, because the attorney was not compensated for his advice.

Answer choice C is correct. A lawyer may counsel a non-lawyer who wishes to proceed pro se in a legal matter. MRPC 5.5, cmt 3. Thus, the attorney would not be subject to discipline for assisting the plaintiff. Answer choice A is not correct, as a lawyer may assist a non-lawyer who is representing himself in a legal matter. Answer choice B is not correct, as there was a close personal relationship between the plaintiff and the attorney, and the attorney was not speaking to the plaintiff out of a desire for pecuniary gain. Answer choice D is not correct, as the attorney would not be subject to discipline for assisting the plaintiff even if he was compensated for his advice.

An attorney adeptly represented the buyer in a real estate transaction. As part of the transaction, the attorney was responsible for obtaining title insurance for the property. The title insurance company required the buyer to sign an indemnification agreement before the company would issue the insurance. The company sent the agreement to the attorney, who forwarded it to the buyer, but the buyer neglected to return the agreement to the attorney. More than one year passed before the attorney contacted the buyer and title insurance on the property was obtained. No one has asserted a claim to the property. Were the attorney's actions with respect to the real estate transaction proper? Yes, because the client did not suffer harm. Yes, because the attorney did not act intentionally. No, because the attorney failed to act with reasonable diligence and promptness in representing the buyer. No, because the attorney failed to provide competent representation to the buyer.

Answer choice C is correct. A lawyer must act with reasonable diligence and promptness in representing a client. MRPC 1.3. While the failure to promptly sign the agreement rests with the buyer, the attorney had an obligation to follow up on the matter and ascertain whether the buyer's failure was due to a mere oversight that could be corrected by the attorney's follow-up. Answer choice A is incorrect because harm to a client is not a requirement for establishing a violation of the Model Rules of Professional Conduct. Answer choice B is incorrect because a lawyer may violate a Model Rule inadvertently, particularly the rule requiring a lawyer to act with reasonable diligence. Answer choice D is incorrect because, while a lawyer must provide competent representation to a client (MRPC 1.1), the attorney here acted with competence with regard to the overall transaction, "adeptly" representing the buyer, and specifically with regard to the important document in question, one necessary to obtain title insurance, by sending it to the buyer. As noted with regard to answer choice C, the attorney's failure was one of diligence and promptness rather than one of competence.

An attorney represented a buyer in the purchase of a restaurant. The contract, which was drafted by the seller's lawyer, specified that a portion of the purchase price ($25,000) was to be held in escrow by the attorney until certain conditions were satisfied, including the transfer of a liquor license. The attorney received a check from the buyer for $25,000 of the purchase price. The attorney placed the check in the trust account that he maintained for his clients' funds. Pursuant to the terms of the contract, the attorney timely filed the documents necessary to transfer the liquor license from the seller to the buyer. Upon learning from the proper local authorities that the transfer of the license had been approved, the attorney transferred $25,000 from the trust account to the seller. The seller did not ask and the attorney did not provide the seller with an accounting of the funds. Were the attorney's actions proper? Yes, because the attorney performed his duties in a timely fashion. Yes, because the attorney, as the buyer's legal representative, did not owe a duty to the seller. No, because the attorney failed to place the $25,000 in a separate escrow account. No, because the attorney did not provide the seller with an accounting of the retained portion of the purchase price.

Answer choice C is correct. A lawyer must hold property of a third party as well as property of a client that is in the lawyer's possession in connection with a representation with the care required of a professional fiduciary. In addition to separation of such property from his own property, an attorney is required to set up a separate trust account when warranted. MRPC 1.15, cmt. 1. Here, the attorney acquired possession of the $25,000 in connection with his representation of the buyer in the purchase of the restaurant. Since the contract required that $25,000 be placed in escrow to be held for the seller if the conditions were met or for the buyer if they were not, placement of this amount in the attorney's clients' trust account potentially and unnecessarily subjected it to claims by other clients of the attorney. Answer choice A is incorrect because, although a lawyer is required to perform his duties promptly, a lawyer must also take the actions necessary to safeguard property that belongs to a client or third party. Answer choice B is incorrect because a lawyer has a duty not only to a client, but also to third parties with respect to property held by the lawyer in connection with a representation. Answer choice D is incorrect because a lawyer is required to provide a third party or client with an accounting of the property that the lawyer has held for the third party or client only if the third party or client requests it. MRPC 1.15(d). Moreover, as a practical matter, since the seller received the full amount of the purchase that the attorney retained, an accounting would serve no purpose.

A lawyer provides both estate planning services and financial planning services to his clients. One client asks the lawyer to draft her will, but first she wants him to provide her with some financial planning advice to optimize her estate. If the lawyer provides such services to the client, would the lawyer be subject to discipline for providing both legal and non-legal services to the same client? Yes, unless the lawyer provides the financial planning services and the estate planning services in two separate meetings. Yes, unless the lawyer owns the law-related services entity with others, and not just by himself. No, unless the lawyer fails to inform the client that the financial planning services are not legal services, and are not protected by the lawyer-client privilege. No, not under any circumstances.

Answer choice C is correct. A lawyer must take reasonable measures to ensure that a person obtaining non-legal services knows that the services are not legal services and that the protections of the lawyer-client privilege do not apply. MRPC 5.7(a)(2). Answer choice A is incorrect, because as long as the client understands the distinction, as noted above with regard to answer choice C, nothing in the Model Rules indicates that the services may not be provided together. Answer choice B is incorrect because the Model Rules apply to the lawyer regardless of whether the lawyer owns the entity providing law-related services alone or with others. Answer choice D is incorrect because the assurances described above with regard to answer choice C are necessary.

An employee contacted a newly licensed attorney about pursuing an appeal with respect to the denial of a workers' compensation claim. The attorney explained to the employee her lack of experience in handling such a matter, promised nevertheless to provide the employee with competent representation, and offered to represent the employee for a lower fee than other lawyers that the employee had approached about handling the matter. The engagement letter prepared by the attorney and signed by the employee included a clause that required the arbitration of any legal malpractice claims. The attorney discussed both the scope and effect of this clause with the client. The attorney diligently prepared for the appeal and competently represented the employee at the hearing on the workers' compensation claim. However, the denial of the employee's workers' compensation claim was upheld. Is the attorney subject to discipline for her handling of this matter? Yes, because the attorney lacked the requisite competence to handle the matter when she agreed to undertake it. Yes, because the engagement letter contained an agreement to arbitrate any malpractice claims. No, because the attorney achieved the necessary level of competence prior to the appeal. No, because, as a licensed lawyer, the attorney was not subject to any restrictions on cases she could take.

Answer choice C is correct. A lawyer who lacks the requisite competence to handle a matter may nevertheless agree to represent a client when the requisite level of competence can be achieved by reasonable preparation or association. MRPC 1.1. The attorney did lack such competence with regard to workers' compensation claims, but through preparation she was able to acquire the requisite competence in a timely fashion. Answer choice A is incorrect because, as noted, an attorney need not possess the necessary competence at the time that she undertakes the matter. Answer choice B is incorrect because, while a lawyer may not enter into an agreement that prospectively limits the lawyer's liability for malpractice unless the client is independently represented, an agreement to arbitrate future malpractice disputes is permissible so long as the attorney discusses both the scope and the effect of an arbitration clause with the client. MRPC 1.8(h). Answer choice D is incorrect because any lawyer is subject to the requirement that the lawyer provide competent representation to a client. Being licensed to practice law in a state does not ensure that the lawyer is competent with respect to a particular legal matter.

An attorney acting as a mediator in a child custody dispute properly explained his role to each of the parents involved in the dispute, how that role differed from that of a lawyer for a litigant, and that the lawyer-client privilege did not apply. Through the attorney's mediation efforts, the parents settled their dispute and came to a custody agreement that was approved by the court. Subsequently, the mother, wanting to modify the terms of the custody agreement, employed a partner at the attorney's law firm to handle the matter. The partner immediately gave written notice to the father and to the court of her employment by the mother, but she did not seek the approval of either. The attorney who served as a mediator was timely screened from participation in the matter and did not receive any part of the fee from this representation. Is the partner's representation of the mother proper? No, because the father did not give his written consent for the partner to represent the mother. No, because another member of the partner's firm had served as mediator with regard to the same matter. Yes, because the partner timely notified the father and the court in writing of her representation of the mother, and the firm properly dealt with the attorney who had acted as a mediator. Yes, because the attorney who acted as a mediator had properly informed the parents about the difference between a mediator and a lawyer for a litigant.

Answer choice C is correct. A lawyer who personally and substantially serves as a mediator with regard to a matter is disqualified from serving as an attorney for anyone in connection with the matter unless all parties give their written, informed consent. This disqualification is imputed to other members of the lawyer's law firm. However, another member of the law firm may represent one of the parties to the mediation in the matter if the other parties to the mediation and any appropriate tribunal are given timely written notification of the representation, and the attorney who served as a mediator is timely screened from participation in the matter and does not receive any part of the fee from this representation. Because these conditions were met here, the partner's representation of the mother is proper. Answer choice A is incorrect because, although the father would have been required to give his written, informed consent if the attorney who acted as a mediator had sought to represent the mother, here the partner representing the mother is another attorney in the attorney-mediator's firm. Answer choice B is incorrect because the fact that a member of a law firm served as a mediator with regard to a matter does not disqualify another member of the law firm from representing a party to the mediation with regard to that matter, provided the proper steps enumerated in answer choice C are taken. Answer choice D is incorrect because, although the attorney who acted as a mediator is required to properly inform the parties to the mediation about the role of the mediator, that action alone does not protect another member of the attorney's firm from disqualification.

A local school board is considering the purchase of new equipment. At a public meeting during which the purchase is discussed, an attorney speaks in favor of the purchase. The attorney truthfully states that she is the parent of a student who attends one of the schools that would receive the new equipment. The attorney, acting in accordance with her client's instructions, does not reveal that she has been employed by a manufacturer of the equipment to speak on behalf of the purchase at the meeting. Is the attorney subject to discipline with regard to addressing the board? No, because the attorney has been instructed by her client not to reveal the representation. No, because the proceeding was not an adversarial one. Yes, because the attorney failed to reveal that she was representing the client. Yes, because the attorney's action, while not improper, has the appearance of impropriety.

Answer choice C is correct. A lawyer who represents a client before a legislative body or administrative agency in a non-adjudicative proceeding must disclose that the appearance is in a representative capacity. MRPC 3.9. Answer choice A is incorrect because, while a lawyer may be compelled by the duty of confidentiality from revealing the identity of the client that the lawyer represents, a lawyer is required to reveal that the lawyer's appearance is in a representative capacity. Answer choice B is incorrect because, as noted with respect to answer choice C, the duty of disclosure specifically applies to a lawyer representing a client in a non-adjudicative proceeding. Answer choice D is incorrect because the attorney's action is improper under the Model Rules of Professional Conduct, as noted above with regard to answer choice C. Moreover, unlike a judge, an attorney does not have a duty to avoid the appearance of impropriety.

A law firm represented residents who were opposed to the rezoning of a nearby undeveloped parcel of land from rural to retail. A newly hired associate was assigned to work on the case. The associate had previously worked for another law firm, where she had participated in the representation of the landowner in securing the environmental permits necessary for the development of the parcel. The associate, after discussing the matter with the supervisory partner at her new firm, accepted the assignment. Even though the associate believed that accepting the assignment violated the conflict-of-interest rules, the partner concluded that there was not a conflict of interest and that, consequently, the consent of the landowner was not required. When the landowner learned that the associate was working with the residents, he notified the appropriate disciplinary authority. Subsequently, the partner was disciplined for violation of the conflict-of-interest rules. Is the associate also subject to discipline? No, because the associate was acting at the direction of her supervisor. No, because the associate acted in accordance with the partner's resolution of a question of professional duty. Yes, because the associate's participation in the representation of the residents clearly constituted a conflict of interest. Yes, because the associate took actions that she believed violated a professional duty.

Answer choice C is correct. A subordinate attorney is bound by the Model Rules of Professional Conduct even if acting under the direction of a supervising attorney. A subordinate attorney is not in violation of the Model Rules if she acts in accordance with a supervising attorney's reasonable resolution of an arguable question of professional duty. However, in this instance, the associate's participation in the representation of the residents constituted a clear conflict of interest with her former client, the landowner. The associate had formerly represented the landowner in a substantially related matter, and the residents' interests are materially adverse to the landowner's interest. Without the landowner's informed consent, confirmed in writing, there is a clear conflict of interest in the associate's participation in the representation of the residents. Consequently, the associate cannot rely on the partner's resolution of this issue to shield herself from discipline for violation of the conflict-of-interest rules. Answer choice A is incorrect, because a subordinate attorney may be subject to discipline even if she is acting at the direction of a supervising attorney when it is clear that a course of action to be undertaken by the subordinate attorney violates the Model Rules of Professional Conduct. Answer choice B is incorrect because a subordinate attorney is protected from discipline for violation of a Model Rule when there is an arguable question as to the subordinate attorney's professional duty and when the supervising attorney's resolution of the question is reasonable. Here, neither of those conditions is met. Answer choice D is incorrect because a subordinate attorney is not subject to discipline for violation of the Model Rules if she acts in accordance with the supervising attorney's reasonable resolution of an arguable question of professional duty, even if the subordinate attorney believes that such action violates a professional duty. To determine whether the resolution is reasonable, the associate must undertake her own analysis of the applicable law.

A toy manufacturer was sued by the parent of a child injured by one of its products. As the manufacturer's attorney was preparing to respond to a discovery request from the plaintiff, the attorney found a document that was very damaging to his client's case. Prior to complying with the discovery request and turning over the document, the attorney called his opposing counsel and offered to settle the case. The attorney stated that although he believed his client was very likely to win a summary judgment motion, they would settle the case for a modest amount to save the costs of litigation. In fact, the attorney believed his client had no chance of winning a summary judgment motion and was also likely to lose at trial based on the document he had found. The opposing counsel declined the attorney's offer. The attorney turned over the document, and the case proceeded to trial, where judgment was awarded to the plaintiff. Were the attorney's statements to the opposing counsel proper? No, because the attorney did not believe in the truthfulness of the statement. No, because the attorney owed a duty of candor to the opposing counsel. Yes, because the attorney's statement did not constitute a statement of fact. Yes, because the opposing counsel did not accept the attorney's offer.

Answer choice C is correct. An attorney is not permitted to make a false statement of material fact. Statements that constitute "puffing" (i.e., opinions or judgments not made as a representation of fact) are permissible as part of a negotiation. Subjective statements, such as statements about the relative merits of a case, are not statements of material fact. In this case, the attorney's statements regarding the merits of the case would constitute puffing and are thus permissible. Answer choice A is incorrect because the attorney did not make a false statement of material fact; this statement constituted permissible puffing. Answer choice B is incorrect because although an attorney is prohibited from making false statements of material fact, an attorney does not owe the duty of candor to opposing counsel that she owes to the court. Answer choice D is incorrect because although the opposing counsel did not accept the attorney's offer and hence the plaintiff was not harmed by the attorney's statements, the absence of harm does not prevent an attorney's action from being a violation of the Model Rules of Professional Conduct.

An attorney represented a corporation in a suit brought by one of its competitors for misappropriation of trade secrets. Shortly before trial but after the discovery deadlines had passed, the corporation's chief engineer told the attorney that he had found a memorandum he had written to himself the previous year. The statements in the memorandum aligned with the chief engineer's version of the events in question and directly contradicted the competitor's claims. The attorney was skeptical of the document's authenticity because it had not been produced during the discovery process, and he believed that it may have been fabricated for trial purposes. Although the chief engineer assured the attorney that the document was authentic, the attorney continued to have doubts. Nonetheless, the attorney offered the document into evidence during his examination of the chief engineer at trial. Was the attorney's action in offering the document into evidence proper? No, because the attorney reasonably believed that the evidence was false. No, because doubts about authenticity should be resolved in favor of protecting the integrity of the judicial process. Yes, because the attorney did not know the evidence was false. Yes, because the prohibition on offering false evidence does not apply to a civil action.

Answer choice C is correct. An attorney is prohibited from knowingly offering false evidence and may refuse to offer evidence that the attorney reasonably believes is false. The term "knows" means actual knowledge of the fact in question. In this case, although the attorney believed that the evidence might have been false, he did not know if it was, and thus it was proper for the attorney to offer it. He may have refused to offer the evidence but was not acting improperly if he did choose to offer it. Answer choice A is incorrect because although an attorney may generally refuse to offer evidence that the attorney believes is false, he is not required to do so. Answer choice B is incorrect because doubts about truthfulness should be resolved in favor of the client, although an attorney may not ignore an obvious falsehood. Answer choice D is incorrect because the prohibition on offering false evidence applies to civil as well as criminal actions.

An attorney took his grandmother out to dinner. Over dessert, the attorney asked his grandmother if she had updated her will recently. The grandmother stated that she had not, and the attorney offered to draft a new will for her. They met the following day and drafted a will in which she bequeathed to the attorney a rare book collection, which he had always admired, as well as a sizeable monetary gift. She left the remainder of her estate to her son and a charitable organization. Were the attorney's actions in offering to prepare and preparing a will proper? No, because an attorney may not prepare a will that gives a substantial gift to the attorney. No, because an attorney may not solicit an individual to prepare a will that gives a substantial gift to the attorney. Yes, because an attorney may solicit or prepare a will that gives a substantial gift to the attorney if the client is a family member. Yes, because an attorney may prepare a will that gives a substantial gift to the attorney as long as the attorney does not exert undue influence over the client.

Answer choice C is correct. An attorney may not generally solicit or prepare an instrument (such as a will, trust agreement, or deed) that gives a substantial gift to the attorney or a person related to the attorney. There is an exception if the attorney is related to the client, however. In this case, the attorney was permitted to solicit and prepare the will for his grandmother. Answer choice A is incorrect because an attorney may prepare a will that gives him a substantial gift if he is related to the person executing the will. Answer choice B is incorrect because an attorney may solicit a will for a related person. Answer choice D is incorrect because unless the attorney is related to the client, the attorney may not solicit or prepare a will that gives the attorney a substantial gift, even in the absence of undue influence.

A new client has asked his attorney to write a letter recommending his nephew for admission to the bar. The client has told his attorney that he has no direct contact with his nephew, but that his sister (the nephew's mother) has assured him that the nephew is industrious and honest. Which of the following would be proper for the attorney? Write the letter on the basis of the client's assurance. Write the letter on the basis of the client's assurance if the attorney has no unfavorable information about the nephew. Make an independent investigation and write the letter only if the attorney is thereafter satisfied that the nephew is qualified. All of the above would be proper.

Answer choice C is correct. Answer choices A and B are incorrect, as it would be improper for the attorney to recommend an applicant to the bar without independently assuring herself that the applicant is fit to practice law. The attorney may not properly rely on the client's assurance or upon her own lack of unfavorable information about the nephew. Answer choice C is correct, as it provides for an independent investigation without reliance on the client's assurances.

A company's president telephoned his city's best-known employment attorney and asked her to represent the company in a dispute that had just arisen with the company's chief financial officer. The attorney, who had never previously represented the company, agreed. At the president's insistence, she immediately commenced the representation. A few days later, during a meeting with the president, the attorney first revealed the amount of her customary hourly fee and then explained that the company would also be responsible for reimbursing her expenses. The president responded that her fee was higher than he had expected but that he would be happy for the company to pay it, given her excellent work to date. Although the attorney intended to follow up with a confirming letter, she never did so. For several more months, she assisted the company in resolving its employment dispute. Afterward, she sent the company a bill accurately reflecting her hourly fee and expenses, which were reasonable. Is the attorney subject to discipline? Yes, because she did not disclose the basis of her fee before commencing the representation. Yes, because she did not confirm her fee agreement in writing. No, because she disclosed the basis of her fee within a reasonable time after commencing the representation. No, because she was not required to advise the client of her customary hourly fee, unless requested to do so.

Answer choice C is correct. As required by the Model Rules, the lawyer disclosed the basis of the fee within a reasonable time after commencing the representation. Answer choice A is incorrect because MRPC 1.5(b) says that the lawyer's basis for the fee must be disclosed before commencing or within a reasonable time after commencing a representation. Answer choice B is incorrect, as MRPC 1.5(b) states that a fee arrangement shall be communicated to the client, preferably in writing, but written confirmation is not mandatory. Answer choice D is incorrect; because she was taking on a new representation in this case, the lawyer was required by the Model Rules to inform the client about the basis for her fee.

After both parties had completed the presentation of evidence and arguments, the judge took under advisement a case tried without a jury. The case involved a difficult fact issue of causation and a difficult issue of law. After the case was under advisement for several weeks, the plaintiff's attorney heard rumors that the judge was having difficulty determining the issue of factual causation and was uncertain about the applicable law. Immediately after hearing these rumors, the attorney telephoned the judge, told her of the rumors he had heard, and asked the judge if she would like to reopen the case for additional evidence and briefing from both parties. Thereafter the judge reopened the case for further testimony and requested supplementary briefs from both parties. Was it proper for the attorney to communicate with the judge? Yes, because both parties were given full opportunity to present their views on the issues in the case. Yes, because the attorney did not make any suggestion as to how the judge should decide the matter. No, because the attorney communicated with the judge on a pending matter without advising opposing counsel. No, because the attorney caused the judge to reopen a case that had been taken under advisement.

Answer choice C is correct. Ex parte communications with the judge are prohibited by MRPC 3.5. The attorney should have approached the judge jointly with opposing counsel or through a formal submission to the court with a copy to opposing counsel. Answer choices A and B are incorrect; the communication was improper. MRPC 3.5 prohibits a lawyer from communicating ex parte with a judge unless authorized to do so by law or court order, neither of which occurred here. Answer choice D is incorrect; although the content of the attorney's communication was unobjectionable, MRPC 3.5 prohibits a lawyer from communicating ex parte with a judge unless authorized to do so by law or court order, neither of which occurred here. In this case, the lawyer participated in an unauthorized ex parte communication concerning the subject of the litigation.

A client hired an attorney to file a lawsuit against the client's former employer for wrongful discharge. The attorney filed the suit in federal district court based upon three grounds. It turned out that a unanimous U.S. Supreme Court decision had recently eliminated the third ground as a theory available to plaintiffs in wrongful discharge cases. The opposing attorney filed a motion alleging that the complaint was based upon a theory (the third ground) that is no longer supported by existing law and cited the new decision. Within ten days after the filing of the complaint, the attorney withdrew the third ground and continued with the litigation. Is the attorney subject to litigation sanction? Yes, unless the attorney discussed the adverse legal authority with the client before filing the complaint. Yes, because the attorney should have cited the U.S. Supreme Court decision in the complaint. No, because the attorney withdrew the third ground within ten days after filing the complaint. No, unless the attorney knew or should have known of the recent decision when the complaint was filed.

Answer choice C is correct. F.R.C.P. 11 imposes sanctions for filing frivolous pleadings and taking frivolous positions. Here, while the attorney initially filed a pleading in federal district court that contained a frivolous ground, by withdrawing the ground there is no violation of F.R.C.P. 11, and the attorney would not be subject to litigation sanction. Answer choice A is not correct, as there is no requirement that the attorney specifically discuss the adverse legal authority with the client before filing the complaint in order to avoid sanctions under F.R.C.P. 11. Answer choice B is not correct, as the attorney withdrew the ground on which the Supreme Court decision was relevant. Answer choice D is not correct, as even if the attorney knew or should have known of the decision, by withdrawing the ground, the attorney rectified the error and can avoid litigation sanction.

An attorney brought an action to recover fees for services rendered to a client in a complex civil litigation matter involving the ownership of a business. The client filed a counterclaim against the attorney alleging negligent malpractice in the prior litigation. The client contended that the attorney failed, in the prior trial, to introduce into evidence documents obtained by the client as to the value of the business. Although the documents were relevant and favorable to the client, the documents were not produced in response to a court discovery order due to having been misfiled by the attorney's otherwise competent, and properly supervised, paralegal. As a sanction, the court prohibited the documents from being introduced into evidence, even though they were subsequently located. In the prior action, the court found, based on other evidence, that the value of the business exceeded the amount supported by the documents, but the court reduced the amount that the client expected to receive for other reasons. Is the attorney subject to civil liability for the inability to introduce the documents into evidence? Yes, because the documents were relevant and favorable to the client. Yes, because the attorney is responsible for acts and omissions committed by the paralegal within the ordinary course of business. No, because the client did not suffer a harm. No, but only because the attorney did not commit a violation of the Model Rules of Professional Conduct.

Answer choice C is correct. For a lawyer to be subject to civil liability for malpractice based on negligence, the client must suffer harm; damages are not presumed. Since the value of the business as determined by the court was greater than the amount supported by the documents, the client did not actually suffer harm from the inability to introduce the documents. Answer choice A is incorrect because, although the fact that the documents were relevant and favorable to the client suggests that the failure to introduce such documents harmed the client, the client was not adversely affected by the prohibition on the introduction of the documents into evidence, as noted with respect to answer choice C. Answer choice B is incorrect because, even though the attorney, as a principal, can be liable for the actions of an employee, such as a paralegal, that are taken in the ordinary course of business, and even though the employee's carelessness with regard to the documents in question constituted a breach of the attorney's duty to handle the client's representation competently, the client did not suffer a harm from this breach. Answer choice D is incorrect because, even though the attorney did not violate the Model Rules (because the attorney had properly supervised a competent paralegal), the attorney nevertheless could have been subject to civil liability had the client suffered harm as a consequence of the employee's action.

An attorney is skilled in trying personal injury cases. The attorney accepted the representation of a plaintiff in a personal injury case on a contingent fee basis. While preparing the case for trial, the attorney realized that the direct examination and cross-examination of the medical experts would involve medical issues with which the attorney was not familiar and, as a consequence, the attorney might not be able to represent the plaintiff competently. Without informing the plaintiff, the attorney consulted a second attorney, who is both a lawyer and a medical doctor and who is a recognized specialist in the care and treatment of injuries of the type sustained by the plaintiff. The two attorneys agreed that the second attorney would participate in the trial to the limited extent of conducting the direct examination and cross-examination of the medical experts and that the original attorney would divide the fee in proportion to the services performed and the responsibility assumed by each. Was the arrangement between the two attorneys proper? Yes, because the fee to be paid by the plaintiff was not increased by reason of the second attorney's association. Yes, because the fee would be divided in proportion to the services performed and the responsibility assumed by each. No, because the plaintiff was not advised of the association of the second attorney. No, unless, upon conclusion of the matter, the original attorney provides the plaintiff with a written statement setting forth the method of determining both the fee and the division of the fee with the second attorney.

Answer choice C is correct. For the two attorneys to split a fee, MRPC 1.5 requires that: (i) the fee be in proportion to the services rendered by each lawyer or that joint responsibility is assumed for the representation, (ii) the client agree in writing to the fee-splitting arrangement, and (iii) the total fee charged be reasonable such that a client is not charged more just because an additional lawyer is working on the case. Here, there is no indication that the plaintiff agreed in writing to the fee-splitting arrangement, so it would be improper. Answer choices A and B are not correct, as each represents only one of the three requirements for a fee-splitting arrangement. Since the plaintiff did not agree in writing, the arrangement is still improper. Answer choice D is not correct, as the plaintiff's consent was required prior to the two attorneys entering into their association on the plaintiff's case.

An attorney engaged in litigating a negligence action against a toy manufacturer on behalf of a client who had been injured playing with the toy sought the help of a private investigator to assist in developing the case. Knowing that the attorney was working on a contingent fee basis, the private investigator proposed that the attorney enter into a reasonable compensation agreement under which one-third of any fee that the attorney obtained as a result of the litigation would be paid to the private investigator. In the event that no fee was obtained by the attorney, the private investigator would not be paid. Is it proper for the attorney to enter into such an agreement with the private investigator? Yes, because the contingent fee is a permissible compensation arrangement. Yes, because the fee arrangement is reasonable. No, because the attorney is not permitted to share his fee with a non-lawyer. No, because the fee arrangement is not reasonable.

Answer choice C is correct. Generally, a lawyer is prohibited from sharing legal fees with a non-lawyer. The proposed agreement would require the attorney to share his fee for the litigation with the private investigator. Answer choice A is incorrect, as, even though a contingent fee may be proper for a lawyer, the lawyer is not permitted to share his fee with a non-lawyer. Answer choice B is incorrect, as it is irrelevant whether the private investigator's fee arrangement with the lawyer is reasonable. A lawyer is not permitted to share his fee with a non-lawyer. Answer choice D is incorrect for the same reason.

As part of a routine conflict check conducted upon the hiring of new personnel, a law firm learned that an attorney who was joining the firm as a lateral associate had previously represented an insurance company in an ongoing reinsurance contract dispute with another insurance company, the firm's second biggest client. The firm immediately notified the attorney that she would be subject to the firm's detailed existing screening procedures with regard to the case and would not be allowed to participate in the matter or share any part of the fee. The firm and the attorney then sent a letter to the attorney's former client, describing the attorney's prior representation and stating that the client's material confidential information had not been disclosed or used and that the attorney and the firm were in compliance with all required confidentiality rules. The letter indicated that the attorney was being screened from the firm's representation of its client, described the screening procedures used, indicated that judicial review might be available to the former client, and provided that the firm would be willing to respond promptly to any written questions or objections by the client about the screening procedures. In response, the attorney's former client sent a letter to the firm objecting to the firm's continued representation of its client on the basis of the attorney's conflict of interest. Is the firm's continued representation of its client proper? No, because the attorney's former client objected to the representation. No, because the firm's current client was not notified about the attorney's conflict. Yes, because the firm screened the attorney from its representation of its client and notified the attorney's former client of what was being done. Yes, because the attorney is merely an associate and not a partner with the new firm.

Answer choice C is correct. If a lawyer switches firms and has a conflict of interest because of the lawyer's previous representation of a client or the prior law firm's previous representation of a client, the new law firm will not have the disqualified lawyer's conflict imputed to it if the disqualified lawyer is timely screened from any participation in the matter, the lawyer is apportioned no part of the fee, and written notice is promptly given to any affected former client so that the client can determine if there has been compliance with the Model Rules. MRPC 1.10(a)(2). If the requirements of the Rule are met, the former client's consent is not required. Thus, answer choice A is incorrect. Answer choice B is incorrect because only the former client, not the current client, must be notified. Answer choice D is incorrect because it is irrelevant whether the attorney was an associate or a partner. The rule limiting imputed disqualification applies to an associate as well as a partner.

An attorney specializing in tax law was contacted by a man who requested some advice with regard to filing his federal income tax return for the year. The attorney invited the man to come to her office for an initial interview. During the interview, the man indicated to the attorney that he suspected that certain money he had received from an uncle was stolen in a recent bank robbery, and he wanted to know whether to report the money as income on his tax return. The bank robbery had been in the news, but no suspects had yet been arrested. The attorney advised the man to report the receipt of the money to the police and then told the man that she would not be taking him on as a client. When the man left her office, the attorney called the police to inform them that she had information about the bank robbery. She gave them the man's name and told the police what had been said at the meeting. Were the attorney's actions in reporting the information to the police proper? Yes, because the man was never actually the attorney's client. Yes, because as an officer of the court, the attorney had a duty to report to the police information concerning unsolved cases. No, because the attorney-client privilege applied to protect the information provided by the man to the attorney. No, because the attorney should have waited until the man's current tax issues were resolved before informing the police of her involvement.

Answer choice C is correct. If an individual communicates information to a lawyer for the purpose of seeking legal advice, that communication is privileged, even if the lawyer does not give legal advice or agree to the representation. As such, it is irrelevant that the man was not the attorney's client, and thus, answer choice A is incorrect. Answer choice B is incorrect because an attorney is never under an obligation to disclose attorney-client communications, although she is permitted to do so if the communication relates to the future commission of what the client knew or should have known was a fraud. Answer choice D is incorrect because the attorney-client privilege continues indefinitely.

An accountant contacted an attorney to determine whether several matters in which the attorney was assisting a client should be considered liabilities on the client's books. The client authorized the evaluation. The attorney had not yet had the time to consider all of the client's records that would be relevant to the accountant's inquiry, and the client refused to produce the records in a timely manner, telling the attorney that there was information in the records he wanted the attorney to see only after he provided his evaluation to the accountant. When the attorney provided the report of the evaluation to the accountant, he made no mention of the unexamined records. Were the attorney's actions proper? Yes, because the attorney provided no false statements of material fact or law in providing the evaluation. Yes, because informing the accountant of the unexamined records would have violated the attorney's duty of confidentiality. No, because the attorney should have described the unexamined records in the attorney's report to the accountant. No, because the attorney should have refused to perform the evaluation until he had examined all relevant records.

Answer choice C is correct. If the terms of an attorney's evaluation are limited in any relevant way, such limitations that are material to the evaluation must be described in the lawyer's report. Answer choice A is incorrect because, while it is true that a lawyer is not permitted to knowingly make a false statement of material fact or law in providing an evaluation, the lawyer must still inform the third party of any material limitations. Answer choice B is incorrect because, while information relating to an evaluation is confidential, an attorney may disclose information as authorized in connection with the report of the evaluation. Nothing in the facts here indicates that the client ordered the attorney not to disclose the existence of the records. Answer choice D is incorrect because an evaluation may be limited in scope for some reason, so long as any material limitations are disclosed in the report.

An attorney represented a police officer in an action brought against the officer for violation of the plaintiff's constitutional rights at the time of the plaintiff's arrest. In the attorney's closing arguments, the attorney honestly stated, "As you, members of the jury, do, I believe in the Constitution of the United States. I also believe that my client did not violate any of the plaintiff's rights arising from that Constitution. I know my client personally, and I believe that he is a good police officer who would never violate a person's constitutional rights." Is the attorney subject to discipline for this statement? No, because the attorney honestly stated his beliefs. No, because the statements were made in the attorney's closing arguments. Yes, because the statements were based on the attorney's personal beliefs. Yes, because an attorney may not comment on the culpability of a civil litigant.

Answer choice C is correct. In addressing the jury, a lawyer is prohibited from stating a personal opinion as to the culpability of a civil litigant. Answer choice A is incorrect because the prohibition on a lawyer's statement to a jury of a personal opinion as to the culpability of a civil litigant is not subject to a good-faith or honestly held exception. Answer choice B is incorrect because the prohibition on a lawyer's statement to a jury of a personal opinion as to the culpability of a civil litigant applies when the lawyer is addressing a jury, such as during closing arguments. Answer choice D is incorrect because a lawyer may comment on the culpability of a civil litigant if such comment is based on the evidence in the case rather than made in the form of the lawyer's personal opinion as to such culpability.

A judge and an attorney were formerly law partners and during their partnership acquired several parcels of real property as co-tenants. After the judge was elected to the trial court in the county, she remained a co-tenant with the attorney, but left the management of the properties to the attorney. The judge's term of office will expire soon and she is opposed for re-election by two members of the bar. The attorney, who has not discussed the matter with the judge, intends to make a substantial contribution to her campaign for re-election. The judge is one of fifteen judges sitting as trial court judges in the county. Is the attorney subject to discipline if he contributes $10,000 to the judge's re-election campaign? Yes, if the attorney frequently represents clients in cases tried in the trial court of the county. Yes, because the judge and the attorney have not discussed the matter of a campaign contribution. No, if the contribution is made to a campaign committee organized to support the judge's re-election. No, because the attorney and the judge have a long-standing personal and business relationship.

Answer choice C is correct. Lawyers have a right to participate fully in the political process, which includes making and soliciting political contributions to candidates for judicial and other public office. MRPC 7.6, cmt 1. The Code of Judicial Conduct permits reasonable campaign contributions to a judge's election campaign, so long as they are made through a campaign committee. Answer choice A is not correct, as representation of clients in courts in the county where a judge holds office would not prohibit a campaign contribution. Answer choice B is not correct, as there is no requirement that a candidate for judicial office and a lawyer discuss a campaign contribution in order for it to be made. Answer choice D is not correct, as the long-standing personal and business relationship is not relevant and would not prevent a proper contribution under the ethics rules.

An attorney, who had worked for a corporation for 10 years, was in charge of supervising several new attorneys. The attorney assigned several small contract matters to each of the new attorneys as a training exercise, and told the new attorneys to come to him if they had any questions about their assignments. All of the contracts went out without the attorney having reviewed any of them. One of the new attorneys negligently failed to include an important choice-of-law clause in one of the contracts, and when the corporation was sued on that contract, the lack of a choice-of-law provision meant that the plaintiff was able to bring an action in a very unfavorable jurisdiction, costing the corporation millions of dollars. Which of the following statements is true? The attorney is not subject to discipline because he did not ratify the new attorney's conduct. The attorney is not subject to discipline under the Model Rules for failing to supervise the new attorneys because he is not a "partner." The attorney is subject to discipline for failing to make reasonable efforts to ensure that the supervised attorneys' conduct conformed to the Model Rules. The attorney is subject to discipline because supervising attorneys are subject to discipline for all misconduct committed by the attorneys they supervise, so long as the misconduct was not willful.

Answer choice C is correct. Lawyers with direct supervisory authority are required to make reasonable efforts to ensure that the supervised persons comply with the Model Rules. So, the attorney here would be subject to discipline. MRPC 5.1(b). Answer choice A is incorrect because ratification is not the only way a supervisor can be held liable for her associates' actions. Answer choice B is incorrect because a lawyer who has supervisory authority in a corporate enterprise is considered a "partner" for the purposes of MRPC 5.1. Answer choice D is incorrect because a supervising attorney is not liable for all negligent behavior of subordinates, if, for example, the subordinate is adequately supervised.

An attorney represents a client in commercial litigation that is scheduled to go to trial in two months. Over the past several weeks, the client has disagreed with almost every tactical decision that the attorney has made. Frustrated, the attorney finally said to the client that if she didn't like the way he was handling the lawsuit, perhaps she should get another lawyer. The client was upset at the suggestion and accused the attorney of trying to get out of the case. Reasonably believing that he could no longer work effectively with the client, the attorney sought the client's permission to withdraw from the representation, and the client reluctantly agreed. After giving the client sufficient notice to obtain replacement counsel, the attorney requested the court's permission to withdraw from the litigation, but the court denied the request. May the attorney withdraw from the representation? Yes, because the client agreed, and the attorney gave the client sufficient notice to obtain replacement counsel. Yes, because the client had made it unreasonably difficult for the attorney to carry out the representation effectively. No, because the court denied the attorney's request to withdraw. No, because the attorney's withdrawal would cause material prejudice to the client, and the client's agreement was not voluntary.

Answer choice C is correct. MRPC 1.16(c) states that the lawyer must continue the case if ordered to do so by the court. Answer choice A is incorrect because in all court proceedings, the judge must consent to the withdrawal and substitution of counsel. Sufficient notice and client consent are not enough for withdrawal if the court does not agree. MRPC 1.16(c) states that the lawyer must continue the case if ordered to do so by the court. Answer choice B is incorrect; the client's conduct is a reason for the lawyer to move for withdrawal, but the lawyer must get the court's permission before withdrawal would be proper. MRPC 1.16(c) states that the lawyer must continue the case if ordered to do so by the court. Answer choice D is incorrect because there is no evidence that the client would be prejudiced by the withdrawal. Withdrawal is improper because the court refused to grant permission to withdraw.

An attorney represented a husband and wife in the purchase of a business financed by contributions from their respective separate funds. The business was jointly operated by the husband and wife after acquisition. After several years, a dispute arose over the management of the business. The husband and wife sought the attorney's advice, and the matter was settled on the basis of an agreement drawn by the attorney and signed by the husband and wife. Later, the wife asked the attorney to represent her in litigation against the husband based on the claim that the husband was guilty of fraud and misrepresentation in the negotiations for the prior settlement agreement. Is it proper for the attorney to represent the wife in this matter? Yes, if all information relevant to the litigation was received by the attorney in the presence of both the husband and wife. Yes, if there is reason to believe the husband misled both wife and the attorney at the time of the prior agreement. No, because the attorney had previously acted for both parties in reaching the agreement now in dispute. No, unless the husband is now represented by independent counsel.

Answer choice C is correct. MRPC 1.9(a) provides that a lawyer who has formerly represented a client in a matter shall not thereafter represent another person in the same or a substantially related matter in which that person's interests are materially adverse to the interests of the former client unless the former client gives informed consent, confirmed in writing. Unless the husband, a former client of the attorney, gives informed consent in writing, it would not be proper for the attorney to represent the wife in the matter. Answer choices A, B, and D are not correct, as the fact that the attorney represented both clients in a substantially related matter controls.

An attorney represents a famous politician in an action against a newspaper for libel. The case has attracted much publicity, and a jury trial has been demanded. After one of the pretrial hearings, as the attorney left the courthouse, news reporters interviewed her. In responding to the questions, the attorney truthfully stated that the judge upheld their right to subpoena the reporter involved and question the reporter on her mental impressions when she prepared the article. The attorney named the reporter involved. Is the attorney subject to discipline for making this statement? Yes, because the attorney identified a prospective witness in the case. Yes, because prospective jurors might learn of the attorney's remarks. No, because the statement relates to a matter of public record. No, because the trial has not commenced.

Answer choice C is correct. MRPC 3.6(b)(2) specifically permits statements concerning information contained in a public record. Answer choice A is incorrect because although MRPC 3.6 prohibits making public communications that have a substantial likelihood of materially prejudicing an adjudicative proceeding, there is no indication that the attorney's communication in this case would be materially prejudicial. Answer choice B is incorrect, as there is no indication that this communication would be materially prejudicial, and MRPC 3.6(b)(2) specifically permits statements concerning information contained in a public record. Answer choice D is incorrect. The prohibition in the rule against making prejudicial extrajudicial statements is not limited to statements regarding trials that have commenced.

Attorney Alpha currently represents a building contractor who is the plaintiff in a suit to recover for breach of a contract to build a house. The builder also has pending before the zoning commission a petition to rezone property she owns. The builder is represented by Attorney Beta in the zoning matter. A neighbor owning property adjoining that of the builder has asked Alpha to represent him in opposing the builder's petition for rezoning. The neighbor knows that Alpha represents the builder in the contract action. Is it proper for Alpha to represent the neighbor in the zoning matter? Yes, if there is no common issue of law or fact between the two matters. Yes, because one matter is a judicial proceeding and the other is an administrative proceeding. No, because Alpha is currently representing the builder in the contract action. No, if there is a possibility that both matters will be appealed to the same court.

Answer choice C is correct. Representation of the neighbor would constitute a concurrent conflict of interest with regard to the builder pursuant to MRPC 1.7(a), in that the representation of the neighbor will be directly adverse to the builder. Such a conflict can be waived, with informed consent, if the lawyer reasonably believes that he will be able to provide competent and diligent representation to each affected client and the representation is not prohibited by law and does not involve the assertion of a claim by one client against another client represented by the lawyer in the same litigation. The facts here do not establish that any of that has occurred. Thus, it would not be proper for Alpha to represent the neighbor in the zoning matter. Answer choice A is not correct, as there is a conflict under MRPC 1.7(a) because the representation of the neighbor is directly adverse to the builder. Answer choice B is not correct, as the nature of the proceedings does not matter. What controls is that the representation of the neighbor will be directly adverse to a current client. Answer choice D is not correct, as the place of appeal is irrelevant to whether or not representation of the neighbor is proper. The representation will be directly adverse to the interests of a current client, and without a proper waiver, it is prohibited under MRPC 1.7(a).

An attorney currently represents the owner of a local hardware store in contract negotiations to extend the lease of the building in which the store is located. An authorized representative of a large corporation that owns and operates a nationwide chain of hardware stores has contacted the attorney about representing the store in negotiations with the neighboring property owner for an easement to gain additional access to the public roads. Without obtaining the written, informed consent of both the owner of the local store and the corporation, can the attorney agree to represent the corporation if the attorney reasonably believes that he will be able to provide competent and diligent representation to each client? No, because the conflict-of-interest rules apply to transactional representation as well as to representation in adversarial proceedings before a tribunal. No, because the written informed consent of each client is required when a concurrent conflict of interest exists. Yes, because simultaneous representation in unrelated matters of clients whose interests are only economically adverse does not typically constitute a conflict of interest. Yes, because the attorney reasonably believes that the attorney will be able to provide competent and diligent representation to each client.

Answer choice C is correct. Simultaneous representation in unrelated matters of clients whose interests are only economically adverse, such as representation of competing economic enterprises in unrelated litigation, does not ordinarily constitute a conflict of interest, and, as such, does not require the consent of the respective clients. MRPC 1.7, cmt. 6. While the attorney's existing client and possible new client are economic competitors, the legal matters of each are unrelated. Consequently, there is not a concurrent conflict of interest and the attorney need not seek the consent of each in order to represent the other. Answer choice A is incorrect because, although the conflict-of-interest rules do apply to transactional representation (MRPC 1.7, cmt. 7), these rules do not apply, as noted above, to simultaneous representation in unrelated matters of clients whose interests are only economically adverse. Answer choice B is incorrect because, while the written informed consent of each client is generally required when a concurrent conflict of interest exists, such consent is not required when such a conflict does not exist. Answer choice D is incorrect because, although it is true that if there is a conflict of interest, a lawyer may not represent each affected client unless the lawyer reasonably believes that the lawyer will be able to provide competent and diligent representation to each client, there is no conflict of interest between the attorney's representation of the owner of the hardware store and the attorney's representation of the corporation.

An attorney was employed by an insurance company as a lawyer solely to handle fire insurance claims. While so employed, she investigated a fire loss claim of a claimant against the insurance company. The attorney is now in private practice. The claim has not been settled and the claimant consults the attorney and asks the attorney either to represent him or refer him to another lawyer for suit on the claim. Which of the following would be proper for the attorney to do? Represent the claimant. Refer the claimant to an associate in her law firm, provided the attorney does not share in any fee. Give the claimant a list of lawyers who the attorney knows are competent and specialize in such claims. None of the above is proper.

Answer choice C is correct. The attorney was formerly employed on this matter by the insurance company and may not represent the claimant unless the insurance company gives informed consent, confirmed in writing. MRPC 1.9(a). Even though there is a conflict and the attorney cannot represent the claimant, there is no rule prohibiting the attorney from providing the claimant with a list of qualified lawyers who may be able to represent him. Because there is no indication that the insurance company has given informed consent, answer choice A is incorrect. Answer choice B is incorrect because the firm of a lawyer who is personally disqualified under MRPC 1.9(a) from representing a client cannot represent that client unless the disqualified lawyer is timely screened from any participation in the matter and is apportioned no part of the fee, and written notice, including a description of the screening procedures employed by the firm, is promptly given to the affected former client to enable the former client to ascertain compliance with the Model Rules. MRPC 1.10(a)(2). Here, referral to an associate in the attorney's law firm would not be proper unless the screening and written notice requirements are met, in addition to the restriction on the apportionment of the fee mentioned in the answer choice. Answer choice D is incorrect because answer choice C provides a proper course of action.

An attorney served two four-year terms as the state's governor immediately prior to reopening his law office in the state. The attorney printed and mailed an announcement of his return to private practice to members of the bar, previous clients, and personal friends whom he had never represented. The printed announcement stated that the attorney had reopened his law office, gave his address and telephone number, and added that he had been the state's governor for the past eight years. Is the attorney subject to discipline for the announcement? Yes, because the mailing included persons who had not been his clients. Yes, because his service as governor is unrelated to his ability as a lawyer. No, because the information in the announcement was true. No, because all of the information was already in the public domain.

Answer choice C is correct. The communication is an advertisement. MRPC 7.1, which covers lawyer advertising, forbids only false or misleading communications, and this communication was neither false nor misleading. Answer choice A is incorrect because although MRPC 7.3 prohibits lawyers from making in-person, live telephone, or real-time electronic contact to solicit for pecuniary gain, this communication does not fall within the banned behavior. It is an advertisement governed by MRPC 7.1 and 7.2. As a result, the communication is permissible so long as it does not contain any false or misleading statements, which this announcement did not. Answer choice B is incorrect, as MRPC 7.1 forbids only false or misleading communications in lawyer advertising. A truthful statement about the attorney's prior service as governor is not likely to create any unjustified expectations and is therefore not misleading. Answer choice D is also incorrect. Information in the public domain can still be either false or misleading. As such, communication of that information might be prohibited under MRPC 7.1, which forbids false or misleading communications in lawyer advertising.

An attorney represented a baker in a claim involving a breach of the baker's employment contract. The case was settled without suit being filed. The proceeds of the settlement were paid directly to the baker, who subsequently paid the attorney in full for the attorney's fee and expenses. Thereafter, the attorney did no other work for the baker. The baker is now being audited by the Internal Revenue Service (IRS). The IRS has asked the attorney for details of the settlement, including the amount claimed for each item of damage and the amounts paid for the items. The attorney reported the request to the baker, who told the attorney not to provide the information to the IRS. Is it proper for the attorney to furnish the information to the IRS? Yes, if the information does not involve the attorney's work product. Yes, because the attorney no longer represents the baker. No, because the baker told the attorney not to provide the information. No, unless the attorney believes the disclosure would be beneficial to the baker.

Answer choice C is correct. The ethical duty of confidentiality under MRPC 1.6 prohibits the attorney from furnishing the information in the absence of the baker's consent. While a lawyer may reveal confidential information concerning the representation of a client to the extent the lawyer reasonably believes it necessary to comply with other law or a court order, here there is no indication of any law or court order requiring such disclosure. Moreover, unless the client provides informed consent to do otherwise, the lawyer must assert on behalf of the client all non-frivolous claims that the order is not authorized by other law or that the information sought is protected against disclosure by lawyer-client privilege or other applicable law. Answer choice A is not correct, as all information from the representation of the baker, not merely attorney work product, must be kept confidential in the absence of the baker's consent, pursuant to MRPC 1.6. Answer choice B is not correct, as the ethical duty of confidentiality continues even after the representation ends. Answer choice D is not correct, because even if the attorney believes the disclosure would be beneficial to the baker, the attorney is not permitted to provide the information in the absence of the baker's consent, pursuant to MRPC 1.6.

An attorney filed a personal injury suit on behalf of a plaintiff against a defendant. The defendant was personally served with process. The attorney knows that the defendant is insured by an insurance company and that a different attorney has been retained by the insurance company to represent the defendant. No responsive pleading has been filed on behalf of the defendant, and the time for filing expired over ten days ago. Is the plaintiff's attorney subject to discipline if he proceeds to have a default judgment entered? Yes, because he knew that an attorney had been retained by the insurance company to represent the defendant. Yes, because he failed to extend professional courtesy to another lawyer. You Selected: No, because he is properly representing his client's interests. No, because any judgment will be satisfied by the insurance company.

Answer choice C is correct. The plaintiff's attorney has a professional duty to be dedicated and committed to his client's interests, which would include filing a motion to have a default judgment entered in the case. Answer choice A is not correct, as the plaintiff's attorney has a professional obligation to file the default motion in the case. Answer choice B is incorrect, as the plaintiff's attorney is duty-bound to file the motion in order to properly represent his client's interests. This duty to act with diligence neither prohibits extending professional courtesy nor requires it. Answer choice D is not correct, as the fact that the judgment would be satisfied by the insurance company is not relevant to whether or not the plaintiff's attorney has a duty to move for a default judgment in the case.

Under a state law, the court's appointment of lawyers as special masters in certain proceedings is discretionary with the court. A judge decided to appoint special masters in all such proceedings, regardless of their nature and complexity, and decided that compensation for such appointees would be at a reasonable hourly rate. The judge believed that this practice would ensure competent and impartial handling of every proceeding. The judge further decided to use published law directories to compile a list of qualified prospective appointees. Is the judge's proposed practice proper? Yes, because it results in competent and impartial handling of the proceedings. Yes, because the appointees will be compensated at a reasonable hourly rate. No, because the practice may result in unnecessary appointments. No, because the judge cannot use law directories to compile a list of qualified prospective appointees.

Answer choice C is correct. The practice of making appointments without reference to need results in unnecessary appointments and is therefore prohibited by 2007 CJC Rule 2.13(A)(2). Answer choice A is incorrect because achieving competence and impartiality does not justify making special master appointments in all cases before a judge. According to 2007 CJC Rule 2.13(A)(2), a judge must avoid making unnecessary appointments. Answer choice B is incorrect because even if a special master charges a reasonable rate, the judge may not make unnecessary appointments. According to 2007 CJC Rule 2.13(A)(2), a judge must avoid making unnecessary appointments. Answer choice D is incorrect because nothing in the code prohibits a judge from using a law directory to make appointments.

A trial court judge had instructed his court clerk and his secretary that one of them should be present in the office during working hours to answer the telephone. One day, however, the secretary was out sick. The judge was in his office when his court clerk was at lunch, and when the telephone rang, the judge answered it. The call was from a lawyer in a case presently pending before the judge. The lawyer was calling to attempt to reschedule a pretrial conference set for the next day because of a sudden family emergency. The lawyer had tried to call opposing counsel on the case, but she was not answering his calls. The judge agreed to reschedule the pretrial conference for the following week. When the judge's court clerk returned from lunch, the judge instructed the clerk to contact opposing counsel to inform her of the telephone call and the fact that the pretrial conference had been rescheduled. Did the judge act properly? No, because the judge participated in an ex parte communication. No, because there was still time for the calling lawyer to notify opposing counsel in order to reach agreement on rescheduling the pretrial conference. Yes, because the ex parte communication was for scheduling purposes only and did not deal with substantive matters or issues. Yes, because there was no one else in the office to take the lawyer's call.

Answer choice C is correct. Under CJC Rule 2.9(a)(1), a judge or court official may speak ex parte with one of the litigants or a lawyer on a matter of scheduling or any other administrative issue. Answer choice A is incorrect, as the judge did speak ex parte with one of the lawyers, but this specific type of communication is expressly permitted by CJC Rule 2.9(a)(1). Answer choice B is incorrect; the ex parte communication was either proper or it was not. A determination as to whether there was time remaining to notify opposing counsel will not affect the propriety of the conduct. Answer choice D is incorrect because the availability of others to take the call does not affect the propriety of the ex parte communication.

A judge has recently resigned from the state trial court bench. When she was a judge, she supervised activity in cases pending before another judge while he was on vacation. In one pending case, she entered an administrative order changing the courtroom in which a particular case was to be tried. After trial and appeal, the case was remanded for a new trial. The plaintiff in that case decided to change lawyers and has asked the recently resigned judge to try the case. Will the judge be subject to discipline if she tries this case on behalf of the plaintiff? Yes, because the judge acted officially as a judge with respect to an aspect of the case. Yes, because the judge would try the case before a judge of the court on which she previously sat. No, because the judge did not act as a judge with respect to a substantial matter in or the merits of the case. No, because any information that the judge learned about the case while acting as a judge was a matter of public record.

Answer choice C is correct. Under MRPC 1.12(a), the former judge may represent the plaintiff because she did not participate "substantially" in the matter while she was an acting judge. She merely entered an incidental administrative order unrelated to the merits of the lawsuit. Answer choice A is incorrect because, although the judge acted officially in the lawsuit before resigning from the bench, she was not restricted by MRPC 1.12(a) because she did not participate "substantially" in the matter when she entered an administrative order changing the courtroom. Comment [1] to MRPC 1.12 provides that "the fact that a former judge exercised administrative responsibility in a court does not prevent the former judge from acting as a lawyer in a matter where the judge had previously exercised remote or incidental administrative responsibility that did not affect the merits." Answer choice B is incorrect because no rule prohibits a former judge from appearing on behalf of a party before a judge of a court on which she previously sat. Answer choice D is incorrect because the fact that information learned by the former judge in her administrative capacity is a matter of public record would not be enough in itself to allow the former judge to represent a party in a matter in which she had previously participated as a judge. If the judge's personal participation had been substantial, which it was not in this case, then MRPC 1.12(a) would forbid her later representation of the plaintiff.

An attorney is employed by a client who is a fugitive from justice under indictment for armed robbery. The attorney, after thorough legal research and investigation of the facts furnished by the client, reasonably believes the indictment is fatally defective and should be dismissed as a matter of law. The attorney advised the client of his opinion and urged the client to surrender. The client told the attorney that she would not surrender. The attorney informed the district attorney that he represented the client and that he had counseled her to surrender but that she refused to follow his advice. The attorney has not advised his client on how to avoid arrest and prosecution and does not know where she is hiding. Is the attorney subject to discipline if he continues to represent the client? Yes, because the client is engaging in continuing illegal conduct. Yes, because the client refused to accept the attorney's advice and surrender. No, because the attorney is not counseling the client to avoid arrest and prosecution. No, because the attorney reasonably believes the indictment is defective.

Answer choice C is correct. Under MRPC 1.2(d), a lawyer may not knowingly counsel or assist a client in illegal conduct, but in this case, the lawyer has not done so. Answer choice A is incorrect because, although it is true that the client is engaging in continuing illegal conduct, the client is nevertheless entitled to the advice of a lawyer. So long as the lawyer does not knowingly counsel or assist the client in the illegal conduct (MRPC 1.2(d)), the lawyer may continue to represent the client. Answer choice B is incorrect because, although MRPC 1.16(b)(4) grants a lawyer permission to withdraw if the client insists on action that the lawyer believes is repugnant or with which the lawyer fundamentally disagrees, the lawyer is not required to do so. Answer choice D is incorrect because even if the indictment is defective, the client's conduct would still be illegal. Nevertheless, the lawyer may continue to represent the client so long as the lawyer does not knowingly counsel or assist the client in the illegal conduct.

An attorney represents a plaintiff in a medical malpractice case. The attorney's contract with the client provides for a contingent fee of 20% of the recovery by settlement and 30% if the case is tried, with a total fee not to exceed $50,000. The attorney associated a second attorney, a sole practitioner, in the case, with the client's written consent and after full disclosure of the fee agreement between the two lawyers. The second attorney is both a medical doctor and a lawyer and is well qualified by experience and training to try medical malpractice cases. The fee agreement between the two attorneys reads as follows: "The total fee in this case is 20% of recovery by settlement and 30%, if tried, with a maximum fee of $50,000. The first attorney will help with discovery and will be the liaison person with the client. The second attorney will prepare the case and try it if it is not settled. The two attorneys will divide the fee, 40% to the first attorney and 60% to the second, sole practitioner attorney." Are the two attorneys subject to discipline for their agreement for division of the fee? Yes, unless the client's consent is in writing. Yes, because the first attorney will not try the case. No, if the division of the fee between the two attorneys is in proportion to actual work done by each. No, because the total fee does not differ from that contracted for by the first attorney with the client.

Answer choice C is correct. Under MRPC 1.5(e), lawyers from different firms may split a fee if: (i) the fee is in proportion to the services rendered by each lawyer or joint responsibility is assumed for the representation, (ii) the client agrees in writing to the fee-splitting arrangement, and (iii) the total fee charged must be reasonable such that a client is not charged more just because additional lawyers are working on the case. All three conditions are met under answer choice C. Answer choice A is incorrect, because, although the client's consent must be in writing under the rule, this is not the only requirement for permissible fee-splitting. Therefore, A is not the best answer. Answer choice B is incorrect, as there is no requirement under the rule that the first attorney try the case. Answer choice D is incorrect because, though the client has not been charged more as a result of the addition of the second attorney, this is not the only requirement for permissible fee-splitting. With the proviso of answer choice C, all requirements would be met, making answer choice C the best choice.

An attorney is employed in the legal department of a public utility company and represents that company in litigation. The company has been sued by a consumer group that has accused the company of various acts in violation of its charter. Through its general counsel, the company has instructed the attorney not to negotiate a settlement but to go to trial under any circumstances because a precedent needs to be established. The attorney believes the defense can be supported by a good faith argument, but also believes the case should be settled if possible. Must the attorney withdraw as counsel in this case? Yes, because the company is controlling the attorney's judgment in refusing to settle the case. Yes, because a lawyer should endeavor to avoid litigation. No, because the company's defense can be supported by a good faith argument. No, because, as an employee, the attorney is bound by the instructions of the general counsel.

Answer choice C is correct. Under MRPC 3.1, it is not frivolous to begin a lawsuit or assert a defense if there is a good faith argument that will support the prosecution or defense. Answer choice A is incorrect; although the attorney had an obligation to use independent judgment in advising the client, MRPC 1.2(a) provides that a lawyer must abide by a client's decision concerning the objectives of the representation. Answer choice B is incorrect; litigation is often necessary to vindicate a client's rights under law, and, as a result, nothing in the text or comments of the Model Rules suggests that lawyers should as a general matter endeavor to avoid litigation that is not frivolous. Answer choice D is incorrect; even when an attorney is an employee of a company, the attorney is required to exercise independent judgment and could not, for example, file a frivolous lawsuit on the company's behalf. In prosecuting lawsuits that are not frivolous and in defending lawsuits in which there is a good faith argument for the defense, however, the lawyer must abide by the client's decision concerning the objectives of the representation regardless of whether the lawyer is employed or retained.

Attorney Alpha, a partner in the law firm of Alpha & Beta, was retained by the plaintiff in a personal injury action. The jury rendered a verdict in favor of the defendant, and Alpha filed an appeal on the plaintiff's behalf. Alpha reviewed the trial transcript and wrote the brief. The brief stated, "It is uncontroverted that the defendant failed to signal before turning left into the intersection." In fact, a witness called by the defendant testified that the defendant did signal before turning. Alpha was aware of this testimony, having read it while reviewing the trial transcript. Three days before the appeal was scheduled to be argued in the state's intermediate appellate court, Alpha suffered a heart attack. Attorney Beta, one of Alpha's partners, agreed to argue the appeal. Beta knew nothing about the case and had no opportunity to confer with Alpha. In preparing for the argument, Beta read Alpha's brief thoroughly and read as much of the trial transcript as was possible in the limited time available, but did not read the witness's testimony. In oral argument, Beta stated to the court, "Your honors, as stated in our brief, it is uncontroverted that the defendant failed to signal before turning left into the intersection." Beta assumed that Alpha's statement in the brief to that effect was correct. Is Beta subject to discipline for making this statement during oral argument? Yes, because the statement was false. Yes, because Beta did not know whether or not the statement was true. No, because Beta did not know that the statement was false. No, because all Beta did was truthfully recount the statement made by Alpha in the brief.

Answer choice C is correct. Under MRPC 3.3, a lawyer must not knowingly make a false statement of fact to a tribunal. Here, Beta did not know the statement was false and therefore is not subject to discipline. Answer choice A is not correct, as even though the statement was false, Beta did not know it was false. Answer choice B is not correct, as MRPC 3.3 subjects a lawyer to discipline only for knowingly making a false statement. Answer choice D is not correct, as factually this answer choice is wrong. Beta did not merely recount the statement made in the brief, Beta relied on it as correct in oral argument, thus asserting the statement again. Even so, Beta did not know that the statement was false, so Beta is not subject to discipline under MRPC 3.3.

An attorney was engaged under a general retainer agreement to represent a corporation involved in the uranium industry. Under the agreement, the attorney handled all of the corporation's legal work, which typically involved regulatory issues and litigation. The corporation told the attorney that a congressional committee was holding hearings concerning the extent of regulation in the copper industry. Because the corporation was considering buying a copper mine during the next fiscal year, the corporation wanted the attorney to testify that the industry was overregulated. The attorney subsequently testified before the relevant congressional committee. The attorney registered his appearance under his own name and did not disclose that he was appearing on behalf of a client. Afterward, the attorney billed the corporation for fees and expenses related to his testimony. Was the attorney's conduct proper? Yes, because the duty of confidentiality prevented the attorney from disclosing the identity of his client. Yes, because the attorney-client evidentiary privilege prevented disclosure of the identity of his client in this context. No, because the attorney failed to disclose that he was appearing and testifying in a representative capacity. No, because the attorney accepted compensation in return for his testimony.

Answer choice C is correct. Under MRPC 3.9, a lawyer who represents a client before a legislative body must disclose that the appearance is in a representative capacity. As the attorney did not do that here, his conduct was improper. Answer choice A is not correct, as MRPC 3.9 requires such disclosure. Answer choice B is not correct, as the attorney-client evidentiary privilege is irrelevant. There is no indication in the facts that the client wanted the attorney not to reveal the representation. Even if that was the case, a lawyer may not mislead a legislative body regarding the fact that the lawyer appears in a representative capacity. To comply with the rule, the lawyer could simply disclose that he is appearing on behalf of another, whose identity is privileged. Answer choice D is not correct, as a lawyer may accept compensation for assisting a client in trying to influence legislative action. The lawyer must disclose the fact that he is appearing in a representative capacity, however, to be in compliance with the Model Rules.

Attorney Alpha is recognized as an expert in securities regulation law. A corporation retained Alpha's law firm to qualify the corporation's stock for public sale. After accepting the matter, Alpha decided that he preferred to spend his time on cases with larger fee potential, so he assigned responsibility for the corporation matter to Attorney Beta, an associate in Alpha's office who had recently been admitted to the bar. Beta protested to Alpha that he, Beta, knew nothing about securities regulation law and that he had too little time to prepare himself to handle the corporation's matter competently without substantial help from Alpha. Alpha responded, "I don't have time to help you. Everyone has to start somewhere." Alpha directed Beta to proceed. Was Alpha's conduct proper in this matter? Yes, because as a member of the bar, Beta is licensed to handle any legal matter. Yes, because Alpha may withdraw from a case if work on it would cause him unreasonable financial hardship. No, because Alpha knew Beta was not competent to handle the matter, and Alpha failed to provide supervision adequate to protect the client's interests. No, because the corporation had not given Alpha permission to assign Beta to work on the matter.

Answer choice C is correct. Under MRPC 5.1, a lawyer having direct supervisory authority over another lawyer shall make reasonable efforts to ensure that the other lawyer conforms to the Model Rules. Here, Alpha knew that Beta was not competent to handle the securities matter and he failed to provide adequate supervision to protect the interests of the client. Answer choice A is not correct, as the fact that Beta is a licensed attorney is not sufficient to render Alpha's conduct proper under the Model Rules. Securities matters are complex and expertise is required. Alpha was not permitted to delegate responsibility for the matter without ensuring that Beta was competent to do the work or adequately supervising Beta. Answer choice B is not correct, as Alpha did not withdraw from the case, but simply delegated the matter to an associate, Beta, who was not competent to do the work. Answer choice D is not correct, as Alpha did not need the corporation's permission to assign Beta to work on the matter. A lawyer may generally delegate responsibilities to associates, so long as the lawyer ensures adequate supervision.

Attorney Alpha represents a wife in a marriage dissolution proceeding that involves bitterly contested issues of property division and child custody. The husband is represented by Attorney Beta. After one day of trial, the husband, through Beta, made a settlement offer. Because of the husband's intense dislike for Alpha, the proposed settlement requires that Alpha agree not to represent the wife in any subsequent proceeding, brought by either party, to modify or enforce the provisions of the decree. The wife wants to accept the offer, and Alpha believes that the settlement offer made by the husband is better than any award the wife would get if the case went to judgment. Is it proper for Alpha to agree that Alpha will not represent the wife in any subsequent proceeding? Yes, because the restriction on Alpha is limited to subsequent proceedings in the same matter. Yes, if Alpha believes that it is in the wife's best interests to accept the proposed settlement. No, because the proposed settlement would restrict Alpha's right to represent the wife in the future. No, unless Alpha believes that the wife's interests can be adequately protected by another lawyer in the future.

Answer choice C is correct. Under MRPC 5.6(b), a lawyer must not make or offer an agreement in which a restriction on the lawyer's right to practice is part of the settlement of a client controversy. Answer choice A is not correct, as MRPC 5.6(b) applies to any restriction on the lawyer's right to practice. Answer choice B is not correct, as MRPC 5.6(b) restricts Alpha regardless of whether the settlement is or is believed by Alpha to be in the wife's best interests. Answer choice D is not correct, as it is not relevant that Alpha may believe the wife's interests can be adequately protected by another lawyer in the future. MRPC 5.6(b) controls and prohibits Alpha from making or offering an agreement that would restrict Alpha's right to practice.

Attorneys Alpha and Beta had been political opponents. Alpha was elected to the state legislature after a bitter race in which Beta had managed the campaign of Alpha's opponent. Alpha had publicly blamed Beta at that time for what Alpha reasonably believed were illegal and unethical campaign practices and later had publicly objected to Beta's appointment as a judge. Alpha represented a client in a widely publicized case tried in Judge Beta's court. At the conclusion of the trial, Beta ruled against Alpha's client. Alpha then held a press conference and said, "All that you reporters have to do is check your files and you will know what I think about Judge Beta's character and fitness." Is Alpha subject to discipline for making this statement? You Selected: Yes, if Alpha's statement might lessen confidence in the legal system. Yes, because Alpha's past accusations were unrelated to Beta's legal knowledge. No, because Alpha reasonably believed that the statements about Beta were true. No, if Beta had equal access to the press.

Answer choice C is correct. Under MRPC 8.2(a), a lawyer must not make a statement that the lawyer knows to be false or with reckless disregard as to its truth or falsity concerning the qualifications or integrity of a judge. Here, Alpha told the reporters to check their files, referring to his prior statements about what he reasonably believed were illegal and unethical campaign practices by Beta. As he reasonably believed that these statements were true, Alpha is not subject to discipline. Answer choice A is not correct, as Alpha's reasonable belief in the truth of his statements renders them permissible under the Model Rules. Answer choice B is not correct, as it is irrelevant that Alpha's past accusations were unrelated to Beta's legal knowledge. To be subject to discipline for a statement about a judge's integrity, a lawyer must know the statement to be false or make the statement with reckless disregard for its truth. Answer choice D is not correct, as it is not necessary for Beta to have had equal access to the press for Alpha not to be subject to discipline.

A defendant, who has been indicted for auto theft, is represented by an attorney. The prosecutor reasonably believes that the defendant committed the offense, but, because of the defendant's youth, it is in the interest of justice to permit the defendant to plead guilty to the lesser offense of "joy-riding" in return for an agreement by the prosecutor to recommend probation. The prosecutor has so advised the attorney, but the attorney told the prosecutor she would not plea bargain and would insist on a jury trial. The attorney informed the defendant of the prosecutor's offer and advised the defendant not to accept it. The defendant followed the attorney's advice. The attorney is a candidate for public office, and the prosecutor suspects that the attorney is insisting on a trial of the case to secure publicity for herself. Assume for the purposes of this question that the defendant was tried, convicted, and sentenced to prison for two years. Must the prosecutor report to the disciplinary authority his suspicions about the attorney's conduct in the case? Yes, because the defendant suffered a detriment from the attorney's refusal to plea bargain. Yes, if the attorney in fact received widespread publicity as a result of the trial. No, unless the prosecutor has knowledge that the attorney's refusal to plea bargain was due to personal motives. No, if the attorney zealously and competently represented the defendant at the trial.

Answer choice C is correct. Under MRPC 8.3, a lawyer generally must report misconduct by another lawyer to the appropriate professional authority when the lawyer has actual knowledge of the misconduct. Only if the prosecutor knew that the attorney's refusal to accept the plea bargain was based on an improper motive would the prosecutor be required to report the matter. Answer choice A is not correct, as misconduct must only be reported when there is actual knowledge of the misconduct. The fact that the defendant merely suffered a detriment is not enough to require reporting. Answer choice B is not correct, as only actual knowledge of misconduct imposes a duty to report. The fact that the attorney received widespread publicity does not, by itself, indicate that the attorney did anything wrong and does not require a report to the disciplinary authority. Answer choice D is not correct, as it is irrelevant to the question asked. The only issue is whether the prosecutor has actual knowledge of misconduct that he is required to report.

An attorney and a client entered into a written retainer and hourly fee agreement that required the client to pay $5,000 in advance of any services rendered by the attorney and that required the attorney to return any portion of the $5,000 that was not earned. The agreement further provided that the attorney would render monthly statements and withdraw her fees as billed. The agreement was silent as to whether the $5,000 advance was to be deposited in the attorney's Clients' Trust Account or in a general account. The attorney deposited the entire fund in her Clients' Trust Account, which also contained the funds of other persons, which had been entrusted to the attorney. Thereafter, the attorney rendered monthly progress reports and statements for services to the client after services were rendered, showing the balance of the client's fee advance. However, the attorney did not withdraw any of the $5,000 advance until one year later, when the matter was concluded to the client's complete satisfaction. At that time, the attorney had billed the client reasonable legal fees of $4,500. The attorney wrote two checks on her Clients' Trust Account: one to herself for $4,500, which she deposited in her general office account, and one for $500 to the client. Was the attorney's conduct proper? Yes, because the attorney deposited the funds in her Clients' Trust Account. Yes, because the attorney rendered periodic and accurate billings. No, because the attorney's failure to withdraw her fees as billed resulted in an impermissible commingling of her funds and the client's funds. No, because the attorney required an advanced payment against her fee.

Answer choice C is correct. Under the fee agreement, the attorney agreed to withdraw her fees as billed. The fact that she failed to withdraw her fees as billed over the year meant that she commingled her funds with the client's because all the money remained in a Clients' Trust Account. MRPC 1.15(a) prohibits commingling of a lawyer's personal funds with client funds. Answer choice A is not correct, as the attorney was required to withdraw her fees as billed. She could not leave them in the Clients' Trust Account. Answer choice B is not correct because the fact that the billing was accurate is not at issue. The attorney commingled personal and client funds by not withdrawing her fees as billed. Answer choice D is not correct, as advance payment of fees is not improper so long as the lawyer deposits the funds into a Clients' Trust Account to be withdrawn by the lawyer only as fees are earned or expenses incurred in the representation. MRPC 1.15(c).

While presiding over the trial of a highly publicized antitrust case, a judge received in the mail a lengthy letter from a local attorney. The letter discussed the law applicable to the case. The judge knew that the attorney did not represent either party. The judge read the letter and, without mentioning its receipt to the lawyers in the pending case, filed the letter in his general file on antitrust litigation. Later, after reading the trial briefs, the judge concluded that the attorney's letter better explained the law applicable to the case pending before him than either of the trial briefs. The judge followed the attorney's reasoning in formulating his decision. Was it proper for the judge to consider the attorney's letter? Yes, because the judge did not initiate the communication with the attorney. Yes, if the attorney did not represent any client whose interests could be affected by the outcome. No, unless the judge, prior to rendering his decision, communicated its contents to all counsel and gave them an opportunity to respond. No, because the attorney is not of record as counsel in the case.

Answer choice C is correct. While CJC Rule 2.9(A)(2) provides that a judge is permitted to obtain the written advice of a disinterested expert on the law applicable to a proceeding before the judge, it requires that the judge give advance notice to the parties of the person to be consulted and the subject matter of the advice, and provide them with a reasonable opportunity to object and respond to the notice and to the advice received. Answer choice A is not correct, as a judge may consider unsolicited advice, but is required to communicate its contents to the parties and provide them with notice and an opportunity to respond to the advice. Answer choices B and D are not correct, as it is irrelevant that the attorney did not represent any client whose interests could be affected by the outcome or is not of record as counsel in the case. The judge can look at the advice of a disinterested expert, so long as the judge gives advance notice to the parties of the person being consulted and provides them with a reasonable opportunity to object and respond to the notice.

The secretary to a state trial court judge asked the judge to write a letter to the secretary's bank describing her good employment record for the judge and recommending her for a mortgage. The judge wrote the letter on official judicial letterhead, indicating that the reference was personal, truthfully describing the secretary's exemplary work record, and stating that she believed the secretary was a good risk as a borrower. The bank provided a mortgage to the secretary at a rate of interest that would have been made available on the same terms to similarly situated persons who did not work for a state trial court judge. Did the judge act properly? No, because the judge abused her official position to advance the secretary's personal economic interests. No, because it was improper to use official letterhead for a personal reference. Yes, because a judge may use official letterhead for a reference for someone else if the judge indicates that the reference is personal and there is no likelihood it would reasonably be perceived as an attempt to gain favorable treatment. Yes, because the judge was not using her official position to gain a personal advantage, but rather an economic advantage for someone else.

Answer choice C is correct. While a judge must not abuse the prestige of judicial office to advance her own personal or economic interests, a judge is permitted to provide a reference for another person based on the judge's personal knowledge. A judge must not use judicial letterhead to gain an advantage in conducting her personal business, but may write a reference letter on official letterhead for another, if the judge indicates that the reference is personal and there is no likelihood that the use of the letterhead would reasonably be perceived as an attempt to use the judge's official position to exert pressure. Answer choice A is incorrect because the judge did not abuse her official position; she merely provided a common employment reference. Although the reference advanced the secretary's economic interests, such action is permitted. Answer choice B is incorrect because official letterhead may be used to provide a reference if the judge indicates that it is personal and there is no likelihood that it would reasonably be perceived as an attempt to gain favorable treatment. Answer choice D is incorrect because it is not enough that the judge was simply trying to help another person obtain an economic advantage rather than using her official position for personal gain. To be proper, there must also be no likelihood that the judge's behavior would reasonably be perceived as an attempt to gain favorable treatment.

A client, during a conference with her attorney in his office about the client's pending divorce, threatened to kill her husband, from whom she was separated. Based on his knowledge of the client, the attorney believed his client's threat to be credible. The attorney warned the husband of the client's threat. Despite the warning, the husband did not take the threat seriously. Shortly thereafter, the client shot her husband, seriously wounding him. Is the attorney subject to discipline for warning the husband of the threat? No, because the attorney had a duty to warn his client's husband. No, because the attorney's failure to warn the client's husband was not the proximate cause of the husband's harm. No, because the attorney was reasonably certain that the disclosure was necessary to prevent death or bodily harm. Yes, because the attorney owed his client a duty of confidentiality.

Answer choice C is correct. While a lawyer has a duty of confidentiality to his client, which includes protecting confidential information received from the client, a lawyer may reveal such information to the extent that the lawyer reasonably believes disclosure is necessary to prevent reasonably certain death or bodily harm. MRPC 1.6(b)(1). Consequently, although the client's threat to kill her husband was received by the attorney in the course of his representation of the client in a divorce, and as such was subject to the ethical duty of confidentiality, the "prevention of death or substantial bodily harm" exception permits the attorney to reveal the client's threat to the husband. Answer choice A is incorrect because, while a lawyer may reveal confidential information received from a client concerning the representation to the extent that the lawyer reasonably believes disclosure is necessary to prevent reasonably certain death or bodily harm, a lawyer is not obligated to do so. Answer choice B is incorrect because the issue of the proximate cause of the husband's injury is not relevant in a disciplinary action. Answer choice D is incorrect because, as noted, while the attorney owed a duty of confidentiality to his client, there is an exception that permitted the attorney to disclose the client's threat to the husband.

A woman who claimed she had been severely injured on a construction job site asked an attorney to represent her in a civil lawsuit against the owners of the site and the construction company. The woman indicated that she was recently unemployed and that she had no ready money to pay for the costs of the action. She said that she could mortgage her home or sell some family heirlooms, but that coming up with funds to begin the litigation would take her some time. The attorney told her that he would advance the costs of filing suit, including the costs to obtain a medical examination to determine the extent of damages the woman had suffered, but that such costs would be recoverable by him from any judgment he secured on her behalf, along with his reasonable contingent attorney's fee. The woman signed a costs and fee agreement assenting to these terms. Are the attorney's actions with regard to the costs proper? No, because the attorney obtained an improper proprietary interest in the woman's litigation. No, because the costs of a medical examination cannot be advanced to a client. Yes, because advancement of these particular costs on a contingent basis is permissible. Yes, because costs of filing suit can be waived for indigent clients.

Answer choice C is correct. While financial assistance to a client is generally not permitted for pending or planned litigation, a lawyer is permitted to advance litigation costs, including the costs of a medical examination and the costs of obtaining and presenting evidence, to the client. Repayment may be made contingent on the outcome of the case. Answer choice A is incorrect because the lawyer has not obtained an improper proprietary interest. Answer choice B is incorrect, as the costs of a medical examination can also be advanced. Answer choice D is incorrect, because while this is a true statement, it is not relevant to this question. The client is not indigent, only unable to immediately pay the litigation costs.

An attorney is representing the plaintiff in a personal injury case on a contingent fee basis. The client is without resources to pay for the expenses of the investigation and the medical examinations necessary to prepare for trial. The client asked the attorney to pay for these expenses. The attorney declined to advance the funds but offered to guarantee the client's promissory note to a local bank in order to secure the funds needed to cover those expenses. The client has agreed to reimburse the attorney in the event she incurs liability on the guaranty. Is the attorney subject to discipline if she guarantees the client's promissory note? Yes, because the attorney is lending her credit to the client. Yes, because the attorney is helping to finance litigation. No, because the funds will be used for trial preparation. No, because the attorney took the case on a contingent fee basis.

Answer choice C is correct. While financial assistance to a client is generally prohibited with respect to pending or planned litigation, a lawyer is permitted to advance litigation costs (including the expenses of medical examination and the costs of obtaining and presenting evidence) to the client under MRPC 1.8(e). The attorney will therefore not be subject to discipline for guaranteeing the client's promissory note. Answer choices A and B are not correct, as MRPC 1.8(e) allows a lawyer to advance litigation costs to the client. Answer choice D is not correct, as MRPC 1.8(e) allows the repayment of litigation costs to be contingent on the outcome of the matter.

An attorney experienced several instances when clients failed to pay their fees in a timely manner, but it was too late in the representation to withdraw without prejudicing the clients. To avoid a recurrence of this situation, the attorney has drafted a stipulation of consent to withdraw if fees are not paid according to the fee agreement. She proposes to have all clients sign the stipulation at the outset of the representation. Is it proper for the attorney to use the stipulation to withdraw from representation whenever a client fails to pay fees? Yes, because a lawyer may withdraw when the financial burden of continuing the representation would be substantially greater than the parties anticipated at the time of the fee agreement. Yes, because the clients consented to the withdrawal in the stipulation. No, because a client's failure to pay fees when due may be insufficient in itself to justify withdrawal. No, unless clients are provided an opportunity to seek independent legal advice before signing the stipulation.

Answer choice C is correct. While in some cases failure to pay fees may be sufficient to justify a withdrawal under MRPC 1.16, in other cases it will not be sufficient. Under MRPC 1.16(b)(5), a lawyer may withdraw if the client refuses to fulfill an obligation regarding the representation, which can include nonpayment of fees. In such cases, however, the client must first be given reasonable warning and an opportunity to satisfy the obligation. The use of an advance-consent stipulation allowing withdrawal in all circumstances in which fees are not paid immediately would not allow for a proper warning or provide an opportunity for the client to satisfy her obligation as required by the rule. Answer choice A is not correct. While a lawyer may seek to withdraw when the representation will result in an unreasonable financial burden on the lawyer, this stipulation attempts to allow the attorney to withdraw whenever fees are not paid according to the fee agreement. A court may determine that one instance of nonpayment does not necessarily constitute an unreasonable burden. Answer choice B is not correct, as even if a client consents to a withdrawal, a court may nevertheless order a lawyer to continue representation. Answer choice D is not correct, as the requirement for seeking independent legal advice applies to the settlement of malpractice claims, not in this context.

Attorney Alpha, a member of the bar, placed a printed flyer in the booth of each artist exhibiting works at a county fair. The face of the flyer contained the following information: "I, Alpha, am an attorney, with offices in 800 Bank Building, telephone (555) 555-5555. I have a J.D. degree from State Law School and an M.A. degree in fine arts from State University. My practice includes representing artists in negotiating contracts between artists and dealers and protecting artists' interests. You can find me in the van parked at the fair entrance." All factual information on the face of the flyer was correct. There was a retainer agreement on the back of the flyer. At the entrance to the fair, Alpha parked a van with a sign that read "Alpha - Attorney at Law - 800 Bank Building." For which, if any, of the following is Alpha subject to discipline? Placing copies of the flyer in the booth of each artist. Including a retainer agreement on the back of the flyer. Parking the van with the sign on it at the fair entrance. Alpha is not subject to discipline for any of the above.

Answer choice D is correct because answer choices A, B, and C are all proper. Answer choice A is proper because a lawyer is permitted to advertise his services through written communication, so long as the communication regarding such services is not false or misleading in violation of MRPC 7.1, or in violation of the rules against solicitation of clients under MRPC 7.3. MRPC 7.2(a). In addition, any advertisement must include the name and office address of at least one lawyer or law firm responsible for its content. MRPC 7.2(c). Here, the flyer is a written communication that is not false or misleading, and it included the attorney's name and office address. The attorney was not engaging in solicitation, because unless a prospective client has made known to the lawyer a desire not to be solicited, the lawyer may target the client with written communications so long as the communications satisfy the Model Rules with regard to advertising and communications by lawyers. Answer choice B is proper, as a lawyer may use a preprinted retainer form. Answer choice C is proper, as there is no prohibition against the use of a billboard or outdoor sign as advertising, so long as it meets the requirements of MRPC 7.2.

A member of the bar is a candidate for judicial office in an election. The candidate personally asked several of his friends to contribute $1,000 each to kick off his campaign. After the candidate's friends made the contributions, the candidate, who was elated by the support, formed a committee to collect more contributions. The candidate then turned over the contributions to the committee and began campaigning in earnest. Is the candidate subject to discipline? No, because the candidate turned over the funds to his committee. No, unless the committee includes lawyers likely to practice before the candidate. Yes, unless none of the original contributors was a lawyer. Yes, because the candidate personally solicited funds.

Answer choice D is correct. A candidate for judicial office is not permitted to personally solicit or accept campaign contributions, except through a campaign committee authorized by CJC Rule 4.4. Here, the candidate personally asked for contributions and then turned the money over to his campaign committee, which subjects him to discipline under the Code of Judicial Conduct. Answer choice A is not correct, as subsequently turning over campaign contributions that he personally solicited does not meet the requirements of CJC Rule 4.4. Answer choice B is not correct, as the issue of who is on the committee is irrelevant. What controls the issue is that the candidate personally solicited campaign contributions, rather than only soliciting them through a campaign committee. Answer choice C is not correct, as it does not matter whether any of the original contributors was a lawyer. As the candidate personally solicited the funds, he is subject to discipline for violating CJC Rule 4.4.

A trial court judge serves as the president of a not-for-profit charitable organization that provides relief to victims of natural disasters. It is unlikely that the organization will be engaged in proceedings that would ordinarily come before the judge or the court of which the judge is a member. As president, the judge receives reasonable compensation that is commensurate with the judge's service as president. Following a spate of natural disasters that have depleted the organization's funding, the organization decides to seek additional funding through a grant from a charitable trust. The grant application requires the president of the organization to sign the grant application on behalf of the organization. The judge signs the application, indicating that she is president of the organization, but does not indicate that she is a judge. Are the judge's actions with regard to the not-for-profit charitable organization improper? No, because the judge fully complied with the Code of Judicial Conduct with regard to participation in charitable organizations and activities. Yes, because a judge may never receive compensation for extrajudicial activities. Yes, because a judge is not permitted to serve as an officer of a not-for-profit organization. Yes, because the judge may not engage in the fundraising activities on behalf of the not-for-profit organization as described in these facts.

Answer choice D is correct. A judge generally may not engage in fundraising activities on behalf of a not-for-profit charitable organization. The limited exceptions that permit a judge to solicit contributions to such an organization from members of the judge's family or from other judges over whom the judge does not have supervisory or appellate authority do not apply to this judge. CJC Rule 3.7(A)(2). Answer choice A is therefore incorrect. Answer choice B is incorrect because a judge may receive reasonable compensation for extrajudicial activities that are otherwise permitted by the Code of Judicial Conduct or other law unless such acceptance would appear to a reasonable person to undermine the judge's independence, integrity, or impartiality. CJC Rule 3.12. Answer choice C is incorrect because a judge may generally serve as an officer of a not-for-profit charitable organization, subject to certain restrictions, including not engaging in fundraising activities on behalf of the organization. CJC Rule 3.7.

A judge is presiding in a case that has, as its main issue, a complicated point of commercial law. The lawyers have not presented the case to the judge's satisfaction, and the judge believes she needs additional legal advice. The judge's former partner in her previous law practice is an expert in the field of law that is at issue. The former partner has no interest in the case. Is it proper for the judge to consult her former partner? Yes, because her former partner has no interest in the case. Yes, if the judge believes that her former partner's advice is needed to serve the interests of justice. No, unless all parties in the case first give their written consent to the judge's consultation with her former partner. No, unless the judge informs the parties of the former partner's identity and the substance of the former partner's advice, and asks for their responses.

Answer choice D is correct. A judge is permitted to obtain the written advice of a disinterested expert on the law applicable to a proceeding before the judge, if the judge gives advance notice to the parties of the person to be consulted and the subject matter of the advice to be solicited, and gives them a reasonable opportunity to object and respond to the notice and to the advice received. CJC Rule 2.9(A)(2). Answer choices A and B are not correct, as the Code of Judicial Conduct requires more than what these answers provide. The judge must inform the parties in advance and give them a reasonable opportunity to object and respond. Answer choice C is not correct, as the judge does not need the parties' consent.

A judge who deals exclusively with criminal matters is named in the will of a deceased uncle as the personal representative of the uncle's estate. The judge was very close to the uncle while growing up, but the judge has had almost no contact with the uncle for the past five years. The judge would like to serve as both the personal representative of the estate and as the lawyer for the estate in probate court. Which of the following is a correct statement regarding the judge's ability to serve as the personal representative of the uncle's estate and as the lawyer for the estate in probate court? It is proper for the judge to serve as both personal representative and lawyer for the uncle's estate. It is proper for the judge to serve as the personal representative, but not as the lawyer for the uncle's estate. It is proper for the judge to serve as the lawyer for his uncle's estate, but not as personal representative. It is not proper for the judge to serve as either personal representative or lawyer for the uncle's estate.

Answer choice D is correct. A judge may serve as the personal representative for the estate of a decedent only when the decedent was a member of the judge's family. CJC Rule 3.8. A member of the judge's family includes only the judge's spouse or domestic partner, child, grandchild, parent, grandparent, or other relative or person with whom the judge maintains a close familial relationship. CJC, Terminology. Here, the judge did not maintain the close familial relationship that she once had with the uncle. Thus, the judge cannot serve as personal representative of the uncle's estate. In addition, while a judge may act pro se, a judge may not serve as a lawyer, even for a family member, in any forum. CJC Rule 3.10. For the reasons noted with respect to answer choice D, answer choices A, B, and C are incorrect.

A judge, prior to her appointment to the probate court, was a partner in a law firm. The law firm had an extensive probate practice. At the time of the judge's appointment, the law firm had pending before the court to which the judge was appointed numerous matters in which requests were being made for allowances for attorney's fees. When the judge left the law firm, she was paid a cash settlement. She has no further financial interest in any matter handled by the law firm. The judge is now being asked to rule on these requests for allowances for attorney's fees. Is it proper for the judge to rule on these requests? Yes, because the judge has no financial interest in the outcome of these cases. Yes, if these requests are not contested. No, unless the judge notes on the record in each case her prior association with the law firm. No, because the judge was associated with the law firm when these matters were pending.

Answer choice D is correct. A judge must disqualify herself if she was associated with a lawyer who participated substantially as a lawyer in the matter during such association. CJC Rule 2.11(A)(6). Answer choice A is not correct because even if the judge has no financial interest in the outcome of the cases, the Code of Judicial Conduct requires recusal when the judge was associated with a lawyer who participated substantially as a lawyer in the matter during such association. Answer choice B is not correct, as the Code of Judicial Conduct requires recusal even if the requests are not contested. Similarly, answer choice C is not correct, as recusal is required and cannot be avoided merely by noting the judge's prior association with the law firm for the record.

At a local restaurant, a justice of the peace notices another customer sitting at a table next to the table at which the justice of the peace is sitting. The justice of the peace does not know the other customer. After that customer leaves, the waiter for the table at which the justice of the peace is sitting complains to the justice of the peace that the customer at the next table had left without paying his bill. The waiter indicates that he knows the identity of his nonpaying customer, since the customer has behaved similarly on at least one other occasion, but refuses to reveal the customer's identity to the justice of the peace. Despite the urging of the justice of the peace to legally pursue the matter, the waiter states that he is not going to do so, but instead will refuse to wait on the customer if he comes to the restaurant again. During the following week, the justice of the peace, after seeing a picture in a state bar association publication, does not know, but believes that there is substantial likelihood that a particular licensed attorney in the state was the nonpaying customer and that, under state law, the failure to pay constituted misdemeanor theft. The justice of the peace takes no action with respect to the attorney's conduct at the restaurant. Assuming that the justice of the peace is herself not under a duty to report the customer's behavior to the police, is the justice of the peace nevertheless subject to discipline as a judicial official for her failure to take any action with respect to the attorney? No, because she, as a justice of the peace, is not subject to the Code of Judicial Conduct. No, because the attorney's misconduct did not involve the representation of a client. Yes, because the justice of the peace must inform the appropriate authorities of the attorney's criminal act because it raises a substantial question regarding the lawyer's honesty, trustworthiness, or fitness as a lawyer. Yes, because there was a substantial likelihood that the attorney committed a theft.

Answer choice D is correct. A judge who receives information indicating a substantial likelihood that a lawyer has committed a violation of the Model Rules of Professional Conduct must take appropriate action. CJC Rule 2.15(D). The commission of a criminal act that reflects adversely on the lawyer's honesty, trustworthiness, and fitness as a lawyer constitutes professional misconduct. In addition, conduct that involves dishonesty, fraud, deceit, or misrepresentation can constitute professional misconduct. MRPC 8.4(b), (c). Consequently, the justice of the peace had a duty to take appropriate action with regard to the attorney's theft. Answer choice A is incorrect because a justice of the peace is subject to the Code of Judicial Conduct. CJC, Application I(A). Answer choice B is incorrect because commission of a criminal act or conduct that reflects adversely on the lawyer's honesty need not arise out of the representation of a client. MRPC 8.4(b), (c), cmt. 2. Answer choice C is incorrect because, while a judge does have a duty to inform the appropriate authorities of a lawyer's criminal action when it raises a substantial question regarding the lawyer's honesty, trustworthiness, or fitness as a lawyer, that duty only arises when the judge possesses actual knowledge of the lawyer's professional misconduct. The justice of the peace did not have actual knowledge of the attorney's misconduct, but only information that indicated a substantial likelihood that such misconduct had occurred.

An attorney has tried many contested cases before a certain judge up for reelection. The attorney believes the judge is lacking both in knowledge of the law and in good judgment and that a different attorney would make an excellent judge. The attorney wishes to defeat the judge and assist the other attorney in getting elected. The attorney intends to contribute $5,000 to the other attorney's campaign. Is it proper for the attorney to do so? Yes, the attorney may give $5,000 to the other attorney personally for his campaign. Yes, if the attorney's contribution to the other attorney is made anonymously. No, because the attorney is practicing before the court to which the other attorney seeks election. No, unless the attorney gives the $5,000 to a committee formed to further the other attorney's election.

Answer choice D is correct. A judicial candidate is not permitted to solicit or accept campaign contributions other than through a campaign committee. CJC Rule 4.1. Thus, unless the attorney's contribution is made to a campaign committee, it would be improper. Answer choice A is incorrect, as CJC Rule 4.1 prohibits a judicial candidate from personally accepting campaign contributions. Answer choice B is incorrect because, even if made anonymously, a contribution to a candidate for judicial office must be made through a campaign committee established to accept contributions on behalf of the candidate. Answer choice C is incorrect, as neither the Model Rules nor the Code of Judicial Conduct prohibits campaign contributions by lawyers who practice before a court to which a candidate for judicial office seeks election.

An attorney's recorded radio advertisement stated: "For a fee of $600, I will represent a party to a divorce that does not result in a court trial of a contested issue of fact." The attorney had the advertisement prerecorded and approved by the appropriate bar agency for broadcast. A client, who had previously agreed with her husband to an uncontested dissolution of their marriage, heard the broadcast and called on the attorney in his office. The client told the attorney that she had heard the broadcast and asked the attorney to represent her. The attorney agreed to represent the client. Because of the nature of the parties' property, the attorney spent more time on the tax aspects of the case than the attorney anticipated. The time expended by the attorney, if charged at a reasonable hourly rate, would have resulted in a fee of $2,000. After the decree was entered, the attorney billed the client for $2,000. Is the attorney subject to discipline? No, because the attorney's fee was a reasonable charge for the time expended. No, because the attorney, when the representation was accepted, did not anticipate the tax problems. Yes, unless the client pays the fee without protest. Yes, because the attorney charged a fee in excess of the advertised fee.

Answer choice D is correct. A lawyer is generally permitted to advertise his services through advertising, including radio advertisements, so long as the communications regarding such services are not false or misleading in violation of MRPC 7.1 or in violation of the rules against solicitation of clients under MRPC 7.3. Here, the attorney charged a fee in excess of what was advertised, which means that the advertisement was false and misleading, and the attorney would therefore be subject to discipline. Answer choice A is not correct because even though the $2,000 fee would have been reasonable, the attorney had advertised a fee of $600. The attorney is subject to discipline for a false and misleading advertisement. Answer choice B is not correct, as the attorney's advertisement offered representation for a flat fee of $600. The fact that the attorney charged more than that fee means that the attorney is subject to discipline for a false and misleading advertisement. Answer choice C is not correct, as it is irrelevant whether or not the client pays the fee without protest. The attorney charged a fee that is in excess of what was advertised, thus subjecting the attorney to discipline for violating the Model Rules.

A law firm learned that one of its partners had recently begun a sexual affair with the vice president of the legal department of a corporate client she was representing in a high-profile, multimillion-dollar contract defense case. When questioned about the affair by the governing committee of the firm, the attorney produced a signed waiver from the chairman of the board of the corporate client indicating that the board of directors had been advised of the potential conflict of interest by the vice president and waived any conflict. Is the attorney subject to discipline? No, because the client gave informed consent to waive any conflict. No, because there is no prohibition on representation under these circumstances in a civil action. Yes, because the case is high-profile and involves millions of dollars. Yes, because the sexual relationship is prohibited, and the conflict cannot be waived.

Answer choice D is correct. A lawyer is prohibited from engaging in sexual relations with a client, unless a consensual sexual relationship existed between the lawyer and client when the lawyer-client relationship commenced. The rule does not permit the client to waive the conflict. If the client is an organization, the lawyer for the organization is prohibited from having a sexual relationship with a constituent of the organization who supervises, directs, or regularly consults with the lawyer with regard to the organization's legal matters. The vice president of the client's legal department would be such a constituent. Answer choice A is incorrect because the Model Rules do not permit a client to waive such a conflict, even through informed consent. Answer choice B is incorrect because there is a prohibition on such conduct, regardless of whether the action is civil or criminal. Answer choice C is incorrect because it is irrelevant whether the case is high-profile and involves millions of dollars. A sexual relationship is prohibited in any case, unless the relationship predates the commencement of the lawyer-client relationship.

An attorney, upon graduation from law school and passing the bar exam, was admitted to practice in State X. Immediately following graduation, the attorney served as a law clerk for a judge on the highest court of State X. After completion of her employment as a law clerk, and without the knowledge of the judge for whom she clerked, the attorney sent a letter to all trial lawyers who practiced in the state. The letter recounted the attorney's legal experience, truthfully indicated that the attorney intended to specialize in appellate law, and offered the attorney's services to handle the appeal of cases that had been litigated by the trial lawyer. State X's rules of professional conduct for lawyers are identical in all relevant respects to those of the American Bar Association's Model Rules of Professional Conduct. Was the attorney's letter proper? No, because the attorney did not notify the judge for whom she clerked before sending the letter. No, because the attorney was not certified in appellate law. Yes, because the letter was directed only to other lawyers. Yes, because the statements in the letter were neither false nor misleading as to the attorney or her services and because the attorney is permitted to communicate an area of the law in which she specializes.

Answer choice D is correct. A lawyer may not make a false or misleading communication about the lawyer or the lawyer's services. A lawyer is permitted to communicate the areas of the law in which the lawyer specializes. MRPC 7.1, 7.2, cmt. 9. Here, the statements in the letter were truthful, including the attorney's identification of the area of the law in which she intended to specialize. Accordingly, the letter was proper under the Model Rules of Professional Conduct. Answer choice A is incorrect because, although a lawyer who currently serves as a law clerk to a judge may not negotiate employment with a party or lawyer involved in a matter in which the lawyer is participating personally and substantially without first notifying the judge (MRPC 1.12(b)), there is not a similar restriction on a former law clerk. Consequently, even assuming that some of the trial lawyers who received the attorney's letter had represented clients before the state's highest court in a matter in which the attorney had personally and substantially participated as a law clerk, there is no requirement that the attorney notify the judge for whom she used to clerk. Answer choice B is incorrect because a lawyer may state that she specializes in an area of the law without being certified. MRPC 7.2, cmt. 9. Answer choice C is incorrect because the requirement that a communication about a lawyer and her services be neither false nor misleading applies to all such communications; this requirement is not limited to communications to non-lawyers. MRPC 7.1.

An attorney brought a products liability action against the manufacturer of an appliance on behalf of an individual who was seriously injured while using the appliance. The attorney and his client agreed to a fee arrangement. Under this arrangement, the attorney's fees were contingent on the outcome of the case, and the attorney agreed to pay the litigation expenses as they arose, with the client being obligated to repay the attorney only if the action was resolved in a manner favorable to the client. Immediately before the action was to be tried, the manufacturer proposed a settlement. As part of the settlement, the manufacturer agreed to pay the attorney's legal fees and to reimburse the attorney for litigation expenses incurred in pursuing the action. In addition, as a condition of the settlement, the attorney was obligated to return this amount to the manufacturer in the event that the attorney represented another person injured by the same type of appliance in an action against the manufacturer within the next three years. After discussing the terms of the settlement agreement with the attorney, the client directed the attorney to accept it. Both parties and the attorney signed the settlement agreement. Is the attorney subject to discipline for agreeing to the terms of the settlement? No, because the decision as to whether to accept a settlement offer rests with the client. No, because the attorney-client arrangement made the payment of the attorney's fees and litigation expenses contingent on an outcome favorable to the client. Yes, because the attorney may not accept compensation for representation of a client from another person unless the client gives informed consent. Yes, because the agreement restricts the attorney's right to practice.

Answer choice D is correct. A lawyer may not make an agreement in which a restriction on the lawyer's right to practice is part of the settlement of a client controversy. MRPC 5.6(b). While the settlement agreement here does not specifically prevent the attorney from pursuing another case against the appliance manufacturer, the agreement has the effect of discouraging the attorney from doing so and thereby serves as a restriction on the attorney's right to practice. Answer choice A is incorrect because, while a lawyer must generally abide by a client's decision as to whether to settle a matter (MRPC 1.2(a)), a lawyer cannot be compelled by a client to enter into a settlement agreement that would violate another Model Rule of Professional Conduct. Answer choice B is incorrect because, while a lawyer may take a products liability case on a contingency fee basis and may advance litigation expenses, the repayment of which are contingent on the outcome of the matter (MRPC 1.5(c), 1.8(e)(1)), the settlement agreement here does not violate the contingency fee rules. Answer choice C is incorrect because, while a lawyer may not accept compensation for representation of a client from another person unless the client gives informed consent (MRPC 1.8(f)), the attorney did discuss the terms of the settlement agreement with the client, and the client did agree to its terms, which called for the manufacturer to pay the attorney's compensation.

An attorney is a sole practitioner whose practice is largely in the areas of tax, wills, estates, and trusts. The attorney learned of a new Internal Revenue Service (IRS) regulation that probably affects the trust provisions in a will she prepared for a testatrix two years ago. The attorney has not represented the testatrix since she drew the will. Is the attorney subject to discipline if she calls the testatrix and advises her of the new IRS ruling and the need to revise the will? Yes, if the attorney has any reason to believe that the testatrix has another lawyer. Yes, because the attorney would be soliciting legal business from a person who is not a current client. No, provided the attorney does not thereafter prepare a new will for the testatrix. No, because the testatrix is a former client of the attorney.

Answer choice D is correct. A lawyer may properly solicit professional employment from a person with whom the lawyer has had a prior professional relationship. MRPC 7.3(a). Answer choice A is incorrect because the testatrix is a former client of the attorney and it is immaterial whether she has another lawyer. Answer choice B is incorrect because MRPC 7.3(a) permits a lawyer to solicit business from a former client. Answer choice C is incorrect because it is permissible for the attorney to prepare a new will if the testatrix desires her to do so.

A client hires an attorney to represent her in a civil commitment proceeding initiated by the state. The client is now undergoing psychiatric evaluation to determine whether civil commitment should be ordered. The client told the attorney that she intends to commit suicide as soon as the tests are completed, and the attorney believes that the client will carry out this threat. Both suicide and attempted suicide are crimes in the state. The attorney considers informing the authorities of the client's threat, but decides not to do so. The client commits suicide the evening the tests are completed. Is the attorney subject to discipline for failing to report a future crime that ultimately resulted in the client's death? Yes, because the attorney was under an obligation to report information of a future crime that was likely to result in imminent harm to or death of a person. Yes, because as an officer of the court, the attorney was under an obligation to report knowledge of a future crime. No, because disclosure would have breached the attorney's ethical duty of confidentiality. No, because disclosure in this case was discretionary on the part of the attorney.

Answer choice D is correct. A lawyer may reveal confidential information concerning the representation of a client to the extent that the lawyer reasonably believes disclosure is necessary to prevent reasonably certain death or bodily harm. However, the lawyer is under no duty to disclose such information; it is discretionary. Thus, answer choice A is incorrect. Answer choice B is also incorrect because, while communications involving a future crime or fraud are not necessarily covered by the attorney-client privilege, attorneys are under no duty to alert the authorities of these communications. Answer choice C is incorrect because the Model Rules of Professional Conduct provide that such disclosures do not constitute breaches of the duty of confidentiality, as discussed above, and are in the discretion of the attorney.

A business owner hired an attorney solely to prepare a contract for the sale of the business. Shortly after the sale was completed, the attorney learned from an employee of the business that the owner had falsified the business's records in order to make the business, which had been losing money for several years, appear profitable. When the attorney confronted the owner about the owner's actions, the owner acknowledged the fraud but refused to take any action. The attorney contacted the buyer of the business, who had used his retirement savings to purchase the business, and revealed the owner's fraud. Is the attorney subject to discipline for this revelation to the buyer of the business? Yes, because the attorney breached the duty of confidentiality the attorney owed to the business owner. Yes, because disclosure was not necessary to prevent reasonably certain death or bodily harm. No, because the attorney did not learn of his client's fraud during the course of a litigation. No, because the owner had used the attorney's services in the sale of the business.

Answer choice D is correct. A lawyer may reveal confidential information concerning the representation of a client to the extent that the lawyer reasonably believes it is necessary to rectify substantial injury to the financial interests of another that is reasonably certain to result from the client's commission of a fraud, provided that the client has used the lawyer's services in furtherance of the fraud. Here, the business owner employed the attorney to prepare the contract of sale of the business in furtherance of the owner's fraud, and the buyer, by purchasing an unprofitable business, is reasonably certain to suffer substantial financial injury. Although the lawyer is not required to make such a revelation, the lawyer is permitted to do so. Answer choice A is incorrect. A lawyer does owe a duty of confidentiality with regard to information relating to the representation of a client, even when such information comes from a source other than the client, and even with regard to information learned after the termination of the representation, so long as the information is not generally known. However, there is an exception to this duty in this situation that permits the attorney to disclose the client's fraud to the buyer. Answer choice B is incorrect because, although there is an exception to the duty of confidentiality that permits a lawyer to disclose confidential information to the extent that the lawyer reasonably believes disclosure is necessary to prevent reasonably certain death or bodily harm, that circumstance is not the only exception to the duty of confidentiality. Answer choice C is incorrect because the duty of confidentiality is not confined to information learned during the representation of a client in a litigated matter; it applies to any situation in which the lawyer is representing a client.

An attorney, who was duly licensed and authorized to practice law only in State X, represented a local builder in defending a breach of contract action brought in State X by a corporation that was incorporated in State Y and had its headquarters in State Y. The attorney made arrangements to take the deposition of a vice president of the corporation in State Y. A paralegal from the attorney's staff accompanied the attorney to the deposition in State Y. Near the end of the deposition, the attorney became ill and left the site of the deposition. Before leaving, the attorney directed the paralegal to finish the deposition, which the paralegal did. The paralegal had worked closely with the attorney on the case and had, subject to the attorney's supervision, drafted the questions to be asked during the deposition. In State Y, the practice of law includes the taking of a deposition. Is the attorney subject to discipline for her actions? No, because the attorney was duly licensed and authorized to practice law in State X. No, because the attorney properly supervised the paralegal in preparing the questions. Yes, because the attorney was not duly licensed and authorized to practice law in State Y. Yes, because the attorney directed the paralegal to complete the deposition.

Answer choice D is correct. A lawyer must not assist a person who is not admitted to practice in a jurisdiction in the unauthorized practice of law. MRPC 5.5(a). Here, the attorney directed the paralegal to complete the deposition, which, in State Y, constitutes the practice of law. Answer choice A is incorrect. While a lawyer who is duly licensed and authorized to practice law in one jurisdiction may temporarily engage in conduct in another jurisdiction in which the lawyer is authorized to appear in connection with pending litigation, including taking depositions, a lawyer may not assist a non-lawyer in the practice of law. Answer choice B is incorrect because, while a lawyer is required to properly supervise the work of a non-lawyer on a legal matter, the lawyer may not assist a non-lawyer in the practice of law. Answer choice C is incorrect because, as noted with respect to answer choice A, a lawyer who is not duly licensed and authorized to practice law in a jurisdiction may, under the appropriate conditions, temporarily practice law in that jurisdiction.

An attorney is properly certified as a specialist in family relations law by a national organization accredited by the American Bar Association. On his website, the attorney identifies himself as a "Certified Specialist in Elder Law," but he does not identify the certifying organization. In addition, on the website, the attorney states that he does not provide representation in criminal law matters. Is the attorney subject to discipline for his website? No, because the attorney is a certified specialist. No, because the attorney may exclude areas of the law from his practice. Yes, because the attorney does not practice in patent, trademark, or admiralty law. Yes, because the attorney failed to identify the certifying organization.

Answer choice D is correct. A lawyer who is certified as a specialist in a field of law by an accredited organization that has been approved by an appropriate state authority or by the American Bar Association may state that fact, but he must also clearly identify the name of the certifying organization. MRPC 7.4(d)(2). Since the attorney here failed to identify the name of the certifying organization, the attorney has not complied with the Model Rules and is subject to discipline. Answer choice A is incorrect because, although the lawyer is entitled to state that he is a certified specialist in family relations law, he failed to also identify the certifying organization. Answer choice B is incorrect because, while the attorney may communicate those areas of the law in which he does not practice (MRPC 7.4(a)), the attorney must also comply with the Model Rule regarding communications about the attorney's status as a certified specialist. Answer choice C in incorrect because, although a lawyer who is admitted to engage in patent or trademark practice before the U.S. Patent and Trademark Office may use the designation "patent attorney," and a lawyer who engages in Admiralty practice may use the designation "Admiralty" or "Proctor in Admiralty" (MRPC 7.4(b), (c)), a lawyer is not prohibited from truthfully indicating other areas of specialty.

A judge hearing a divorce case appointed an attorney to represent the couple's 12-year-old child. Under state law, the attorney will be compensated by one or both parents at the court's discretion. The child told the attorney that she planned to falsely accuse her father of sexual abuse at the behest of her mother. Although the attorney counseled the child not to make the accusation, the child insisted that she intended to do so. As a result of the attorney's advice, the child indicated that she no longer trusted the attorney, and she wanted the court to appoint another guardian. Disgusted by the child's stated course of action, the attorney sought the judge's permission to withdraw. Both parents object, and the court refuses such permission. Must the attorney continue to serve as the child's attorney? No, because a client has the absolute right to discharge an attorney. No, because the attorney finds the child's course of action repugnant. Yes, because the child's parents object to the attorney's withdrawal. Yes, because the court denied the attorney permission to withdraw from the case.

Answer choice D is correct. A lawyer who is denied permission by a court to withdraw from representation of a client must continue to represent the client, even though there is otherwise good cause for the lawyer to withdraw. Answer choice A is incorrect because, although a client generally enjoys an absolute right to discharge a lawyer, a lawyer is required to continue representing a client when required to do so by the court. In addition, since the client is a minor, there is an issue as to whether the child has the legal capacity to discharge the attorney. Answer choice B is incorrect because, even though the attorney would otherwise be permitted to withdraw because he finds the child's course of action (i.e., falsely accusing her father of child abuse) repugnant, the attorney may not withdraw in the face of the court's refusal to grant its permission. Answer choice C is incorrect because, even though one or both of the parents will be responsible for compensating the attorney, the child is the attorney's client.

Hoping to gain favorable judicial appointments, a law firm made campaign contributions to the campaign committees of all of the state trial court judges up for re-election. After one of these judges was re-elected, the judge appointed an attorney from the firm as an uncompensated special master in a property dispute. Is it proper for the attorney to accept the appointment from the judge? No, because the attorney's firm made a monetary contribution to the judge's re-election campaign for the purpose of securing judicial appointments. No, because the firm made contributions to the campaigns of all of the state trial judges up for re-election. Yes, because the campaign contribution was made to the judge's campaign committee. Yes, because the appointment is uncompensated.

Answer choice D is correct. Although a lawyer or law firm may not accept an appointment by a judge if the lawyer or law firm makes a political contribution for the purpose of obtaining or being considered for such an appointment, an appointment to provide substantially uncompensated services is excluded from this prohibition. Answer choice A is therefore incorrect. Answer choice B is incorrect, as it is irrelevant that the firm also made contributions to the campaigns of all other state trial judges up for re-election. Acceptance of the appointment is permissible because the appointment would be uncompensated. Answer choice C is incorrect because while the judge was required to establish such a committee in order to accept contributions, the fact that the contribution was made to a campaign committee does not prevent the subsequent acceptance of an appointment from being a violation of the professional responsibility rules.

An attorney's law firm regularly represented a large company in its international business transactions. The company became involved in a contractual dispute with a foreign government. The company invoked a mandatory arbitration procedure contained in the contract. Under the arbitration clause, each party was allowed to choose a partisan arbitrator and the partisan arbitrators were to choose an additional arbitrator to sit on the panel. The company selected the attorney to be on the arbitration panel. Neither the attorney nor his law firm had represented the company in connection with the contract with the foreign government. The arbitration was completed, and the company was awarded the sum of $100,000. The company then hired the attorney to enforce the award. The attorney obtained the consent of the other arbitrators before accepting the representation. He was successful in enforcing the award. Is the attorney subject to discipline? Yes, because the attorney should not have represented the company in a matter in which the attorney had been an arbitrator. Yes, because the attorney should have declined the arbitration assignment in view of his law firm's regular representation of the company. No, because the attorney obtained the consent of the other arbitrators before accepting the representation. No, because the attorney was appointed to the arbitration panel as a partisan arbitrator.

Answer choice D is correct. Although a lawyer who served as a neutral arbitrator would be prohibited from undertaking the representation, MRPC 1.12(d) expressly permits a lawyer selected as a partisan of a party in a multi-member arbitration panel to subsequently represent that party in the same matter. Answer choice A is incorrect; under MRPC 1.12(d), an arbitrator selected as a partisan of a party in a multi-member arbitration is not prohibited from subsequently representing that party in the same matter. Answer choice B is incorrect because there is no rule that prohibits a party from choosing as a partisan arbitrator a member of the law firm that regularly represents the company in matters other than the subject of the arbitration. Answer choice C is incorrect; MRPC 1.12(d) does not require a lawyer who has served as a partisan arbitrator to obtain the consent of the other arbitrators before representing one of the parties to the arbitration. Moreover, if the representation was otherwise prohibited, as when a lawyer who served as a neutral arbitrator subsequently represents one of the parties in the same matter, the consent of the arbitrators would not cure the conflict.

An attorney represented a shopkeeper who was trying to sell his business, and was approached by an interested buyer. The attorney told the potential buyer that she believed the opportunity to purchase the business would be brief because the business was being offered at a very low price. In fact, the attorney believed that the business was priced too high, and that the shopkeeper would have difficulty selling it for that reason. Was the attorney's statement to the potential buyer proper? No, because the attorney did not believe in the truthfulness of her statement. No, because the attorney, as a negotiator, owed a duty of candor to the potential buyer. Yes, because as a negotiator, the attorney owes a duty of zealous representation to her client. Yes, because the attorney's statement did not constitute a statement of fact.

Answer choice D is correct. An attorney is not permitted to make a false statement of material fact. Statements that constitute "puffing" (i.e., opinions or judgments not made as a representation of fact) are permissible as part of a negotiation. Subjective statements, such as estimates of price, are not statements of material fact. In this case, the attorney gave an opinion about the value of the business. Her statements constituted puffing, and they were not prohibited. Answer choice A is incorrect because, while an attorney is prohibited from making a false statement of material fact to others, the attorney's statement here falls into the permissible category of "puffing." Answer choice B is incorrect because an attorney serving as a negotiator does not owe a duty of candor to potential buyers. Although an attorney is prohibited from making a false statement of a material fact, the attorney may engage in puffing. Answer choice C is incorrect because an attorney, whether serving as a negotiator or in another role, does not owe a duty of zealous representation to a client.

An attorney was appointed to represent a criminal defendant. Prior to trial, the defendant's girlfriend called the attorney without the defendant's knowledge. The girlfriend told him that she had found evidence of the crime at her house and believed the defendant had hidden it there. The defendant's girlfriend asked the attorney whether she should tell the police about the evidence. The attorney, unsure of how to proceed, told the defendant's girlfriend that he would call her back. The attorney called his former law school ethics professor, who agreed to provide confidential legal advice as to the attorney's professional responsibility obligations. The attorney then told the professor about his discussion with the defendant's girlfriend and asked the professor for advice. The attorney did not mention the call to the professor to the defendant. Was the attorney's action in revealing the discussion with the defendant's girlfriend proper? No, because the attorney's discussion with the defendant's girlfriend was protected by attorney-client privilege. No, because the attorney failed to secure the defendant's consent before consulting the professor. Yes, because the defendant's girlfriend was not the attorney's client. Yes, because the attorney was seeking to obtain confidential legal advice about his ethical obligations.

Answer choice D is correct. An attorney's duty of confidentiality does not prohibit an attorney from obtaining confidential legal advice about the attorney's responsibility to comply with the Model Rules of Professional Conduct. Because the attorney was seeking confidential legal advice about his ethical obligations from the professor, he was permitted to reveal his conversation with the defendant's girlfriend. Answer choice A is incorrect because the conversation was not protected by the attorney-client privilege. The defendant's girlfriend was not the attorney's client, nor did she make her disclosure to the attorney at the defendant's behest. Answer choice B is incorrect because the exception to the duty of confidentiality for a disclosure made for the purpose of seeking confidential legal advice regarding ethical obligations does not require the client's consent. Answer choice C is incorrect because although the defendant's girlfriend was not the attorney's client, the information she disclosed to the attorney related to the attorney's representation of the defendant and therefore was covered by the duty of confidentiality.

Alpha and Beta are members of the bar in the same community but have never practiced together. Beta is a candidate in a contested election for judicial office. Beta is opposed by Delta, another lawyer in the community. Alpha believes Beta is better qualified than Delta for the judiciary and is supporting Beta's candidacy. Which of the following would be proper for Alpha to do? Solicit public endorsements for Beta's candidacy by other attorneys in the community who know Beta, including those who are likely to appear before Beta if Beta becomes a judge. Solicit contributions to Beta's campaign committee from other attorneys in the community, including those who are likely to appear before Beta if Beta becomes a judge. Publicly oppose the candidacy of Delta. All of the above would be proper.

Answer choice D is correct. Answer choice A is proper, as a lawyer may solicit endorsements for a candidate for judicial office. Answer choice B is proper, as lawyers may contribute to a judicial office candidate's campaign through a campaign committee authorized by CJC Rule 4.4. Answer choice C is proper, as a lawyer may publicly oppose a candidate for judicial office. Therefore, answer choice D is the best choice, as it includes all of the proper courses of action.

An attorney places an advertisement in the local newspaper that includes only true information. For which, if any, of these statements is the attorney subject to discipline? My credentials are: B.A., magna cum laude, Eastern College; J.D., summa cum laude, State Law School; LL.M., Eastern Law School. My offices are open Monday through Friday from 9:00 a.m. to 5:00 p.m., but you may call my answering service twenty-four hours a day, seven days a week. I speak modern Greek fluently. The attorney is not subject to discipline for any of these statements.

Answer choice D is correct. Answer choice A will not subject the attorney to discipline, as a lawyer is generally permitted to advertise her services through a newspaper, so long as the communications regarding such services are not false or misleading or in violation of the rules against solicitation of clients. MRPC 7.2(a). These are true statements about the attorney's credentials and do not violate the rules. Answer choice B will not subject the attorney to discipline, as MRPC 7.2 permits public dissemination of information concerning office hours and a lawyer's availability through an answering service. Answer choice C will not subject the attorney to discipline, as MRPC 7.2 permits a lawyer to disseminate a true statement about her foreign language ability. Therefore, because all of the statements in the attorney's advertisement are permissible, answer choice D is the best answer.

After retirement, a former government attorney represented various clients on a pro bono basis, including an immigrant from a foreign country who sought to change her immigration status from visitor to resident alien. As part of the process, the immigrant had to return to her country of origin. Fearful of being unable to return to the United States due to the machinations of her husband, who continued to live in the foreign country, the immigrant illegally sought a United States passport, which she could use to reenter this country should a problem develop. The immigrant obtained a birth certificate from a friend who was born in the United States. The attorney accompanied the immigrant to apply for a passport and signed an affidavit as an identifying witness that the immigrant was, in fact, the friend. It is a crime to knowingly and willfully make a false statement in a passport application or supporting documents. Is the attorney subject to criminal liability for his actions in this matter? No, because a lawyer may serve as a witness in a representation that does not involve an adjudication. No, because the attorney owed a duty of confidentiality to the client. Yes, because the rules of professional responsibility apply to a pro bono representation. Yes, because the attorney made a false statement in a document supporting a passport application.

Answer choice D is correct. As with any other person, a lawyer is personally subject to criminal liability for illegal conduct. MRPC 8.4, cmt. 2. The attorney in question violated the law by falsely swearing that his client was another person in order to aid his client's attempt to obtain a passport. Answer choice A is incorrect because, although the prohibition on a lawyer acting as an advocate when the lawyer is likely to be called as a necessary witness is limited to a trial, a lawyer who acts as a witness is not absolved from the duty imposed on all witnesses to testify truthfully. Answer choice B is incorrect because, although a lawyer does owe a duty of confidentiality to a client, this duty does not sanction the lawyer lying about his client's identity. Answer choice C is incorrect because, although the Model Rules of Professional Conduct do generally apply to a pro bono representation, violation of a Model Rule does not automatically trigger criminal liability.

An attorney who practiced in a partnership with another attorney agreed to represent a pedestrian in a personal injury action against the driver of an automobile. The written representation agreement, signed by both the attorney and the client, provided that the attorney's fee would be contingent on the success of the litigation and spelled out the manner in which the fee would be calculated, the various anticipated expenses for which the client was liable, and that such expenses would be deducted before the contingency fee was calculated. The client's court costs and litigation expenses were to be paid initially by the attorney, and the client's reimbursement of the attorney for these items was also made contingent on the success of the litigation. Neither in the agreement itself nor in any communication by the attorney to the client did the attorney reveal that, pursuant to the partnership agreement, her partner was entitled to a share of any fee earned by the attorney. The attorney won a sizeable judgment for the client. In a timely manner thereafter at a conference in the attorney's office, the attorney orally explained the amount of the recovery to which the client was entitled pursuant to the contingency agreement and ensured that the client promptly received that amount. Were the attorney's actions with regard to her fee improper? No, because a contingency fee is allowed in a personal injury action as long as the fee agreement is in writing. No, because a client's payment of court costs and litigation expenses may be made contingent on the success of the litigation. Yes, because the attorney failed to disclose to the client that the attorney was obligated to share her fee with her partner. Yes, because, at the conclusion of the case, the attorney failed to provide the client with a written statement stating the outcome and showing the method of determining the client's portion of the recovery.

Answer choice D is correct. At the conclusion of a successful contingent fee matter, the lawyer must provide the client with a written statement stating the outcome of the matter and, if there is a recovery, showing the remittance to the client and the method of its determination. MRPC 1.5(c). The attorney in this case failed to provide this information in written form to the client. Answer choice A is incorrect because, while a contingency fee arrangement that is in writing is permitted in a personal injury action, a lawyer must still provide the disclosures required at the conclusion of a contingency fee matter in written format. MRPC 1.5(c). Answer choice B is incorrect, because, although a client's payment of court costs and litigation expenses may be made contingent on the success of the litigation (MRPC 1.8(e)), the required disclosures were not provided in written form. Answer choice C is incorrect because, while a lawyer is required to disclose a fee-sharing arrangement with another attorney who is not within the same firm as the lawyer, there is not a disclosure requirement with respect to a fee-sharing arrangement between lawyers in the same firm. MRPC 1.5(e).

An attorney represented a small business owner in a contract suit. The attorney advised his client that he had a 50 percent chance of winning the case and should accept any settlement offer over $20,000. The client said that he was open to settlement and would think about an acceptable amount. One week before trial, the opposing counsel told the attorney that his client would be willing to pay $25,000 to avoid trial and that the offer would remain open until the end of the day. The attorney could not reach his client to confirm whether the client wished to accept the settlement offer. At the end of the day, the attorney called the opposing counsel and accepted the offer. The attorney did not mention that he had not spoken to his client. Is the attorney subject to discipline? No, because the attorney had apparent authority to accept the settlement. No, because the acceptance of a settlement offer is a tactical decision that falls within the attorney's authority. Yes, because the attorney did not have apparent authority to accept the settlement. Yes, because the attorney accepted the settlement offer without his client's consent.

Answer choice D is correct. Generally, a lawyer must communicate all bona fide offers of settlement to the client, and the ultimate decision as to whether to accept rests with the client. An exception exists when the client has indicated in advance that a proposal will be acceptable or unacceptable. An attorney who agrees to a settlement without the client's consent and authorization is subject to discipline. Accordingly, the attorney would be subject to discipline in this case because the client had not expressed a willingness to accept a settlement of $25,000. Answer choice A is incorrect because although the attorney likely had apparent authority in this case, he did not have actual authority to accept the settlement offer on behalf of his client, and thus he would be subject to discipline. The settlement may be enforceable, however, because the attorney acted with apparent authority. Answer choice B is incorrect because acceptance of a settlement offer is not a tactical decision within the attorney's authority. Rather, the client must approve of the terms of settlement. Answer choice C is incorrect, because whether the attorney had apparent authority is irrelevant for purposes of determining whether he would be subject to discipline.

A new attorney was retained by a client to incorporate the client's business, which previously had been operated as a sole proprietorship. The new attorney noticed in the client's file copies of some correspondence from the client to a previous attorney concerning the possibility of that previous attorney incorporating the client's business. The new attorney questioned the client to make certain that any attorney-client relationship between the previous attorney and the client had been terminated. The client told the new attorney, "It certainly has been terminated. When I discussed the matter with my previous attorney six months ago, he asked for a retainer of $1,000, which I paid him. He did absolutely nothing after he got the money, even though I called him weekly, and finally, last week when I again complained, he returned the retainer. But don't say anything about it because my previous attorney is an old friend of my family." Is the new attorney subject to discipline if she does not report her knowledge of the previous attorney's conduct to the appropriate authority? Yes, if the new attorney believes the previous attorney clearly was guilty of professional misconduct. Yes, unless the new attorney believes the previous attorney does not usually neglect matters entrusted to him. No, if the client was satisfied by the previous attorney's return of the retainer. No, unless the client agrees that the new attorney may report the information.

Answer choice D is correct. Generally, a lawyer must report misconduct by another lawyer to the appropriate professional authority when the lawyer has actual knowledge of the misconduct. However, if the lawyer learns of misconduct through a communication protected by the duty of confidentiality, the lawyer is not required to report the misconduct. MRPC 8.3(c). Here, the client has specifically asked the new attorney to keep the information confidential, and she would be subject to discipline for violating the duty of confidentiality if she did report the misconduct. If the client consents to the reporting, however, the new attorney could properly report the misconduct. Answer choice A is not correct, as MRPC 8.3(c) prohibits the disclosure as a violation of the duty of confidentiality. Answer choice B is not correct, as it does not matter whether the previous attorney is or is not usually responsible as an attorney. MRPC 8.3(c) prohibits the disclosure without the client's consent. Answer choice C is not correct because it is irrelevant whether the client was satisfied by the previous attorney's return of the retainer. The new attorney would generally be required to report the misconduct, but she must not in this instance because it would violate her duty of confidentiality to the client.

An attorney entered into an agreement with a resident alien to represent her in a deportation hearing before the Immigration Court. The attorney filed a notice of entry of appearance with the court. Prior to the hearing, the attorney received a letter from the resident alien discharging the attorney. The attorney's conduct had not given the resident alien a reason to discharge the attorney. In the letter, the resident alien indicated she would secure the services of another lawyer, but did not disclose the identity of this lawyer. Acknowledging the discharge, the attorney sent a termination letter by certified mail, restricted delivery, to the resident alien at the only address that the attorney had for her. The letter, which contained the information mandated by Immigration Court rules, including the date, time, and place of the hearing, and the consequences of failing to attend the hearing, was returned to the attorney as undeliverable. Other attempts to contact the resident alien proved fruitless. Despite the resident alien's expressed intent to obtain alternate counsel, the attorney was not served with a motion for substitution of counsel by another lawyer. Consequently, the attorney appeared at the scheduled hearing, which the resident alien did not attend, and orally moved to withdraw as the attorney of record. The court refused to permit the attorney to withdraw. The attorney presented the available evidence in the light most favorable to the resident alien, but the court ordered the deportation of the resident alien in absentia. Was the attorney's representation of the resident alien at the hearing proper? No, because the resident alien had discharged the attorney. No, because the attorney had acknowledged the resident alien's discharge in a termination letter. Yes, because the resident alien's discharge of the attorney was without cause. Yes, because the Immigration Court refused to permit the attorney to withdraw from representing the resident alien.

Answer choice D is correct. Generally, a lawyer must withdraw from representation of a client if the lawyer is discharged by the client. MRPC 1.16(a). However, when ordered to continue representation by a tribunal, the lawyer must continue representation despite the existence of good cause for terminating the representation. MRPC 1.16(c). Answer choice A is incorrect because, as noted with respect to answer choice D, a lawyer who has been discharged by the client must continue to represent the client when ordered to do so by a tribunal. Answer choice B is incorrect because, although a lawyer, upon termination of representation, may be required to take steps to the extent reasonably practicable to protect a client's interests, such as warning of the consequences of failing to attend adversarial proceedings (MRPC 1.16(d)), the lawyer cannot terminate representation of a client, despite being aware that the client has discharged the lawyer, when ordered to continue by a tribunal. Answer choice C is incorrect because, while a lawyer's right to compensation for services rendered may turn on whether the discharge is with or without cause, a client has the right to discharge a lawyer at any time, even without cause. MRPC 1.16, cmt. 4.

An attorney who practices as a member of a limited liability firm has been retained by a corporation to prepare a securities offering. After working on the matter for one month, the attorney began a consensual sexual relationship with an assistant to the vice-president of the corporation. While the attorney has regularly consulted the assistant about the offering, the assistant does not have supervisory authority over the attorney. May the attorney continue representation of the corporation? Yes, because the attorney is not an employee of the corporation. Yes, because the assistant does not have supervisory authority over the attorney. No, because the sexual relationship is consensual. No, because the attorney has engaged in a sexual relationship with a corporate employee with whom the attorney regularly consults about the securities offering.

Answer choice D is correct. If the client is an organization, the lawyer for the organization is prohibited from having a sexual relationship with a constituent of the organization who supervises, directs, or regularly consults with the lawyer with regard to the organization's legal matters. MRPC 1.8(j), cmt. 19. Answer choice A is incorrect because the Rule applies whether the lawyer is inside or outside counsel for the client. Answer choice B is incorrect because a lawyer is prohibited from having a sexual relationship with a constituent of the organization who regularly consults with the lawyer on legal matters, even if that constituent does not supervise the lawyer. Answer choice C is incorrect because the prohibition applies to consensual sexual relationships, unless such a relationship predated the existence of the lawyer-client relationship.

An attorney was convinced that his client was suffering from dementia. The attorney spoke to his client's family physician and the client's only daughter to determine whether a guardian should be appointed to monitor the client's finances. These were the only discussions the attorney had ever had with either the physician or the daughter. In these discussions, the attorney revealed confidential information about a bank account maintained by the client before learning that the daughter and her mother were estranged because the daughter had stolen from her mother in the past. Was the attorney's revelation of the confidential information proper? Yes, because the attorney was trying to determine whether his client needed a guardian. Yes, because the daughter had relevant information to help determine whether the client needed a guardian. No, because the attorney should not have disclosed confidential information about a client to others without prior court approval. No, because the attorney did not first determine whether either the doctor or his client's daughter might act adversely to his client's interests.

Answer choice D is correct. Information that relates to the representation of a client with diminished capacity is protected by the duty of confidentiality. Thus, an attorney may not generally disclose such information without authorization. When taking protective action such as seeking the appointment of a guardian, however, an attorney is impliedly authorized to reveal information about the client, but only to the extent reasonably necessary to protect the client's interests. The attorney should determine whether it is likely that the person consulted will act adversely to the client's interests before discussing matters related to the client. In this case, the attorney did not act properly because he did not make such a determination before revealing confidential information. Answer choice A is incorrect because, while the attorney may disclose confidential information when taking protective action for a client with diminished capacity, he should first consider whether it is likely that the person consulted will act adversely to the client's interests. Here, the daughter, based on her past interaction with her mother, could take action that would be antithetical to her mother's financial interests. Answer choice B is incorrect because, although the daughter likely had relevant information, whether the information is relevant is not the only consideration. Answer choice C is incorrect because prior court approval is not required.

A judge has served on a trial court of general jurisdiction for almost three years. During that time, he was assigned criminal cases almost exclusively. Several months ago, however, the judge was assigned an interesting case involving a constitutional challenge to a statute recently passed by the state legislature. The statute permitted any local public school district with an overcrowding problem to purchase educational services for its students in any other public or private school within fifteen miles. Although the briefs submitted by the parties were excellent, the judge was not confident that he had a good grasp of the issues in the case. Accordingly, he took one of his more experienced colleagues on the trial court out to lunch and discussed the case with her in great detail. The colleague was far more conservative than the judge, but he agreed with her and eventually ruled in accord with her views. The case is now on appeal. Is the judge subject to discipline? Yes, because the judge sought an ex parte communication on the merits of a case pending before him. Yes, because the judge initiated a discussion with a colleague that may have influenced his judgment in the case. No, because the judge is permitted to obtain the advice of a disinterested expert on the law. No, because the judge was permitted to consult about a pending case with another judge

Answer choice D is correct. Judges are permitted to consult with other judges in the court about pending cases as long as they do not abrogate their responsibilities to decide the matters and as long as they make reasonable efforts to avoid receiving factual information not part of the record. 2007 CJC Rule 2.9(A)(3). Answer choice A is incorrect because a conversation with another judge about a pending matter is not an ex parte communication. Answer choice B is incorrect, as a judge may consult with another judge in the court about a case as long as the judge to whom the case is assigned does not abrogate his responsibilities. Answer choice C is incorrect because under the 2007 CJC Rule 2.9(A)(2), a judge may obtain written advice from a disinterested expert provided that the judge gives advance notice to the parties about details of the consultation and the opportunity to object and respond to the advice. This rule, however, is not applicable to a situation in which a judge consults with another judge in the court.

While working on a complex matter for a client, an attorney in a law firm identified a particularly difficult issue of law that could prove decisive in the dispute. The attorney had not encountered this issue before and was uncertain of its effect. The attorney called his partner and asked her for assistance. Was it proper for the attorney to consult with his partner? No, unless the total fee is not increased by the consultation. No, because the client's consent was not previously obtained. Yes, unless the attorney identified the client to his partner. Yes, because the two attorneys are partners in the same firm.

Answer choice D is correct. Lawyers in a firm may, in the course of the firm's practice, disclose to each other information relating to a client of the firm, unless the client has instructed that particular information be confined to specified lawyers. MRPC 1.6, cmt 5. Answer choice A is not correct, as the fee increase would be relevant only in the context of a lawyer dividing his fee with a lawyer outside of a firm. Here, the Model Rules would allow the attorney to disclose the information to a lawyer in Alpha's own firm, and the fee issue is irrelevant. Answer choice B is not correct, as the client's consent is not needed to share information within a firm. Answer choice C is not correct, as the attorney is permitted to consult with a partner in the same firm and can reveal the name of the client.

An attorney had been employed as an assistant prosecutor in the district attorney's office during the time that an investigation of a defendant was being conducted by that office. The attorney took no part in the investigation and had no knowledge of the facts other than those disclosed in the press. Two months ago, the attorney left the district attorney's office and formed a partnership with a second attorney. Last week, the defendant was indicted for offenses allegedly disclosed by the prior investigation. The defendant asked the first attorney to represent him. The first attorney declined to do so, but suggested the second attorney. Is the second attorney subject to discipline if she represents the defendant? Yes, because the first attorney was employed in the district attorney's office while the investigation of the defendant was being conducted. Yes, unless the district attorney's office is promptly notified and consents to the representation. No, unless the first attorney participates in the representation or shares in the fee. No, because the first attorney had no responsibility for or knowledge of the facts of the investigation of the defendant.

Answer choice D is correct. MRPC 1.11(a) prohibits a former government lawyer from representing a client in a matter in which the lawyer participated personally and substantially as a government lawyer, unless the appropriate government agency gives its informed consent, confirmed in writing, to the representation. MRPC 1.11(a). Only if the former government lawyer is disqualified from representation will her firm potentially be disqualified from representation. Here, the first attorney did not participate personally and substantially in the investigation. Thus, the second attorney would be permitted to represent the defendant. Answer choice A is not correct, as mere employment by the district attorney's office is not enough to disqualify the first attorney and therefore the second attorney. The first attorney did not personally and substantially participate in the investigation of the defendant. Answer choice B is not correct, as no consent is required since the first attorney did not personally and substantially participate in the defendant's investigation. Answer choice C is not correct, as the first attorney would not need to be screened from the representation, because she did not personally and substantially participate in the investigation of the defendant.

A woman has asked an attorney to represent her in obtaining compensation for a tract of land that is being condemned by the state department of transportation to build a new highway. Two years ago, the attorney had been employed by the department and had been assigned to search title on several tracts of land, including the one owned by the woman. The attorney remembers a department engineer had drafted a confidential memorandum advising against running a new highway across the woman's land because of a potential adverse environmental impact. Because of this information, the attorney believes it is possible to prevent the condemnation of the woman's land or to increase the settlement amount. What is the proper action for the attorney to take? Represent the woman on the issue of damages only and not disclose the information that might prevent the condemnation. Represent the woman and attempt to prevent the condemnation by using the information about the adverse environmental impact. Refuse to represent the woman but disclose to her the information about the adverse environmental impact. Refuse to represent the woman and not disclose the information about the adverse environmental impact.

Answer choice D is correct. MRPC 1.11(a)(1) provides that a lawyer who was formerly employed by a government agency may not later represent a client "in connection with a matter in which the lawyer participated personally and substantially" as a public employee, unless the government agency consents. Having participated personally and substantially in an early stage of the condemnation matter by conducting a title search on the woman's property, the attorney may not now represent the woman in opposing or seeking additional damages for the condemnation. Further, the attorney may not disclose the information learned while working at the department because, under MRPC 1.11(a)(2) and MRPC 1.9(c), a lawyer who was formerly employed by the government must not reveal information relating to a former representation. Answer choices A and B are incorrect, as having participated personally and substantially in an early stage of the condemnation matter by conducting a title search on the woman's property, the attorney may not now represent the woman in opposing or seeking additional damages for the condemnation. Answer choice C is incorrect, as the attorney may not disclose information learned while working at the department because, under MRPC 1.11(a)(2) and MRPC 1.9(c), a lawyer who was formerly employed by the government must not reveal information relating to a former representation.

A client retained an attorney to appeal his criminal conviction and to seek bail pending appeal. The agreed-upon fee for the appearance on the bail hearing was $100 per hour. The attorney received $1,600 from the client, of which $600 was a deposit to secure the attorney's fee and $1,000 was for bail costs in the event that bail was obtained. The attorney maintained two office bank accounts: a fee account, in which all fees collected from clients were deposited and from which all office expenses were paid, and a clients' trust account. The attorney deposited the $1,600 in the clients' trust account the week before the bail hearing. She expended six hours of her time preparing for and appearing at the hearing. The effort to obtain bail was unsuccessful. Dissatisfied, the client immediately demanded return of the $1,600. What should the attorney do with the $1,600? Transfer the $1,600 to the fee account. Transfer the $600 to the fee account and leave $1,000 in the clients' trust account until the attorney's fee for the final appeal is determined. Transfer $600 to the fee account and send the client a $1,000 check on the clients' trust account. Send the client a $1,000 check and leave $600 in the clients' trust account until the matter is resolved with the client.

Answer choice D is correct. MRPC 1.15(e) requires that when the lawyer has a dispute with a client over funds in the lawyer's possession, the lawyer must deliver to the client the funds that undisputedly belong to the client and hold the remainder in the clients' trust account until the dispute is resolved. Therefore, the attorney is required to hold in the clients' trust account the amount of disputed legal fees ($600) and return the remaining $1,000 to the client. Answer choice A is incorrect. When legal fees are paid in advance, they must be kept in the clients' trust account until earned. When the representation ends, the lawyer must return the unearned portion (here, $1,000) to the client. The lawyer may withdraw fees as earned except when, as in this case, they are disputed, in which case they must be kept in the clients' trust account until the dispute is resolved. Answer choice B is incorrect because both steps are wrong. The $600, which represents the amount in dispute, must be kept in the clients' trust account until the dispute is resolved. The remaining $1,000, which represents unearned fees, belongs to the client, and it must therefore be returned to the client. Answer choice C is incorrect. Although the lawyer is right to disburse $1,000 from the clients' trust account to the client (since that amount represents unearned fees and therefore undisputedly belongs to the client), the $600 that is in dispute must be kept in the clients' trust account until the dispute is resolved.

A defendant was on trial for the murder of a victim killed during a barroom brawl. In the course of closing arguments to the jury, the prosecutor said, "The defendant's whole defense is based on the testimony of a witness who said that the victim attacked the defendant with a knife before the defendant struck him. No other witness testified to such an attack by the victim. I don't believe the witness was telling the truth, and I don't think you believe him either." Was the prosecutor's statement proper? Yes, if the prosecutor accurately stated the testimony in the case. Yes, if the prosecutor, in fact, believed the witness was lying. No, because the prosecutor alluded to the beliefs of the jurors. No, because the prosecutor asserted his personal opinion about the witness's credibility.

Answer choice D is correct. MRPC 3.4 provides that a lawyer shall not state a personal opinion regarding the credibility of a witness. Answer choice A is not correct, as even if the statement of the testimony is true, the prosecutor is improperly stating a personal opinion regarding the credibility of a witness. Answer choice B is not correct, as the Model Rules prohibit a lawyer from making a personal statement regarding the credibility of a witness. Answer choice C is not correct, as the statement regarding the jurors' beliefs is irrelevant. The statement is not proper because it is the expression of the prosecutor's personal opinion regarding the credibility of the witness.

An attorney represents a plaintiff in a personal injury action. An eyewitness to the accident lives about 500 miles distant from the city where the case will be tried. The attorney interviewed the witness and determined that the witness's testimony would be favorable for the client. The witness asked the attorney to pay him, in addition to the statutory witness fees while attending the trial, certain other expenses. If the attorney agrees to pay the witness the following, for which, if any, is the attorney subject to discipline? Reimbursement for lodging expenses while attending the trial. Reimbursement for lost wages while present at the trial. Reimbursement for personal meals while attending the trial. An amount equal to 5% of any recovery in the matter.

Answer choice D is correct. Paying a witness a portion of the amount recovered by the plaintiff would constitute an inducement to a witness that is prohibited by law, pursuant to MRPC 3.4(b). Answer choices A, B, and C all include reasonable expenses that a lawyer may reimburse to a non-expert witness in return for testifying.

An attorney represents a well-known contractor before a state administrative agency. The agency has ordered the client to show cause why the client's license as a contractor should not be revoked for violation of agency regulations. In a newspaper interview prior to the administrative hearing, the attorney made a variety of statements. Which of these statements, if any, would be improper? "My client denies the charge made by the agency that she engaged in conduct constituting grounds for revocation of her license as a contractor." "The next step in the administrative process is the administrative hearing; if the agency is successful, we will appeal, and the agency still cannot revoke my client's license until a court affirms the finding for the agency." "My client needs witnesses who are aware of the incidents that are the subject of the hearing." All of these statements are proper

Answer choice D is correct. Statement A is proper, as a public statement of the defense involved is permissible under MRPC 3.6(b)(1). Statement B is proper, as a statement of the schedule of the litigation is permissible under MRPC 3.6(b)(4). Statement C is also proper, as a request for assistance in obtaining evidence and information necessary to the case is permissible under MRPC 3.6(b)(5). Therefore, answer choice D is correct, as it includes all three statements.

An attorney decided to obtain a master's degree in taxation, but lacked the funds required for tuition and expenses. The attorney consulted one of his clients, a wealthy banker, for advice about obtaining a loan. To the attorney's surprise, the client offered the attorney a personal loan of $10,000. The attorney told the client that he would prepare the required note without charge. Without further consultation with the client, the attorney prepared and signed a promissory note bearing interest at the current bank rate. The note provided for repayment in the form of legal services to be rendered by the attorney to the client without charge until the value of the attorney's services equaled the principal and interest due. The note further provided that if the client died before the note was fully repaid, any remaining principal and interest would be forgiven as a gift. The attorney mailed the executed note to the client with a transmittal letter encouraging the client to look it over and call with any questions. The client accepted the note and sent the attorney a personal check for $10,000, which the attorney used to obtain his master's degree. A month after the degree was awarded, the client was killed in a car accident. The attorney had not rendered any legal services to the client from the date of the note's execution to the date of the client's death. Thereafter, in an action brought by the client's estate to recover on the note, the court ruled that the note was discharged as a gift. Was the attorney's conduct proper? Yes, because the client, without having been requested by the attorney to do so, voluntarily made the loan. Yes, because the court ruled that the note had been discharged as a gift. No, because a lawyer may never accept a loan from a client. No, because the attorney did not comply with the requirements for entering into a business transaction with a client.

Answer choice D is correct. The attorney violated MRPC 1.8(a), which provides that a lawyer may not enter into a business transaction with a client unless the transaction is fair and reasonable, and unless the lawyer complies with procedural requirements, such as giving written advice about the desirability of seeking independent legal advice and obtaining informed consent. Answer choice A is incorrect because the lawyer failed to comply with the procedural requirements of MRPC 1.8(a) and was required to do so even though the client initiated the transaction by offering to make the loan. Answer choice B is incorrect because, at the time of the transaction, the client was making a loan, not giving a gift, and therefore the lawyer was obligated, but failed, to comply with the procedural requirements of MRPC 1.8(a) regarding business transactions with clients. Had this been a gift, not a loan, the lawyer's conduct would have been improper under MRPC 1.8(c), because a lawyer may not prepare an instrument on behalf of a client that gives the lawyer a substantial gift, unless the lawyer and client are relatives. Answer choice C is incorrect; a lawyer may accept a loan from a client as long as the lawyer complies with the requirements of MRPC 1.8(a). In this case, the attorney violated MRPC 1.8(a).

An attorney represented a client in an action against the client's former partner to recover damages for breach of contract. During the representation, the client presented the attorney with incontrovertible proof that the former partner had committed perjury in a prior action which was resolved in the partner's favor. Neither the attorney nor the client was involved in any way in the prior action. The attorney believes that it would be detrimental to the client's best interests to reveal the perjury because of the implication that might be drawn from the former close personal and business relationship between the client and the former partner. Would it be proper for the attorney to fail to disclose the perjury to the tribunal? No, because the information is unprivileged. No, because the attorney has knowledge that the former partner perpetrated a fraud on the tribunal. Yes, because neither the client nor the attorney was involved in the prior action. Yes, because the attorney believes that the disclosure would be detrimental to the client's best interests.

Answer choice D is correct. The information about perjury committed by the client's former partner is "information relating to the representation," which generally may not be disclosed without client consent. Confidential information may be disclosed under MRPC 1.6(a) when "the disclosure is impliedly authorized in order to carry out the representation," but there would not be implied authorization when, as here, the disclosure would be likely to prejudice the client, not advance the client's objectives. Answer choice A is incorrect because the lawyer's duty to protect client information under MRPC 1.6(a) covers more than just privileged information. It covers all "information relating to the representation of a client." Answer choice B is incorrect; a lawyer has a duty under MRPC 3.3(b) to take remedial measures when the lawyer knows that a person is engaged in criminal conduct related to a proceeding in which the lawyer represents a client. But that duty does not apply here, because the attorney did not represent the client in the proceeding in which the client's former partner committed perjury. Therefore, the duty to maintain confidentiality applies to this information. Answer choice C is incorrect; a lawyer has a duty under MRPC 3.3(b) to take remedial measures when the lawyer knows that a person is engaged in criminal conduct related to a proceeding in which the lawyer represents a client. But the duty continues only to the end of that proceeding. Here, the proceeding in which the former partner committed perjury is over. Therefore, even if the lawyer had represented the client in that proceeding, the lawyer would have no duty at this time to disclose the former partner's perjury.

An attorney represents a client in a personal injury case resulting from a car accident in which the client sustained injuries. The information the attorney has collected while preparing the case includes a copy of the police report—a public record—completed at the scene of the accident. The client is also involved in a workers' compensation case for unrelated injuries, and has retained a separate lawyer for that case. The workers' compensation attorney contacts the personal injury attorney and asks for a copy of the police report, to save him the trouble of contacting the court and obtaining a copy himself. Would providing a copy of the police report to the workers' compensation attorney subject the personal injury attorney to discipline? No, because the workers' compensation attorney also has an attorney-client relationship with the client. No, because the police report, a public document, is not covered by the attorney-client privilege. Yes, because the police report constitutes work product. Yes, unless the personal injury attorney receives informed consent to do so from the client.

Answer choice D is correct. The police report is subject to the lawyer's professional obligation of confidentiality. Under the Model Rules of Professional Conduct, a lawyer is prohibited from disclosing information relating to the representation of a client, unless the client gives informed consent, the disclosure is impliedly authorized to carry out the representation, or one of several non-applicable exceptions applies. Answer choice A is incorrect because the two cases are unrelated; the client's separate relationship with the workers' compensation attorney has no bearing on the personal injury case. While answer choice B includes a correct statement that the police report is not covered by the attorney-client privilege, this choice is incorrect because the lawyer's professional duty of confidentiality is broader than the attorney-client privilege, and covers all information relating to the representation of a client, even non-privileged information. Answer choice C is incorrect because the report does not constitute work product.

Attorney Alpha filed a complaint on behalf of a client against a corporation, alleging that the corporation had breached a valid oral contract, entered into on the corporation's behalf by the president and chief executive officer of the company, to sell the client certain merchandise for a specified price. Attorney Beta, representing the company, has filed an answer denying the contract and asserting the Statute of Frauds as a defense. Attorney Beta has given notice to Alpha that he will take the deposition of the president on the grounds that the president will be out of the country on the date the case is set for trial. The president is not a shareholder of the corporation. Alpha would like to interview the president, prior to the taking of the deposition, to better prepare her cross-examination. Is Alpha subject to discipline if she interviews the president without Beta's knowledge and consent? No, unless the president will be personally liable to the corporation for damages in the event judgment is rendered against it. No, because the president allegedly entered into the contract on behalf of the corporation. Yes, because the president is being called as an adverse witness. Yes, because the president is the president of the corporation.

Answer choice D is correct. The president of the corporation is considered a client represented by Attorney Beta. In the case of a represented organization, a lawyer is prohibited from communicating with a constituent of the organization who supervises, directs, or regularly consults with the organization's lawyer concerning the matter or has authority to obligate the organization with respect to the matter, or whose act or omission in connection with the matter may be imputed to the organization for purposes of civil or criminal liability. MRPC 4.2, cmt 7. Under MRPC 4.2, Attorney Alpha is not permitted to communicate about the subject of the representation with a person the lawyer knows to be represented by another lawyer in the matter, without the consent of the other lawyer or authorization by law or a court order. Answer choice A is not correct, as it does not matter whether the president will be personally liable to the corporation for damages. What controls is that the president is considered a client of Attorney Beta under MRPC 4.2. Answer choice B is not correct, as the fact that the president may have entered into the contract on behalf of the corporation does not control the issue. The president is considered a client of Attorney Beta with regard to the matter, and Attorney Alpha cannot interview the president without Beta's consent. Answer choice C is not correct because even if the president is called as an adverse witness, the president is considered Beta's client under MRPC 4.2 and cannot be interviewed by Attorney Alpha without Beta's consent.

The judicial district in which a judge sits has a rule that allows litigants two postponements as a matter of right. After that, a litigant who moves for a postponement must convince the presiding judge that a postponement is appropriate. The judge routinely grants additional postponements because, in her view, "What harm is done if one of the litigants wants a postponement? The worst that can happen is that the parties have more time to negotiate and thus are more likely to settle." Are the judge's actions proper? Yes, because the judge is exercising her judicial discretion. Yes, because a party objecting to a postponement can seek appellate review. No, because judges have no official obligation to encourage private settlements. No, because the judge should expedite the determination of matters before her.

Answer choice D is correct. Under CJC Rule 2.5(A), a judge is required to perform judicial and administrative duties diligently, which includes monitoring and supervising cases in ways that reduce or eliminate dilatory practices, avoidable delays, and unnecessary costs. CJC Rule 2.5, cmt 4. A blanket policy granting additional postponements conflicts with the judge's duty to expedite the determination of matters before her. Answer choice A is not correct because the judge's discretion is circumscribed by a duty to expedite the determination of matters before her. Answer choice B is not correct, as it is irrelevant to the determination of whether the judge's actions are proper under the Code of Judicial Conduct that a party can seek appellate review. What controls is the judge's ethical duty as a judge to expedite the determination of matters before her. Answer choice C is not correct, as it is not improper for a judge to attempt to encourage private settlements. The difficulty here is that the judge has imposed a blanket policy of granting additional postponements that may lead to avoidable delays and unnecessary costs, which conflicts with the judge's duty under CJC Rule 2.5(A).

A judge needed to obtain a loan to be secured by a second mortgage on his house. A bank offered him a loan at a very favorable interest rate. The vice president at the bank told the judge: "Frankly, we normally don't give such a large loan when the security is a second mortgage, and your interest rate will be 2% less than we charge our other customers. But we know that your salary is inadequate, and we are giving you special consideration." Is it proper for the judge to accept the loan? Yes, if the judge does not act in any case involving the bank. Yes, if the bank is not likely to be involved in litigation in the court on which the judge sits. No, unless the same terms are available to all judges in the state. No, because the amount of the loan and the interest rate were not available to persons who were not judges.

Answer choice D is correct. Under CJC Rule 3.13, a judge may accept commercial or financial opportunities and benefits, including loans from lending institutions in their regular course of business, only if the same opportunities and benefits or loans are made available on the same terms to similarly situated persons who are not judges. Here, the vice president of the bank indicated that the deal was a special deal offered to the judge. It is therefore improper for the judge to accept the loan. Answer choice A is not correct, as even if the judge does not act in any cases involving the bank, CJC Rule 3.13 prohibits acceptance of the loan. Answer choice B is not correct, as regardless of whether the bank is likely to be involved in litigation in the court on which the judge sits, CJC Rule 3.13 prohibits acceptance of the loan. Answer choice C is not correct, as CJC Rule 3.13 does not merely require that the terms of the loan be available to all judges in the state, but that the terms are available to similarly situated persons who are not judges.

Four years ago, a judge in a state court of general jurisdiction heard a civil case in which the plaintiff prevailed and secured a judgment for $50,000, which was sustained on appeal. Since then, the judge has resigned from the bench and returned to private practice. The defendant has filed suit to enjoin enforcement of the judgment on the grounds of extrinsic fraud in its procurement. The plaintiff has now asked the former judge to represent her in defending the suit to enjoin enforcement. Is it proper for the former judge to accept the representation of the plaintiff in this matter? Yes, because the former judge would be upholding the decision of the court. Yes, if the plaintiff's conduct of the first trial will not be in issue. No, unless the former judge believes the present suit is brought in bad faith. No, because the former judge had acted in a judicial capacity on the merits of the original case.

Answer choice D is correct. Under MRPC 1.12(a), a lawyer may not represent anyone in connection with a matter in which the lawyer participated personally and substantially as a judge, unless all parties to the proceeding give informed, written consent. MRPC 1.12(a). Here, the former judge acted as a judge in the original case on the merits. Accordingly, it would not be proper for him to now accept representation of the plaintiff. Answer choice A is not correct, as MRPC 1.12(a) prohibits the former judge from representing the plaintiff because the former judge previously participated personally and substantially as a judge in the original case and all parties have not given informed, written consent to the representation. Answer choice B is not correct for the same reason. Answer choice C is not correct, because even if the present suit was brought in bad faith, it would not permit the former judge to avoid the prohibition of MRPC 1.12(a).

A client was an experienced oil and gas developer. The client asked an attorney for representation in a suit to establish the client's ownership of certain oil and gas royalties. The client did not have available the necessary funds to pay the attorney's reasonable hourly rate for undertaking the case. The client proposed instead to pay the attorney an amount in cash equal to 20% of the value of the proceeds received from the first-year royalties the client might recover as a result of the suit. The attorney accepted the proposal and took the case. Is the attorney subject to discipline? Yes, because the agreement gave the attorney a proprietary interest in the client's cause of action. Yes, unless the fee the attorney receives does not exceed that which the attorney would have received by charging a reasonable hourly rate. No, because the client rather than the attorney proposed the fee arrangement. No, because the attorney may contract with the client for a reasonable contingent fee.

Answer choice D is correct. Under MRPC 1.8(i), a lawyer shall not acquire a proprietary interest in the cause of action or subject matter of litigation the lawyer is conducting for a client, except that the lawyer may contract with a client for a reasonable contingent fee in a civil case. A contingent fee must be reasonable, and it must be reasonable to charge any form of contingent fee under the circumstances. Here, 20% of the first year's proceeds is reasonable, and the fact that the client cannot pay the attorney's hourly fee makes it reasonable to charge a contingent fee. Answer choice A is not correct, as MRPC 1.8(i) does allow a proprietary interest as part of a reasonable contingent fee arrangement in a civil case. Answer choice B is not correct, as a contingent fee must be reasonable but may exceed what a lawyer would have received by charging a reasonable hourly rate. What is reasonable depends on the circumstances. Answer choice C is not correct, as a client may propose a fee arrangement.

An attorney represented a plaintiff in the plaintiff's action for defamation against a defendant. After the defendant's lawyer had filed and served an answer, the attorney, at the plaintiff's direction, hired a licensed private investigator and instructed him to attempt to interview the defendant without revealing his employment. The investigator succeeded in interviewing the defendant privately and obtained an admission from the defendant that the statements she had made were based solely on unsubstantiated gossip. Is the attorney subject to discipline for obtaining the statement from the defendant in this matter? No, because the attorney was following the plaintiff's instructions. No, because the statement obtained was evidence that the defendant's allegations were unfounded. Yes, because the attorney should have interviewed the defendant personally. Yes, because the attorney instructed the investigator to interview the defendant.

Answer choice D is correct. Under MRPC 4.2, the attorney or a representative of the attorney is prohibited from talking to a party represented by a lawyer about the subject matter of the representation. Answer choice A is not correct, as the interview of the defendant violated MRPC 4.2 even if it was at the instruction of the plaintiff. Answer choice B is not correct, as the mere fact that the interview took place violated MRPC 4.2. Answer choice C is not correct, as MRPC 4.2 is violated by the attorney or a representative of the attorney interviewing a party represented by a lawyer.

An attorney placed an associate recently admitted to the bar in complete charge of the work of the paralegals in the attorney's office. That work consisted of searching titles to real property, an area in which the associate had no familiarity. The attorney instructed the associate to review the searches prepared by the paralegals, and thereafter to sign the attorney's name to the required certifications of title if the associate was satisfied that the search accurately reflected the condition of the title. This arrangement enabled the attorney to lower office operating expenses. The attorney told the associate that she should resolve any legal questions that might arise and not to bother the attorney because he was too busy handling major litigation. Is it proper for the attorney to assign the associate this responsibility? Yes, if the paralegals are experienced in searching titles. Yes, because the attorney is ultimately liable for the accuracy of the title searches. No, unless it enables the attorney to charge lower fees for title certification. No, because the attorney is not adequately supervising the work of the associate.

Answer choice D is correct. Under MRPC 5.1(b), a lawyer having direct supervisory authority over another lawyer must make reasonable efforts to ensure that the other lawyer conforms to the Model Rules. The attorney supervises the associate and is requiring the associate to oversee work about which she has no familiarity and is not competent to oversee. The attorney is required to adequately supervise the associate's work and is not doing so. Answer choice A is not correct, as the fact that the paralegals are experienced in searching titles does not make up for the fact that the associate is inexperienced and is being told by the attorney to handle the title review work on her own. Answer choice B is not correct because although the attorney will be ultimately liable for the accuracy of the title searches, this fact does not absolve the attorney of the duty to properly supervise the associate in carrying out the title responsibilities the attorney has assigned. Answer choice C is not correct, as the ability to charge lower fees to clients does not excuse the attorney from the duty to adequately supervise the associate and the other employees in their work.

An in-house attorney for a large corporation was asked by the president of the company to investigate an employee's claim that a division of the company was illegally bribing foreign officials in order to gain significant contracts for the company. During the course of the investigation, the attorney discovered evidence that it had been the president of the company who had ordered certain employees to bribe the foreign officials and that the officials had provided substantial financial benefits in return. The bribes violated both foreign and U.S. law and could potentially result in significant criminal and civil liability for the company. Believing she was acting in the best interests of the corporation, the attorney immediately reported the evidence she had discovered to the corporation's board of directors, which promptly suspended the president, pending a full investigation by the appropriate foreign and U.S. authorities. Were the attorney's actions in reporting the misconduct proper? No, because the attorney was required to report the misconduct to the appropriate foreign and U.S. authorities. No, because the president of the company was the attorney's client, and reporting the evidence constituted a breach of the ethical duty of confidentiality. Yes, but only because the bribes could result in criminal liability for the corporation. Yes, because the board of directors was a higher authority within the corporation than the president, and the attorney believed she was acting in the best interests of the corporation.

Answer choice D is correct. Upon learning of an action by a person associated with the corporation that will likely cause substantial injury to the organization, a lawyer for the corporation must proceed as is reasonably necessary in the best interests of the organization. Unless the lawyer reasonably believes that it is not necessary in the best interests of the organization to do so, the lawyer must refer the matter to higher authority in the organization, including, if warranted by the circumstances, to the highest authority that can act on behalf of the organization. The in-house attorney believed she was acting in the best interests of the corporation by reporting the misconduct, and she reported it to the highest authority that could properly act on the matter, the board of directors. Answer choice A is incorrect, as a lawyer is not required to report the matter to outside authorities under these circumstances. Answer choice B is incorrect, because the organization, rather than the president, was the attorney's client, and the attorney was required to report the misconduct to a higher authority in the organization. Answer choice C is incorrect, as even if there were only civil penalties at stake, that fact could cause substantial injury to the corporation. Thus, it is not merely because there was potential criminal liability at issue that the attorney was required to report the evidence of misconduct.

An attorney successfully represented a plaintiff in a personal injury action to recover damages incurred by the plaintiff. The defendant paid the amount of the judgment by check to the attorney. After the verdict, but before the attorney received the check, the client's health insurance company contacted the attorney. The company asserted that, by contract with the client, the company had a right to be reimbursed for payments it made pursuant to the insurance policy to medical care providers who treated the client for injuries received as a consequence of the accident. The amount for which the company sought reimbursement was approximately 25 percent of the amount of the verdict. The attorney discussed the company's claim with her client, who insisted that the attorney pay the full amount of the settlement less the amount due to the attorney to the client. Aware that, under her contract with the client, she was entitled to her fee as well as reimbursement for litigation expenses and court cost that she had advanced to the client, all of which were reasonable, the attorney, upon receipt of the check, cashed it and retained her fee as well as an amount equal to the litigation expenses and court costs. The amount retained by the attorney was equal to approximately 35 percent of the verdict. Also aware that the applicable law was clear that the company was entitled to be reimbursed, but unsettled as to the amount of the reimbursement, and aware that the attorney had a duty to protect the company's claim from wrongful interference by the client, the attorney placed the remainder of the recovery in an escrow account. She promptly and properly informed both the company and client of her actions and provided a full accounting to each. Subsequently, the attorney filed an interpleader action with respect to the funds in escrow and named both the client and the company as claimants. Is the attorney subject to discipline for her actions? No, because the amount of the recovery retained by the attorney was reasonable. No, because the attorney did not commingle the funds to which she was entitled with the funds due to the client and the company. Yes, because the attorney failed to comply with her client's instructions regarding dispersal of the recovery. Yes, because the attorney failed to pay the funds that were not in dispute to the client.

Answer choice D is correct. Upon receiving funds that are due to a client or third party, a lawyer must promptly notify the client or third party and deliver to the client or third party any funds that the client or third party is entitled to receive. MRPC 1.15(d). While the attorney in question did notify both the client and the company of the receipt of the defendant's check, the attorney failed to deliver to the client the uncontested portion of the recovery, which was approximately 40 percent of the recovery (total recovery less 35 percent to the attorney and less 25 percent disputed by the company). Answer choice A is incorrect because, even though the attorney's fee, litigation expenses, and court costs were reasonable in accordance with MRPC 1.5(a), the attorney, as noted with respect to answer choice A, failed to properly disperse the undisputed portion of the recovery to the client. Answer choice B is incorrect because, even though the attorney did not commingle funds to which she was entitled with funds due to the client or the company in accordance with MRPC 1.15(a), the attorney, as noted with respect to answer choice A, failed to properly disperse the undisputed portion of the recovery to the client. Answer choice C is incorrect because, while a lawyer generally must comply with a client's instructions regarding property that belongs to the client, a lawyer cannot ignore legal obligations to third parties to which the lawyer is subject. MRPC 1.15, cmt. 4.

An attorney learns from a third party that a longtime personal friend whom the attorney knew was in financial difficulty has filed a personal bankruptcy petition pro se. The attorney calls his friend and offers his legal services to the friend at a reduced rate. The friend rejects the attorney's offer and requests that the attorney not contact him again about the matter. Subsequently, the attorney and the friend both attend a local sporting event. Noticing the friend, the attorney initiates a conversation. During the course of the conversation, the attorney inquires about the status of the friend's bankruptcy matter and renews his offer to provide legal services. Reconsidering the issue, the friend accepts the attorney's offer. Are the attorney's actions proper? Yes, because the prospective client was a close personal friend. Yes, because the friend accepted the attorney's offer of representation. No, because the attorney engaged in in-person solicitation of his friend's legal business. No, because the attorney failed to honor his friend's request not to contact the friend again about legal representation.

Answer choice D is correct. While a lawyer may not generally solicit business through in-person contact, a lawyer may do so when the individual is a close personal friend of the lawyer. However, a lawyer may not contact a prospective client when the prospective client has made known to the lawyer a desire not to be solicited by the lawyer. MRPC 7.3(a)(2), (b)(1). Here, the attorney solicited his friend's legal business after being requested by the friend not to do so. Answer choice A is incorrect because, while, as noted with respect to answer choice D, a lawyer may solicit legal business from a close personal friend for pecuniary gain through in-person contact, the attorney may not continue to do so after the friend requests otherwise. Answer choice B is incorrect because whether the friend eventually agreed to the attorney's representation is irrelevant to the issue of whether the attorney's solicitation of the friend's legal business was proper. Answer choice C is incorrect because, as noted with respect to answer choice D, a lawyer may engage in in-person solicitation of a close personal friend's legal business.

In representing a client in litigation involving a boundary dispute, an attorney, after consultation with and approval by the client, employed a surveyor. The attorney, who had used and compensated the surveyor in previous, similar situations for other clients, described the purpose of the survey and the party she represented to the surveyor. The retainer agreement between the attorney and the client specified that the client was responsible for payment of all litigation expenses. The surveyor performed a survey of the disputed boundary and submitted an invoice to the attorney for the agreed-upon amount. Prior to payment of this invoice, the client, in direct conversation with his neighbor, reached an agreement over the boundary between their properties. The client paid the attorney her fee as agreed upon in the retainer agreement but refused to pay the attorney for the cost of the survey. Is the attorney likely subject to civil liability to the surveyor for the unpaid invoice? No, because the client was responsible for the payment of all litigation expenses. No, because the client was consulted about and approved the hiring of the surveyor. Yes, because an attorney is primarily responsible for litigation expenses. Yes, because of the nature of the services rendered by the surveyor.

Answer choice D is correct. While generally an agent who enters into a contact with a third party on behalf of a disclosed principal is not liable on the contract, an attorney who acts on behalf of a client can be liable for the payment of goods and services typically used by lawyers, such as a stenographer, appraiser, or surveyor, because such persons are likely to reasonably rely on the attorney's, rather than the client's, credit. Answer choice A is incorrect because, although the client was contractually obligated to pay litigation expenses (as is generally required by the Model Rules of Professional Conduct except in the case of a contingency fee arrangement or an indigent client), the attorney is liable to the surveyor; note, though, that the attorney is entitled to seek reimbursement from the client for the payment of the surveyor's bill. Answer choice B is incorrect because, although the client was consulted about and approved the hiring of the surveyor, the attorney is liable to the surveyor, even though the attorney is entitled to seek reimbursement from the client for payment of the surveyor's bill. Answer choice C is incorrect because the Model Rules of Professional Conduct generally require the client, not the attorney, to bear responsibility for the payment of litigation expenses.

An attorney was contacted by a family member of an individual who was near death to prepare a will. After speaking briefly with the individual, the attorney drafted a document and oversaw its execution. After the death of the testator, the validity of the will was successfully challenged. The attorney was then sued for malpractice based on her alleged negligence. Which of the following standards is applied to determine whether the attorney's conduct complied with the duty of care? Did her conduct violate the Model Rules of Professional Conduct as adopted in the applicable jurisdiction? Was her conduct at least average as judged by lawyers with similar skill and knowledge? Did she act in good faith in light of the emergency situation? Did she act with the competence and diligence normally exercised by lawyers in similar circumstances?

vAnswer choice D is correct. For a lawyer, as a member of a profession, the standard of care is defined by normal professional practice. Answer choice A is incorrect because, while violation of the Model Rules of Professional Conduct may serve as evidence of negligence, it does not create a presumption that the lawyer has breached her legal duty. Answer choice B is incorrect because a lawyer's duty of care is not judged by an average standard. Otherwise, lawyers who are not as skillful or as knowledgeable as their fellow lawyers would automatically be liable for malpractice. Answer choice C is incorrect because, while the circumstances in which a lawyer renders legal services can affect the competence and diligence with which the lawyer performs those services, and even an attorney who lacks the skill ordinarily required to give advice or provide assistance may be able to render limited assistance in such circumstances, a lawyer is not judged by a good faith standard even in an emergency situation.

A law firm, a professional corporation with five lawyer shareholders, employs twenty-five additional lawyers. Which of the following is proper? Employees who are members of the bar are not made shareholders until they have been with the law firm ten years. An office manager, who is not a member of the bar, is executive vice president of the law firm. A widow, whose husband was a lawyer shareholder in the law firm until his death two years ago, continues to hold her husband's shares in the law firm, distributed in his estate, until their child completes a law school education. For the first two years of employment, new associates of the law firm must have all of their work approved by a supervisor who is not a lawyer.

Answer choice A is correct. A law firm is permitted to set its own rules regarding membership in the firm by lawyers. Answer choice B is incorrect, since under MRPC 5.4(d), a lawyer is not permitted to practice with or in the form of a professional corporation if a non-lawyer is an officer thereof. Answer choice C is incorrect, as the widow, a non-lawyer, may hold the shares in the professional corporation only for a reasonable period of time during administration, not until her child has completed law school. Answer choice D is incorrect, as it is not permissible for a non-lawyer to direct or control the professional judgment of a lawyer practicing in a professional corporation. MRPC 5.4(d).

The owner of undeveloped real property entered into a contract to sell the property to an investor. The investor's obligation to purchase the property was conditioned on the title to the property being marketable. A lawyer employed by the investor investigated the validity of the seller's title to the property and reported to the investor that there was a cloud on the seller's title, which made the property unmarketable. As a consequence, on the date set for closing, the investor did not tender the purchase price. On that date, due to independent economic events, the fair market value of the property was less than the purchase price. Subsequently, it was uncovered that the lawyer misread a key document and the seller had clear title to the property. Is the lawyer subject to discipline? Yes, because the lawyer failed to competently investigate the validity of the seller's title to the property. Yes, because the seller suffered an economic loss when the buyer refused to complete the sale. No, because due to the drop in the fair market value of the property, the investor did not suffer an economic loss in foregoing the purchase of the property. No, because the lawyer's error was not intentional.

Answer choice A is correct. A lawyer is required to provide competent representation to a client. Here, the lawyer failed to provide such representation by negligently conducting the title search. Answer choice B is incorrect because, although the seller presumably suffered an economic loss when the seller eventually sold the property since, at the time of closing, the property's fair market value had dropped below the contract price, it is not necessary that a third party suffer harm for a lawyer to be subject to discipline for violation of a rule. Answer choice C is incorrect because, although the investor did not suffer an economic loss by not completing the sale since the fair market value of the property had dropped below its contract price, it is not necessary that a client suffer harm for a lawyer to be subject to discipline for a violation of a rule. Answer choice D is incorrect because, while the punishment imposed for violation of a rule of professional conduct may be less when the lawyer's violation was not willful, a lawyer is nevertheless subject to discipline for violation of a rule even though the violation was inadvertent rather than intentional.

An attorney is engaged by a testator to prepare a codicil to a will, which was drafted by another lawyer who is now deceased. The testator, who is a new client, has been referred to the attorney by another client, whom the attorney has represented in a variety of business transactions in the past, and currently represents in negotiations to purchase land as an investment. The testator is, in fact, the aunt of the referring client. In discussing her testamentary desires with the attorney, the testator reveals that she has changed her mind about leaving any property to her niece, who in the will is named as the residuary beneficiary and otherwise would be entitled to her aunt's property under the laws of intestate succession, but she instead wishes to leave her entire estate to a charity. The attorney, upon learning the scope of the changes to the testator's existing will, suggests that the testator draft a new will, but otherwise follows her directions in drafting the will. The attorney does not give and the testator does not ask for the lawyer's advice regarding the advisability of any of her actions. The attorney's fee is paid in full by the testator. The attorney does not reveal the substance of the will to the niece or otherwise discuss his representation of the aunt in this matter. Is the attorney subject to discipline for drafting the aunt's will? No, because there is not a conflict of interest between the attorney's two clients. No, because the aunt left her estate to charity. Yes, because the niece recommended the attorney to her aunt. Yes, because the aunt disinherited her niece, who is a current client of the attorney.

Answer choice A is correct. A lawyer must not represent a client when doing so is directly adverse to the interests of another client or there is a significant risk that the representation of the client will be materially limited by the lawyer's responsibilities to the current client, without each client's informed consent. MRPC 1.7. For direct adversity to exist between clients, there must be a conflict as to the legal rights and duties of the clients. Here, the niece did not have a legal claim to inherit her aunt's estate; she had a mere expectancy. In addition, since the attorney was asked by the aunt to prepare a testamentary instrument, any duties owed by the attorney to the niece with regard to the business transactions on which the attorney was representing the niece did not pose a significant risk that the attorney would be unable to perform his duties for the aunt. Answer choice B is incorrect because the charitable nature of the beneficiary of the will is irrelevant as to whether the attorney breached his ethical duties to his clients. Answer choice C is incorrect. While a lawyer must not allow a person who recommends his legal services to another to direct or regulate the lawyer's professional judgment in rendering those services (MRPC 5.4(c)), the will drafted here by the attorney did not favor the niece, rather, it resulted in her being disinherited. Answer choice D is incorrect because a lawyer who drafts a will does not, by doing so, owe any duties to a potential beneficiary other than to effect the testator's intent. Moreover, as noted with respect to answer choice A, any duties owed by the attorney to the niece with regard to her legal representation in various business transactions did not pose a significant risk that the attorney would be unable to perform his duties for the aunt.

A prominent politician contacted a well-respected and successful lawyer about representing her in a divorce action. During the initial consultation, the politician revealed that she was having an affair. At the conclusion of the consultation, the lawyer declined to represent her. The politician responded, "Good. I only wanted to make sure you couldn't represent my husband." Shortly afterwards, the lawyer, in order to discredit the politician, revealed to a reporter that the politician was having an affair. Is the lawyer subject to discipline? No, because the lawyer did not owe a duty of confidentiality to the politician. No, because the lawyer declined to represent the politician. Yes, because the lawyer revealed information learned from a consultation with a prospective client. Yes, because the lawyer revealed the politician's affair in order to discredit her.

Answer choice A is correct. Although a lawyer owes a duty of confidentiality to a prospective client, a person who contacts a lawyer for the purpose of preventing the lawyer from representing another party in a matter (i.e., to disqualify the lawyer) is not treated as a prospective client. Consequently, the lawyer did not owe a duty of confidentiality with regard to the information about the politician's affair. Answer choice B is incorrect because the duty of confidentiality with regard to information learned during an interview with a prospective client about representation of the prospective client applies, even though the lawyer does not accept the representation. Answer choice C is incorrect because, as noted with regard to answer choice A, although a lawyer owes a duty of confidentiality to a prospective client, a person who contacts a lawyer for the purpose of preventing the lawyer from representing another party in a matter (i.e., to disqualify the lawyer) is not treated as a prospective client. Answer choice D is incorrect. Although the duty of confidentiality generally applies to the disclosure of any information learned during a consultation with a potential client, and it includes information used to the disadvantage of a client, the duty does not apply when a person contacts a lawyer for the purpose of preventing the lawyer from representing another party in a matter (i.e., to disqualify the lawyer).

In an attempt to grow her business, a newly licensed attorney accepted nearly all cases brought to her. After a few months, her workload became so heavy that she did not have sufficient time to devote to the cases, and she subsequently hired a paralegal and a secretary to alleviate some of the work. She also telephoned each of her clients and informed them that her secretary and paralegal would complete much of the legal legwork because she simply did not have time to do so herself. The attorney subsequently missed a major deadline, and a client filed a malpractice suit against her. During discovery, the attorney stated that her paralegal was responsible for tracking deadlines and that her excessive workload prevented her from being able to work full-time on each case herself. Were the attorney's actions with regard to managing her workload proper? No, because she had a duty to manage her workload more effectively. No, because the attorney did not inform her clients in writing that her staff would take on some of the work. Yes, because it was not possible for the attorney to work on all details of each case in light of her workload. Yes, because the attorney adequately supervised her staff.

Answer choice A is correct. An attorney cannot justify a lack of diligence based on illness, subordinates, personal animosity, or the inability to balance other work. An attorney must control her workload to ensure that she can handle all matters competently. Here, the attorney took on more cases than she could handle, and she was unable to competently handle her cases, despite the fact that she hired additional staff to compensate for having an unmanageable caseload. Answer choice B is incorrect because an attorney is permitted to employ paraprofessionals, such as secretaries and paralegals, to help the attorney with certain duties, so long as the attorney properly supervises the paraprofessionals. There is no requirement that the attorney inform her clients in writing that she delegated duties to such paraprofessionals. Answer choice C is incorrect because an excessive workload is not a valid excuse to justify a lack of diligence. Answer choice D is incorrect because, although the attorney supervised her staff, she could not allow them to take sole responsibility for all deviations from reasonable diligence.

An attorney represented an incorporated delivery business in a negligence lawsuit stemming from an accident involving a driver employed by the business. The attorney was selected and directed by the president and sole shareholder of the business. After interviewing another employee who witnessed the accident, the attorney engaged in sexual relations with this witness-employee. The attorney did not discuss his professional responsibility obligations regarding sexual relations with the witness-employee. Is the attorney subject to discipline for engaging in sexual relations with the witness-employee? No, because the witness-employee did not supervise, direct, or regularly consult with the attorney concerning the delivery business's legal matters. No, because the conflict of interest arising from sexual relations with a client does not apply when the client is an entity rather than an individual. Yes, because the attorney engaged in a sexual relationship with an employee of the attorney's client. Yes, because the attorney did not seek the informed consent of the witness-employee before engaging the sexual relations.

Answer choice A is correct. An attorney is generally prohibited from engaging in sexual relations with a client. When the client is an organization, this prohibition applies to an employee of the client who supervises, directs, or regularly consults with the attorney concerning the organization's legal matters. Since the witness-employee was not such an employee, the prohibition against sexual relations does not apply. Answer choice B is incorrect because the prohibition of engaging in sexual relations with a client may apply to employees of a client that is an organization. Answer choice C is incorrect because while an attorney may be subject to discipline for engaging in a sexual relationship with an employee of a client, a conflict of interest does not arise with respect to all employees, but only those employees who supervise, direct, or regularly consult with the attorney. Answer choice D is incorrect because a client cannot waive this prohibition through informed consent.

A college student was arrested very late one night for attempted robbery. When he arrived at the police station, he called his uncle, who was an attorney. The uncle specialized in trusts and estates law, and had never practiced criminal law. The uncle went to the police station, where he represented himself as his nephew's attorney. He was ushered back to a room where two officers were interrogating his nephew. The uncle told the police officers that he was his nephew's attorney, and then he instructed his nephew not to speak any further. The nephew protested, explaining to his uncle that the officers had promised to speak to the district attorney if he would explain what had happened, but the uncle was insistent. The next morning, the uncle called a local criminal defense attorney, who visited the nephew and agreed to take over his representation. Is the uncle subject to discipline for his representation of his nephew? No, because a referral to or consultation with another attorney would have been impractical under the circumstances. No, because an attorney need not have knowledge of or competence in criminal defense in order to represent a criminal defendant. Yes, because an attorney may not represent a family member without informed consent. Yes, because the uncle did not possess the legal knowledge and skill reasonably necessary for the representation.

Answer choice A is correct. An attorney is obligated to provide competent representation to the client and must possess the legal knowledge, skill, thoroughness, and preparation reasonably necessary for the representation. In an emergency, an attorney may give advice or assistance in a matter in which the attorney does not have the skill ordinarily required if referral to or consultation or association with another attorney would be impractical. Assistance should be limited, however, to what is reasonably necessary under the circumstances. In this case, it would have been impractical to consult with or provide a referral to another attorney in the middle of the night. Moreover, the uncle provided only the level of assistance that was reasonably necessary. Answer choice B is incorrect because although an attorney can accept representation when the level of competence can be achieved by preparation, an attorney must have the legal knowledge, skill, thoroughness, and preparation reasonably necessary for that representation. There is an exception, however, when the circumstances constitute an emergency. Answer choice C is incorrect because an attorney is not generally prohibited from representing a family member. Answer choice D is incorrect because although the uncle may not have had the necessary knowledge or skills to represent his nephew, the circumstances constituted an emergency.

A partner in a law firm was a witness to an assault that occurred in his neighborhood. The partner will likely have to testify in the civil action brought by the victim of the assault for damages. An associate at the partner's firm has agreed to represent the victim in the assault action. Is the associate subject to disqualification? No, because there is no conflict of interest. No, because the action is not a criminal case. Yes, because the partner is likely to be called as a witness. Yes, because the possibility of the partner being called as a witness creates a conflict that is imputed to the associate.

Answer choice A is correct. An attorney is permitted to act as an advocate in a trial in which another attorney in his firm is likely to be called as a witness, unless the conflict-of-interest rules otherwise preclude the attorney from doing so. Here, the facts do not indicate that there is any conflict, and the fact that the partner may be called as a witness does not prohibit a member of his firm from representing that client. Answer choice B is incorrect because the advocate-witness rule applies to civil as well as criminal actions. Answer choice C is incorrect because, unless some other conflict would prevent the associate from representing the client, the likelihood of the partner being called as a witness would not prohibit the associate from serving as the client's advocate. Answer choice D is incorrect because absent a conflict of interest regarding another matter, the partner's disqualification from serving as an attorney for the victim is not imputed to an attorney who is a member of the partner's law firm.

A well-known defense attorney met with a criminal defendant regarding representation of the defendant in a highly publicized case. During the meeting, the attorney told the defendant, who was indigent, that she would represent him if he agreed to grant her movie rights regarding the representation. The client agreed to these terms, and the attorney provided him with a written consent form setting forth the terms of the representation and advising him to seek independent counsel. The attorney met with the defendant several days later, at which time he returned a signed copy of the written consent form. He told her that he had not consulted with another attorney. The attorney succeeded in obtaining an acquittal at trial and began shopping a movie based on the case to television studios shortly thereafter. Were the attorney's actions in securing the movie rights based on the case proper? A. No, because the attorney negotiated for movie rights prior to the conclusion of the representation. B. No, because the defendant did not obtain independent legal counsel before signing the consent form. C. Yes, because the defendant signed a written consent form after being advised of his right to seek independent counsel. D. Yes, because the attorney negotiated for movie rights as a replacement for fees.

Answer choice A is correct. An attorney is prohibited from negotiating for literary or media rights relating to representation of a client prior to the conclusion of the representation. Because the attorney negotiated for movie rights before the representation concluded in this case, her actions were improper. Answer choice B is incorrect because the prohibition on making or negotiating an agreement prior to the conclusion of the representation is a blanket prohibition that cannot be waived by the client regardless of whether the client sought or obtained independent legal counsel. Answer choice C is incorrect because an attorney may not negotiate for media rights before the conclusion of representation, even if the client consents. Answer choice D is incorrect because an attorney may not negotiate for literary or media rights before the conclusion of representation, even if such rights serve as the attorney's compensation.

An attorney represented an incarcerated, indigent criminal defendant on a pro bono basis. At their initial meeting, the defendant was extremely fidgety. The defendant explained that she was a smoker but had not had a cigarette since being imprisoned because she had no money to buy cigarettes in the facility. The attorney gave the defendant $50 to buy cigarettes without securing from the defendant a promise to repay the money. Nevertheless, the defendant subsequently did repay the attorney. Was the attorney's loan to his client proper? No, because an attorney may not provide financial assistance to a client for the client's living expenses. No, because an attorney must take steps to ensure that a client will repay any monetary advances made by the attorney before making such an advance. Yes, because an attorney may provide financial assistance to an indigent client. Yes, because the defendant repaid the money to the attorney.

Answer choice A is correct. Financial assistance to a client is prohibited with respect to pending or planned litigation, except that an attorney may advance litigation expenses and court costs. The $50 in this case did not relate to such expenses and costs, and therefore it is prohibited. Answer choice B is incorrect because an attorney may not generally provide money to a client regardless of whether there is a prior agreement by the client to repay it. Answer choice C is incorrect because although an attorney representing an indigent client may advance court costs and litigation expenses regardless of whether the funds will be repaid, an attorney may not provide other forms of financial assistance. The money in this case did not relate to court costs or litigation expenses, and thus it would be prohibited. Answer choice D is incorrect because, even though the client repaid the attorney, an attorney is prohibited from providing a client with funds for living expenses.

As required by a purchase agreement, a consumer and a retailer submitted a dispute to an arbitration panel. The consumer and retailer each named an attorney to be a partisan member of the panel and together those two panel members chose the third member of the panel. After listening to both sides, the panel ruled 2-1 in favor of the retailer, with the panel member named by the consumer voting in favor of the consumer. Without seeking the consent of the retailer, the consumer employed the attorney he had appointed to the panel to challenge the arbitration of the dispute. Is it proper for the attorney to accept this employment? Yes, because the attorney served as a partisan member of the three-member arbitration panel. Yes, because the attorney voted against the decision reached by the panel. No, because the retailer did not consent to the attorney's subsequent representation of the consumer with regard to the arbitrated matter. No, because the attorney personally and substantially participated as a member of the arbitration panel.

Answer choice A is correct. Generally, an attorney who personally and substantially participates as an arbitrator may not represent anyone in connection with the matter arbitrated, unless all parties to the proceeding give informed, written consent. However, this restriction does not apply to an arbitrator selected as a partisan of a party in a multi-member arbitration panel. Answer choice B is incorrect because there is no requirement that an attorney who serves as an arbitrator vote against the panel's decision in order to be able to represent a party in connection with the matter arbitrated. Answer choice C is incorrect. As noted with respect to answer choice A, while generally each party to an arbitration must give informed, written consent for an attorney who personally and substantially participated as an arbitrator to represent anyone in connection with the matter arbitrated, this restriction does not apply to an arbitrator selected as a partisan of a party in a multi-member arbitration panel. Answer choice D is incorrect because, although the attorney personally and substantially participated as a member of the arbitration panel by listening to both parties to the arbitration present their cases and voting on the matter being arbitrated, the attorney could nevertheless represent the consumer in an action challenging the arbitration of the dispute because the attorney was selected as a partisan of the party in the multi-member arbitration panel.

During the trial of an age discrimination lawsuit, both the judge and the attorney referenced the plaintiff's age and political affiliation multiple times in closing argument and jury instructions. The judge stated that "while members of an older generation may be more sensitive to being passed over for promotions, the law does not allow for actual discrimination." He also stated that individuals of the plaintiff's political party might face additional challenges in the particular field, which tends to be composed mostly of members of an opposing political party, and that this fact may have made the plaintiff feel alienated by his peers. Is it proper for the judge to reference the plaintiff's age and political affiliation in the above-referenced manner? No, as to both age and political affiliation Yes, as to both age and political affiliation Yes, as to age only Yes, as to political affiliation only

Answer choice A is correct. In performing judicial duties, a judge must not by words or conduct manifest bias or prejudice. Examples of manifestations of bias or prejudice include, but are not limited to: negative stereotyping; attempted humor based upon stereotypes; suggestions of connections between race, ethnicity, or nationality and crime; and irrelevant references to personal characteristics. The requirements of the Code of Judicial Conduct do not preclude judges or attorneys from making legitimate reference to race, sex, gender, religion, national origin, ethnicity, disability, age, sexual orientation, marital status, socioeconomic status, or political affiliation, or similar factors, when they are relevant to an issue in a proceeding. Here, the judge's reference to age reflected negative stereotyping, and was not permitted. Political affiliation was not relevant at all to an age discrimination issue, so the reference was not a legitimate exercise of judicial authority. Answer choice B is incorrect because the judge's comments for both age and political affiliation were improper. Answer choice C is incorrect because the judge's comments about age reflected inappropriate stereotyping. Answer choice D is incorrect because the judge should not have referenced the plaintiff's political affiliation, which was not a relevant issue.

An attorney represented a client as a plaintiff in a personal injury matter under a standard contingent fee contract. The client agreed to settle the case for $1,000,000, from which funds the attorney would receive $250,000. The client informed the attorney that she planned to take $25,000 of the settlement funds and spend the money purchasing lottery tickets. The attorney told the client that he disagreed with this plan and encouraged the client to take some classes on investing money. The client agreed to take the classes, but still insisted on playing the lottery. The attorney received the check for $1,000,000 three days before the client was to attend the investing classes. The attorney held the check for one week, giving the client at least a few days of classes. The attorney then informed the client of the receipt of the funds, disbursed the funds according to the agreement, and also furnished the client with an accounting. The attorney told the client that he had delayed notice to allow time for the client to come to her senses. The client laughed and said, "I guess your plan worked, because these classes have convinced me to invest my money in the stock market instead of playing the lottery." Is the attorney subject to discipline? You Selected: Yes, because the attorney had a duty to promptly notify the client of the receipt of the $1,000,000. Yes, because the attorney gave unsolicited advice about non-legal matters. No, because the client did not object to the withholding of the notice and funds. No, because the attorney acted in the client's best interest.

Answer choice A is correct. MRPC 1.15(d) requires the lawyer to "promptly notify the client" upon receiving funds in which the client has an interest. Here, the attorney deliberately delayed notifying the client for one week, until the client had been to classes in investing. Answer choice B is incorrect because the lawyer was permitted to give this unsolicited advice. MRPC 2.1 permits a lawyer who is advising a client to "refer not only to law but to other considerations such as moral, economic, social, and political factors that may be relevant to the client's situation." Answer choice C is incorrect because the lawyer's failure to promptly notify the client of the lawyer's receipt of client funds and to promptly deliver the funds to the client, as required by MRPC 1.15(d), is not excused by the client's later failure to object. Answer choice D is incorrect because MRPC 1.15(d) provides that the lawyer must promptly notify the client when the lawyer receives funds belonging to the client. There is no exception when the lawyer thinks that it would be best for the client to delay notification.

An attorney represented the plaintiff in a personal injury matter. The attorney had heard that the defendant in the matter was anxious to settle the case and reasonably believed that the defendant's lawyer had not informed the defendant about the attorney's recent offer of settlement. The attorney instructed her non-lawyer investigator to tell the defendant about the settlement offer so that the attorney could be sure that the defendant's lawyer did not force the case to trial merely to increase the defendant's lawyer's fee. Is the attorney subject to discipline? Yes, because the defendant was represented by counsel. Yes, because the attorney was assisting the investigator in the unauthorized practice of law. No, because the investigator is not a lawyer. No, because the attorney reasonably believed that the defendant's lawyer was not keeping the defendant informed.

Answer choice A is correct. MRPC 4.2 prohibits a lawyer from communicating about a matter with a person known to be represented in the matter unless the person's lawyer consents to the communication or the communication is authorized by law. Under MRPC 8.4(a), a lawyer may not violate the rules directly or through the acts of another. Here, the attorney used a non-lawyer agent to violate the no-contact rule. Answer choice B is incorrect because the investigator merely relayed a communication from the attorney to the defendant and did not thereby engage in the unauthorized practice of law. Answer choice C is incorrect because of MRPC 4.2, as discussed above. Such communication is impermissible regardless of whether it is done by the attorney or by an agent of the attorney. Answer choice D is incorrect also because of MRPC 4.2; the defendant's lawyer did not consent to the communication, and the fact that the defendant's lawyer was not relaying settlement offers did not otherwise justify the communication.

An attorney represents a defendant in bitter and protracted litigation. The attorney, at the defendant's request, has made several offers of settlement to the plaintiff's lawyer, all of which have been rejected. During a week's recess in the trial, the defendant's attorney and the plaintiff were both present at a cocktail party. The plaintiff went over to the defendant's attorney and said: "Why can't we settle that case for $50,000? This trial is costing both sides more than it's worth." Which of the following is a proper response by the defendant's attorney? "I can't discuss the matter with you." "If that's the way you feel, why don't you and the defendant get together." "I agree. We already have made several offers to settle this matter." "That's a good idea. Call my office tomorrow so that we can discuss the details."

Answer choice A is correct. MRPC 4.2 prohibits a lawyer from communicating about the subject of the representation with a person the lawyer knows to be represented by another lawyer in the matter, unless the lawyer has the consent of the other lawyer or is authorized to do so by law or a court order. Here, the attorney knows that the plaintiff is represented by a lawyer and must immediately terminate communication. Answer choices B, C, and D are not proper, as there has been no consent by the plaintiff's attorney or other lawful authorization that allows the defendant's attorney to communicate with the plaintiff about the subject of the attorney's representation of the defendant.

A seller was engaged in negotiations to sell his interest in a large tract of land to a buyer who was unrepresented in the transaction. Before the seller went out of town for a few days, he told the buyer to call his attorney if the buyer had any questions about the property. The buyer called the seller's attorney, asked certain questions about the size of the tract, and expressed hesitations concerning the high asking price for the tract. The attorney responded that, based on his experience handling real estate transactions in the neighborhood, the buyer would be getting a lot of property for the price. At the time the attorney spoke to the buyer, the attorney knew that there was a defect in the title and that the buyer's attempt to purchase the seller's interest in the tract would not result in the buyer's acquisition of any interest in the property. Relying on the attorney's assurance, the buyer agreed to make the purchase. Shortly after the sale closed, the buyer discovered that his acquisition was worthless. Is the attorney subject to civil liability to the buyer? Yes, because the attorney knowingly made false representations of fact to the buyer. Yes, because the attorney implied that his opinion regarding the value of the property was a disinterested opinion. No, because the attorney's statement that the buyer would be getting a lot of property for the money was a statement of opinion regarding the value of the property. No, because the buyer was not a client of the attorney.

Answer choice A is correct. Normally, lawyers do not owe a duty to the opposing side of a transaction. But a lawyer may not make intentional or negligent misrepresentations to a third person. In this case, the attorney made a misrepresentation when he falsely told the buyer that he would be getting a lot of property for the price, because he knew that the buyer's attempt to purchase the seller's interest would not result in the buyer's acquisition of any interest in the property. Answer choice B is incorrect because whether the attorney was or was not disinterested does not change the fact that the attorney made a misrepresentation to the buyer. Answer choice C is incorrect; the attorney's statement could not be construed as an "opinion." The attorney told the buyer that he would be getting a lot of property for the price, knowing that the buyer's investment would be compromised because of the title issue. Answer choice D is incorrect. Normally, attorneys do not owe a duty to the opposing side of a transaction. In this case, however, the attorney made a statement to the buyer that was patently false, exposing himself to civil liability.

An attorney has recently started her own law firm with four other lawyers as associates. The law firm has moved into offices in a new building that is owned by a bank. The attorney has borrowed heavily from the bank to finance her new law firm. In addition, the bank provides the law firm with accounting services through its computer. At the bank's suggestion, an employee of the bank, who is not a lawyer, serves as a part-time office manager for the law firm without compensation from the firm. The duties of the office manager are to advise the firm generally on fees and time charges and on program matters for the computer services, and to consult with the attorney on accounting and billing practices to ensure solvency. Is the arrangement with the bank proper? Yes, unless secrets or confidences of clients may be disclosed to the bank. Yes, because the office manager is paid by the bank. No, because a non-lawyer will be advising the law firm on fees and time charges. No, because the bank will be involved in the practice of law.

Answer choice A is correct. The bank employee is serving as an office manager. There is no violation of the Model Rules when a lawyer has offices in a creditor's building, obtains accounting services from a creditor, or hires a creditor's employee, paid by the creditor. If the bank employee discloses secrets or confidences of the attorney's clients, however, that would violate the attorney's ethical duty of confidentiality under MRPC 1.6 and the attorney's responsibility under MRPC 5.1 to ensure that subordinates, including non-lawyers, follow the requirements of the Model Rules. Answer choice B is incorrect, because the fact that the office manager is paid by the bank is not relevant, as she is not practicing law at the firm, but performing clerical and office tasks. Answer choice C is incorrect, as advising the firm on fees and time charges is not improper under the Model Rules. It does not constitute the unauthorized practice of law by the office manager. Answer choice D is incorrect, as the office manager is not practicing law and therefore her activities, if attributed to the bank, her employer, would not be improper.

A business attorney entered into a partnership with a certified public accountant. The partnership provided legal and other assistance to clients in connection with business and tax planning, tax filings, and other personal and corporate business matters. The accountant performed only work that she was authorized to perform as a certified public accountant. The attorney made reasonable efforts to ensure that the accountant did not interfere with the attorney's compliance with his professional obligations as a lawyer. Is the attorney subject to discipline? Yes, because some of the activities of the partnership consisted of the practice of law. Yes, because lawyers may not form partnerships with non-lawyers. No, because the accountant performed only work that she was authorized to perform as a certified public accountant. No, because the attorney made reasonable efforts to ensure that the accountant did not interfere with the attorney's compliance with his professional obligations as a lawyer.

Answer choice A is correct. This fact pattern presents the classic multi-disciplinary practice of law issue. Lawyers and non-lawyers may not practice together in a single entity co-owned by the two professionals or sets of professionals if the entity delivers legal services. MRPC 5.4. Answer choice A is correct because it zeroes in on the fact that the partnership delivers legal services. Answer choice B is incorrect, as there is no absolute ban on lawyer and non-lawyer partnerships. The rule prohibits only those partnerships in which all or part of the services delivered are legal services. The lawyer could own part of this entity with the accountant if, for example, the partnership delivered only accounting services. Answer choice C is incorrect, as the accountant is a partner with the lawyer in the partnership, and that fact gives the accountant a portion of the profits of the entity and control over the law aspects of the business. What the accountant does in terms of work is not determinative. Answer choice D is incorrect because, although it is true that the accountant should not interfere with the judgment of the lawyer, lack of interference is not enough. The accountant as a partner in the partnership has legal and managerial control over the attorney's practice of law, which is prohibited behavior. The lawyer would therefore be subject to discipline.

An attorney was recently admitted to practice and was hired as a new associate of a large law firm. The attorney was working late one night when he received a telephone call from his cousin. The cousin said that he was calling from the police station because he had just been arrested for possession of cocaine with intent to distribute. He was permitted to make only one phone call, and the attorney was the only one he knew. The attorney responded that he had no criminal law experience and that his firm did not handle criminal cases. Nevertheless, the cousin pleaded with the attorney to come to the police station and see what he could do to get him out on bail. The attorney replied that he would do what he could. The attorney went to the police station and used what information he recalled from his criminal law and procedure courses to attempt to get his cousin released on bail. However, as a result of his inexperience, the attorney was unable to secure his cousin's release that night. The next morning, the attorney found an experienced criminal lawyer for his cousin, who obtained his release within one hour. Was the attorney's conduct proper? Yes, because neither referral to another lawyer nor consultation with another lawyer was practical under the circumstances. Yes, because the attorney was a close relative. No, because the attorney had no special training or experience in criminal cases. No, because the attorney did not have the requisite level of competence to accept representation in the case.

Answer choice A is correct. Under MRPC 1.1, a lawyer may give advice on an emergency basis when referral or consultation with another lawyer is not possible. An emergency phone call from jail is one of those kinds of cases in which a lawyer may give advice even though the lawyer does not know criminal law. But for any additional work, the lawyer must become competent or find another lawyer to take on the matter, which the attorney did in this case. Answer choice B is incorrect because nothing in the rules makes conduct proper merely because the services are performed for relatives. That is, the work would have been acceptable even if the attorney was unrelated to the client because this was an emergency situation. Answer choice C is incorrect; although it is true that the attorney did not have experience in criminal law, because this was an emergency case, the attorney was permitted to give advice. Answer choice D is incorrect because the attorney did not take on the case; he only gave limited legal advice in an emergency situation, which is permitted by the rules.

Attorney Alpha represents a plaintiff in a personal injury action against a defendant, who is represented by Attorney Beta. Alpha had heard that the defendant was anxious to settle the case and believed that Beta had not informed the defendant of a reasonable settlement offer made by Alpha. Alpha instructed Alpha's non-lawyer investigator to tell the defendant about the settlement offer so Alpha could be sure that Beta does not force the case to trial merely to increase Beta's fee. The investigator talked to the defendant as instructed. Is Alpha subject to discipline? Yes, because the defendant was represented by counsel. Yes, because Alpha was assisting the investigator in the unauthorized practice of law. No, because the investigator is not a lawyer. No, if Alpha reasonably believed Beta was not keeping the defendant informed.

Answer choice A is correct. Under MRPC 4.2, Alpha or Alpha's representative is prohibited from talking to a party represented by a lawyer about the subject matter of the representation. MRPC 5.3 makes a lawyer subject to discipline for violations of the Model Rules by a non-lawyer assistant when the lawyer orders the conduct in violation of the Model Rules. Answer choice B is not correct, as there was no unauthorized practice violation. Alpha could properly hire a non-lawyer assistant. Alpha, though, is liable for any misconduct under the Model Rules by the non-lawyer assistant that Alpha orders. Answer choice C is not correct, as it does not matter that the investigator is not a lawyer. Alpha has supervisory authority over the investigator and is therefore responsible to ensure that the investigator engages in conduct consistent with the Model Rules. Answer choice D is not correct, as it is irrelevant whether Alpha reasonably believed that Beta was not keeping the defendant informed. MRPC 4.2 forbids a lawyer from talking to a party represented by a lawyer about the subject matter of the representation.

An attorney is a lawyer for a city and advises the city on all tort claims filed against it. The attorney's advice is limited to recommending settlement and the amount thereof. If a claim is not settled and suit is filed, defense of the suit is handled either by lawyers for the city's insurance carrier or by outside counsel specially retained for that purpose. In connection with any notice of claim and before suit is filed, the attorney arranges for an investigator to call upon the claimant at the claimant's home and, with no one else present, to interview the claimant and endeavor to obtain a signed statement of the claimant's version of the facts. The claimant has filed a notice of claim against the city. The attorney has sent an investigator to interview the claimant. Is the attorney subject to discipline for arranging an interview with the claimant? Yes, if the claimant was known by the attorney to be represented by counsel. Yes, if the statement taken is later used to the claimant's disadvantage. No, because the claimant had not filed suit at the time of the interview. No, because the attorney would not be representing the city in any subsequent litigation on the claimant's claim.

Answer choice A is correct. Under MRPC 4.2, in representing a client, a lawyer or someone acting on behalf of the lawyer is not permitted to communicate about the subject of the representation with a person the lawyer knows to be represented by another lawyer in the matter, unless the lawyer has the consent of the other lawyer or is authorized to do so by law or a court order. Here, if the claimant was known by the attorney to be represented by counsel, the attorney would have violated MRPC 4.2. Answer choice B is not correct, as the subsequent use of the statement is irrelevant and would not by itself subject the attorney to discipline. Answer choice C is not correct, as the fact that a suit has or has not been filed is not determinative of whether the attorney is subject to discipline. Answer choice D is not correct, as the attorney is representing the city at the time of the interview.

An attorney hired a recent law school graduate as an associate. For the first six months, the associate was assigned to draft legal documents that the attorney carefully reviewed and revised before filing. However, shortly after the associate was admitted to the bar, the attorney told the associate that he would be going on vacation the following week and was assigning her the representation of the landlord in a housing case that was going to trial while he was away. The associate had never conducted or observed a trial before and, because she had not previously worked on any housing cases, she was unfamiliar with the relevant law and procedure. She did not believe that she would have enough time to learn everything that she needed to know, but she was reluctant to decline the assignment. Before the trial began, she met with the landlord and disclosed that this would be her first trial, but the landlord did not object. Although the associate prepared diligently, the landlord lost the trial. Is the attorney subject to discipline? Yes, because the attorney did not ensure that the associate was competent to conduct the trial on her own. Yes, because the landlord lost the trial. No, because the attorney could reasonably assume that, having been admitted to the bar, the associate was capable of conducting the trial. No, because the landlord did not object to the associate's representation.

Answer choice A is correct. Under MRPC 5.1, a newly admitted lawyer in a firm must have her work properly supervised by a more experienced lawyer. Under MRPC 1.1, a law firm owes a client a duty of competence, and when the firm uses less experienced lawyers to perform client work, the partners must supervise that work. In this case, the associate, who had never conducted or observed a trial and had not worked on landlord-tenant cases before, needed supervision. There was not enough time for the associate to become familiar with the relevant law and procedure, nor did she have the experience to competently handle the case. The attorney did not take the proper precautions to make sure that the lawyer was adequately prepared to carry out the assignment. Answer choice B is incorrect. The outcome of the trial is not relevant to whether the associate was properly supervised as required by MRPC 1.1 and 5.1. Answer choice C is incorrect. The associate's admission to the bar did not relieve the attorney of the responsibility to properly supervise and train her. Answer choice D is incorrect. The consent of the landlord to this representation would not mitigate the violation of MRPC 1.1 and 5.1, which impose a responsibility to properly train and supervise less experienced attorneys.

A certified public accountant has proposed to an attorney, a recognized specialist in the field of tax law, that she and the attorney form a partnership for the purpose of providing clients with tax-related legal and accounting services. Both the accountant and the attorney have deserved reputations of being competent, honest, and trustworthy. The accountant further proposes that the announcement of the proposed partnership, the firm stationery, and all public directory listings clearly state that the accountant is a certified public accountant and that the attorney is a lawyer. Is the attorney subject to discipline if he enters into the proposed partnership with the accountant? Yes, because one of the activities of the partnership would be providing legal services to clients. Yes, because the attorney would be receiving fees paid for other than legal services. No, because the partnership will assure to the public high-quality services in the fields of tax law and accounting. No, if the attorney is the only person in the partnership who gives advice on legal matters.

Answer choice A is correct. Under MRPC 5.4(b), a lawyer is prohibited from entering into a partnership with a non-lawyer if any of the partnership's business will include the practice of law. Answer choice B is not correct, as the Model Rules do not prohibit a lawyer from receiving fees for services other than legal services. They do, however, prohibit a lawyer from entering into a partnership with a non-lawyer who will share in the fees from the practice of law. Answer choice C is not correct, as it is irrelevant whether the partnership will assure to the public high-quality services in both tax law and accounting. The Model Rules prohibit a partnership between a lawyer and a non-lawyer when any of the partnership's business will include the practice of law. Answer choice D is not correct, as the Model Rules prohibit a partnership if any of the business of the partnership will include the practice of law, even if the lawyer is the only person providing legal advice.

An attorney was retained to represent a client charged with assault. The attorney interviewed several of the client's friends and colleagues in a search of character witnesses. In one such interview, the client's secretary revealed that the client, who was a banker, often skirted legal and ethical lines. After the client was acquitted, the attorney's friend called to congratulate him. The attorney mentioned that it might not be long before the client found himself in legal trouble again, and he told his friend about the conversation with the client's secretary. Was the attorney's action in revealing what he learned from the client's secretary proper? No, because the attorney was bound by the duty of confidentiality. No, because the conversation with the secretary was protected by attorney-client privilege. Yes, because the attorney's representation of the client had concluded. Yes, because the conversation was not protected by the attorney-client privilege.

Answer choice A is correct. Under the Model Rules of Professional Conduct, an attorney is prohibited from disclosing information relating to the representation of a client unless the client consents, the disclosure is impliedly authorized, or certain exceptions apply. The ethical duty of confidentiality applies to all information relating to the representation regardless of the source, including third-party communications. Because the attorney learned the information during the course of representation, he was bound by the duty of confidentiality not to disclose it. Answer choice B is incorrect because the attorney-client privilege applies only to communications between an attorney and a client, and only to those communications in which the client expresses a desire for confidentiality. Because this conversation involved a third party, rather than the client, the privilege would not apply. Answer choice C is incorrect because the duty of confidentiality applies to a former client. Answer choice D is incorrect because although attorney-client privilege would not apply, the attorney would be bound by the duty of confidentiality.

A woman had a meeting with an attorney to discuss the attorney's possible representation of her in a divorce. They discussed the facts and circumstances of the divorce, as well as the attorney's fees. At the conclusion of the meeting, the woman told the attorney that she looked forward to working with him. The attorney sent her home with a representation agreement, which he told her to sign and return to him with the discussed retainer. Later that day, the woman's husband, who was a senior manager of a large corporation, asked the attorney to represent him. The attorney realized that he might be able to get other business from the husband and agreed to represent him. When the wife returned the signed representation agreement, the attorney informed her that he was now representing her husband. Is the attorney's representation of the husband proper? No, because the wife had discussed the facts and circumstances of her divorce with the attorney. No, because the attorney gave the wife a representation agreement to sign. Yes, because the wife and the attorney did not enter into an attorney-client relationship. Yes, because the wife had not paid a retainer when the attorney agreed to represent the husband.

Answer choice A is correct. While the wife may be able to successfully argue that she entered an attorney-client relationship with the attorney, at the very least, the wife was a prospective client of the attorney. A lawyer may not represent a client with interests materially adverse to those of a prospective client in the same or a substantially related matter if the lawyer received information from the prospective client that could be significantly harmful to that person in the matter. Here, the attorney and the wife discussed the details of the case, which would certainly be harmful to the wife. Accordingly, it was improper for the attorney to accept representation of the husband. Answer choice B is incorrect because, while the representation agreement is certainly evidence of an attorney-client relationship, which would make representation of the husband improper, the agreement alone does not make the representation improper. Additionally, the representation would be improper even if the attorney had not given the wife an agreement. Accordingly, answer choice A is a better answer. Answer choice C is incorrect because, even if no attorney-client relationship existed, an attorney may be prohibited from representing a client whose interests are materially adverse to a prospective client. Here, the wife was a prospective client of the attorney. Because the attorney and the wife discussed material details of the case, which would be harmful to the wife, the attorney's representing the husband is improper, even though no attorney-client relationship existed between the wife and the attorney. Answer choice D is incorrect because the payment of a fee is unnecessary to create an attorney-client relationship.

An attorney was subpoenaed by a grand jury and asked questions regarding the whereabouts of her client. The attorney considered whether the lawyer-client privilege applied to the revelation of this information. Determining that it was more likely than not that the privilege did not apply, the attorney answered the grand jury questions. Is the attorney subject to discipline for disclosing her client's whereabouts? Yes, because the attorney failed to assert a claim that the lawyer-client privilege applied. Yes, because an attorney may not reveal the whereabouts of her client. No, because the attorney determined that it was more likely than not that the lawyer-client privilege did not apply. No, because the duty of confidentiality does not apply when an attorney is called as a witness.

Answer choice A is correct. While there is an exception to the duty of confidentiality if the lawyer reasonably believes it necessary to comply with a court order or other law, this exception states that the lawyer should assert all non-frivolous claims that the information sought is protected against disclosure by the lawyer-client privilege or other applicable law. Even though the attorney did not think that the attorney-client privilege applied, asserting the claim on the client's behalf would have been non-frivolous. Answer choice B is incorrect because, while the client's whereabouts is typically information protected by the duty of confidentiality, a lawyer is permitted to reveal such information to the extent that the lawyer reasonably believes it necessary to comply with the law or a court order. Answer choice C is incorrect because the "compliance with other law or court order" exception to the duty of confidentiality does not apply unless the lawyer has first asserted all non-frivolous claims that the information sought is protected against disclosure by the lawyer-client privilege or other applicable law. Answer choice D is incorrect because there is no blanket exception to the duty of confidentiality when a lawyer is called as a witness.

An attorney is a member of the bar and a salaried employee of a bank's trust department. As part of his duties, he prepares a monthly newsletter concerning wills, trusts, estates, and taxes that the bank sends to all of its customers. The newsletter contains a recommendation to the customer to review his or her will in light of the information contained and, if the customer has any questions, to bring the will to the bank, where the attorney will review the customer's will and answer the customer's legal questions. The bank provides the attorney's services to its customers for no charge. Is the attorney subject to discipline for the foregoing? Yes, because by sending out the newsletter the attorney is giving legal advice to persons who are not his clients. Yes, because the attorney is assisting the bank in the unauthorized practice of law. No, because no charge is made for the attorney's advice. No, because the attorney is a member of the bar.

Answer choice B is correct. A bank may not provide legal services to its customers through a salaried lawyer because banks are not authorized to practice law. By providing these services, the attorney is assisting the bank in the unauthorized practice of law. Answer choice A is incorrect, as a newsletter does not provide legal advice about a particular client's problem; it merely provides generalized legal information. Answer choice C is incorrect; a bank may not provide legal services to its customers through a salaried lawyer because banks are not authorized to practice law. By providing these services, the attorney is assisting the bank in the unauthorized practice of law, whether or not the customer is charged for the work. Answer choice D is incorrect; a bank may not provide legal services to its customers through a salaried lawyer because banks are not authorized to practice law. The attorney may do legal work for the bank but may not do legal work for bank clients.

An attorney was approached by a husband and a wife who had decided to dissolve their marriage. They had no children and had worked out a tentative mutual property settlement. They did not want to retain separate lawyers because they hoped to save money and believed that working with one attorney was more likely to result in a reasonably amicable dissolution. Before coming to the attorney, they had drafted and each had signed a written agreement not to run up the costs and increase the adversarial nature of the dissolution by retaining separate lawyers. The attorney believed that he was able to provide competent and diligent representation to both the husband and the wife. The attorney consulted with both independently concerning the implications of the common representation, including the advantages and risks involved and the effect on their respective attorney-client privileges. The attorney reduced the disclosures to writing in the form of a written retainer agreement and gave them each several days to consult independent legal counsel if they so desired. The husband and the wife each chose not to consult independent counsel. After six months of reasonably amicable negotiations, the wife announced that she had changed her mind about the representation and had decided to retain her own lawyer. However, after the husband and the attorney insisted that she was obligated to adhere to her prior written agreement, she reluctantly agreed to abide by it. The attorney was then able to draft a property settlement agreement satisfactory to both parties. Is the attorney subject to discipline for his conduct in the representation? Yes, because the attorney should not have undertaken to represent both the husband and the wife in the first place. Yes, because the attorney insisted that the wife not hire another lawyer. No, because both the husband and the wife initially consented to all aspects of the representation. No, because the husband and the wife independently made the agreement that neither would retain separate counsel.

Answer choice B is correct. A client who has given consent to a potential conflict of interest may revoke the consent and may, like any other client, terminate the lawyer's representation at any time. It was improper for the attorney to insist that the wife adhere to her prior agreement in derogation of her absolute right to terminate the joint representation. Answer choice A is incorrect as, under MRPC 1.7, the initial representation was proper because it was reasonable for the attorney to believe that he was able to provide competent and diligent representation to both the husband and the wife, and the clients' consent was informed and in writing. Answer choice C is incorrect because, even though the clients initially consented to the joint representation, each had an absolute right at any time to revoke the consent and, like any other client, to terminate the attorney's representation. It was improper for the attorney to insist that the wife had given up that right. Answer choice D is incorrect because, even though the clients initially agreed between themselves to be jointly represented, each had an absolute right at any time to revoke the consent and, like any other client, to terminate the attorney's representation. It was improper for the attorney to insist that the wife had given up that right.

An attorney and a prospective client met to discuss whether the attorney would represent the client in a contractual dispute. During the conversation, the potential plaintiff spoke to the attorney about her litigation objectives and how much she would be able to pay the attorney. As they were wrapping up the meeting, the client noticed a picture of the attorney's teenaged son on the wall. The client confided in the attorney that she had a son the same age, but she had given him up for adoption because she was an unwed teenager when he was born. She told the attorney that no one except her family knew about the adoption, and she asked the attorney to keep it confidential. Is the information about the client's pregnancy protected by the attorney-client privilege? A. No, because the woman had not retained the attorney when the conversation took place. B. No, because the communication was not relayed for legal advice. C. Yes, because the woman had a reasonable expectation that she had established an attorney-client relationship with the attorney. D. Yes, because the woman had a reasonable expectation that she had established an attorney-client relationship with the attorney, and she stated that the communication was confidential.

Answer choice B is correct. A confidential communication between a client and her attorney is privileged. The evidentiary attorney-client privilege covers the client's communication to an attorney whom the client reasonably believes represents the client, and the circumstances indicate a desire by the client for confidentiality. Although the attorney does not need to give advice or agree to the representation for the privilege to exist, the communication must be for the purpose of seeking legal advice or representation. Although the statement about the client's child was given in confidence and would be protected by the attorney's ethical duty of confidentiality, it is not subject to the attorney-client privilege because it was not relayed for the purpose of obtaining legal advice. Answer choice A is incorrect because, although the woman was a potential client with regard to the contract matter, the privilege would not apply to the statement regarding the adoption because the statement was not made for the purpose of obtaining legal advice. Answer choice C is incorrect because the communication was not made for the purpose of obtaining legal advice, so it is not privileged even if the woman had a reasonable expectation of representation. Answer choice D is incorrect because, although the communication would have been privileged if it had been relayed for the purpose of obtaining legal advice, it was not relayed for that purpose.

A rising second-year law school student was employed as a summer intern at a law firm. The intern attended a deposition of a client who was represented by an associate attorney of the firm. The associate cautioned the intern not to discuss any aspect of the deposition with anyone. The intern discussed his own personal observations of the client's demeanor and his own conclusions about the client's truthfulness with his roommate, which did not result in any harm to the client. Of the following, which would likely provide the associate with the best defense to a disciplinary action based on the intern's actions? The intern only discussed his own personal observations and conclusions with his roommate. The associate cautioned the intern not to discuss any aspect of the deposition with anyone. The intern was not a licensed attorney. The client did not suffer harm as a consequence of the intern's conversation with his roommate.

Answer choice B is correct. A lawyer must make reasonable efforts to prevent the inadvertent or unauthorized disclosure of information relating to the representation of a client by persons who are participating in the representation of the client or who are subject to the lawyer's supervision. The associate's warning to the intern not to discuss any aspect of the disposition with anyone likely constitutes reasonable precautions. Answer choice A is incorrect because the duty of confidentiality extends beyond communications made by a client and her agents to a lawyer or his staff to any information relating to the representation, and it includes personal observations. Answer choice C is incorrect because, although the intern is not subject to discipline under the Model Rules of Professional Conduct because he is not a lawyer, the associate is responsible for taking reasonable efforts to prevent the inadvertent or unauthorized disclosure of information relating to the representation and for taking reasonable precautions to safeguard such information from disclosure by non-lawyer employees and agents of the firm who are under the associate's supervision. Answer choice D is incorrect because a lawyer is subject to discipline for violation of a Model Rule of Professional Conduct even though the client does not suffer harm as a consequence of the violation.

An attorney filed a motion for summary judgment on behalf of his client despite knowledge of directly adverse case law from the highest court in the state. Since the issuance of that decision, however, courts in other states had reached the opposite conclusion. In the motion, the attorney cited the decisions from other states but did not mention the decision from the state's highest court. The opposing attorney filed a brief in opposition to the motion for summary judgment that did not cite the decision from the state's highest court. The court did not hold oral argument but granted summary judgment based on the briefs. Were the attorney's actions in not disclosing the decision of the state's highest court proper? No, because the attorney was required to disclose the decision in his initial brief supporting the motion for summary judgment. No, because the attorney was required to disclose the decision after the opposing counsel failed to disclose it. Yes, because an attorney does not have a duty to disclose cases that are adverse to his client's position. Yes, because disclosure of the case would violate the attorney's duty to zealously advocate his client's position.

Answer choice B is correct. An attorney has a duty of candor to the tribunal, which requires the attorney to disclose to the tribunal legal authority in the controlling jurisdiction known to the attorney to be directly adverse to the position of the client that was not disclosed by opposing counsel. In this case, the attorney was required to disclose the case when the opposing counsel failed to disclose it. Answer choice A is incorrect because an attorney is required to disclose controlling authority only when the opposing counsel fails to disclose it. Accordingly, the attorney would not be required to cite the decision in his initial brief. Answer choice C is incorrect because an attorney has a duty to disclose adverse legal authority in some circumstances. Answer choice D is incorrect because although the Preamble to the Model Rules of Professional Conduct recognizes that an attorney may zealously assert his client's position when acting as an advocate, the attorney nevertheless has the duty of candor to the tribunal that is imposed by the Rules.

An attorney represented an actor in a contract dispute with the producer of a play. During the dispute, the attorney and the actor had a brief sexual relationship. Prior to engaging in sexual relations, the attorney informed the actor that the attorney was prohibited from having sex with a client. The actor dismissed the attorney's concerns, saying that it didn't matter. Sexual relations between the two terminated before the dispute was resolved. The dispute was resolved to the satisfaction of the actor. Was the attorney's action regarding the relationship proper? No, because an attorney may not engage in a sexual relationship with a client. No, because a client may not waive a conflict stemming from a sexual relationship with the attorney. Yes, because the client consented to the sexual relationship despite knowledge of the prohibition. Yes, because the contractual dispute was resolved to the satisfaction of the client after the sexual relationship with the attorney terminated.

Answer choice B is correct. An attorney is generally prohibited from engaging in sexual relations with a client. This conflict of interest may not be waived by the client through informed consent. Answer choice A is incorrect because an attorney may engage in a sexual relationship with a client if the sexual relationship existed before the attorney-client relationship began. Answer choice C is incorrect because a conflict arising from a sexual relationship may not be waived by informed consent. Answer choice D is incorrect because neither the termination of the sexual relationship before the termination of the representation nor the resolution of the matter to the satisfaction of the client excuses an attorney's sexual relationship with a client.

An attorney accepted a libel case, though he had no experience in that type of action and had little knowledge related to the issues involved. He informed his client of his inexperience but promised to work with a fellow competent attorney to handle the claim effectively. He received the client's written consent acknowledging this fact and allowing the attorney to proceed with the representation. The attorney then called his friend, a fellow attorney who had graduated at the top of her law school class but who likewise had no libel experience or knowledge herself, for advice on how to proceed. The friend provided good advice on general litigation techniques, and the attorney heeded this advice but took no other action. Would the attorney's representation of the client subject him to discipline? Yes, because he lacked the knowledge or experience to successfully represent the client. Yes, because he lacked the knowledge or experience to competently represent the client. No, because he consulted his friend on the matter. No, because the client consented in writing to the representation.

Answer choice B is correct. An attorney lacking the necessary knowledge or experience must: (i) decline or withdraw from representing the client, (ii) become competent without unreasonable delay, or (iii) associate with competent counsel. An attorney should not accept representation in a matter unless it can be performed competently. An attorney who represents a client incompetently is subject to discipline, regardless of causation or any financial loss to the client. Here, the attorney was not knowledgeable in the area of libel, and advice on general litigation tactics would not provide him with the knowledge he needed to competently represent his client in that area. Answer choice A is incorrect because an attorney does not have to win the case; he must merely be competent. Answer choice C is incorrect because the attorney's consultation with his friend would not constitute an association with competent counsel, as she had no knowledge or experience with libel cases either. Answer choice D is incorrect because a client cannot consent to incompetent representation.

An attorney regularly represented oil companies that supported drilling in the state in which he lived and worked. The attorney diligently represented his clients and did not let his personal beliefs interfere with his ability to represent them, even though he was morally opposed to drilling. He also belonged to a nonprofit legal services agency organized to represent the residents of cities and towns affected by hazardous environmental agents in any legal action based on the effects of those agents. The group also raised community awareness on the dangers of drilling on various segments of the community, including children. The attorney attended all of the group's meetings and events but abstained from voting on all matters related to the organization's mission and objectives. Is the attorney subject to discipline for his involvement with the agency? No, because he did not serve as a director or an officer of the agency. No, because he did not knowingly participate in decisions incompatible with his obligation to his clients or the agency's clients. Yes, because the agency's efforts might have an adverse effect on the attorney's clients. Yes, because the attorney knowingly participated in an agency whose efforts might have an adverse effect on the attorney's clients.

Answer choice B is correct. An attorney may serve as a director, an officer, or a member of a legal services organization, apart from the law firm in which the attorney practices, even if the organization serves persons having interests that are adverse to a client of the attorney. The attorney, however, is not permitted to knowingly participate in a decision or action of the organization if such participation would be incompatible with the attorney's obligations to a client under the Model Rules of Professional Conduct concerning conflicts of interest, or when the decision or action could have a material adverse effect on the representation of a client of the organization whose interests are adverse to a client of the attorney. Answer choice A is incorrect because the attorney may serve as an officer or director of such an agency, provided he does not knowingly participate in decisions incompatible with his obligations to a client and there is no other conflict of interest. Answer choice C is incorrect because even if the organization and the client have adverse interests, the attorney may still serve as a director, an officer, or a member if he does not participate in decision-making that has such an effect. Answer choice D is incorrect because the attorney may participate in such an organization even if he knows of the adverse interests.

An attorney litigated a non-jury case before a judge. After losing the case, the attorney filed an appeal arguing that the judge misapplied the law. After also losing the appeal, the attorney, attending a continuing legal education class, stood up and made a statement to everyone present that the judge, identifying her by name, "is crazy. She has some sort of mental disorder. She is not fit to sit on the bench." Is the attorney subject to discipline for this statement? Yes, because it was a statement affecting an individual's professional reputation that reflected a reckless disregard for the truth. Yes, because it reflected a reckless disregard for the truth about the qualifications or integrity of a judge. No, because the attorney was among fellow attorneys rather than the general population or the media. No, because his statement was based on the judge's misapplication of the law.

Answer choice B is correct. An attorney must not make a statement that the attorney knows to be false or with reckless disregard as to its truth or falsity concerning the qualifications or integrity of a judge, adjudicatory officer, or public legal officer, or of a candidate for election or appointment to judicial or legal office. Here, the attorney made a very strong statement about the judge's mental health and qualification to remain on the bench without any basis of truth. Answer choice A is incorrect because a statement affecting another individual's professional reputation made with a reckless disregard for the truth might not automatically subject the attorney to discipline, but because the statement pertained to the integrity or qualifications of the judge, it would subject him to such discipline. Answer choice C is incorrect because the audience receiving the message is not relevant in determining whether the attorney's statement would subject him to discipline. Answer choice D is incorrect because, even if the judge had misapplied the law, the statement regarding the judge's mental health and ability to serve on the bench demonstrated a reckless disregard for the truth.

An attorney decided to accept a case although a verdict in favor of his client would cause his own property to decrease substantially in value. The attorney explained the situation to the client, and the client consented in writing to the representation. After the court ruled against the attorney, the client filed a complaint with the disciplinary board alleging that the attorney should not have accepted the case. By what standard should the attorney's action be judged to determine whether he violated the conflict-of-interest rules? The attorney must have honestly believed that he could provide competent and diligent representation to the client. The attorney must have reasonably believed that he could provide competent and diligent representation to the client. The client must have believed that the attorney could provide competent and diligent representation to him. The attorney must have provided competent and diligent representation to the client.

Answer choice B is correct. An attorney must not represent a client if the representation of the client may be materially limited by the attorney's own interests, unless, in addition to securing the client's informed consent, confirmed in writing, the attorney reasonably believes that he will be able to provide competent and diligent representation to the affected client. The attorney's belief in his ability to provide competent and diligent representation is viewed under both a subjective and an objective standard. Answer choice A is incorrect because it is not enough for the attorney to have honestly believed that he could provide competent and diligent representation; such belief must also be reasonable. Answer choice C is incorrect because, although the client must give informed consent, confirmed in writing, it is the attorney who must possess a reasonable belief as to his ability to provide competent and diligent representation. Answer choice D is incorrect because, while the attorney may be disciplined for his failure to provide competent and diligent representation to his client, whether the attorney violated the conflict-of-interest rules turns on whether the attorney reasonably believed that he would be able to provide competent and diligent representation.

A client approached an attorney to represent her in a products liability case because the attorney claimed to be a specialist in products liability actions. He told the client that the National Council on Products Liability had certified him as a specialist in products liability resolution, even though he had lost half of the 100 products liability actions he had taken. The client consented to representation in writing after this discussion. The attorney lost the case on a technicality that would have been known to any specialist in products liability, though a general practice attorney would not have recognized the issue. The entire case was lost based on this error. After demonstrating that she suffered damages, would the client be likely to recover in a civil liability action against the attorney? Yes, because the attorney lost her case. Yes, because the attorney deviated from the applicable standard of care for products liability attorneys. No, because the attorney did not deviate from the applicable standard of care of a general attorney. No, because the client consented to the representation in writing after full disclosure.

Answer choice B is correct. An attorney owes a duty of care to the client. The duty of care is generally the competence and diligence exercised by attorneys of similar experience under similar conditions. If an attorney represents to the client that he has specialized expertise, then the attorney will be held to the standard of care of a specialist. Here, the standard of a specialist would apply, and the attorney likely deviated from this standard. Answer choice A is incorrect because losing a case alone is not a basis for civil liability. What is relevant is the standard of care that the attorney exercised in representing the client. Answer choice C is incorrect because while the attorney may not have deviated from a general attorney's standard of care, he did deviate from the standard of care of a specialist. Answer choice D is incorrect because if the attorney was negligent, then the client's consent would not mitigate his liability.

A law firm agreed to represent a client in various business matters. The written retainer agreement called for the client to pay the firm's hourly rates of $180 per hour for a partner's time and $110 per hour for an associate's time. The representation proceeded. The firm submitted monthly bills, which the client paid promptly. After two years, the firm decided to increase its hourly rates by $10.00. The firm thereafter billed the client at the new rates, but did not specifically inform the client of the increase. The client continued to pay monthly bills promptly. Are the firm's partners subject to discipline? Yes, because the entire original fee agreement was required to be in writing. Yes, because the client was not consulted about the increase. No, if the $10.00 hourly increase is reasonable. No, because the client agreed in writing to pay the firm's hourly rate.

Answer choice B is correct. Any changes in the basis or rate of an agreed fee must be specifically communicated to the client. MRPC 1.4 and 1.5(b). Answer choice A is not correct, as an hourly fee agreement need not be in writing, so long as the basis for the fee has been communicated to the client. Answer choice C is not correct, as whether or not the increase is reasonable is not all that controls whether the partners of the firm are subject to discipline. The Model Rules require that a change in a fee rate be communicated to the client. Answer choice D is not correct, as the client only agreed in writing to pay the original hourly rate contained in the retainer agreement.

An attorney represented ten plaintiffs in a case against a railroad in which the plaintiffs were injured when a train derailed. The railroad offered the attorney a $500,000 lump sum settlement for the ten plaintiffs. The attorney allocated the $500,000 among the ten plaintiffs with the amount paid each plaintiff dependent on the nature and extent of that plaintiff's injuries. The attorney reasonably believed the division was fair to each plaintiff. The railroad would not settle any of the claims unless all were settled. The attorney told each plaintiff the total amount the railroad was prepared to pay, the amount that the individual would receive, and the basis on which that amount was calculated. The attorney did not tell any plaintiff the amount to be received by any other plaintiff. The attorney believed that if she revealed to each plaintiff the amount of each individual settlement, it might jeopardize the settlement. Each of the plaintiffs agreed to his or her settlement amount and was satisfied with that amount. Is the attorney subject to discipline for entering into this settlement? Yes, because the attorney was aiding the lawyer for the railroad in making a lump sum settlement. Yes, because no individual plaintiff knew the amount to be received by any other plaintiff. No, because disclosing all settlements to each plaintiff might have jeopardized the entire settlement. No, because the attorney reasonably believed that the division was fair and each plaintiff agreed to his or her settlement and was satisfied with the amount.

Answer choice B is correct. Because this is an aggregate settlement of the clients' claims, MRPC 1.8(g) requires the attorney to obtain the clients' informed consent, after disclosing "the existence and nature of all the claims" and "the participation of each person in the settlement." Comment [13] states that the necessary disclosure includes information about "all the material terms of the settlement, including what the other clients will receive." Answer choice A is incorrect; MRPC 1.8(g) allows a lawyer to participate in an aggregate settlement of civil claims, as in this case, as long as the lawyer complies with the requirement of informed consent by all clients, which the attorney failed to do here. Answer choice C is incorrect; MRPC 1.8(g) and Comment [13] to the Rule require disclosure of the material terms of the settlement regardless of whether the settlement will thereby be jeopardized, because all of the clients are entitled to material information regarding their decision on whether to accept the settlement offer, and each must give informed consent. Answer choice D is incorrect. Whether or not the lawyer regards the aggregate settlement offer as fair, MRPC 1.8(g) requires a lawyer to make full disclosure to the clients so that they can each make an informed decision about whether to accept the offer. The attorney did not make the disclosure required by Comment [13] to MRPC 1.8 because she did not disclose "all the material terms of the settlement, including what the other clients will receive."

An attorney regularly represents a certain client. When the client planned to leave on a world tour, she delivered to the attorney sufficient money to pay her property taxes when they became due. The attorney placed the money in his clients' trust account. When the tax payment date arrived, the attorney was in need of a temporary loan to close the purchase of a new personal residence. Because the penalty for late payment of taxes was only 2 percent while the rate for a personal loan was 6 percent, the attorney withdrew the client's funds from the clients' trust account to cover his personal check for the closing. The attorney was confident that the client would not object. Ten days later, after the receipt of a large fee previously earned, the attorney paid the client's property taxes and the 2 percent penalty, fully satisfying the client's tax obligation. After the client returned, the attorney told her what he had done, and the client approved the attorney's conduct. Is the attorney subject to discipline? Yes, because the attorney failed to pay the client the 10 days of interest at the fair market rate. Yes, because the attorney used the client's funds for a personal purpose. No, because the client was not harmed and the attorney reasonably believed at the time he withdrew the money that the client would not object. No, because when the attorney told the client what he had done, the client approved his conduct.

Answer choice B is correct. Client funds must be held for safekeeping in an account that is separate from the lawyer's funds and may be used only in accordance with the client's instructions, not for the lawyer's personal benefit. Further, the attorney was required to use the funds for the client's designated purpose, that is, for payment of the taxes when they became due, and for no other purpose. Answer choice A is incorrect because even if the lawyer had paid the higher rate of interest to the client, the lawyer's personal use of the client's funds and his failure to carry out the client's instructions would have been improper. Answer choice C is incorrect because whether or not the client might be expected to acquiesce, a lawyer may not violate the client's instructions about how to use funds given to the lawyer for safekeeping, nor may the lawyer use those funds for personal purposes. Answer choice D is incorrect because a client's later approval of the action does not excuse a lawyer's unethical use of client funds for personal purposes.

An attorney was engaged to create a trust for a client. The attorney, working to acquire an LL.M. in taxation, had taken several advanced tax law classes. The attorney did not advertise her advanced knowledge of the tax law because she had not been certified as a tax specialist by a state-approved or ABA-accredited organization, and the client was not aware that the attorney had any training in tax law. The trust created by the attorney ran afoul of tax provisions that a lawyer with her special knowledge of the tax law should have been able to avoid. As a consequence, the settlor incurred a tax liability that he otherwise would not have incurred. May the attorney be subject to civil liability to the settlor? Yes, because the attorney could not disclaim her possession of specialized knowledge of the tax law. Yes, because the attorney failed to exercise the special skill in tax law that she possessed. No, because the client was not aware and therefore did not rely on the attorney's special tax training. No, because the attorney did not hold herself out as a tax specialist.

Answer choice B is correct. In determining a lawyer's duty of care, the lawyer's special experience with or knowledge of the law is taken into account. Because the attorney should have been able to avoid the adverse tax consequences to the settlor by using her special knowledge of the tax law, the attorney has breached her duty of care and may be liable to the settlor for the tax liability incurred by the settlor. Answer choice A is incorrect because, at least in some circumstances, a lawyer may disclaim competency in excess of the ordinary or the possession of specialized knowledge. However, no disclaimer was made here. Answer choice C is incorrect because a lawyer has a duty to exercise any special skills or to apply special knowledge when acting on behalf of her client, unless she properly disclaims such skills or knowledge. Note: Such duty does not extend to a non-client who is not aware of the lawyer's skills or knowledge and who therefore does not rely on them. Answer choice D is incorrect because, although the attorney properly did not hold herself as a tax specialist, the attorney was required to use the specialized knowledge that she possessed.

The following advertisement appeared in a daily newspaper in a state in which both parties are members of the bar: A. ALPHA, M.D., J.D. and B. BETA, J.D. Attorneys at Law 1000 "A" Street, City, State, 00000 Telephone (555) 555-5555. Are Alpha and Beta subject to discipline? No, because both law and medicine are licensed professions. No, if they possess the degree(s) stated. Yes, because the reference to the M.D. degree is self-laudatory. Yes, unless they limit their practice to areas in which a medical degree is relevant.

Answer choice B is correct. Lawyers are permitted to advertise through public media under the Model Rules, so long as their advertisements are not false, misleading, or in violation of the rules against client solicitation. MRPC 7.2. This rule permits public dissemination of information concerning a lawyer's name or firm name, address, and telephone number; the kinds of services the lawyer will undertake; the basis on which the lawyer's fees are determined, including prices for specific services and payment and credit arrangements; a lawyer's foreign language ability; names of references; and other information that might invite the attention of those seeking legal assistance. MRPC 7.2, cmt 2. Any advertisement must also include the name and office address of at least one lawyer or law firm responsible for its content. MRPC 7.2(c). Here, all of these rules are satisfied. So long as Alpha and Beta have the degrees they list in the advertisement, there would be nothing false or misleading, and they would not be subject to discipline under the Model Rules. Answer choice A is not correct, as it does not matter that both law and medicine are licensed professions. The fact that Alpha and Beta both possess the degrees that they advertised, so that the advertisement is not false or misleading, is what controls. Answer choice C is not correct, as it is not improper under the Model Rules for a lawyer to truthfully list his educational degrees. Answer choice D is not correct, as there is no requirement under the Model Rules that they limit their practice to areas in which a medical degree would be relevant.

An attorney represented a client who was injured when the television antenna he was attempting to erect in his yard came in contact with a power line. As part of its defense, the manufacturer of the antenna claimed that the antenna came with a warning label advising against erecting the antenna near power lines. The client told the attorney that he had not seen a warning label. The client's wife told the attorney that she had kept the antenna and the box it came in and that she saw no warning label anywhere. When called by the attorney as witnesses, both the client and his wife testified that they had never seen a warning label. After their testimony, but while the trial was still in progress, the attorney learned from the wife's sister that there indeed had been a warning label on the box, but that the wife had removed and destroyed it. When the attorney confronted the wife with her sister's statement, the wife admitted destroying the label but insisted that her husband knew nothing about it. The attorney continued the trial, but made no reference to the absence of a warning label in his summation to the jury. Instead, the attorney argued that the warning label, even if seen, was insufficient to advise his client of the serious consequences that would ensue if the warning was not heeded. The jury found in favor of the manufacturer. Is the attorney subject to discipline? Yes, because the attorney called the wife as a witness and she gave perjured testimony. Yes, because the attorney failed to take reasonable remedial action after he realized that the wife had given perjured testimony. No, because the jury apparently disbelieved the wife's testimony. No, because the attorney did not rely on the wife's testimony once he discovered the perjury.

Answer choice B is correct. MRPC 3.3(a)(3) requires that when a lawyer learns that a witness called by the lawyer has offered material evidence that is false, the lawyer must take reasonable remedial measures. That duty continues to the conclusion of the proceeding. MRPC 3.3(c). Because the proceeding was not yet concluded when the attorney learned that the wife's testimony was false, the attorney had a duty to take reasonable remedial measures. Such measures would include meeting with the client and informing him of the attorney's obligation of candor to the court and asking the client to correct the fraud. If the client refused and if the attorney could not undo the prejudicial effect of the fraud, the attorney would need to disclose the fraud to the court. Answer choice A is incorrect; MRPC 3.3(a)(3) prohibits a lawyer from offering evidence that the lawyer knows to be false, but the attorney did not know that the wife's testimony would be perjured when the attorney called her as a witness. Answer choice C is incorrect; MRPC 3.3(a)(3) requires the lawyer to take reasonable remedial measures when the lawyer learns that the testimony was false, because the lawyer cannot know at that time the effect the false testimony will have on the fact finder. Answer choice D is incorrect because it is insufficient for the attorney to refuse to rely on the false testimony in summation. MRPC 3.3(a)(3) requires the lawyer to take reasonable remedial measures, including, if needed, disclosure to the tribunal.

An attorney represented a man in a case set for a jury trial. After the list of potential jurors was made available, the attorney hired a private investigator to interview the potential jurors and their family members concerning their relevant past experiences related to the subject matter of the action. The investigator did not inform the jurors or their family members that he was working on behalf of the attorney. The interviews were entirely voluntary and were not harassing. The attorney did not provide the report of the interviews to opposing counsel. He used the report to make decisions regarding jury selection. Is the attorney subject to discipline? Yes, because the attorney did not provide the report of the interviews to opposing counsel. Yes, because the investigator, at the attorney's direction, communicated with potential jurors prior to trial. Yes, because the investigator did not inform the jurors or their family members that he was working on behalf of the attorney. No, because the interviews were entirely voluntary and not harassing.

Answer choice B is correct. MRPC 3.5(b) prohibits a lawyer from communicating ex parte with prospective jurors during the proceeding unless authorized to do so by law or court order. The attorney could not use a private investigator to do what the attorney could not do directly. MRPC 8.4(a). Answer choice A is incorrect because providing a report of the interviews to opposing counsel would not have cured the violation here. Answer choice C is incorrect, as the attorney's conduct in this case was clearly prohibited by the rule, and informing the jurors or their family members that the contact was being made on behalf of the attorney would not cure the violation. Answer choice D is incorrect, as the attorney's conduct in this case was clearly prohibited by the rule even if the interviews were entirely voluntary and not harassing.

A recently graduated attorney began a plaintiffs' personal injury practice, but was having a difficult time attracting clients. The attorney hired an advertising agency to prepare a television commercial in which the attorney appeared to be arguing a case before a jury. In the commercial, the jury brought back a large award for the attorney's client. The voice-over stated that results would vary depending upon particular legal and factual circumstances. The attorney's only experience at the time the commercial was filmed was in moot court. As a result of airing the commercial, the attorney received several significant cases. Is the attorney subject to discipline? Yes, because the commercial created an unjustified expectation about the results that could be achieved in court. Yes, because the commercial implied that the attorney had successfully argued a case to a jury. No, because commercial speech is protected under the First Amendment. No, because the commercial contained an express disclaimer about the results a client could expect.

Answer choice B is correct. MRPC 7.1 prohibits false or misleading statements in lawyer advertising. Although the advertisement does not expressly state that the attorney has successfully tried a case, the communication would lead a reasonable person to believe that he has. As a result, the communication is misleading and is prohibited by MRPC 7.1. Answer choice A is incorrect. An advertisement may truthfully report a lawyer's achievement on behalf of a former client unless it would lead a reasonable person to form an unjustified expectation that the same results could be obtained for other clients. The use of a prominent disclaimer, such as the voice-over in the ad, avoids such an expectation. But, the issue here is that the communication was misleading. Answer choice C is incorrect because, although it is true that commercial speech is protected under the First Amendment, the First Amendment does not prohibit states from banning false or misleading communications in advertisements by professionals such as lawyers. Answer choice D is incorrect; the voice-over disclaimer avoids unjustified expectations regarding the results a client could expect, but it does not avoid the misleading implication that the attorney had successfully argued a case to a jury.

An attorney worked at a law firm that encouraged its attorneys to provide at least 100 hours of pro bono service per year. The attorney typically surpassed this goal. Last year, however, the attorney was involved in a long, complex trial, and he did not provide any pro bono service. The attorney did not make any contributions to nonprofit legal organizations during the year. Would the attorney's failure to provide pro bono services or to contribute to nonprofit legal organizations subject him to discipline? No, because the Model Rules allow pro bono hours exceeding the 50-hour minimum to be carried forward to subsequent years. No, because the Model Rules indicate only that an attorney should aspire to perform 50 hours of pro bono services annually. Yes, because the Model Rules require an attorney to provide a minimum of 50 hours of pro bono legal services each year. Yes, because the Model Rules require an attorney to either render a minimum of 50 hours of pro bono legal services each year or make a monetary contribution to nonprofit legal organizations in lieu of such service.

Answer choice B is correct. The Model Rules of Professional Conduct set forth an aspirational goal for an attorney to accept representation of clients who are unable to pay, suggesting that attorneys offer at least 50 hours of pro bono legal services per year. This goal is not enforced through the disciplinary process. Accordingly, the attorney would not be subject to discipline. Answer choice A is incorrect because an attorney may not be disciplined for failure to perform pro bono service, and thus it is irrelevant whether hours can be carried forward. Answer choice C is incorrect because the guidelines regarding pro bono service in the Model Rules of Professional Conduct are voluntary, and thus the attorney would not be subject to discipline for failure to meet the suggested minimum. Answer choice D is incorrect because the Model Rules of Professional Conduct do not subject an attorney to discipline for the failure to either provide pro bono service or to make a monetary contribution to a nonprofit legal organization.

An attorney represented a client on a minor personal injury claim against an uninsured motorist. The attorney represented the client on a 30% contingent fee basis. Pursuant to a negotiated settlement in the amount of $2,000, the driver agreed to send the attorney a $100 check, made payable to the attorney, in each of the ensuing twenty months. Which of the following dispositions of each monthly check would be improper for the attorney? Endorse the check and immediately forward it to the client, with a request that the client pay the attorney $30. Deposit the check into her office account and immediately write the client a check for $70 from that account. Deposit the check into a separate account established for the client and immediately request the client to pay the attorney $30. Deposit the check into a trust account in which funds belonging to all the attorney's clients are deposited and immediately write the client a check for $70 and herself a check for $30 from that account.

Answer choice B is correct. This option is improper, as MRPC 1.15 requires that a lawyer hold funds that are in a lawyer's possession in connection with representation separately from the lawyer's own funds. By depositing the check in her office account, the attorney would be commingling funds improperly, even if only for a short period. Answer choice A is incorrect because the action is one that would be proper for the attorney to perform. It is proper because a lawyer who receives funds or other property in which a client has an interest must promptly notify the client and is required to pay over the funds. Answer choice C is a proper action (and thus an incorrect answer), as the funds will be in a client trust account separate from the attorney's personal funds. The attorney could properly request 30% or $30 from each monthly payment under the contingent fee agreement. Finally, answer choice D is also a proper action, as a lawyer may generally keep all funds of clients in one trust account. MRPC 1.15, cmt 1. As the agreement called for the attorney to receive 30% of the amount recovered as a contingent fee, it is proper for the attorney to write herself the $30 check and the client the $70 check. Thus, answer choice D is incorrect.

After a high-profile defendant was acquitted of murder, the prosecutor met with the media on the courthouse steps. When asked whether he was satisfied with the verdict, the prosecutor responded: "No; he got away with murder." Is the attorney subject to discipline with regard to addressing the media? No, because he was responding to a media inquiry. No, because the trial was over at the time of the statement. Yes, because the prosecutor made an extrajudicial statement that the lawyer knows or reasonably should know will be disseminated by means of public communication. Yes, because the prosecutor made an extrajudicial statement about the guilt or innocence of the defendant.

Answer choice B is correct. Under MRPC 3.6(a), a lawyer who is participating or has participated in the investigation or litigation of a matter is not permitted to make an extrajudicial statement that the lawyer knows or reasonably should know will be disseminated by means of public communication and will have a substantial likelihood of materially prejudicing an adjudicative proceeding in the matter. However, since the trial was over at the time the prosecutor made this statement, and the prosecution may not appeal the verdict here, the statement can no longer prejudice the proceeding. Answer choice A is incorrect because the fact that the prosecutor was responding to a media inquiry is irrelevant. The relevant fact is that the trial was over. Answer choice C is incorrect because, while such a statement is prohibited during a trial, it was made here after the trial was over. Answer choice D is incorrect because the prosecutor may make such a statement after the conclusion of a trial.

An attorney represents the defendant, a prominent businessman, in a civil paternity suit brought by a plaintiff, who was formerly the defendant's employee. Blood tests did not exclude the defendant's paternity, and the case is being tried before a jury. The result turns on questions of fact. The defendant has steadfastly denied that he had sexual relations with the plaintiff, while the plaintiff has testified that they had sexual relations while on business trips and in her home. The trial has generated great public interest and is closely followed by the news media. When the plaintiff completed her testimony, the attorney was interviewed by a newspaper reporter. Which of the following statements, if believed by the attorney to be true, would be improper for the attorney to make? "As stated in our pleadings, we expect to prove that other men could be the father of the plaintiff's child." "We have scientific medical tests proving that the defendant is sterile." "We have been unable to locate several people whose testimony will be helpful to us, and I implore them to contact me immediately." None of these statements is improper.

Answer choice B is correct. Under MRPC 3.6(a), a lawyer who is participating or has participated in the investigation or litigation of a matter is not permitted to make an extrajudicial statement that the lawyer knows or reasonably should know will be disseminated by means of public communication and will have a substantial likelihood of materially prejudicing an adjudicative proceeding in the matter. The comment to MRPC 3.6 indicates that a statement about the performance or results of any examination or test is likely to be improper under the rule. Given that the case is going to be tried before a jury, a newspaper headline with this statement could materially prejudice the adjudicative proceeding. Statement A is incorrect because it is proper, as it is a statement of the defense involved and repeats information that is contained in a public record, the pleadings in the case. Statement C is proper, as MRPC 3.6(b) allows a statement making a request for assistance in obtaining evidence.

A sole practitioner who specialized in family law shared office space with a sole practitioner who specialized in criminal law. Since they frequently referred matters to one another, they decided to formalize this arrangement by entering into a reciprocal referral agreement under which they each agreed to refer matters that fit within their respective specialties to each other. They agreed that the arrangement would remain in effect for one year, at which point they would review it. The agreement did not prohibit either attorney from referring cases to other attorneys, and neither hesitated to recommend a different attorney whom they believed would better handle a particular case. When making referrals to each other, neither attorney informed clients of their agreement. Were the attorneys' actions regarding the referral agreement proper? No, because reciprocal referral agreements are prohibited by the Model Rules of Professional Conduct. No, because they did not inform clients of the arrangement. Yes, because the arrangement was not exclusive. Yes, because the arrangement did not provide for either attorney to be paid for referring clients.

Answer choice B is correct. Under certain circumstances, an attorney may enter into a reciprocal referral agreement whereby each attorney agrees to refer clients to the other. The agreements must not be exclusive or of an indefinite duration, and the client must be informed of the nature and existence of the agreement. In this case, the clients were not informed of the agreement, and thus the attorneys did not act properly. Answer choice A is incorrect because reciprocal referral agreements are permitted by the Model Rules of Professional Conduct. Answer choice C is incorrect because although the agreement was not exclusive, the attorneys acted improperly when they failed to notify the clients of the agreement. Answer choice D is incorrect because although the attorneys did not pay each other for the referrals, they acted improperly by failing to inform clients of the agreement.

An attorney, who was recently admitted to the bar, has been appointed by the court as counsel for an indigent defendant charged with a felony. After consulting with the defendant and attempting for two days to prepare the case for trial, the attorney became convinced that he lacked the knowledge and experience to represent the defendant effectively. Which of the following would NOT be proper for the attorney? Request permission of the court to withdraw from representing the defendant because the attorney knows that he is not competent to handle the case. Request the court to appoint experienced co-counsel and grant a continuance to enable co-counsel to prepare the case. Explain the circumstances to the defendant and, if the defendant consents, proceed to represent him alone to the best of his ability. None of the above would be proper.

Answer choice C is correct because it is the only improper course of action. A lawyer's duty of competence may not be waived by the client. Answer choice A is incorrect, because, as an appointed lawyer who knows that he is not competent to represent a client, it is proper for the attorney to attempt to obtain the court's permission to withdraw from representation. Answer choice B is incorrect because it is proper for the attorney to seek to provide competent representation through the association with a lawyer of established competence in the field. MRPC 1.1, cmt 2. Answer choice D is incorrect because options A and B are proper courses of action for the attorney to take.

A woman applied for a fellowship after completing her graduate degree. The woman asked a state trial court judge to provide a letter of recommendation for her application. In the judge's letter, which was written on his official judicial letterhead, the judge explained that he, as the woman's godfather, had known her for her entire life. He went on to describe her work ethic and explained that she had shown a strong commitment to the issues supported by the fellowship for years. Were the judge's actions in writing the letter of recommendation proper? No, because a judge may not provide a letter of recommendation on official letterhead. No, because a judge may serve as a reference only for former employees. Yes, because a judge may use official letterhead for a personal reference letter. Yes, because there are no restrictions on the use of official letterhead for reference letters.

Answer choice C is correct. A judge may provide a reference or recommendation for an individual based upon the judge's personal knowledge. The judge may use official letterhead for a reference letter if the judge indicates that the reference is personal and if there is no likelihood that the use of the letterhead would reasonably be perceived as an attempt to use the judge's official position to exert pressure. In this case, the judge indicated that the reference was personal, and there is no indication that the judge was trying to exert inappropriate pressure on the fellowship selection committee. Accordingly, the judge's actions were proper. Answer choice A is incorrect because a judge may write a letter of reference on official letterhead as long as the conditions described above are satisfied. Answer choice B is incorrect because a judge may write a personal reference letter for individuals who never worked for the judge. Answer choice D is incorrect because a reference letter written on official letterhead must satisfy the conditions described above.

An attorney who maintained a solo law practice entered into substantive discussions with a law firm about joining the firm. As part of these discussions, the attorney revealed the identities of her clients, their status as current or former clients, and the matters for which she represented them. The purpose of these revelations, which did not compromise the attorney-client privilege or otherwise prejudice the clients, was to detect conflicts of interest that would arise from the attorney joining the firm. The attorney did not seek the informed consent of her clients before making these revelations. Due to an inability to resolve a potential conflict of interest, the attorney did not join the law firm. Were the attorney's revelations proper? No, because the attorney did not seek the informed consent of her clients before making these revelations. No, because the attorney did not join the law firm. Yes, because the purpose of the revelations was to detect conflicts of interest that would arise from the attorney joining the law firm. Yes, because the duty of confidentiality does not apply to disclosures to another attorney of information relating to the representation of a client.

Answer choice C is correct. Although an attorney has a duty of confidentiality with respect to information relating to the representation of a client, there is an exception for such disclosures that are made to detect and resolve conflicts of interest that occur when a lawyer is engaged in substantive discussions about changing employment or there is substantive discussion about a change in the composition or ownership of a firm. Answer choice A is incorrect because an attorney seeking to associate with a firm may make limited disclosure of confidential information without first seeking the informed consent of her clients unless the revelations would compromise the attorney-client privilege or otherwise prejudice the clients. Because these limitations did not apply here, the attorney did not need the informed consent of her clients before making the disclosures. Answer choice B is incorrect because the "detection of conflicts of interest" exception to the duty of confidentiality applies even when the attorney does not change employment, provided the disclosures are not made until substantive discussions regarding the new relationship have occurred. Answer choice D is incorrect because there is no blanket exception to the duty of confidentiality for disclosures regarding a client made by an attorney to another attorney. In fact, most such disclosures would violate this duty.

A criminal defendant who was to be a witness at his own trial told his publicly provided attorney that he intended to testify that he was not at the scene of the crime. The attorney, without knowledge of the defendant's whereabouts at the time of the crime but reasonably believing, based on other evidence, that such testimony would be a lie, counseled the defendant about his obligation to testify truthfully. At trial, even though the defendant indicated that he had not had a change of heart, the attorney called the defendant as a witness, and the defendant testified that he was not at the scene of the crime. Is the attorney subject to discipline for permitting the defendant to testify about his whereabouts at the time of the crime? Yes, because the attorney reasonably believed that the client was going to lie. Yes, because the attorney has a duty of candor toward the tribunal. No, because the witness was a criminal defendant. No, because the attorney was publicly provided.

Answer choice C is correct. An attorney for a criminal defendant cannot refuse to offer the defendant's testimony when the attorney reasonably believes, but does not know, that the testimony will be false. Consequently, the attorney in this question was required to permit the defendant to testify, and therefore he is not subject to discipline for doing so. Answer choice A is incorrect because, while an attorney may generally refuse to offer evidence that the attorney reasonably believes is false, an attorney is subject to discipline only when the attorney knowingly offers false evidence. Answer choice B is incorrect because, although an attorney has a duty of candor toward the tribunal, the attorney does not breach this duty by offering evidence that the attorney reasonably believes, but does not know, is false. Answer choice D is incorrect because a publicly provided attorney is not subject to special rules with regard to the duty of candor.

An attorney represented the defendant in an aggravated assault action. While the client was in jail, he was attacked by another inmate and received a concussion. The attorney had his client immediately evaluated by a physician to ensure that the concussion did not impact the defendant's ability to recount the events of the assault. Hours before the client was to testify, the physician called the attorney to tell him that the concussion did not cause any memory problems but that a secondary psychiatric opinion noted that the defendant did have a personality disorder. The psychiatrist noted that the disorder would not affect the client's decision-making process or ability to recount events, but it should be treated with intensive, in-patient therapy as soon as possible to improve quality of life. The attorney believed that the trial would end in the next day or two if they proceeded, and because he had a trial-heavy few months after the trial in question, he did not want to delay the trial further by immediately sending his client for treatment. He was confident that delaying treatment for a few days would not harm his client and that the client was competent to testify, as the trial was nearly over. The attorney therefore waited to tell his client about the diagnosis until after the trial was complete. Is the attorney subject to discipline for withholding this information from the client? No, because the attorney did not believe that the delay would harm the client. No, because the attorney followed the correct process for evaluating a client's mental status. Yes, because the attorney withheld the information to serve his own interests. Yes, because the attorney withheld information relating to a psychiatric diagnosis.

Answer choice C is correct. An attorney may be justified in delaying transmission of information to a client when the client would be likely to react imprudently to an immediate communication. Thus, an attorney might withhold a psychiatric diagnosis of a client when the examining psychiatrist indicates that such disclosure would harm the client. An attorney may not, however, withhold information to serve the attorney's own interests or convenience or the interests or convenience of another person. Here, the attorney withheld the information because a delay in the trial would be inconvenient for his own schedule, and thus, his actions are improper. Answer choice A is incorrect because the attorney's motivation in withholding the information was related to his own interests rather than concern over harming the client. Answer choice B is incorrect because the attorney's actions following his receipt of the results of the client's evaluation did not follow the correct process. Answer choice D is incorrect because an attorney may, in some circumstances, withhold certain information from a client, including information about a psychiatric diagnosis. In this case, however, the attorney's motivation for withholding it would subject him to discipline.

An author engaged an attorney to represent him in a copyright infringement action. The representation agreement contained a provision that required the parties to submit all disputes, including malpractice disputes, to arbitration. The attorney did not discuss this provision with the author, nor did he advise him to seek independent counsel with regard to it. The infringement action was successful, and no dispute arose between the author and the attorney. Consequently, the provision was not enforced. The applicable jurisdiction recognizes the validity of arbitration in this context. Is the attorney subject to discipline? No, because the validity of arbitration is recognized by the applicable jurisdiction. No, because the provision was not enforced. Yes, because the attorney failed to inform the author as to the scope and effect of the arbitration section of the representation agreement. Yes, because the attorney failed to advise to the author to seek independent counsel with regard to the arbitration section of the representation agreement.

Answer choice C is correct. An attorney may enter into an arbitration agreement with a client with regard to a malpractice dispute if such an agreement is permitted by the applicable jurisdiction. However, the client must be fully informed as to the scope and effect of the agreement. Because the attorney failed to provide the client with such information, the attorney is subject to discipline for this failure. Answer choice A is incorrect because, although an arbitration provision is only enforceable if it is permitted by the applicable jurisdiction, in order to adhere to his ethical obligations, an attorney must ensure that the client is fully informed as to the scope and effect of the agreement. Answer choice B is incorrect because the lack of enforcement of the provision can affect whether the attorney is sanctioned for the violation, but it does not erase the attorney's failure to adhere to his ethical obligations. Answer choice D is incorrect because, while the client must be fully informed as to the scope and effect of the arbitration agreement, the attorney is not required to advise the client to seek independent counsel.

An elderly client hired an attorney to amend his will to provide the client's nephew with a specific tract of heavily wooded property that he knew the nephew loved to use for hunting. The wooded property was one of many tracts of land on a vast stretch of property owned by the client. After the client died, the nephew discovered that the tract of land actually left to him was a patch of rocky shore abutting a large lake two plots down from the wooded property. The error occurred because the attorney had mistakenly listed the incorrect address for the property that the client sought to transfer to his nephew. The nephew, having been told by his grandfather that he would receive the wooded property, filed an action to have the wooded property transferred to him. During the estate proceedings, the court held that there was insufficient evidence to establish that the client intended to transfer the wooded property to the nephew. They instead awarded him the rocky land abutting the lake. The nephew filed a negligence action against the attorney. The attorney moved to dismiss the nephew's claim and argued that he owed no duty to the nephew. Would the nephew be entitled to file such a claim under a negligence theory? No, because the attorney did not owe a duty to the nephew. No, because the attorney did not represent the nephew. Yes, because the attorney knew that his client wanted the will to provide the nephew with the wooded property. Yes, because the attorney breached a duty to his client.

Answer choice C is correct. An attorney may owe a duty to a non-client when the attorney knows that the client intends the attorney's services to benefit the non-client. An attorney will owe a duty to a non-client to use care when and to the extent that: (i) the attorney knows that the client intends as one of the primary objectives of the representation that the attorney's services benefit the non-client, (ii) such duty would not significantly impair the attorney's performance of obligations to the client, and (iii) the absence of such duty would make enforcement of those obligations to the client unlikely. Here, the attorney knew that the client hired him to draft a will to benefit the nephew, and nothing about the arrangement seems to have impaired the attorney's ability to represent the client adequately. Answer choice A is incorrect because in this instance, the attorney would owe a duty to the nephew. Answer choice B is incorrect because attorneys may owe duties to parties other than their clients. As explained above, the attorney in this case did owe a duty to the nephew even though the nephew is not his client. Answer choice D is incorrect because the nephew, a third party, would not recover only because the attorney breached a duty to his client, the uncle. Rather, the nephew would be able to recover under a negligence theory in his own right, because the attorney owed a duty to the nephew as an intended beneficiary of the representation.

An attorney was at a party talking with his friend when the friend confessed that while serving as trustee of a now defunct charitable trust, she had embezzled funds from the trust. The attorney urged the friend, who was also a lawyer, to set matters straight, but otherwise took no action. The friend did not take any remedial action. Was the attorney's conduct proper? Yes, because the friend was serving as a trustee rather than an attorney at the time that the friend embezzled the funds, so the friend's conduct did not constitute a violation of the Model Rules of Professional Conduct. Yes, because the attorney did not learn of his friend's misconduct as a legal representative. No, because the attorney failed to inform the appropriate authority that another lawyer had embezzled trust funds. No, because the friend failed to take remedial action.

Answer choice C is correct. An attorney must inform the appropriate authority that another lawyer has committed a violation of the Model Rules of Professional Conduct that raises a substantial question as to that lawyer's honesty, trustworthiness, or fitness as a lawyer when the attorney has actual knowledge of the misconduct. Here, the friend's admission to the commission of the crime of embezzlement raises a substantial question as to her honesty and trustworthiness as a lawyer. Answer choice A is incorrect because the substantial question as to an attorney's honesty and trustworthiness need not arise from the attorney's conduct as an attorney, but can stem from any other activities, such as serving as a trustee. Answer choice B is incorrect because an attorney's duty to report another attorney's misconduct can arise from information learned by an attorney while engaged in nonlegal activities, such as talking with a friend. Answer choice D is incorrect because an attorney's duty to disclose another attorney's misconduct is not impacted by any remedial action taken by the attorney who participated in the misconduct.

An attorney and her client met to discuss a discovery request from the opposing party. One item requested was a computer disk that contained evidence of the client's fraudulent behavior. The defendant told her attorney that if the attorney turned over the disk, the client would go "straight to jail." The attorney told the client that her words were protected by attorney-client privilege, and therefore, he would not reveal what she just stated. The attorney told the client to destroy the disk because the attorney was afraid that if he saw the information included on the disk, he might have to withdraw from representing her. Would the attorney's advice to his client subject him to discipline? No, unless the client actually destroyed the disk. No, because the attorney did not know if the disk included any relevant information. Yes, because the attorney counseled the client to destroy the disk. Yes, because the attorney did not report the information to the appropriate legal authority.

Answer choice C is correct. An attorney must not unlawfully obstruct another party's access to evidence or unlawfully alter, destroy, or conceal a document or other material having potential evidentiary value. An attorney must also not counsel another person to do or assist another person in doing any such act. This rule applies to evidentiary material generally, including computerized information. By telling the client to destroy the disk, which had clear evidentiary value, the attorney subjected himself to discipline. Answer choice A is incorrect because merely advising the client to destroy the disk would subject the attorney to discipline, regardless of whether the client followed the advice. Answer choice B is incorrect because the attorney was aware that the disk had evidentiary value, particularly in light of the client's comments. Answer choice D is incorrect because the attorney was not required to report this information.

An attorney represented a client who the attorney reasonably believed had a severe mental disorder affecting her ability to make rational personal and business decisions. Rather than consulting a physician or other mental healthcare professional about how to deal with his client, the attorney sought the advice of another attorney, who specialized in representing other attorneys in malpractice actions, about his professional obligations to his client. During his discussions with the other attorney, the attorney disclosed information that the client had revealed to him. Was the attorney's disclosure of information learned from his client proper? No, because the disclosed information was acquired during the course of the attorney's representation of the client. No, because the lawyer failed to consult a physician or mental healthcare professional first. Yes, because the lawyer may disclose this information to obtain advice regarding his professional obligations. Yes, because the lawyer for a client under a disability must disclose information necessary for protective action.

Answer choice C is correct. An attorney's duty of confidentiality does not prohibit an attorney from obtaining confidential legal advice about the attorney's responsibility to comply with the Model Rules of Professional Conduct. Generally, disclosing information to obtain such advice would be impliedly authorized for the attorney to carry out the representation. Even if not impliedly authorized, the Model Rules of Professional Conduct allow such disclosure. Answer choice A is incorrect because even if the information was acquired in the course of the representation, the attorney may still disclose the information to obtain confidential legal advice. Answer choice B is incorrect because the attorney does not have such an obligation before securing confidential legal advice on how to proceed in accordance with the Model Rules of Professional Conduct. Answer choice D is incorrect because the attorney does not have to reveal information about a client under a disability in order to take protective measures for that client. However, the Rules permit the attorney to make such disclosures to the extent reasonably necessary to protect the client's interests.

During a conversation among friends in his home, an attorney called into question whether a candidate for the office of attorney general met the statutory requirements. Specifically, the attorney stated that he had heard a rumor that the candidate had not been licensed to practice law in the state for the requisite number of years mandated by statute. When asked whether he had investigated this matter, the attorney replied that he didn't need to check the facts, it had to be true. The candidate had, in fact, been licensed to practice law for the statutorily required time period. Is the attorney subject to discipline? No, because the statement did not concern a current member of the judiciary or a candidate for judicial office. No, because the statement was not made in a public forum. Yes, because the false statement was made with a reckless disregard for its truth or falsity. Yes, because the statement concerned a candidate for public office.

Answer choice C is correct. False statements by a lawyer can unfairly undermine public confidence in the administration of justice. The prohibition on such statements applies to the qualifications or integrity of not only a judge or a candidate for judicial office, but also a public legal officer, such as an attorney general, or a candidate for such office. Consequently, a lawyer must not make a statement that the lawyer knows to be false or with reckless disregard as to its truth or falsity (i.e., a false statement made with actual malice) concerning the qualifications or integrity of a judge, adjudicatory officer, or public legal officer, or of a candidate for election or appointment to judicial or legal office. Because the attorney evidenced a reckless disregard for the truth or falsity of his statement regarding the failure of the attorney general candidate to meet the statutory requirements, the attorney is subject to discipline. Answer choice A is incorrect because, as noted with regard to answer choice C, the prohibition on a false statement made with actual malice about the qualifications or integrity of an individual applies not only to a judge or a candidate for judicial office, but also to a public legal officer, such as an attorney general, or a candidate for such office. Answer choice B is incorrect because the prohibition applies to false statements made with actual malice, regardless of whether the statements are made in private or in public. Answer choice D is incorrect because the prohibition on false statements does not apply to any candidate for a public office, but is limited to candidates for judicial or legal offices, such as judges, attorney generals, prosecuting attorneys, and public defenders.

The chair of one of a state's political parties approached a law firm's senior partner, who was very active within that party, about representing the political party in litigation opposing a ballot measure backed by the state's opposing party. A junior partner at the firm was an active member of the opposing party and had contributed substantial time and money toward getting the measure on the ballot. The firm had never previously represented any political party or organization. After discussing the matter with the firm's management, who were all aware of the junior partner's involvement with getting the measure on the ballot, the senior partner decided to represent the political party. Is the senior partner's representation of the state's political party proper? No, because one of the firm's partners has a conflict of interest. No, because the junior partner's conflict of interest would be imputed to the other partners. Yes, because the junior partner's conflict of interest would not be imputed to the other partners. Yes, because there is no conflict of interest.

Answer choice C is correct. In general, if one attorney in a firm is prohibited by the conflict-of-interest rules from representing a client, that prohibition applies to all other attorneys in the firm. If the prohibition is based on a personal interest of the disqualified attorney and does not present a significant risk of materially limiting the representation of the client by the remaining attorneys in the firm, then the disqualified attorney's conflict will not be imputed to the firm. In this case, although the junior partner had a conflict of interest given his involvement in getting the measure on the ballot, this conflict would be based on his personal interest. Because there is no indication that the junior partner's interest would materially limit the representation of the client by the remaining attorneys in the firm, the conflict would not be imputed to the other attorneys. Answer choice A is incorrect because although the junior partner likely would have a conflict of interest, this would not prevent another attorney from representing the client unless the conflict was imputed to the other attorneys. Answer choice B is incorrect because the conflict would not be imputed to the other attorneys. Answer choice D is incorrect because the junior partner had a conflict of interest based on his personal involvement in placing the measure on the ballot

An attorney admitted to the bar never practiced law, but instead took a job as a financial planner. After several years as a successful financial planner, one of the attorney's clients accused him of lying to her about the entities in which her funds had been invested. The attorney admitted that he had lied to the client but argued he had been acting in her best interest because she was overly risk averse. The attorney pointed out that the client had actually made more money for her than she would have made otherwise. The client filed a negligence action against the attorney and reported him to the local disciplinary commission. Would the attorney's actions as a financial planner subject him to discipline? No, because he was not engaged in the practice of law. No, because the client did not suffer any harm. Yes, because he engaged in conduct involving dishonesty. Yes, because the client filed a negligence action against the attorney.

Answer choice C is correct. It is professional misconduct for an attorney to engage in conduct involving dishonesty, fraud, deceit, or misrepresentation. An attorney need not be engaged in the practice of law to violate the Model Rules of Professional Conduct and be subject to discipline. Because the attorney engaged in conduct involving dishonesty, he would be subject to discipline. Answer choice A is incorrect because an attorney need not be engaged in the practice of law to be subject to discipline. Answer choice B is incorrect because an attorney is subject to discipline if he engages in conduct involving dishonesty, regardless of whether it results in actual harm. Answer choice D is incorrect because the mere fact that a person has filed a legal action against an attorney is not in and of itself evidence of professional misconduct.

An attorney who had represented a client for many years prepared the client's will and acted as one of the two subscribing witnesses to its execution. The will gave 10 percent of the client's estate to her housekeeper, 10 percent to her son and sole heir, and the residue to charity. Upon the client's death one year later, the executor named in the will asked the attorney to represent him in probating the will and administering the estate. At that time, the executor informed the attorney that the son had notified him that he would contest the probate of the will on the grounds that the client lacked the required mental capacity at the time the will was executed. The attorney believes that the client was fully competent at all times and will so testify, if called as a witness. The other subscribing witness to the client's will pre-deceased the client. Is it proper for the attorney to represent the executor in the probate of the will? Yes, because the attorney is the sole surviving witness to the execution of the will. Yes, because the attorney's testimony will support the validity of the will. You Selected: No, because the attorney will be called to testify on a contested issue of fact. No, because the attorney will be representing an interest adverse to the client's heir at law.

Answer choice C is correct. MRPC 3.7(a) states that a lawyer may not serve as an advocate at a trial in which the lawyer is likely to be a necessary witness relating to a contested issue. Answer choice A is incorrect because the fact that the attorney is the sole surviving witness to the will's execution means that the attorney is likely to be a necessary witness regarding the client's mental capacity when she executed the will. The attorney, therefore, may not also represent the executor in connection with the will contest, per MRPC 3.7(a). Answer choice B is incorrect because, under MRPC 3.7(a), a lawyer may not represent a party at a trial in which the lawyer is likely to be a necessary witness relating to a contested issue, even if the testimony supports the party's position. Answer choice D is incorrect, as the fact that the attorney would be representing a party (the executor) who is adverse to the deceased client's heir is irrelevant. But for the fact that the attorney is a likely witness, the representation would be proper.

A home builder employed an attorney to draft a contract of sale for a house the builder had constructed. The builder sold the house to a buyer who assumed a personally significant debt in order to purchase the house. After the sale, a friend of the builder's told the attorney that the builder, in violation of the law, had built the house on a landfill. The attorney contacted the builder and urged the builder to discuss the matter with the buyer. Although the builder admitted to the criminal act, the builder refused. The lawyer took no further action. Subsequently, although the landfill did not represent a health threat, the buyer had to sell the house for less than half of what he had paid for it. Is the attorney subject to discipline for failing to reveal the builder's criminal action to the buyer? Yes, because the client had used the attorney's services in furtherance of a crime. Yes, because the crime resulted in substantial injury to the financial interests of the buyer. No, because of the duty of confidentiality. No, because, at the time that the attorney learned of the crime, it had already occurred.

Answer choice C is correct. Subject to a few exceptions, a lawyer is prohibited from disclosing information relating to the representation of a client, whether that information was obtained from a third party or from the client, and whether it was acquired after the termination of the representation. While there is an exception for the disclosure of confidential information to prevent, mitigate, or rectify substantial financial harm to another based on a client's fraud or crime in furtherance of which the client has used the lawyer's services, the exception allows but does not require disclosure. Accordingly, the attorney is not subject to discipline for keeping the information confidential. Answer choice A is incorrect because, although the client had used the attorney's services in furtherance of a crime, the applicable exception that permits disclosure of confidential information in such circumstances does not require such disclosure. Answer choice B is incorrect because, although the crime resulted in substantial injury to the financial interests of the buyer, disclosure is not required. Answer choice D is incorrect because the exception to the disclosure of confidential information based on a client's fraud or crime encompasses situations in which the attorney learns of the crime after it has occurred. However, the exception does not require the attorney to disclose confidential information, but merely permits it.

An attorney represented a criminal defendant in a murder case. The attorney's client told the attorney that he wanted to testify in his own defense. The attorney tried to dissuade his client from testifying, in part because although he had no evidence to support it, he suspected that the client's version of the events was fabricated. The attorney also believed that the jury would not believe his client. The client insisted, however, that he testify. The attorney told his client that he would call the client to the stand but reminded him that he was under oath and that it was imperative that he answer all questions truthfully. At trial, the client testified in his own defense, and the prosecution subsequently impeached the defendant's testimony. Was the attorney's action in allowing his client to testify proper? No, because the attorney reasonably believed that his client's testimony would be false. No, because the attorney was required to take remedial measures. Yes, because the attorney did not know that the client's testimony would be false. Yes, because an attorney is required to respect a criminal defendant's decision to testify.

Answer choice C is correct. The Model Rules of Professional Conduct do not permit an attorney to refuse to offer the testimony of a client who is a criminal defendant when the attorney reasonably believes but does not know that the testimony will be false. Unless the attorney knows that the testimony will be false, the attorney must honor the criminal defendant's decision to testify. In this case, because the attorney believed but did not know that the defendant's testimony would be false, the attorney was required to allow the defendant to testify. Answer choice A is incorrect because an attorney must honor a defendant's decision to testify when the attorney reasonably believes but does not know that the testimony will be false. Answer choice B is incorrect because an attorney is required to take remedial measures only if the attorney knows of the falsity of the defendant's testimony. Answer choice D is incorrect because an attorney is not required to call the defendant to the stand when the attorney knows that the defendant will testify falsely. In such a case, if the attorney cannot dissuade the defendant from testifying falsely, the attorney must withdraw from the case.

An attorney who is a sole practitioner limits his practice to personal injury cases. He regularly places advertisements in local newspapers, stating that his practice is limited to personal injury cases, including medical malpractice. After seeing one of the attorney's ads, a man approached the attorney for representation in a medical malpractice case. After a 30-minute interview, the attorney told the man that he was too busy to take his case because it appeared quite complicated. He further offered to refer the man to another lawyer who regularly practiced in the field. He reminded the man that he should see another lawyer promptly before the statute of limitations expired and he lost his right to sue. Although the attorney did not charge the man for the interview, the man was upset at wasting 30 minutes of his time. The man did not contact another lawyer until eight months later, when he learned that the statute of limitations on his claim had expired six months after his interview with the attorney. In fact, the man had a meritorious medical malpractice claim. Is the attorney subject to civil liability? Yes, because the attorney falsely advertised his availability for medical malpractice cases. Yes, because the attorney did not advise the man as to the date the statute of limitations would expire. No, because the attorney did not violate any duty owed to the man. No, because the attorney offered to refer the man to another medical malpractice lawyer.

Answer choice C is correct. The attorney talked to the prospective client, declined the representation, gave the man another lawyer's name, and urged him to contact that lawyer on a timely basis. Under MRPC 1.18, the attorney did not owe the man any other duty. Answer choice A is incorrect because advertising in and of itself does not create an attorney-client relationship. The fact that the attorney advertised in this manner did not require that he be available for all medical malpractice cases. Answer choice B is incorrect because the attorney declined to accept the matter and gave the man information about another lawyer and about the need to see that lawyer on a timely basis. The attorney did not have a duty to give the man the exact date the statute would run, unless that date happened to be in a very short time, such as within the next day or two. Answer choice D is incorrect because the referral was not necessary under the rules, so that is not the reason why the attorney is not subject to civil liability. After he declined to accept the matter, the attorney told the man that he should see another lawyer on a timely basis. Under MRPC 1.18, the attorney did not owe the man any other duty.

A plaintiff and a defendant are next-door neighbors and bitter personal enemies. The plaintiff is suing the defendant over an alleged trespass. Each party believes, in good faith, in the correctness of his position. Each is represented by their own attorney. After the plaintiff had retained his attorney, he told her, "I do not want you to grant any delays or courtesies to the defendant or his lawyer. I want you to insist on every technicality." The plaintiff's attorney has served the defendant's attorney with a demand to answer written interrogatories. The defendant's attorney, because of the illness of his secretary, has asked the plaintiff's attorney for a five-day extension of time within which to answer them. Is the plaintiff's attorney subject to discipline if she grants the opposing attorney's request for a five-day extension? Yes, because she is acting contrary to her client's instructions. Yes, unless she first informs the plaintiff of the request and obtains his consent to grant it. No, unless granting the extension would prejudice the plaintiff's rights. No, because the defendant's attorney was not at fault in causing the delay

Answer choice C is correct. The lawyer generally controls the means by which the objectives of representation are to be pursued, but the lawyer must reasonably consult with the client. MRPC 1.2 and 1.4. A lawyer is not required to obey every instruction of the client with regard to the means of representation and would not be subject to discipline for a tactical decision to grant a five-day extension, as long as the client's rights would not be prejudiced as a result of the delay. Answer choice A is not correct, as a lawyer generally controls the decisions with regard to the means by which representation will be pursued and can act contrary to a client's instructions, so long as the client is reasonably consulted. Answer choice B is not correct, as the Model Rules require reasonable consultation with the client, but not necessarily prior consultation, and the Rules do not require the client's consent as to the means by which the objectives of representation are to be pursued. Answer choice D is not correct, as the issue of fault is irrelevant. The plaintiff's attorney has discretion under the Model Rules to grant the extension, so long as it would not prejudice the plaintiff's rights.

An attorney, with her client's approval, settled a claim against a defendant for $60,000. The settlement agreement provided that one-half would be paid by the defendant's primary insurance carrier, and one-half by a co-insurer. The attorney's agreed fee was 30% of the amount of the settlement. The attorney received the primary insurance carrier's check for $30,000 and a letter from the co-insurer advising that its check would be sent in two weeks. The attorney promptly advised the client and deposited the $30,000 in her Clients' Trust Account. The client demanded that the attorney send him the entire $30,000 and take her fee out of the funds to be received from the co-insurer. Which of the following would not be proper for the attorney? Send the client $30,000. Send the client $21,000 and retain $9,000 in her Clients' Trust Account. Send the client $21,000 and transfer $9,000 to her personal account. All of the above would be proper.

Answer choice C is correct. The undisputed portion of the funds must be promptly distributed. MRPC 1.15, cmt 3. The $9,000 in dispute, however, is required to be kept separately from the attorney's own funds. Answer choice A is incorrect because it is proper. The attorney can follow the client's instructions and take her fee from the payment to be made by the co-insurer. Answer choice B is incorrect because is it proper. The attorney has a claim to the $9,000 that is disputed by the client; under such circumstances, she may properly keep the $9,000 in the Clients' Trust Account until the claim is resolved, but she must send any undisputed funds to the client. Answer choice D is incorrect because, as described above, answer choice C would not be proper.

During the closing argument to the jury in a civil tax fraud case, an attorney representing the government quoted a portion of the defendant's testimony and then made the following three statements. Which of these statements, if any, would be improper? "That testimony of the defendant directly contradicts the testimony of two witnesses for the government." "I ask you, who has the reason to lie, the two witnesses for the government or the defendant?" "I can truthfully say I have never seen a witness less worthy of belief." None of these statements is improper.

Answer choice C is correct. This statement is not proper, as it states a personal opinion as to the credibility of a witness in violation of MRPC 3.4. Statement A is proper, as it is drawing conclusions based on specific facts, is relevant, and is supported by admissible evidence. Statement B is proper, as a rhetorical question is permissible so long as a lawyer does not allude to any matter that the lawyer does not reasonably believe is relevant or that will not be supported by admissible evidence, assert personal knowledge of facts in issue, or state a personal opinion as to the justness of a cause, the credibility of a witness, or the culpability of a civil litigant.

An attorney agreed to represent a client in a lawsuit. The attorney and the client executed the attorney's preprinted retainer form that provides, in part: "The client agrees to pay promptly the attorney's fee for services. In addition, the client and the attorney agree to release each other from any and all liability arising from the representation. The client agrees that the attorney need not return the client's file prior to receiving the client's executed release. Upon full payment, the attorney will return the file to the client." Although the attorney recommended that the client consult independent counsel before signing the retainer agreement, the client chose not to do so. The attorney reasonably believes that his fee is fair and that the quality of his work will be competent. Is the attorney's retainer agreement with the client proper? Yes, because the attorney furnished consideration by agreeing to return the client's file. Yes, because the attorney reasonably believes that his fee is fair and that the quality of his work will be competent. No, because the attorney is attempting to limit his liability for malpractice. No, because the attorney uses a pre-printed form for all retainers.

Answer choice C is correct. Under MRPC 1.8(h), a lawyer may not limit his liability prospectively unless the client is independently represented in making such an agreement. Answer choice A is incorrect because whether there is consideration for the agreement is not relevant. Answer choice B is incorrect because the reasonableness of the proposed fee and the quality of the work are not relevant to the propriety of the agreement. Answer choice D is incorrect because the form of the agreement is not relevant to whether the agreement is proper.

Attorney Alpha serves on a bar association committee established to counsel and rehabilitate lawyers who suffer from substance abuse. The day before Alpha was to leave on a fishing trip, Alpha's close friend, Attorney Beta, disclosed to Alpha that, over the preceding two years, Beta had become heavily addicted to cocaine and was afraid he had committed criminal offenses in his banking activities as a result of his addiction. Beta asked Alpha to represent him. Alpha agreed, but explained that Alpha could do little for two weeks and would consult with Beta immediately upon Alpha's return. While on the fishing trip, an accountant, who knew that Alpha represented Beta, told Alpha that the accountant had been retained by the trust department of a commercial bank to audit several substantial trust accounts in which the bank and Beta are co-trustees. The accountant also told Alpha that the audit furnished incontrovertible proof that Beta had embezzled more than $100,000 from the trust accounts. Must Alpha report Beta's embezzlement to the appropriate disciplinary authority? Yes, because Alpha learned of Beta's embezzlement from the accountant. Yes, because Alpha's failure to report would assist the concealment of Beta's breach of trust. No, because Alpha gained the information while representing Beta. No, because the information will probably be made public by the bank.

Answer choice C is correct. Under MRPC 8.3(c), when a lawyer learns of attorney misconduct through a communication protected by the ethical duty of confidentiality, the lawyer is not required to report the misconduct. Here, the information Alpha knows was obtained in the course of representing Beta and is protected by the ethical duty of confidentiality. Thus, Alpha is not required to report the embezzlement. Answer choice A is not correct, as the ethical duty of confidentiality requires information obtained in the course of the representation of a client to be kept confidential, even if it is not obtained directly from the client. Answer choice B is not correct, as the ethical rule of confidentiality controls and does not require disclosure even if the lack of disclosure would conceal past fraud or crime. Answer choice D is not correct, as the fact that the information might be made public by the bank does not override Alpha's ethical duty of confidentiality to protect information learned in the course of representing Beta.

An attorney has a highly efficient staff of paraprofessional legal assistants, all of whom are graduates of recognized legal assistant educational programs. Recently, the statute of limitations ran against a claim of a client of the attorney's when a legal assistant negligently misplaced the client's file and suit was not filed within the time permitted by law. Which of the following correctly states the attorney's professional responsibility? The attorney is subject to civil liability and is also subject to discipline on the theory of respondeat superior. The attorney is subject to civil liability or is subject to discipline at the client's election. The attorney is subject to civil liability but is NOT subject to discipline unless the attorney failed to supervise the legal assistant adequately. The attorney is NOT subject to civil liability and is NOT subject to discipline if the attorney personally was not negligent.

Answer choice C is correct. Under the doctrine of respondeat superior, as the supervisor of the legal assistant, the attorney would be subject to civil liability for malpractice for allowing the statute of limitations to expire. Unless the attorney failed to adequately supervise the legal assistant, however, the attorney would not be subject to discipline under the Model Rules. Under MRPC 5.3(c), a lawyer will be responsible for the conduct of a non-lawyer assistant that would be a violation of the Model Rules if: (i) the lawyer orders or, with the knowledge of the specific conduct, ratifies the conduct involved, or (ii) the lawyer is a partner or has comparable managerial authority in the law firm in which the non-lawyer assistant is employed, or has direct supervisory authority over the person, and knows of the conduct at a time when its consequences can be avoided or mitigated but fails to take reasonable remedial action. Answer choice A is not correct, as the Model Rules do not subject an attorney to discipline on the theory of respondeat superior. A lawyer is subject to discipline only if the lawyer orders or specifically ratifies the conduct. Answer choice B is not correct because even though a client can make a complaint seeking disciplinary action, only the jurisdiction's disciplinary authority can subject a lawyer to discipline. Answer choice D is not correct, as the attorney would be subject to civil liability for malpractice under the doctrine of respondeat superior.

A farmer asked his attorney to draft a letter in connection with a loan transaction. The letter issued by the attorney stated that the farm equipment to be pledged by the farmer to the lender to secure the loan was not subject to prior liens. The letter did not indicate that the attorney had relied on the farmer's statement to that effect and had not performed, as custom would require, a search of the relevant public records. Such a search would have revealed that the farmer's statement was false. At the farmer's direction, the attorney, unaware that the farmer had misinformed him as to the existence of the liens, sent the letter to the lender, who relied on it in making the loan. Subsequently, the farmer defaulted on the loan and, due to the prior liens, the lender was unable to recoup the outstanding balance owed by the farmer on the loan. Is the lawyer likely subject to civil liability to the lender? No, because the lender was not the attorney's client. No, because the attorney relied on his client's statement. Yes, because the attorney failed to comply with his duty of care with regard to the letter. Yes, because, by issuing the letter, the attorney became a guarantor of the loan.

Answer choice C is correct. While generally a lawyer does not owe a duty to a non-client, a lawyer can owe a duty of care to a non-client when the non-client has been invited to rely on the opinion or legal services of the lawyer, and the non-client does so. This is particularly true when the lawyer is aware that his opinion is being used with regard to a specific transaction and is being supplied to a particular person, rather than the public in general. In this instance, since the attorney failed to search publicly available records, as custom would require, he violated that duty of care. Moreover, the lender was harmed by the attorney's breach of this duty. Answer choice A is incorrect because, although generally a lawyer does not owe a duty to a non-client, a lawyer may owe a duty to a non-client when the non-client has been invited to rely on the opinion or legal services of the lawyer, and the non-client does so to her detriment. Answer choice B is incorrect because, although a lawyer who relies on information provided by a client is generally not liable to the client for doing so, such reliance may not protect the lawyer from liability to a third party when, in so relying, the lawyer fails to exercise reasonable care. Answer choice D is incorrect because the attorney, in issuing an opinion, did not assume an obligation with respect to the loan itself.

A client, during a conference with her attorney in his office about the client's pending divorce, threatened to kill her husband, from whom she was separated. Based on his knowledge of the client, the attorney believed his client's threat to be credible. The attorney took no action to warn the client's husband. Shortly thereafter, the client shot her husband, seriously wounding him. Is the attorney subject to discipline for his failure to warn his client's husband? Yes, because the attorney had a duty to warn his client's husband. Yes, because the attorney's failure to warn the client's husband was the cause of the husband's harm. No, because, since the client's husband did not die, the client did not accomplish her threat. No, because the attorney owed his client a duty of confidentiality.

Answer choice D is correct. A lawyer has an ethical duty of confidentiality to his client, which includes protecting confidential communications received from a client. MRPC 1.6(a). Since the communication regarding the client's intention to kill her husband was received by the attorney in the course of his representation of the client in a divorce, the attorney had a duty not to reveal the client's threat, even to the subject of the threat, the client's husband. While the Model Rules allow a lawyer to reveal confidential information concerning the representation of a client to the extent that the lawyer reasonably believes disclosure is necessary to prevent reasonably certain death or bodily harm, the Model Rules do not require disclosure. Accordingly, the attorney here would not be subject to discipline for failing to warn the client's husband. Answer choice A is incorrect because, while a lawyer may reveal confidential information received from a client concerning the representation to the extent that the lawyer reasonably believes disclosure is necessary to prevent reasonably certain death or bodily harm, a lawyer is not obligated to do so. MRPC 1.6(b)(1). Answer choice B is incorrect because, while the causal connection between the attorney's failure to warn the client's husband and the husband's harm would be relevant in a civil action that seeks to impose liability on the attorney, it is not relevant in a disciplinary action based on that failure. Answer choice C is incorrect because whether the client succeeded in accomplishing her threat is irrelevant to the issue of whether the attorney had a duty to warn the client's husband of that threat.

An attorney wants to make it easier for her clients to pay their bills for her fees. Which of the following would NOT be proper for the attorney? Accept bank credit cards in payment of the attorney's fees. Allow clients to pay the attorney's fees in monthly installments. Arrange for clients to obtain bank loans for the purpose of paying the attorney's fees. If a case is interesting, suggest that the client give the attorney publication rights concerning the case as partial payment of the fee.

Answer choice D is correct. A lawyer is prohibited from negotiating for literary or media rights relating to representation of a client prior to the conclusion of the representation. MRPC 1.8(d). Answer choices A, B, and C are incorrect, because there is no prohibition on a lawyer accepting payment of a fee by credit card, permitting clients to make payments over time, or arranging a bank loan for a client for the purpose of paying the lawyer's fee.

An attorney opened a trust account at a local bank into which she deposited a check from a client that constituted an advance payment for future services to be rendered by the attorney. Subsequently, the attorney received an engagement fee from another client, which the attorney also deposited into the account. The engagement fee represented payment to the attorney for accepting the case, being available to handle the case, and agreeing not to represent another party in the case. The fee did not require the attorney to perform specific legal services. The attorney maintained records related to all account transactions in accordance with the state rules of professional conduct, which were identical to the ABA Model Rules of Professional Conduct, but she did not pay bank service charges on the account from her own funds. Is the attorney subject to discipline with regard to her actions concerning the trust fund account? No, because the attorney properly maintained the records related to the account. No, because the attorney maintained a trust fund account to keep her property separate from her client's property. Yes, because the attorney failed to pay the bank service charges on the account from her own funds. Yes, because the attorney did not keep her client's property separate from her own property.

Answer choice D is correct. A lawyer must hold funds that are in a lawyer's possession in connection with a representation separate from the lawyer's own funds. Funds must be kept in a separate client trust account, which generally must be maintained in the state in which the lawyer's office is situated. MRPC 1.15(a). The engagement fee was the attorney's property since the attorney was not required to perform future legal services. By placing the engagement fee into the trust account, the attorney improperly commingled her property with her client's property. Answer choice A is incorrect because, even though the attorney properly maintained the records related to the account, the attorney improperly deposited the engagement fee into the account. Answer choice B is incorrect because, although the attorney properly created the trust account to keep her client's property separate from her own, the attorney improperly deposited the engagement fee into the account. Answer choice C is incorrect because a lawyer is permitted but not required to pay the bank service charges on a client trust account from the lawyer's own funds.

An attorney met with a client on a Monday morning about the client's pending divorce. The client told the attorney that if his wife did not agree to a property settlement by the end of the week, he "was going to kill [her]." The attorney thought his client was simply being dramatic but was concerned because he knew that his client already owned a gun and had made similar statements before. The attorney worried that the client might be serious about the threat and decided that, if the case had not settled by Friday, he would call the client's wife and warn her. The client shot his wife on Thursday. Is the attorney subject to discipline for not disclosing the client's threat to kill his wife? Yes, because the attorney reasonably believed that the client was going to harm his wife if the case did not settle. Yes, because the attorney owed a duty of fairness to the wife as the other party in the civil action. No, because the threat was not imminent since the client claimed he would kill his wife at the end of the week if a settlement was not reached. No, because the attorney had no obligation to disclose information he thought was necessary to prevent future bodily harm.

Answer choice D is correct. An attorney may reveal confidential information concerning the representation of a client to the extent the attorney reasonably believes disclosure is necessary to prevent reasonably certain death or bodily harm. The attorney is not obligated to do so, however. Here, the attorney thought his client was simply being dramatic, and the client had not taken steps to harm his wife despite making similar statements in the past. Answer choice A is incorrect because the fact that the belief was reasonable does not require the attorney to disclose the information: it simply permits him to do so. Answer choice B is incorrect because although an attorney does owe a duty of fairness to an opposing party, this duty does not require the attorney to disclose the information learned as a consequence of representing his client. Answer choice C is incorrect because the attorney was not obligated to disclose the information even if the threat was imminent. He would have the option of revealing the information, however.

Two friends were involved in a car accident and hired an attorney to represent them as co-parties in a claim for damages related to the accident. Each client provided the attorney with the minimum amount he would be willing to accept to settle the case. The clients suggested vastly different amounts. The attorney received a large settlement offer that met the individual requirements of each party, though each party would receive significantly different amounts in the settlement. He called both clients into his office separately to discuss settlement. Both clients immediately consented in writing to the agreement, though neither was aware of the amount received by the other. The attorney then accepted the offer on behalf of both of the clients. The clients met for dinner the following night to celebrate their win, and during dessert, one of the clients disclosed to the other the amount he had received in the settlement. The other client, who received a far smaller settlement, immediately called the attorney to express his dissatisfaction with the settlement. The attorney responded by saying that he accepted the amount his client had told him was acceptable. Would the attorney's actions regarding the settlement subject him to discipline? No, because he did not accept an amount below each client's specified amount. No, because both clients consented in writing to the settlement. Yes, because the parties received different amounts in the settlement. Yes, because the attorney did not disclose the extent of both settlement offers to both clients.

Answer choice D is correct. An attorney representing co-parties may make an aggregate settlement only if both clients consent in writing after full consultation and disclosure by the attorney, including disclosure of the nature and extent of all claims and pleas, and the participation of each party in the settlement. Here, the attorney did not disclose the nature and extent of the offer and the participation of each party in the settlement before accepting the settlement offer. Answer choice A is incorrect because the attorney did not make a full disclosure about the settlement to each party before he accepted the offers; it is irrelevant whether the amount was aligned with the original amount specified by each party. Answer choice B is incorrect because the attorney did not make a full disclosure regarding the settlement to the clients before they accepted the settlement in writing. Answer choice C is incorrect because an attorney may represent co-parties receiving different settlement amounts as long as the attorney makes a full disclosure to each client.

An attorney successfully represented a woman charged with operating a house of prostitution. Subsequently, a famous politician who was charged with soliciting a prostitute within the alleged brothel sought to hire the attorney to represent him. The woman told the attorney that she did not object to his representation of any of the men charged with solicitation, but she refused to sign a written statement to that effect because she no longer wanted to be linked to the charges in any written document. Would it be proper for the attorney to accept the politician as a client? No, because the attorney represented the woman previously. No, because the politician's claim is substantially related to the matter in which the attorney represented the woman. Yes, because the attorney received the woman's informed oral consent. Yes, although the attorney did not receive the woman's informed consent in writing.

Answer choice D is correct. An attorney who has previously represented a client in a matter must not subsequently represent another person in the same or a substantially related matter in which that person's interests are materially adverse to the interests of the former client, unless the former client gives informed consent, confirmed in writing. Here, while the matters were substantially related, the interests of the politician and the woman were not adverse because both wanted to disprove that any illegal acts related to prostitution occurred. Answer choice A is incorrect because even though the attorney represented a former client in a substantially related matter, the interests of the woman and the politician are not adverse. Answer choice B is incorrect because the former and current clients do not have adverse interests. Answer choice C is incorrect because the woman's informed consent, oral or written, is not required because the clients do not have adverse interests.

An attorney was formerly employed by an insurance company as a lawyer solely to handle fire insurance claims. While so employed, she investigated a fire loss claim of a claimant against the insurance company. The attorney is now in private practice. The original claim was settled. One year after the attorney left the employ of the insurance company, the claimant slipped and fell in the insurance company's office. The claimant now asks the attorney to represent him or refer him to another lawyer for suit on the "slip and fall" claim. Which of the following would be proper for the attorney to do? Refuse to discuss the matter with the claimant. Represent the claimant. Give the claimant a list of lawyers who the attorney knows are competent and specialize in such claims. All of the above are proper.

Answer choice D is correct. Answer choice A is proper, as a lawyer is never required to discuss a matter with a potential client and generally is not under a duty to accept representation of any client (except for court appointments when no good cause exists to decline). Answer choice B is proper, because the Model Rules only prohibit a lawyer who has previously represented a client in a matter from subsequently representing another person in the same or a substantially related matter in which that person's interests are materially adverse to the interests of the former client and do not prohibit representation in an entirely different matter. MRPC 1.9(a). Answer choice C is proper, as there is no rule prohibiting the attorney from providing the claimant with a list of qualified lawyers who may be able to represent him. Answer choice D includes all of the proper options, and is therefore the best choice.

Attorney represented Client in a personal injury action against the driver of the car in which Client was injured while a passenger. The personal injury action was settled, and Attorney received a check in the amount of $10,000 payable to Attorney. Attorney deposited the check in her Clients' Trust Account. One day later, Attorney received a letter from Bank, which had heard of the settlement of the personal injury lawsuit. Bank informed Attorney that Client had failed to make his monthly mortgage payments for the last three months and demanded that Attorney immediately release $900 of the proceeds of the settlement to Bank or Bank would institute mortgage foreclosure proceedings against Client. Attorney informed Client of Bank's letter. Client responded: "I don't care what Bank does. The property is essentially worthless, so let Bank foreclose. If Bank wants to sue me, I'll be easy enough to find. I don't think they'll even bother. You just take your legal fees and turn the rest of the proceeds over to me." Is Attorney subject to discipline if she follows Client's instructions? Yes, if Client does not dispute the $900 debt to Bank. Yes, because Attorney knew that client was planning to force Bank to sue him. No, unless Attorney had reason to believe that Client would not have sufficient funds to pay any subsequent judgment obtained by Bank. No, because Bank has no established right to the specific proceeds of Client's personal injury judgment.

Answer choice D is correct. Bank has not established any right to the specific proceeds of Client's personal injury judgment as would be needed to put the funds in dispute under MRPC 1.15. There is no state lien or other legal right that has been demonstrated to Attorney, who would therefore have no duty to Bank and would not be subject to discipline for following Client's instructions. Answer choice A is not correct, as in the absence of Bank demonstrating a legal right to the specific funds at issue, Attorney may properly release the funds to Client. Answer choice B is not correct, as in the absence of being shown a legal right to the specific proceeds of the judgment, Attorney owed no duty to Bank with regard to the proceeds. Answer choice C is incorrect, as it is irrelevant whether Client would have sufficient funds to pay any subsequent judgment obtained by bank. Attorney is not subject to discipline because Bank has not established a legal right to the specific proceeds of the personal injury judgment.

An attorney participates in a prepaid legal services plan providing legal services to the plan's members. The attorney is not involved in the solicitation of persons to become members, but he does manage the plan for the plan's sponsor. In this role, the attorney acts to ensure that the plan sponsor adheres to the rules governing lawyer advertising and solicitation. Employees of the plan's sponsor, none of whom are attorneys, do solicit by phone individuals to become members of the plan, but they do not target persons who are known to need legal services in a particular matter. Is the attorney subject to discipline? No, because the attorney is not involved in the solicitation of persons to become members of the plan. No, because the attorney acts to ensure that the plan sponsor adheres to the rules on lawyer advertising and solicitation. Yes, because employees of the plan's sponsor engage in real-time solicitation. Yes, because the attorney manages the legal services plan.

Answer choice D is correct. Generally, a lawyer is prohibited from engaging in in-person or other real-time contact to solicit professional employment for pecuniary gain. A lawyer may provide legal services through a prepaid or group legal services plan that solicits membership in the plan through real-time contact, but not if the plan is owned or directed by a lawyer or law firm that participates in the plan. Answer choice A is incorrect because, even though the attorney is not involved in the solicitation of persons to become members of the plan, the attorney's management of a plan that engages in real-time solicitation of members, coupled with the attorney's participation in the plan as a provider of legal services, violates the solicitation limits on lawyers. Answer choice B is incorrect because, although an attorney who participates in a prepaid or group legal services plan is charged with being reasonably assured that the plan sponsors are in compliance with the advertising and solicitation rules, the attorney in this case is violating those rules by being both a participant in and manager of the plan. Answer choice C is incorrect because a prepaid or group legal services plan may engage in real-time solicitation of individuals to be members of the plan, provided that individuals who are known to need legal services in a particular matter are not targeted.

A witness was subpoenaed to appear and testify at a state legislative committee hearing. The witness retained an attorney to represent her at the hearing. During the hearing, the attorney, reasonably believing that it was in the witness's best interest not to answer, advised the witness not to answer certain questions on the grounds that the witness had a constitutional right not to answer. The committee chairperson directed the witness to answer and cautioned her that refusal to answer was a misdemeanor and that criminal prosecution would be instituted if she did not answer. Upon the attorney's advice, the witness persisted in her refusal to answer. The witness was subsequently convicted for her refusal to answer. Is the attorney subject to discipline? Yes, because his advice to the witness was not legally sound. Yes, because the witness, in acting on the attorney's advice, committed a crime. No, if the offense the witness committed did not involve moral turpitude. No, if the attorney reasonably believed the witness had a legal right to refuse to answer the questions.

Answer choice D is correct. If the attorney gave the best advice he could and reasonably believed that the witness had a legal right to refuse to answer, the attorney would not be subject to discipline under the Model Rules for such advice. Answer choice A is not correct, as the accuracy of the attorney's advice is irrelevant. What controls is whether or not it was reasonable for the attorney to believe that his advice to the witness was correct. Answer choice B is not correct, as it does not matter for purposes of subjecting the attorney to discipline under the ethics rules that the advice provided by the attorney resulted in the client having committed a crime. What controls is whether or not it was reasonable for the attorney to believe his advice was correct. Answer choice C is not correct, as the issue of whether the offense committed by the witness involved moral turpitude has no relevance to whether the attorney should be subject to discipline under the Model Rules.

Attorney Alpha, a sole practitioner, recently suffered a heart attack and was advised that she could not return to work for six months. Alpha delivered all of her clients' files to Attorney Beta, who is also a sole practitioner. Beta agreed to review each client's file promptly, take any action necessary to protect each client's interests, and treat the information in the files as confidential. Alpha then wrote her clients, advising each client that the client's file had been delivered to Beta for review and for any action necessary to protect the client's interest, and that the client was free to select another lawyer. Alpha knows that Beta is a competent attorney. Beta did not accept the file of any person whose interests were, or could be, adverse to the interests of any of Beta's own clients. Was it proper for Alpha to deliver the files to Beta for review? Yes, because Alpha knows that Beta is competent to protect the clients' interests. Yes, because Beta agreed to treat the information in the files as confidential. Yes, because given her medical condition, Alpha's delivery of the files was necessary to protect the clients' interests. No, because Alpha did not obtain the prior consent of each client whose file was delivered to Beta.

Answer choice D is correct. In general, MRPC 1.6(a) prohibits a lawyer from revealing information relating to the representation of a client without client consent. A lawyer practicing in a law firm is impliedly authorized to disclose client confidences to other lawyers within the law firm, see MRPC 1.6, cmt. [5], but not to lawyers outside the firm. There is an exception when a lawyer seeks legal advice from another lawyer about how to comply with the ethics rules, but that exception does not apply here. Therefore, the lawyer needed client consent. Answer choice A is incorrect because even if Beta is competent, Alpha needs a client's consent to engage Beta in the representation and to disclose the client's confidences to Beta. The disclosure to Beta was not "impliedly authorized in order to carry out the representation," because a client would not anticipate that his or her lawyer, who is a sole practitioner, would transfer the client's matters to a different lawyer. Answer choice B is incorrect because even if Beta agrees to keep the client's information confidential, the client's consent is needed to engage Beta and to share the client's confidences with Beta. Subject to exception, MRPC 1.6(a) generally prohibits a lawyer from revealing information relating to the representation of a client without client consent. No exception applies here. Answer choice C is incorrect. If Alpha had first secured the clients' consent, it would have been permissible to deliver the files to Beta. But, absent client consent, the disclosure of clients' confidences to Beta was prohibited by MRPC 1.6(a).

An attorney represents a client who is under indictment for homicide. In the course of the representation, the client told the attorney that she had previously killed two other people. These murders are completely unrelated to the murder indictment for which the attorney is providing representation. With the client's consent, the attorney made a tape recording of the client's confession regarding the unrelated homicides. At the attorney's request, the client also drew a map of the remote locations of the victims' graves from the unrelated killings. Those bodies have not been found by the police, and the client is not a suspect in either crime, both of which remain unsolved. Is the attorney subject to discipline if he fails to voluntarily disclose to the authorities his knowledge of the two prior murders and the locations of the victims' bodies? Yes, because as an officer of the court, the attorney must disclose any knowledge that he has, whether privileged or not, concerning the commission of the prior crimes by his client. Yes, because the attorney is impeding the state's access to significant evidence. No, because the attorney did not represent or advise his client with respect to the prior crimes. No, because the information was obtained by the attorney in the course of the representation.

Answer choice D is correct. In general, MRPC 1.6(a) provides that "[a] lawyer shall not reveal information relating to the representation of a client unless the client gives informed consent." Further, no exception to the confidentiality rule allows or requires the attorney to disclose the information. Answer choice A is incorrect because there is no exception to MRPC 1.6(a), the confidentiality rule, permitting or requiring a lawyer to disclose information relating to a client's representation when the information concerns the client's past crimes of violence. There are exceptions that allow the lawyer to disclose client confidences in certain circumstances (e.g., to prevent a future death or to prevent or rectify financial harm caused by a crime or fraud involving the lawyer's services), but these exceptions do not apply here. Answer choice B is incorrect because criminal laws involving obstruction of justice, as well as judicial opinions, require a lawyer under certain circumstances to disclose physical evidence of a crime when the lawyer possesses that evidence. But the laws and opinions regarding obstruction of justice do not require disclosure of the client's confidential information in this case. Answer choice C is incorrect because the attorney's knowledge is information "relating to the representation" of the client in the homicide case, and therefore it must be kept confidential under MRPC 1.6(a), even though the information relates to murders other than the one for which the client is on trial. The information about the undiscovered homicides would potentially be relevant to the attorney in representing the client competently with regard to the crime charged. For example, the lawyer's knowledge of this information might influence the lawyer's advice to the client about whether to plead guilty or whether to testify at trial, and the information might influence the lawyer's strategic decisions.

An assistant district attorney was in charge of the presentation before a grand jury of evidence that led to an indictment charging 32 defendants with conspiracy to sell controlled drugs. Shortly after the grand jury returned the indictments, the attorney resigned as assistant district attorney and became an associate in the law office of a sole practitioner. At the time of such association, the sole practitioner was the attorney for one of the indicted co-defendants. Is it proper for the former sole practitioner to continue to represent the co-defendant? Yes, if the associate attorney does not reveal to the former sole practitioner any confidence or secret learned while an assistant district attorney. Yes, because a public prosecutor must make timely disclosure to the defense attorney of any exculpatory evidence. No, unless the associate attorney agrees not to participate in the representation of the co-defendant. No, because the associate attorney had substantial responsibility for the indictment of the co-defendant.

Answer choice D is correct. MRPC 1.11(a) prohibits a former government lawyer from representing a client in a matter in which the lawyer participated personally and substantially as a government lawyer unless the appropriate government agency gives its informed consent, confirmed in writing, to the representation. If the former government lawyer is disqualified from representation, no lawyer in his firm may knowingly undertake or continue representation in such matter unless: (i) the disqualified lawyer is timely screened from any participation in the matter and gets no part of any fee from the matter, and (ii) written notice is promptly given to the appropriate government agency to enable it to ascertain whether the lawyer and the firm are in compliance with the conflict rules. MRPC 1.11(b). The fact pattern does not indicate that the associate attorney has met the screening and notice requirements for continued representation of the co-defendant by the former sole practitioner. Answer choices A and C are incorrect because while they address the screening requirement of MRPC 1.11(b), they do not address the written notice requirement of MRPC 1.11(b). Answer choice B is incorrect because it is a rule that does not apply to this fact pattern.

An attorney prepared a will for a client and acted as one of the subscribing witnesses to the client's execution of the will. The will left all of the client's estate to the client's son. Later, at the client's request, the attorney prepared a second will for the client and acted as one of the subscribing witnesses to the client's execution of the second will. The second will left one-half of the client's estate to the son and the other one-half to the client's housekeeper. The client died and the housekeeper has offered the second will for probate. If the son requests the attorney to represent him in opposing probate of the second will on the grounds of fraud and undue influence, is it proper for the attorney to do so? Yes, because after the client's death, the attorney may represent the son. Yes, because the son is a beneficiary under both wills. No, because an attorney guarantees the validity of a will that she prepares. You Selected: No, because the attorney would be taking a position adverse to a will she prepared and witnessed.

Answer choice D is correct. MRPC 1.9(a) provides that a lawyer who has formerly represented a client in a matter shall not thereafter represent another person in the same or a substantially related matter in which that person's interests are materially adverse to the interests of the former client unless the former client gives informed consent, confirmed in writing. The son is requesting that the attorney represent him and take a position that would be adverse to the will she prepared and witnessed. This is improper under MRPC 1.9(a). Answer choice A is not correct, as the death of the former client does not override a lawyer's responsibilities under MRPC 1.9(a). Answer choice B is not correct, as even though the son is a beneficiary under both wills, he stands to lose one-half of the estate if the second will is accepted and thus would be materially adverse to the acceptance of that will. Answer choice C is not correct, as even if this were the case, it does not prohibit the attorney from representing the son. MRPC 1.9 prohibits the representation.

An attorney is a well-known tax lawyer and author. During congressional hearings on tax reform, the attorney testified to her personal belief and expert opinion on the pending reform package. She failed to disclose in her testimony that she was being compensated by a private client for her appearance. In her testimony, the attorney took the position favored by her client, but the position was also one that the attorney believed was in the public interest. Was it proper for the attorney to present this testimony without identifying her private client? Yes, because the attorney believed that the position she advocated was in the public interest. Yes, because Congress is interested in the content of the testimony and not who is paying the witness. No, because a lawyer may not accept a fee for trying to influence legislative action. No, because a lawyer who appears in a legislative hearing should identify the capacity in which the lawyer appears.

Answer choice D is correct. MRPC 3.9 requires a lawyer representing a client before a legislative body to disclose that the appearance is in a representative capacity. Answer choice A is incorrect, as the attorney's conduct was improper because MRPC 3.9 requires a lawyer representing a client before a legislative body to disclose that the appearance is in a representative capacity. Answer choice B is incorrect because the congressional members at the hearing would be very interested in knowing whether a witness is possibly biased in her testimony; therefore, MRPC 3.9 requires a lawyer appearing in a representative capacity to disclose that fact to the legislative body. Answer choice C is incorrect; lawyers frequently serve as paid lobbyists. This activity is permissible so long as the lawyer-lobbyist complies with applicable rules, such as the requirement to disclose her representative capacity when appearing before a legislative body.

An attorney represented a seller in negotiating the sale of his ice cream parlor. The seller told the attorney in confidence that, although the business had once been very profitable, recent profits had been stable but modest. As the negotiations proceeded, the buyer appeared to be losing interest in the deal. Hoping to restore the buyer's interest, the attorney stated, "The ice cream business is every American's dream: happy kids, steady profits, and a clear conscience." The buyer bought the ice cream parlor but was disappointed when his own profits proved to be modest. Is the attorney subject to discipline? Yes, because the attorney made a false statement of fact to the buyer. Yes, because the attorney exaggerated the profitability of the business. No, because the attorney represented the seller, not the buyer. No, because the attorney's statement constitutes acceptable puffing in negotiations.

Answer choice D is correct. MRPC 4.1 prohibits lawyers from making false statements of material facts in negotiations. According to the Comment to the Rule, generally accepted conventions in negotiation, including certain types of statements such as estimates of price or value, ordinarily are not taken as statements of material fact. In this case, to the extent that the attorney may have exaggerated the value of his client's business, this exaggeration would not be considered a statement of material fact. Answer choice A is incorrect, as MRPC 4.1 prohibits lawyers from making false statements of material facts in negotiations. Answer choice B is incorrect because an exaggeration such as the one made by the attorney in this case is not ordinarily considered a statement of material fact. Answer choice C is incorrect because MRPC 4.1 prohibits lawyers from making false statements of material fact in negotiations. It is irrelevant which party the attorney represents.

An attorney was retained by a woman to advise her in negotiating a separation agreement with her husband. The husband, who was not a lawyer, had decided to act on his own behalf in the matter. The attorney never met or communicated with the husband during the negotiations. After several months, the woman advised the attorney that the parties had reached agreement and presented him with the terms. The attorney prepared a proposed agreement that contained all of the agreed-upon terms. The attorney mailed the proposed agreement to the husband, with a cover letter stating: "As you know, I represent your wife in this matter and I do not represent your interests. I enclose two copies of the separation agreement that I have drafted in accordance with my client's directions. Please read the agreement and, if it meets with your approval, sign both copies before a notary and return them to me. I will then have your wife sign them and will furnish you with a fully executed copy." Is the attorney subject to discipline? Yes, because the attorney did not suggest that the husband seek the advice of independent counsel before signing the agreement. Yes, because the attorney directly communicated with an unrepresented person. No, because the attorney acted only as a scrivener. No, because the attorney's letter did not imply that the attorney was disinterested and the attorney did not give legal advice to the husband.

Answer choice D is correct. MRPC 4.3 permits a lawyer to contact an unrepresented person so long as the lawyer does not state or imply that the lawyer is disinterested. By clearly identifying the wife as his client, the attorney complied with this rule. Answer choice A is incorrect; MRPC 4.3 applies to a lawyer's dealings with an unrepresented person. It requires that the lawyer not state or imply that the lawyer is disinterested and that the lawyer not give legal advice to the unrepresented person. The attorney did neither here. Although the rule permits the lawyer to advise the unrepresented person to confer with counsel, it does not require the lawyer to do so. Answer choice B is incorrect; MRPC 4.3 expressly permits lawyers to contact unrepresented persons, so long as they follow certain requirements in dealing with those persons. Answer choice C is incorrect because the attorney did not act as a scrivener or otherwise on behalf of the husband. As the attorney clearly stated, the attorney was acting solely on behalf of the wife in this matter. Whether the attorney acted as drafter or scrivener with respect to the proposed agreement is not relevant to the question of whether his contact with the husband was proper.

A state court judge has presided over the pre-trial proceedings in a case involving a novel contract question under the Uniform Commercial Code. During the pre-trial proceedings, the judge has acquired considerable background knowledge of the facts and law of the matter and, therefore, is particularly well qualified to preside at the trial. Shortly before the trial date, the judge discovered that his brother owns a substantial block of stock in the defendant corporation. He determined that his brother's financial interests would be substantially affected by the outcome of the case. Although the judge believed he would be impartial, he disclosed to the parties, on the record, his brother's interest. Is it proper for the judge to hear the case? Yes, because the judge is particularly well qualified to preside at the trial. Yes, because the judge believes his judgment will not be affected by his brother's stockholding. No, because disqualification based on a relative's financial interest cannot be waived. No, unless after proper proceedings in which the judge did not participate all parties and their lawyers consent in writing that the judge may hear the case.

Answer choice D is correct. The Code of Judicial Conduct generally prohibits a judge from hearing a matter when the judge knows that he shares a third-degree relationship or closer to a person who has more than a de minimis interest that could be affected by the proceedings. CJC Rule 2.11(A)(2). A third-degree relationship includes a brother. Here, the judge determined that his brother's financial interests would be substantially affected by the outcome of the case. A judge who is subject to disqualification under the CJC, other than for bias or prejudice, may disclose on the record the basis of the judge's disqualification and may ask the parties and their lawyers to consider, outside the presence of the judge and court personnel, whether to waive disqualification. If, after such disclosure, the parties and lawyers agree, without participation by the judge or court personnel, that the judge should not be disqualified, the judge may participate in the proceeding. CJC Rule 2.11(C). Unless such proceedings take place and written consent is obtained, it would not be proper for the judge to hear the case. Answer choice A is not correct, as the CJC disqualifies the judge from hearing the matter, even if he is well qualified. Answer choice B is not correct, because even if the judge believes his judgment will not be affected, he is still required to disqualify himself under the CJC due to his brother's interest in the corporation. Answer choice C is not correct, as CJC Rule 2.11(C) does permit such disqualification to be waived.

After working for several years in the civil division of a state attorney general's office, an attorney left the attorney general's office and joined a private law firm. The law firm represented a defendant in the appeal of his criminal conviction, which had been obtained by the criminal division of the attorney general's office while the attorney was employed in the civil division. The attorney was assigned to the team representing the defendant. The attorney did not seek the consent of the attorney general's office to the attorney's participation in the firm's representation of the defendant. Is the attorney's participation in the firm's representation of the defendant in an appellate action proper? No, because the attorney did not obtain the attorney general's consent prior to his representation of the defendant. No, because the defendant's conviction was obtained by the attorney general's office while the attorney worked there. Yes, because the attorney did not acquire confidential government information about the client while working at the attorney general's office. Yes, because the attorney did not participate in the defendant's conviction while working at the attorney general's office.

Answer choice D is correct. The Model Rules of Professional Conduct prohibit a former government attorney from representing a client in a matter in which the attorney participated personally and substantially as a government attorney, unless the appropriate government agency gives its informed written consent. In this case, the attorney worked on civil matters exclusively during his government tenure and did not work on the matter in question. Accordingly, his representation of the client would be proper even without informed consent. Answer choice A is incorrect because the attorney general's office did not need to consent, as the attorney did not work on the matter. Answer choice B is incorrect because although the matter was ongoing while the attorney worked in the attorney general's office, the attorney did not participate in the matter and thus would not be prohibited from representing the client. Answer choice C is incorrect. A government attorney who acquires confidential government information about a person may not later represent a private client whose interests are adverse to that person in a matter in which the information could be used to that person's material disadvantage. In this case, even assuming the attorney gained information about the defendant, the defendant was now the attorney's client.

An attorney is defending a client who has been indicted for burglary. During an interview, the client stated to the attorney that before he had consulted her, he had committed perjury while testifying before the grand jury that indicted him. The attorney is subject to discipline if she: Continues to represent the client. Continues to represent the client unless he admits his perjury. Does not inform the authorities of the perjury. Informs the authorities of the perjury.

Answer choice D is correct. The attorney learned the information about the perjury during the course of representing the client; therefore it is protected by the ethical duty of confidentiality under MRPC 1.6, and the attorney will be subject to discipline if she informs the authorities of the perjury. Answer choice A is not correct, as the attorney may continue to represent the client even though she knows of the client's past perjury. Only if the client attempts to commit perjury as part of evidence being offered by the attorney in the future would the attorney have a potential obligation to withdraw from representation. Answer choice B is not correct, as the perjury here was committed in the past, before the attorney was involved in the case. The client does not have to reveal the past perjury for the attorney to continue representation. Only if the client attempts to commit perjury as part of evidence being offered by the attorney in the future would the attorney have a potential obligation to withdraw. Answer choice C is not correct, as the attorney is obligated not to inform the authorities because of the ethical duty of confidentiality under MRPC 1.6.

An attorney is a candidate for a judicial office that has been occupied by an incumbent for six years. The attorney has conducted a thorough investigation of the incumbent's personal and professional life. Assuming that all factual statements are accurate, which of the following statements is improper for the attorney to make during the campaign? "The incumbent has been reversed by the appellate courts more than any other judge in the state during the preceding two years." "The incumbent was publicly censured by the state Judicial Qualification Commission on one occasion for his overbearing conduct in court." "The incumbent was given a poor rating for judicial temperament in a county bar association poll." "During the previous year, the average sentence in armed robbery cases tried in the incumbent's court was 3.5 years, and in murder cases was 8.2 years. If I am elected, I won't be soft on crime."

Answer choice D is correct. The statement is improper, as it appears to be a pledge or commitment by the attorney, as candidate for judicial office, to take a position on cases that are likely to come before the court, in violation of CJC Rule 4.1. Answer choices A, B, and C are all proper, as they are true factual statements.

An attorney was retained by a defendant to represent him in a paternity suit. The defendant's aunt believed the suit was unfounded and motivated by malice. The aunt sent the attorney a check for $1,000 and asked the attorney to apply it to the payment of the defendant's fee. The aunt told the attorney not to tell the defendant of the payment because, "The defendant is too proud to accept gifts, but I know he really needs the money." Is it proper for the attorney to accept the aunt's check? Yes, if the aunt does not attempt to influence the attorney's conduct of the case. Yes, if the attorney's charges to the defendant are reduced accordingly. No, because the aunt is attempting to finance litigation to which she is not a party. No, unless the attorney first informs the defendant and obtains the defendant's consent to retain the payment.

Answer choice D is correct. Under MRPC 1.8(f), a lawyer may not accept payment for representation from someone other than the client, unless (among other things) the client gives informed consent. In the absence of informed consent by the defendant, it is not proper for the attorney to accept the aunt's check. Answer choice A is not correct. MRPC 1.8(f) requires that in order for a lawyer to accept payment for representation from someone other than the client, there must be no interference with the lawyer's professional judgment and there must be informed consent by the client. Answer choice B is not correct, as there is no requirement that the attorney's charges be reduced. All that is required for a lawyer to accept payment for representation from someone other than the client is that: (i) the client gives informed consent, (ii) there is no interference with the lawyer's professional judgment, and (iii) lawyer-client confidentiality is preserved. MRPC 1.8(f). Answer choice C is not correct. MRPC 1.8(f) permits such financing, so long as the above conditions are met.

An attorney's standard retainer contract in divorce cases provides for the payment of a fee of one-third of the amount of alimony or property settlement secured by the attorney. The attorney declines to represent clients who do not agree to this arrangement. Is the attorney's standard retainer contract proper? Yes, because clients often prefer to pay a lawyer a fee based on the outcome of the case. Yes, if a fee of one-third is not excessive. No, because a lawyer may not acquire a proprietary interest in a cause of action. No, because the fee is contingent.

vAnswer choice D is correct. Under MRPC 1.5(d)(1), a lawyer may not charge a fee in a domestic relations case that is contingent on obtaining a divorce or on the amount of support recovered. Answer choice A is not correct, as client preferences do not override the Model Rules, which prohibit a contingent fee under these circumstances. Answer choice B is not correct, as whether the fee is excessive is not relevant, since a contingent fee is prohibited here by the Model Rules. Answer choice C is not correct because the proprietary interest issue is irrelevant. MRPC 1.5(d)(1) prohibits charging a contingent fee under these circumstances.


संबंधित स्टडी सेट्स

Chapter 38: OThe nurse is caring for an older adult client on home oxygen who has dentures, but has quit wearing them stating that the dentures irritate the gums. What nursing action is appropriate?xygen Perfusion

View Set

history of rock test 3 with adam knight

View Set

Learning: Module 1: Section 2_01-2_02

View Set

APES Unit 5 Progress Check A and B

View Set

PSYCH 104 - Ch 8 Hypothesis Testing TEST

View Set